You are on page 1of 227

FARKALEET SERIES

APPLIED CALCULUS
(Revised Edition)

AUTHORS:
Prof. (R) Muhammad Urs Shaikh
Prof. Dr. Asif Ali Shaikh
Dr. Sania Nizamani

CO-AUTHORS:
Muhammad Tahir Wahocho (M.Sc)
Dr. Wajid Ali Shaikh

Edited by:
Prof. Dr. Feroz Shah

JAHANGIR PUBLICATIONS, LARKANA


Cell: 0333-756-5595, 0304-453-2100
PREFACE TO FIRST EDITION
About thirty years ago we wrote a series of books on different subjects of
Engineering Mathematics. The series was named “FARKALEET SERIES”.
FARKELEET means MUHAMMAD (S.A.W.S). This name of our Prophet (S.A.W.S) is
mentioned in the BIBLE.
The series though was hand written but became very much popular among our
students at Mehran University of Engineering and Technology, Jamshoro, Pakistan.
The series also gained popularity in the students of other universities and colleges
of Sindh as well. Many teachers of different universities and colleges also
appreciated such efforts of mine. The reason is that each topic was clearly
explained and that every student as well as teacher enjoyed reading without any
difficulty.
The book “APPLIED CALCULUS” is in your hands. Each topic is explained in well
manner and in detail as well.
We shall be looking forward to hear from the readers any critic that will improve the
standard of the book.
We are indebted to Prof. Dr. Feroz Shah who had edited the book and has tried to
remove any king of typographical mistakes or otherwise.

Prof.(R) Muhammad Urs Shaikh


Prof. Dr. Asif Ali Shaikh
Dr. Sania Nizamani
CO-AUTHORS:
Muhammad Tahir Wahocho (M.Sc)
Dr. Wajid Ali Shaikh
Edited by:
Prof. Dr. Feroz Shah

PREFACE TO SECOND EDITION

In the first edition, the book was containing some typo-graphical mistakes as well
as mathematical errors. These mistakes/errors have been now removed from the
first edition. We hope that, students will be much comfortable with the new revised
edition. However, if still any mistake is found, we will appreciate if the same may be
conveyed to the authors.

Prof. (R) Muhammad Urs Shaikh


CON TENTS

PAGE #
CHAPTER ONE FUNCTIONS AND THEIR APPLICATIONS
What is calculus 1
Contributions of Mathematicians to Calculus 2
Real Numbers 3
Absolute Value 4
Solving Inequalities 6
Introduction to Functions 9
Domain, Co-domain and Range of Real Valued Functions 11
Types of Functions 12
Algebra of Functions 13
Composite Function 13
Graph of a Function 17
Circular and Hyperbolic Functions 23
Physical Applications of Functions 26
Worksheet 01 27
CHAPTER TWO LIMIT AND CONTINUITY OF FUNCTIONS
The concept of Limit 30
Definitions of Limits 31
Formal Definition of Limit 31
Some useful limits 38
Continuous and Discontinuous Functions 47
   Definition of Continuity 49
Case Study Problems 50
Worksheet 02 52
CHAPTER THREE DERIVATIVES
Introduction 55
Procedure to Find the Derivative of a Function 55
One Sided Derivative 57
Derivative at a Point 57
Geometrical Meaning of Derivative 60
Physical Meaning of Derivative 60
Rules of Derivatives 61
Derivatives of Trigonometric Functions 63
Derivatives of Inverse Trigonometric Functions 64
Derivatives of Exponential and Logarithmic Functions 66
Derivatives of Hyperbolic and Inverse Hyperbolic Functions 69
Implicit Differentiation 70
Logarithmic Differentiation 71
Parametric Differentiation 75
Case Study Problems 75
Rates of Change 75
Related Rates 81
Marginal Analysis 82
Higher Derivatives 85
th
Standard n Derivatives 85
Leibniz Theorem 88
Worksheet 03 90
CHAPTER FOUR PARTIAL DERIVATIVES
Introduction 93
Partial Derivatives 94
Geometrical Meaning of Partial Derivative 95
Partial Derivatives of Higher Orders 95
Differentiability 97
Worksheet 04 97
CHAPTER FIVE MEAN VALUE THEOREMS AND INDETERMINATEFORMS
Roll’s Theorem 99
Mean Value Theorem 102
Infinite Series 104
Maclaurin’s Series 105
Taylor’s Series 107
L’ Hopital’s Rule and Indeterminate Forms 110
Applications of Limit, Asymptotes 113
Worksheet 05 116
CHAPTER SIX APPLICATIOONS OF DERIVATIVES
Tangent and Normal 120
Curvature and Radius of Curvature 123
Maxima and Minima of a Function of one Variable 127
Applications of Maxima and Minima 131
Differentials and their Applications 135
Worksheet 06 138
CHAPTER SEVEN CALCULUS OF SEVERAL VARIABLES
Introduction 141
Homogenous Functions 141
Euler’s Theorem on Homogeneous Functions 142
Total Differentials 144
Maxima and Minima of a Function of two Variables 145
Worksheet 07 149

CHAPTER EIGHT INTEGRATION AND IT’S APPLICATIONS


Introduction 151
Methods of Integration 152
Integration by Formulas 152
Integration by Substitution 154
Integration by Parts 158
Integration of Rational Algebraic Functions 160
Integration by Completing the Squares Method 164
Integration of Irrational Algebraic Functions 165
Five Standard Cases 167
Worksheet 08 169
CHAPTER NINE DEFINITE INTEGRATION
Introduction 172
Area and the Definite Integral 172
Fundamental Theorem of Calculus 175
Fundamental Properties of Definite Integration 175
Beta and Gamma Functions 180
Relation between Beta and Gamma Functions 183
Improper Integrals 184
Reduction Formulas 185
Applications of Integration 198
Worksheet 09 201
CHAPTER TEN VECTOR ANALYSIS
Introduction 202
Rectangular Unit Vectors and their Representation 203
Product of Two Vectors 204
Scalar or Dot Product of Two Vectors 204
Geometrical Interpretation of dot product 204
Vector or Cross Product of Two Vectors 205
Scalar Triple Product or Box Product 206
Vector Triple Product 208
Scalar Product of Four Vectors 208
Vector Product of Four Vectors 289
Vector Functions 209
Differentiation of Vector Functions 209
Integration of Vector Functions 209
DEL – The Differential Operator 210
The Gradient 211
Geometrical Interpretation of Gradient 211
Directional Derivative 211
The Divergence of a Vector Function 215
Physical Interpretation of Divergence 216
The Curl of a Vector Function 218
Two Important Theorems 219
Physical Interpretation of curl 220
2
  Operator 221
Worksheet 10 222
CHAPTER ONE FUNCTIONS AND THEIR APPLICATIONS

CHAPTER
ONE FUNCTIONS AND
THEIR APPLICATIONS

WHAT IS CALCULUS
Calculus is the mathematics of motion and change. Where there is motion or growth, where
variable forces are at work producing acceleration, calculus is the right mathematics to apply.
This was true in the beginnings of the subject and it is true today.
Calculus was first invented to meet the mathematical needs of the scientists of the sixteenth and
seventeenth centuries, needs that were mainly mechanical in nature. There are two main branches
of calculus called Differential Calculus and Integral Calculus.
Differential calculus deals with the problem of calculating rates of change. It enabled people to
define slopes of curves, to calculate velocities and accelerations of moving bodies, to find firing
angles that would give cannons their greatest range, and to predict the times when planets would
be closest together or farthest apart. Integral calculus deals with the problems of determining a
function from information about its rate of change. It enabled people to calculate the future
location of a body from its present position and knowledge of the forces acting on it, to find the
areas of irregular regions in the plane, to measure the lengths of curves, and to find the volumes
and masses of arbitrary solids.
Today, applications of calculus and its extensions in mathematical analysis are far reaching
indeed, and the physicists, mathematicians and astronomers who first invented the subject would
surely be amazed and delighted, as we hope you will be, to see what a large number of problems
it solves and what a range of fields now use it in the mathematical models that bring
understanding about the universe and the world around us.
Calculus is widely employed in the physical, biological and social sciences. It is used, for
example, in the physical sciences to study the speed of a falling body, the rates of change in a
chemical reaction, or the rate of decay of a radioactive material. In the biological sciences, a
problem such as rate of growth of a colony of bacteria as a function of time is easily solved using
calculus. In the social sciences calculus is widely used in the study of Statistics, Probability and
optimization problems.
Calculus can be applied to many problems involving the notion of extreme values, such as the
fastest, the most, the slowest or the least. These maximum or minimum amounts may be
described as values for which a certain rate of change (increase or decrease) is zero. By using
calculus it is possible to determine how high a projectile will go by finding the point at which its
change of altitude with respect to time, that is, its velocity, is equal to zero.

-:1:-
CHAPTER ONE FUNCTIONS AND THEIR APPLICATIONS
The invention of calculus had a great impact on technology as well as on the development of
mathematics. Years later, applications of calculus were found in a variety of non – engineering
areas, including business and economics, biology, medicine, sociology and psychology. Calculus
can be used to:
 Determine the average speed at which blood flows through an artery.
 Select the most economical dimensions for packaging.
 Calculate how high a projectile will travel.
 Find the production level that will maximize a company’s
profit.
CONTRIBUTIONS OF MATHEMATICIANS TO CALCULUS
The Englishman Isaac Newton (1642 – 1727) and the German are
the mathematicians credited with inventing calculus. They worked
independently of each other. Newton invented calculus in 1665
but took more than 20 years to publish his results hence Leibniz’s
development of calculus was published first. Furthermore, Leibniz’s
notation was considered superior to Newton’s notation, and it is still
used today.
HOW TO LEARN CALCULUS
Learning calculus is not the same as learning arithmetic, algebra and
geometry. In these subjects, you learn primarily how to calculate with
numbers, how to simplify algebraic expressions and calculate with variables and how to reason
about points lines and figures in the plane. Calculus involves those techniques and skills but
develops others as well, with greater precision and a deeper level. Calculus introduces so many
new concepts and computational operations, in fact, that you will no longer be able to learn
everything you need in class. You will have to learn a fair amount on your own by working with
other students.
 Read the text: You will be able to learn all the meanings and connections you need just
by attempting the exercises. You will need to read relevant passages in the book and
work through examples step-by-step. Speed-reading will not work here. You are reading
and searching for detail in a step-by-step logical fashion. This kind of reading, required
by any deep and technical content, takes attention, patience and practice.
 Do the given work in time, keeping the following principles in mind:
 Sketch diagrams whenever possible.
 Write your solutions in a connected step-by-step logical fashion, as if
you were explaining to someone else.
 Think about why each exercise is there? Why was it assigned? How it is
related to the other assigned exercises?
 Use your calculator and computer whenever possible: Graphs provide insight and
visual representations of important concepts and relationships.
 Try to write your own: Write short descriptions of the key points each time you
complete a section of the text. If you succeed, you probably understand the material. If
you do not, you will know where there is a gap in your understanding.
-:2:-
CHAPTER ONE FUNCTIONS AND THEIR APPLICATIONS
Learning calculus is a process; it does not come all at once. Be patient, keep trying asking
questions, discuss ideas, work with classmates and seek help when you need it. This is the right
away to learn calculus. The rewards of learning calculus will be very satisfying, both
intellectually and professionally.
REAL NUMBERS
The study of elementary calculus requires knowledge of the real number system. The real
numbers can be considered points on a line. To every real number there corresponds one point.
To every point there corresponds one real number. (See figure below)
–5/2 1/2 3 2.63 
     
–4 –3 –2 –1 0 1 2 3 4
Inequalities can be used to compare real numbers. The symbols used are > (greater than),
< (less than),  (greater than or equal to), and  (less than or equal to). Examples of inequalities
are: x > 3, y < 4, x ≥ 5, y ≤ –2.
Finite Intervals
Let a and b be two real numbers with a < b. Then the set of all real numbers x such that
i. a < x < b is called an open interval and is denoted by ]a, b[ or (a, b). The open interval is
defined as: (a, b) = {x | x  R and a < x < b}
Graphically, this is represented by a continuous line on the horizontal axis which include all real
numbers between `a` and `b` with `a` and `b` are excluded.

a . b
ii. a ≤ x ≤ b is called a closed interval and is denoted by [a, b]. The closed interval is defined as
[a, b] = {x | x  R and a ≤ x ≤ b}
Graphically, this is represented by a continuous line on the horizontal axis which include all real
numbers between `a` and `b` with `a` and `b` are also included.

a . b
iii. a < x ≤ b is called half open-half closed interval and is denoted by (a, b]. This interval is
defined as: (a, b] = {x | x  R and a < x ≤ b}
Graphically, this is represented by a continuous line on the horizontal axis which include all real
numbers between `a` and `b` with `a` is excluded and `b` is included.

a . b
iv. a ≤ x < b is called an half closed and half open and is denoted by [a, b). The half open interval
is defined as: [a, b) = {x | x  R and a ≤ x < b}
Graphically, this is represented by a continuous line on the horizontal axis which include all real
numbers between `a` and `b` with `a` is included and `b` is excluded.

It may be noted that: a . b


i. [a, b] – (a, b) = {a, b} ii. (a, b) – [a, b] = φ iii. [a, b]  (a, b) = (a, b)
iv. [a, b]  (a, b) = [a, b] v. {a, b}  (a, b) = [a, b]

-:3:-
CHAPTER ONE FUNCTIONS AND THEIR APPLICATIONS
vi. An interval is said to be finite if its length |b – a| is finite.
vii. Every interval is an infinite set 1
(1) (1, 3) = {x | x  R and 1 < x < 3}
(2) [1, 3] = {x | x  R and 1 ≤ x ≤ 3}.
Graphically, this is represented by a continuous line on the horizontal axis which include all real
numbers between 1 and 3 with 1and 3 are also included.
(3) (1, 3] = {x | x  R and 1 < x ≤ 3} 1 3
(4) [1, 3) = {x | x  R and 1 ≤ x < 3}
(5) (1, 2)  (–1, 3) = (–1, 3) 1 3
(6) (1, 2) – (–1, 3) = φ
(7) (1, 2)  (–1, 3) = (1, 2) 1 3
(8) [-1, 3] – [1, 2] = [-1, 1)  (2, 3]
(9) [1, 5]  [5, 6] = {5}
(10) If U = [1, 5] and A = [2, 5] then A` = [1, 5] - [2, 5] = [1, 2)
Infinite Interval x≥a
An interval is said to be the infinite if its length is infinite. a
(i) The set [a, ∞) = {x | x  R and x ≥ a} is called closed right ray. x≥2
For example, [2, ∞) = {x | x  R and x ≥ 2} 2
(ii) The set (–∞, a] = {x | x  R and x ≤ a} is called closed left ray.
For example, (–∞, 2] = {x | x  R and x ≤ 2} x≤a a
(iii) The set (a, ∞) = {x | x  R and x > a} is called open right ray.
For example, (2, ∞) = {x | x  R and x > 2} x≤2 2
(iv) The set (–∞, a) = {x | x  R and x < a} is called open left ray. a x>a
(iv) The set (–∞, a) = {x | x  R and x < a} is called open left ray.
(iv) The set (–∞, a) = {x | x  R and x < a} is called open left ray. 2 x>2
For example, (–∞, 2) = {x | x  R and x < 2}
It may be noted that: x<a a
i. (–∞,∞) represents the set of real number
ii. (–∞, a)  (b, ∞) = R – [a, b] x<2 2
iii. (–∞, a]  [b, ∞) = R – (a, b)
Examples:
i. (3, ∞)  (–2, ∞) = (- 2, ∞) ii. [2, ∞)  [1, 3] = [2, 3] iii. [3, ∞) – (2, ∞) = φ
iv. (–∞, 3)  (1, ∞) = (–∞, ∞) v. (–∞, 4) – (2, ∞) = (–∞, 2]
Absolute Value
Let x be a real number. The modulus or absolute value or magnitude of x or numerical value of x,
denoted by | x |, is the distance of x from zero. The absolute value of any number x is also defined as
 x if x  0 Graph of y  | x |

|x|  0 if x  0 y  x y  x
 x if x  0

It may be noted that (i) | x  a |  0 is defined as  a  x  a and
(ii) | x  a | 0 is defined as x  a and x  a. 0

-:4:-
CHAPTER ONE FUNCTIONS AND THEIR APPLICATIONS
These two conditions are shown in the following figures.
–a<x<a x<–a x>a

–a 0 a –a 0 a
On the contrary, | x  a |  implies that a –  < x < a +  where  is a positive real number.
Similarly, | x  a |   implies that x < a –  or x > a + 
For example, | x – 2 | < 0.1 implies that 2 – 0.1 < x < 2 + 0.1 or 1.9 < x < 2.1 and
| x – 2 | ≥ 0.3 means x < 2 – 0.3 or x > 2 + 0.3 giving x < 1.7 or x > 2.3.
Now, we discuss the absolute value of a number x by providing suitable examples.
For example, if x = 2 which is positive then, | x | = 2. If x = –2 which is negative then, | x | = | –2 |
= – (–2) = 2 and if x = 0 then | x | = | 0 | = 0.
Geometrically, | x | = 2 means x lies either two units to the right of zero or it lies two units to the
left of zero.
Since the symbol x always denotes the non–negative square root of x, an alternative definition

of x is: | x | x2

For example, | 2 |  (2)2  4  2 and | 2 | 22  4  2.

It is important to remember that | x |  x 2  x 2 .


The absolute values give distances between two points on the number line. Thus, if |x| = 2 then
geometrically it means the distance on the x-axis that starts from –2 and end at + 2.
The absolute value of x is also defined as: | x | = max(– x, x).
For example, | – 2| = max. (–(–2), –2) = max. (2, –2) = 2.
Neighborhood
Any open interval containing `a` is called a neighborhood of `a`. For example if (b, c) is an open
interval and `a` lies inside (b, c) then (b, c) is neighborhood of `a`. b a c

For example, neighborhood of 1 is (0.5, 1.3).


δ-Neighborhood 0.5 1 1.3

If a  R and δ > 0 then the set {x | x  R, a – δ < x < a + δ} = (a – δ, a + δ) is called δ


neighborhood of a with radius δ. a-δ a a+δ
For example, 0.3 neighborhood of 1 = {x | x  R, 1 – 0.3 < x < 1 + 0.3}
= (0.7, 1.3) 1
Deleted δ –Neighborhood 0.7 1.3
If a  R and δ > 0 then the set
{x | x  R, a – δ < x < a + δ, x ≠ a} = { x  R| 0 < | x – a | < δ } = (a – δ, a)  (a, a + δ) is called
deleted δ- neighborhood.
Example 01: a-δ a a+δ 1.7 2 2.3

The deleted 0.3-neighbourhood of 2 =


= {x | x  R, a- δ < x < a + δ, x ≠ a}
= {x | x  R, 2 – 0.3 < x < 2 + 0. 3, x ≠ 2} = (1.7, 2)  (2, 2.3) (see the above figure)

-:5:-
CHAPTER ONE FUNCTIONS AND THEIR APPLICATIONS
Solving Inequalities
In order to solve inequalities, we first convert the inequality into an equation. This equation is
known as “associated equation”. Solutions of the associated equation are called “boundary
numbers” for this inequality. If a rational expression occurs in equality, then the numbers where
the denominator vanishes are not points in the domain of the rational expression. These numbers
are called “free numbers”. These numbers are not the part of the solution.
After finding the boundary numbers, locate them on the real line. The real line will be divided
into a number of distinct “regions” each of which belongs to the solution set in its “entirety”.
Finally, we check the inequality by taking a point selected from each region. Union of all such
regions constitutes a solution set of the inequality.
Example 02: Solve (i) 3x + 5 < x – 9 (ii) |5x + 6 | ≥ 5 (iii) x2 – 5x + 6 < 0
(iv) x2 – 2x + 2 > 0 (v) [(x2 – 2)/(1 – 2x)] > 1 (vi) |x – 2| < 1
Solution: (i) Since 3x + 5 < x – 9; this implies that: 3x – x < – 9 – 5 or 2x < –14 or x < – 7.
Thus solution set is: S = {x | < – 7} = (–∞,–7).
(ii) By definition, if |5x + 6 | ≥ 5 then:
(5x + 6) ≥ 5  5x ≥ 5 – 6  5x ≥ –1 or x ≥ –1/5
Or, – (5x + 6) ≥ 5  5x + 6 ≤ –5  5x ≤ –5 – 6 or x ≤ –11 or x ≤ –11/5.
Thus the solution set is (–∞,–11/5]  [–1/5, ∞). This solution set is graphically shown below. The
thick line shows the solution set of the given inequality.

-∞ –11/5 –1/5 0 ∞
2 2
(iii) x – 5x + 6 < 0: The associated equation is x – 5x + 6 = 0. After solving the quadratic
equation this gives (x – 2)( x – 3) < 0. This means either x – 2 < 0 and x – 3 > 0 OR x – 2 > 0
and x – 3 < 0. Here, x = 2 and x = 3 are the boundary numbers for the given inequality.
Now, if x – 2 < and x – 3 > 0 then x < 2 and x > 3 which is not possible.
Now, if x – 2 > and x – 3 < 0 then x > 2 and x < 3 which is possible.
This shows that there is only one possibility that will satisfy the given inequality which is the
open interval (2, 3). Let us see this through another way.

A B C
0 2 3
Region A: Put x = 0, we get 6 < 0. Result: False
2
Region B: Put x = 2.5, we get (2.5) – 5(2.5) + 6 = – 0.25 < 0 Result: True
2
Region C: Put x = 4, 4 – 5(4) + 6 = 2 < 0 Result: False
Thus only the region B form the solution set. Therefore the solution set is the open interval (2, 3)
where the given inequality is satisfied. It may be noted that if x is replaced by 2 and 3, the
inequality will not be true. Hence the open interval (2, 3) is the only solution which is shown here
in the form of thick line segment.
(iv) x2 – 2x + 2 > 0: The associated equation is x2 – 2x + 2 = 0. Solving we obtain; x = 1 ± i.
These are complex numbers which can not be represented on the real line. Thus there are no
boundary numbers. We have therefore one region, that is, the entire real line. We observe that
-:6:-
CHAPTER ONE FUNCTIONS AND THEIR APPLICATIONS
given inequality is true for all real value of x. For example; if x = –1, we get 5 > 0 (True); if
x = 0, we get 2 > 0 (True); if x = 1, we get 1 > 0 (True) and if x = 0.5, we get 0.25 – 1 + 2
= 1.25 > 0 (True). Thus given inequality is true for all real values of x.
x2  2
(v) [(x2 – 2)/(1 – 2x)] > 1: The associated equation is  1 or x2 – 2 = 1 – 2x or
1  2x
x2 + 2x – 3 = 0. Solving, we get: x = –3, 1.These are the boundary numbers for the given
inequality.
The denominator of this inequality is 1 – 2x is zero for x = 1/2. This is the free boundary number
for the given inequality hence it can not be in the solution set.
The real line shown below is divided by the boundary numbers as well as by free boundary
numbers.

A B C D
-∞ -3 0 1/2

Region A: Put x = –4, we get (16-2)/(1+8) = 1.56 > 1 Result: True


Region B: Put x = –1, we get (1 – 2)/(1+2) = – 1/3 > 1 Result: False
Region C: Put x = 3/4, we get ((9/16) –2))/(1– (3/2)) = (–1.4)( –0.5) = 0.7 > 1
Result: False
Region D: Put x = 2, we get (4-2)/(1-4) = -2/3 > 1 Result: False
The solution set therefore is the open interval (–∞, –3) that satisfies the given inequality.
Graphically, the solution set is shown by thick line segments in the above figure.
(vi) |x – 2| < 1  –1 < x – 2 < 1  –1 + 2 < x < 1 + 2 or 1 < x < 3.
Example 03: Solve the following: (i) |x – 2| = x – 2 (ii) |x2 – x – 2| = 2 + x – x2
Solution: (i) |x – 2| = x – 2 if x – 2 ≥ 0 or x ≥ 2. for x < 2, the inequality is not satisfied. For
example, let x = – 2 then |– 2 – 2| = –2 – 2 which implies that 4 = –4 which is not true. Thus only
solution is the interval [2, ∞).
(ii) |x2 – x – 2| = | – (2 + x – x2 ) |. This implies that x2 – x – 2 ≤ 0 or (x – 2) (x + 1) ≤ 0
or -1 ≤ x ≤ 2.
Example 04: Find the value of x2 if (i) x < 2 (ii) x > –1 (iii) x ≥ 2 (iv) x < –1
Solution: (i) When x < 2, this means x  (, 0)  [0, 2) . Now, if x  [0, 2)  x 2  [0, 4) and

if x  (, 0)  x 2  (0, ). Thus, for x  2, x 2  [0, ).

(ii) When x > – 1 , this means x  (1, 0)  [0, ) . Now, if x  (1, 0)  x 2  (0, 1) and if

x  [0, )  x 2  [0, ). Thus, for x  1, x 2  [0, ).


2
(iii) When x ≥ 2, this means x  [2, )  x  [4, ).
Example 05: Solve the following equations: (i) x2 – |x| – 2 = 0 (ii) |x – 2| = 1
Solution: (i) x2 – |x| – 2 = 0  | x | 2  | x | 2  0  (|x| – 2) (|x| + 1) = 0  (|x| – 2) = 0. (By
factorizing). Since, |x| + 1 ≠ 0 hence, this implies that |x| = 2 or x = ± 2.
(ii) |x – 2 | = 1  x – 2 = ± 1  x = 2 ± 1  x = 1 or x = 3.
-:7:-
CHAPTER ONE FUNCTIONS AND THEIR APPLICATIONS
Example 06: Find the value of 1/x if (i) x > 3 (ii) x < –2 (iii) x  (1, 3)  { 0 }
1 1 1 1 1
Solution: (i) When x > 3, this means 3 < x < ∞     0  .
3 x  x 3
1 1 1 1
(ii) When x < –2, this means x  (,  2)    x  2 or   or   x  0
 x 2 2
1 1 1
(iii) When x  (1, 3)  {0}  x  (1, 0)  (0, 3). Now, if x  (1, 0), we have
 
1 x (0)
1 1 1 1 1 1 1
 1         1. But if x  (0, 3), we have 0  x  3    or   
x x 0 x 3 x 3
1 1
or  x  . Thus,  (,  1)  (1 / 3, ).
3 x
Example 07: Solve (x – 1) (x – 2) (2x – 1) < 0
Solution: Put each term to zero, we get x = 1, x = 2 and x = 1/2. Mark these points on the real
line, as shown below.

0 1/2 1 2 3
Now, if we put:
x < 1/2, given inequality becomes negative which is true.
.

x = 0.7, given inequality becomes 0.156 which positive which is false.


.

x = 1.5, given inequality becomes -0.5 which is negative which is true.


.

x > = 2, given inequality is positive which is false.


.

Thus, (x – 1) (x – 2) (2x – 1) < 0 is true when x  (, 1 / 2)  (1, 2) . This is the solution set of
given inequality.
Example 08: Solve (x – 2)/(x + 2) > (2x – 3)/(4x – 1)
( x  1)( x  4)
Solution: Upon cross multiplication and simplifying, we get: 0
( x  2)( 4x  1)
Put each term to zero, we get x = – 2 , x = 1/4, x = 1 and x = 4. Now, mark these points on the
real line, shown below. 1/4

–2 –1 0 1 2 3 4
At x = – 2 and x = 1/4 the inequality will be infinite. At x = 1 and x = 4 it becomes zero. Thus,
we have to find out those values on the x-axis where the inequality is positive.
Now, if we put:
x  (,  2) , then given inequality is positive.
x  (1/ 4, 1) , then given inequality is also positive.
Finally, if x  (4, ) , then given inequality is positive. Thus, for
(x  1)(x  4)
x  (,  2)  (1/ 4, 1)  (4, ), the inequality  0 is true. This forms the solution
(x  2)(4x  1)
set of given inequality.
Example 09: Solve x1  3 x

-:8:-
CHAPTER ONE FUNCTIONS AND THEIR APPLICATIONS
Solution: The given inequality is meaningful if both x  1 and 3  x are positive. This gives
x > 1 and x < 3. Combining the two inequalities, we get 1 < x < 3.
Now, squaring both sides, we get x – 1 > 3 – x or x > 2. From the two inequalities, the solution is
2 < x < 3.
Example 10: Solve |3x – 2| = x
Solution: If 3x – 2 > 0  3x – 2 = x or 2x = 2 or x = 1. If 3x – 2 ≤ 0  3x – 2 = – x or 4x = 2 or
x = 1/2. Thus, S. S = {1/2, 1}.
Example 11: Solve (i) x2 – 4|x| + 3 < 0 (ii) 0 < |x – 3| ≤ 5 (iii) |x – 3| ≥ 2
(iv) |x| + | x – 2| = 2 (v) (|x – 3 |) / (| x + 1|) ≤ 1
2
Solution: (i) x – 4|x| + 3 < 0  (|x| – 1) (|x| – 3) < 0  1 < |x| < 3  –3 < x < –1 or 1 < x < 3.
Thus, solution set contains is x  (3,  1)  (1, 3).
(ii) Given, 0 < |x – 3| ≤ 5  –5 ≤ x – 3 < 0 or 0 < x – 3 ≤ 5  –2 ≤ x < 3 or 3 < x ≤ 8.
Thus, S. S contains x [2, 3)  (3, 8].
(iii) Given, |x – 3| ≥ 2  x – 3 ≤ –2 or x – 3 ≥ 2  x ≤ 1 or x ≥ 5.
(iv) Given, |x| + | x – 2| = 2  | x – 2 | = 2 – | x |. This is true only if 0 ≤ x ≤ 2.
(v) Given, (|x – 3 |)/ (| x + 1|) ≤ 1  –1 ≤ (x – 3)/ (x + 1) ≤ 1
x 3 x 3 x  3  x 1 4 4
Now, if  1 1  0  0 0  0  x  1
x 1 x 1 x 1 x 1 x 1
x 3 x 3 x  3  x 1 2x  2
And if  1 1  0  0  0  2x  2  0 & x  1  0
x 1 x 1 x 1 x 1
 2x  2 & x  1  0  x  1 & x  1. Now, if we take x  1 say x  2 then value of given
inequality becomes, 5 which is not less than 1. Thus, for x  1 given inequality is not satisfied.
On the other hand if we take x  1 say, x  2 then value of given inequality becomes, 1 / 3 which is
| x  3|
less than 1.Thus, for x  1 given inequality is satisfied. Hence, for x  1,  1 is true.
| x  1|
INTRODUCTION TO FUNCTIONS
2
Recall the formula for the area of a circle having radius equal to r; that is, A = πr . This formula
states that “the area A of the region within a circle is equal to π times the square of the radius r.
2
The equation A = πr defines a correspondence between the two variables, r and A. For every
(non- negative) value of r; there is a corresponding value of A. The formula A = πr2 provides the
rule of correspondence. It indicates how to compute the area A that corresponds to any particular
r value supplied. The correspondence can be viewed as under:
r A = (πr2)
0 0
1 π approx: 3.14
The area example, A = πr2 provides an introduction to the concept of function. The function is a
rule that assigns one of the A value to each of r value. (A is a function of r).
The equation y = 2x + 1 also defines a function; because for every x value supplied; there is one
y value. (y is a function of x).

X Y
y = 2x + 1

0 -:9:- 1
3 7
0.5 2
-2 -3
CHAPTER ONE FUNCTIONS AND THEIR APPLICATIONS

X Y

Definition: A function is a rule of correspondence by which each element of one set (X) is
assigned to exactly one element of the other set (Y).”
In A = πr2 the set of all possible input values for the radius is called the domain of the function.
The set of all output values of the area is the range of the function. Since circles cannot have
negative radii or areas, the domain and range of the function are both the interval [0, ∞)
consisting of all non–negative real numbers.
A Swiss mathematician, Leonard Euler (1707 – 1783) invented a symbolic way to say,
“y is a function of x” by writing: y = f(x). [This is read as y is a function of x]
In this notation, the symbol `f` represents the function. The letter x, called the independent
variable, represents an input value from the domain of f, and y the dependent variable, represents
the corresponding output value in the range of `f`. In the set notation, function is written as:
{(x,f (x)) : x  X,f (x)  Y} 
Symbolically, it is indicated by f :X  Y and is read as “f is a function from the set X to the set
Y”.
NOTE: (i) In calculus we deal with functions from R to R (ii) Functions defined from R to R are
called real-valued functions (iii) Each element y of the set Y is called an image and each element
x of the set X is called co-image of the function f.
Example 01: Which of the following diagrams describe a function from the set X = {1, 2, 3, 4}
into the set Y = {a, b, c, d}.
(i) f (ii) g
X Y X Y

1 a 1 a
2 b 2 b
3 c 3 c
4 d 4 d

Solution: (i) f is not a function from X into Y as the element 4 of the set X is not assigned to any
of the element of the set Y.
(ii) g is not a function from X into Y as the element 2 of the set X is assigned to two different
elements ‘a’ and ‘d’ of set Y and thus the image of the element 2 of X is not unique.
Example 02: Let the function f be defined by f(x) = 5x2 – 4x + 8. Determine each of the
following values: f(0), f(2), f(–3), (d) f(x + 1).
Solution: When x = 0, f(0) = 5(0)2 – 4(0) + 8 = 8.
When x = 2, f(2) = 5(2)2 – 4(2) + 8 = 20. When x = –3, f(0) = 5(–3)2 – 4(–3) + 8 = 65.
When x = x + 1, f(x + 1) = 5(x + 1)2 – 4(x + 1) + 8 = 5x2 + 6x + 9.
Zeros of a Function

-:10:-
CHAPTER ONE FUNCTIONS AND THEIR APPLICATIONS
The zero of a function f is any real number x for which f (x) = 0. Geometrically, it is a point
where the function crosses or touches the x-axis. This is shown in the following figures. It may be
noted that a function may have more than one zero as is shown in the second figure.

Example 03: Find the zeros of each of the following functions: f(x) = 5x – 20 and
f(x) = x2 + 5x – 14.
Solution: Given f(x) = 5x – 20. Now, if f(x) = 0 then 5x – 20 = 0; giving x = 4. Thus the only
zero of this function is 4.
Given f(x) = x2 + 5x – 14. Putting f(x) = 0 then x2 + 5x – 14 = 0. Factorizing, we get
(x + 7) (x – 2) = 0 or x = –7 or x = 2. We conclude that the zeros of this function are –7 and 2.
Domain, Co-domain and Range of Real Valued Functions
Consider a real valued function y = f(x) from the set X to set Y. The set X is called the domain
and set Y is called the co-domain of the function f. The set of those elements of Y which form the
images of the elements of the set X is called the range of the function f.
For example, consider a function f(x) = 2x + 1. When a real number is used for x there will be a
corresponding real value f(x) or y. The domain and range of this function f are, therefore, all real
numbers. The co-domain of this function is also a set of all real numbers.
Similarly consider a function y = f(x) = x2. When a real number is used for x there will be a
corresponding real value f(x) or y. The domain of this function is the set of all real numbers. But
as we know that the square of a real number is always non–negative, therefore the range of this
function will be the set of non–negative real numbers but the co-domain of this function is the set
R. Finally, consider a function A(x) = x(2x + 3) that represents the area of a rectangle. Let x be
the length of one side of a rectangle and (2x + 3) be the length of another side of the rectangle.
Here x cannot be zero or negative; therefore, the domain and range of this function are the set of
positive real numbers.
NOTE: (i) Domain and range of a function f are usually denoted by Df and Rf respectively.
(ii) In addition to the nature of applications, there are other some concepts that can restrict the
domain of a function. These are:
1. Division by zero
Any value of x that creates division by zero can’t be in the domain of a function. There would be
no function f(x) corresponding to such value of x.
2. Square root of negative numbers
Any value of x creating the square root of a negative number can’t be in the domain of a function.
There would be no real f(x) corresponding to such an x.

Types of Functions

-:11:-
CHAPTER ONE FUNCTIONS AND THEIR APPLICATIONS
1. Injective Function: A function f from R to R is said to be an injective function if distinct
elements of Df have distinct images, that is, for all x1, x2  Df;
if x1  x 2  f (x1 )  f (x 2 ) or if x1  x 2  f (x1)  f (x 2 )
To show that a given function is injective, we often use the second condition. An injective
function is also known as One–One function. For example, the function y = f(x) = x3 from R to R
2
is one–one because different values of x have different images in f(x). The function f(x) = x
from R to R on the other hand, is not one–one because different values of x have the same image.
For if x = 1 then y = 1 and if x = – 1 then y = 1. Thus, for two different values of x, y has the
same value. Hence this is not a 1–1 function.
2. Surjective Function: A function f from R to R is said to be surjective if Rf is equal to the
co–domain of the function f. That is, if f : X → Y and Y = Rf then f is a surjective function.
Surjective function is also known as onto function. For example, the function f(x) = x3 from R to
R is onto, whereas the function f(x) = x2 is not onto because Rf ≠ R.
3. Bijective Function: A function f from R to R is said to be a bijective function if it is both
injective and surjective. A bijective function is also known as one–one and onto function. For
3
example, the function f(x) = x and f(x) = x are both bijective functions.
Even and Odd Functions
A function y = f(x) is said to be even function if f(–x) = f(x) and is called odd function if
f(–x) = – f(x). If a function does not satisfy these conditions, it is said to be neither even nor odd
function. For example, the function f(x) = x2 + 1 is even function, because f(–x) = (–x)2 + 1
2
= x + 1 = f(x).
The function f(x) = x3 + x is odd function because f(–x) = (–x)3 + (–x) = – x3 – x = – (x3 + x)
= – f(x). The function f(x) = x3 – x is neither even nor odd function. Readers may verify it.
Algebra of Functions
Let f and g be given functions. The sum f + g the difference f - g the product f.g and the quotient
f/g are functions defined by:
(a) (f  g)(x)  (x)  g(x),  x  Df  Dg (b) (f  g)(x)  f (x)  g(x),  x  Df  Dg
(c) (fg)(x)  f (x)g(x),  x  Df  Dg (d) (f / g)(x)  f (x) / g(x),  x  Df  Dg , g  x   0
(e) The reciprocal of the function f is denoted by 1/f and is defined as
(1 / f )(x)  1 / f (x),  x  Df where f (x)  0
(f) The scalar multiplication of a function f(x) is defined as: (cf )(x)  cf (x),  x  Df , c  R
Example 04: If f(x) = 2x – 1 and g(x) = x2 + 1 where x is real, find
(i) (f + g) (x) (ii) (f – g) (x)
(iii) (f.g) (x) (iv) (1/f)(x)
(v) (f/g)(x) (vi) (–3f)(x)
(vii) f(x + 2) (vii) g(x – 3)
Solution: Sin ce, Df  R,Dg  R  Df  Dg  R. Now,

-:12:-
CHAPTER ONE FUNCTIONS AND THEIR APPLICATIONS
2 2
(i) (f  g)(x)  f (x)  g(x)  2x  1  x  1  x  2x  x(x  2),  x  R
(ii) (f  g)(x)  f (x)  g(x)  (2x  1)  (x 2  1)  x 2  2x  2,  x R
(iii) (fg)(x)  f (x)  g(x)  (2x  1)(x 2  1)  2x 3  x 2  2x  1,  x R
1 1 1
(iv)   (x)   ,  x  R  {1 / 2}
f  f (x) 2x  1
g g(x) x 2  1
(v)   (x)   ,  x  R  {1 / 2}
f  f (x) 2x  1
(vi) (3f )(x)  3f (x)  3  2x  1  6x  3,  x R
(vii) f (x  2)  2(x  2)  1  2x  3
(viii) g(x  3)  (x  3) 2  1  x 2 – 6x  10
Composite function or Composition of functions
If f : A → B and g : B → C are two functions such that range of f is domain of g, then the
function h from A to C that is; h : A → C defined by h(x) = (gof)(x) = g(f(x)) for all real x  A is
called composite function of f and g. This is shown below.

A f(x) B g(x) C
x1 y1 = f(x1) z1= g(y1) =
g(f(x1))
x2 y2 = f(x2) z2= g(y2) =
g(f(x2))

gof
2
Example 04: If f(x) = x and g(x) = x + 1 then find g o f and f o g.
Solution: (i) (gof)(x) = g(f(x)) = g(x2) = x2 + 1
(ii) (fog)(x) = f(g(x)) = f(x + 1) = (x + 1)2
B
A f g C
1 1 2

2 4 5

3 9 10

gof

A g B f C

1 2 4

2 3 9

3 4 16

fog

-:13:-
CHAPTER ONE FUNCTIONS AND THEIR APPLICATIONS
Example 05: If f(x) = 2x + 1 then find f(f(f(x)))
Solution: f(f(f(x))) = f(f(2x + 1)) = f(2(2x + 1) + 1) = f(4x + 3) = 2(4x + 3) + 1 = 8x + 6 + 1
= 8x + 7. It may be noted that f(f(f(x))) = f 3(x).

Example 06: If f  x   x3  3 and g(x) = x2 + 3 find


(i) (f og)(x) (ii) (gof )(x) (iii) (f of )(x) (iv) (gog)(x)

   x 2  3
3
Solution:  i   f g  x   f  g  x    f x 2  3   3  x 6  9x 4  27x 2  24

2
 ii   g f  x   g  f  x    f  x 3  3    x 3  3   3  x 3  3  3  x 3
   

 iii   g g  x   g  g  x    g  x 2  3   x 2  3
2
 3  x 4  6x 2  12

 3 
 iv   f f  x   f  f  x    f  x 3  3     x 3  3   3 
     
 
You may observe that f o g ≠ g o f.
Example 07: Let f(x) = 4x – 3 and g(x) = 5x + 4, then find f + g, f – g, f . g, f/g, 2f.
(i) [f + g](x) = f(x) + g(x) = 4x – 3 + 5x + 4 = 9x + 1.The mapping form of this is shown below.
f . g f+g
1 1 1 9 1 10
2 5 2 14 2 19
3 9 3 19 3 28

(ii) [f – g](x) = f(x) – g(x) = 4x – 3 – 5x – 4 = – x – 7.The mapping form, this is shown below.
f . g f–g

1 1 1 9 1 -8
2 5 2 14 2 -9
3 9 3 19 3 -10

(iii) [f.g](x) = f(x) . g(x) = (4x – 3)(5x + 4) = 20x2 + x -12. The mapping form of this is shown
below.
. f g f.g
1 1 1 9 1 9
2 5 2 14 2 70
3 9 3 19 3 171

(iv) [f/g](x) = f(x)/g(x) = (4x – 3)/(5x + 4), x ≠ – 4/5. The mapping form of this is shown below.

. f g f/g
1 1 1 9 1 1/9
2 5 2 14 2 5/14
3 9 3 19 3 9/19

(v) f(x) = x2 + 1 and k = 2, then kf(x) = 2(x2 + 1). The mapping form of this is shown below.
-:14:-
CHAPTER ONE FUNCTIONS AND THEIR APPLICATIONS
. f 2f
.
1 2 1 4
2 5 2 10
3 10 3 20

1  x when    x  1

Example 08: Let f (x)  1  x when 1  x  2 then
1 when 2  x  

(i) image of zero is: f(0) = 1– 0 = 1 (ii) f(√3) = 1 + √3 (iii) image of 3 is f(3) = 1
Example 09: If f(x) = 5 and f(x) = x + x – 1 then find the value of x.
2

Solution: Equating, x2 + x – 1 = 5, we get x2 + x – 6 = 0 or (x – 2)(x – 3) = 0 giving x = 2


or x = 3.
Example 10: If f(x) = 7x + 3 then find [f(x) – f(2)]/(x – 2).
Solution: Here f(2) = 7(2) + 3 = 17. Thus, [f(x) – f(2)]/(x – 2) = [7x + 3 – 17]/(x – 2)
= (7x – 14)/(x – 2) = 7(x – 2)/(x – 2) = 7.
Example 11: If f(x) = 2x3 + ax2 + 4x – 5 and f(2) = 3 then find a.
Solution: Here f(2) = 2(2)3 + a(2)2 + 4(2) – 5 = 16 + 4a + 8 – 5 = 19 + 4a. Now, it is given that
f(2) = 3. Thus, 19 + 4a= 3 or 4a = –16 giving a = –4.
It may be noted that we have already discussed about the domain and range of a function. The
domain of a function f is also defined as; the set of all real values x for which the function
becomes neither infinite nor complex is called the domain of a function.
Example 12: Find the domain and range of the following functions.
(i) f(x) = x2: Here f(x) is defined for all real values of x. This function neither becomes infinite
nor complex for any real x hence; Df = R. Also for any value of real x, f(x) is always non-
negative. Thus, Rf = set of all non-negative real numbers which includes zero as well as positive
real numbers.
(ii) f(x) = x/(x2 – 4): Here f(x) becomes infinite when x = –2 and x = 2. Thus, Df = R – {–2, 2}.
Moreover, the range of given function is set of all real numbers, i.e; Rf = R.

(iii) f (x)  x 2  9 : The given function becomes complex if x is taken between –3 and 3. Thus,
f(x) is defined only when x2 – 9 ≥ 0 which implies that x ≤ – 3 and x ≥ 3. Thus,
Df = (– ∞, –3]  [3, ∞). Moreover, the Rf = [0, ∞).
x when 0  x  1
(iv) f (x)  
 x  1 when 1  x  2
Here Df = [0, 1]  (1, 2] = [0, 2]. To find the range, we put x = 0, 1, 2 separately in f(x) to get,
f(0) = 0, f(1) = 1 and f(2) = 1. Thus, Rf = [0, 1].
1
(v) f (x) 
| x  3 |  (x  3)

-:15:-
CHAPTER ONE FUNCTIONS AND THEIR APPLICATIONS
 x  3 when x  3 0 when x  3
By definition, | x  3 |  . Thus, | x  3 | (x  3)  
3  x when x  3 6  2x when x  3
Thus, Df = (– ∞, 3).
x3
(vi) f (x) 
(x  3) x 2  4
The domain of f(x) is defined for x + 3 ≠ 0 and x2 – 4 > 0. Thus, Df = (– ∞, -2)  (2, ∞) with
x ≠ –3.
1 x
(vii)  a  f  x   b f  x  c f  x  x 1 d  f  x  3  5x
x4 2
x x
Solution: (a) If we put x = 4, the denominator becomes zero. Since division by zero is not
defined. Thus, four is not in the domain of f. This means that the domain of f is the set of all real
numbers except 4. In other words, Df = R – {4}.
(b) Here the domain of f is the set of all real numbers except 0 and –1. Thus, domain of the
function f(x) is Df = R – {0, –1}.
(c) If x – 1 is negative, the result is the square root of a negative number. Since the square root of
a negative number is not a real number, no f value will be produced for x – 1 < 0 or x < 1. This
means that the domain is all x values for which x ≥ 1. Thus, Df = 1,  
(d) The domain of f is the value of x for which; 3 – 5x ≥ 0. This means 3 ≥ 5x or x ≤ 3/5. In the
interval notation, we may write this as Df = (– ∞, 3/5].
Example 13: Determine the domain of each of the following function.
a f  x  3 x 7x b f  x  (x  4) / (x  1)
Solution: (a) Here f(x) is defined for 3 + x ≥ 0 and 7 – x ≥ 0 or x ≥ -3 and x ≥ 7. Combining the
two conditions imposed on x, the domain of f is Df = [–3, 7].
(b) In the numerator of the function, x – 4 must be non-negative; that is, x – 4 ≥ 0 or x ≥ 4. The
denominator x + 1 must be positive; that is x + 1 ≥ 0 or x ≥ –1. Combining the two relations, the
domain of f is Df = (– ∞, –1) U [4, ∞).
Example 14: Find the range of
(i) f(x) = sin2 x – sin x + 1: f(x) = sin2 x – sin x + 1= (sin2 x – 1/2)2 + 3/4 (1)
[By completing the squares]. Now, – 1 ≤ sin x ≤ 1 or – 1 – (1/2) ≤ sin x – (1/2) ≤ 1 – (1/2)
2
. or –3/2 ≤ sin x – (1/2) ≤ 1/2 Or 0 ≤ [sin x – (1/2)] ≤ 9/4
[Squaring both sides and keeping 9/4 which is greater than 1/4 on the right]
Adding 3/4 on both sides, we get 0 + 3/4 ≤ (sin x – 1/2)2 + 3/4 ≤ 9/4 + 3/4
This gives 3/4 ≤ (sin x – 1/2)2 + 3/4 ≤ 3. Thus, range of given function f(x) is [3/4, 3].
[Observe that we are trying to get the range of expression in (1)]

(ii) f (x)  x  x 2
 x  x, x  0 2x, x  0
Solution: By definition, f (x)  x  x 2  x  | x |  
 x  x, x  0 0, x  0
Thus Rf = [0, ∞).
-:16:-
CHAPTER ONE FUNCTIONS AND THEIR APPLICATIONS
Step Functions or Bracket Functions
Step Functions or Bracket Functions are of two types. They are (i) Greatest Integer Function
which is also known as Floor Function and (ii) Least Integer Function which is also known as
Ceiling Functions. We shall present them in the following sections.
Greatest Integer Function/ /Floor Function
Let f : R → R be defined by f  x    x  and is called greatest integer less than or equal to x, that
is; the greatest integer not greater than x is called Greatest Integer Function.
Here f  x    x  = n if n ≤ x < n + 1. Thus, Floor function
 x  = –2, if –2 ≤ x < –1
= –1, if –1 ≤ x < 0
= 0, if 0 ≤ x < 1
= 1, if 1≤ x < 2
= 2, if 2 ≤ x < 3, etc.
The graph is shown here.
It may be noted that:
i. Df = R and Rf = Z.
.

ii. The right hand end points of the each line segments are not part of the graph
The Least Integer Function or Ceiling Function
Let f : R → R be defined by f  x    x  and denotes the smallest integer greater than or
equal to x. The graph of this function is shown as under.
Here f  x    x  = n + 1 if n < x ≤ n + 1. Thus, Ceiling function

f  x    x  = –1, if –2 < x ≤ –1
= 0, if –1 < x ≤ 0
= 1, if 0 < x ≤ 1
= 2, if 1< x ≤ 2
= 3, if 2 < x ≤ 3, etc.
The graph is shown here.
It may be noted that:
i. Df = R and Rf = Z.
ii. Left hand end points of each line segments are not the part of the segment
For example
 1.1  2 but 1.1  1; 1.1  1 but 1.1  2; 2.9  2 but 2.9  3; 1  1 and 1  1
Graph of a Function
A drawing that shows the relationship between two variables is called a graph. This idea was
developed by the French mathematician Rene Descartes. A graph describes a function in visual
form.
There are various types of graphs. Histograms and the pie charts are used to display numerical
information in a form that is simple and quickly understandable. Others such as scatter diagrams
may be used in analyzing the results of a scientific experiment.

-:17:-
CHAPTER ONE FUNCTIONS AND THEIR APPLICATIONS
In calculus, graphs are used to give a geometric representation of equations. Moreover,
simultaneous equations can be solved by drawing the graphs of the equations and finding the
points of intersection. Graphs are particularly helpful in the study of Analytical Geometry and
Calculus. For this, we must understand the system of coordinates where the graph is to be drawn.
To begin with, we introduce the Cartesian coordinate system in the plane.
The position of all points in a plane can be measured with respect to perpendicular real lines in
the plane at a point called the origin. Moreover, horizontal real line is called the x–axis and the
vertical real line, the y–axis. Both together are called coordinate axes.
If P is any point in the plane, we can draw lines through P perpendicular to the two coordinate
axes. If the lines meet the x–axis at ‘a’ and the y–axis at ‘b’, then ‘a’ is the x–coordinate of P, also
(also known as abscissa) and ‘b’ is the y–coordinate of P (also known as an ordinate). The
ordered pair (a, b) is the coordinate pair of the point P. The x–coordinate of every point on the
y–axis is zero. Similarly, the y–coordinate of every point on the x–axis is zero. The origin is the
point (0, 0). The origin divides the x–axis into the positive x–axis to the right and the negative
x–axis to the left. Similarly, it divides the y–axis into the positive y–axis to the upward direction
and the negative y–axis to the downward direction. The axes divide the plane into four regions
called “quadrants”, numbered counterclockwise as shown in the above figure.
Positive y–axis

P(a, b)
Second quadrant ,  b First quadrant , 
Negative x–axis
Origin

O a x

Third quadrant , 


Positive x–axis
Negative y–axis Fourth quadrant , 
Graph of Linear Function
A function described by the equation y = mx + c is called a linear function. In other words, a
linear function of x is one, which contains no term in x of degree higher than the first. The
general form of a linear function is ax + by + c = 0, where x and y are variables, and a, b and c are
constants.
This function is called linear because its graph in the Cartesian coordinates is always a straight
line.
A straight line is completely determined by two points; therefore to draw the graph of a straight
line we locate at least two points on the line. By joining the two points we obtain the required
graph of the line. Let us take an example now.
Example 15: Draw the graph of the linear function y = 2x + 1.
Solution: We can let x be 0, 1 and 2 (or any other numbers) and then determine each
corresponding y value from the equation. Once two or more points have been obtained, they can
be plotted in the x y – plane and a straight line will be passed through them.

-:18:-
CHAPTER ONE FUNCTIONS AND THEIR APPLICATIONS
. x . y Point
–2 –3 (–2, –3)
–1 –1 (–1, –1)
0 1 (0, 1)
1 3 (1, 3)
-1 0 1
2 5 (2, 5)

Graph of Quadratic Function


A function defined by the equation y = ax2 + bx + c,  a  0  , where a, b, c are constants, is
called a quadratic function. This equation always represents a parabola. The graph of such a
parabola will have one of the shapes as shown below:

when a > 0 when a < 0

Similarly the equation x = ay2 + by + c, where a ≠ 0, represents the parabola having one of the
following shapes:

when a > 0 when a < 0

Thus, if one of the variables in an equation appears as its single power and the other in its square
form, then the graph is a parabola. To draw the graph of such a parabola one needs the vertex and
the direction of branches of the parabola. As noted earlier, there are four basic forms with vertex
at origin.
(i) The branches of y = x2 move upwards since y is non – negative.
(ii) The branches of y = – x2 move downwards since y is non-positive.
(iii) x = y2, x is positive implies that branches move to the right.
(iv) x = – y2, x is negative means the branches move to the left.

2 2 2
y=x x=–y x=y

y = – x2

The Circle
The equation of a circle with radius a and center (0, 0) is

x 2  y 2  a 2 so that y 2  a 2  x 2  y   a 2  x 2
This equation does not represent a function because we see that for one value of x, there are two
values of y. Such a function is called implicit function. The domain of this implicit function is
Df = [– a, a].

-:19:-
CHAPTER ONE FUNCTIONS AND THEIR APPLICATIONS
2 2
Remark: The circle x + y = 1 whose center is at the origin and the radius is 1, is called the unit
circle.
Square root function
Square root function is defined by f  x   x , x  0  The domain of the square root function
consists of all nonnegative real numbers, that is, x ≥ 0 because the square root of a negative
number is not a real number.
The graph of square root function is shown below.

f x  x f(x) = x3

Cube function
Cube function is defined as f(x) = x3, x  R. The domain of the cube function consists of all the
real numbers. The graph of cube function is shown above.
Reciprocal function
The reciprocal function is defined as f(x) = 1/x, x ≠ 0. The graph of reciprocal function is shown
below. This graph represents a curve called rectangular hyperbola.
Constant function
Let f : R  R be defined by f(x) = c for all real x, c being a fixed real number. The graphs
reciprocal and constant functions are shown on the next page.

f(x) = 1/x
.

f(x) = c

f(x) = – c
.

It may be noted that:


(i) Function f(x) = c is a straight line parallel to x – axis. If c is positive then graph lies above the
x-axis and if it is negative, the graph lies below the x-axis.
(ii) In particular, if c = 0 then f(x) = y = 0 represents the x–axis. Similarly, x = a represents a
straight line parallel to y-axis. If a = 0 then x = 0 is the equation of y-axis.
Identity function
A function f : R  R defined by f(x) = x for all real x is called an identity function. y=x
o
Its graph is a straight line passing through the origin and making angle of 45
. with the x–axis. Here Df = R.
Exponential function
If y = ax where a  R+ and a ≠ 1, then y is called an exponential function of x to the base a.
For example, f(x) = 2x, f(x) = (2/3)x, f(x) = (0.2)x, f(x) = (√3)x, f(x) = 1/2x = 2-x are all
exponential functions.
Properties: (i) ax.ay = ax + y (ii) ax/ay = ax-y (iii) (ax)m = amx (iv) a0 = 1 (v) ax.bx = (ab)x.

-:20:-
CHAPTER ONE FUNCTIONS AND THEIR APPLICATIONS
x
(vi) If 0 < a < 1 then y = a is decreasing function. Its graph is shown in fig. (i)
x
(vii) If a > 1 then y = a is increasing function. Its graph is shown in fig. (ii)
y = ax y = ax

Fig. (i) 0<a<1 Fig. (ii) a>1

(viii) Each graph lie above the x-axis hence the function y = ax has the positive values only.
(ix) The graph does not intersect the x-axis hence this function has no zero.
(x) The function y = ex is called “the exponential function“ where e ~ 2.718.
(xi) If a x1  a x 2 then x1  x 2 if a  1 and x1  x 2 if 0  a  1
Logarithmic function
If y = ax with a > 0 and a ≠ 1 then x is called a logarithmic function of y to the base a. It is written
as x = loga y.
For example: (i) 64 = 26 → log2 64 = 6, (ii) 1000 = 103 → log10 1000 = 3
(iii) y = loge x = ln x → ey = x.
Properties: (i) loga(xy) = loga x + loga y (ii) loga(x/y) = loga x – loga y
(iii) loga xm = m. loga x (iv) logb a = 1/ loga b
(v) loga x = logb x. loga b (vi) logb a = (logc a)/(logc b)
y y log a
(vii) a = e
(viii) The log function is inverse of exponential function.
(ix) The function y = loge x is called natural logarithmic function of x. (Also called Napier
Logarithm)
(x) The domain of log function is (0, ∞) and its range is (–∞,∞) = R.
(xi) The y = log10 x is called common logarithmic function. It is also known as Briggs logarithm.
(xii) If loga x1 > loga x2 then x1 > x2 if a > 1 and if x1 < x2 if 0 < a < 1.
(xiii) loga a = 1, loga 1 = 0, provided a ≠ 1.
(xvi) loga 0 = – ∞ if a > 1 and loga 0 = – ∞ if 0 < a < 1.
The graph of exponential function and its inverse logarithmic function are shown as under.
2
Example 16: Solve (i) (1/ 2)x 2x  1/ 4
2 2
Solution: (1/ 2)x 2x  1/ 4  (1/ 2)x 2x  (1/ 2)2 . .

It means x 2  2x  2 or x 2  2x  2  0
Solving the quadratic equation x2 – 2x – 2 = 0, we get
2  12 2  2 3
x   1  3. Thus,
2 2
(x  (1  3))(x  (1  3))  0  x  (1  3)  0 or x  (1  3)  0
Thus, x  (, 1  3)  (1  3, ) .

-:21:-
CHAPTER ONE FUNCTIONS AND THEIR APPLICATIONS
x x
(ii) (1  5 ) / (7  7)  0

Solution: Putting 1  5x  0  5x  1  50  x  0. Now, put 7  x  7  0  7  x  71  x  1


Hence, as per inequality, x  (,  1)  [0, )
1
(iii) If log 8 x  3 then find x.
3
2 10
Solution: log 8 x  log 3/2 x  log 2 x   log 2 x  5 or x  25  32.
2 3 3
1
[Note :log n x  loga x]
(a ) n
Sign function
The sign function is defined as follows:
 1, x 0

sgn  x    0, x 0
1, x0

The graph of sign function is shown above. For x ≠ 0, we can write
x x
sgn  x    , Df  R  {0}
x x
Piecewise Functions
Sometimes a function uses different formulas on different parts of its domain. Such types of
functions are called piecewise defined functions.
For example, consider the function
  x, x0
 2
f x   x , 0  x 1
 1, x 1

is defined on the entire real line but has values given by different formulas depending on the
value of x. The graph of this function is drawn by applying different formulas as given in the
function f(x). Such piecewise functions will be needed for the study of limits and continuity.
Polynomial function
An expression of the form an xn + an-1 xn-1 + …+ a2 x2 + a1 x + a0 is known as a polynomial of
degree n (an ≠ 0). The function of the form
f(x) = an xn + an-1 xn-1 + …+ a2 x2 + a1 x + a0
is known as a polynomial function. The domain of a polynomial function is a set of all real
numbers.
Algebraic function
A function is called an algebraic function if it can be expressed as the sum, difference, products,
quotients, powers or roots of polynomials.
For example, the function f(x) = 2x3 + 5x – 7 is polynomial function and the functions
2
g(x) = (x + 1)/(x-2), h(x)  x 2  2x  5 , k(x) = x (x2 + 5) + 2x  5 are algebraic functions.
Rational function
-:22:-
CHAPTER ONE FUNCTIONS AND THEIR APPLICATIONS
A function f: R → R defined by f(x) = g(x)/h(x) for all real x where g(x) and h(x) are polynomials
with h(x) ≠ 0, is called a rational function.
2
For example, f(x) = (x + 1)/(x + 5), g(x) = (x2 + x + 1)/(x2 + 5x - 7) are rational functions.
Irrational Function
A function f : R → R defined by f(x) = g(x)/h(x) for all real x where g(x) and/or h(x) are
irrational algebraic function with h(x) ≠ 0, is called an irrational function.
2 2 2 1/3
For example, f(x) = 2x  5 /(x + 5), g(x) = (x + x + 1)/(x + 5x – 7) are irrational functions.
Transcendental function
A Function that is not algebraic is known as transcendental functions. The combination of
algebraic and transcendental functions is also a transcendental function. For example,
f(x) = x3 + 2x  5 + cos x – ex + log x
is a transcendental function.
Bounded Function
A function f(x) from R to R is said to be bounded if range of f is bounded otherwise it is

unbounded function. For example, the function f  x   4  x 2 is bounded because its range is
[– 2 , 2]. This function represents the upper half of the circle centered at origin and radius 2.
The functions f(x) = x2 , g((x) = x + 1 are unbounded because the range of f(x) is the set of non-
negative real numbers and that of g(x) is the set of all real numbers. Both sets are unbounded
hence f(x) and g(x) are unbounded functions.
Circular and Hyperbolic functions
(i) Circular Functions
Readers are familiar with all trigonometric functions for example, sin x, cos x, tan x etc. These
functions are also called circular functions as they are having a relation with circle. You have
already known about this in F. Sc course. Here, we develop another idea which will help you to
understand circular functions in a different from, hyperbolic functions and the relation between
them. Circular functions can be expressed in the form of an infinite series of non-negative
increasing powers of x as shown below. Such series are called “power series.” The representation
of some of circular functions and exponential functions is shown as under.
x3 x5 x 7 x 2 x 4 x6 x 2 x3 x 4
sin x  x     , cos x  1     , ex  1  x    
3! 5! 7! 2! 4! 6! 2! 3! 4!
2
Replacing x by ix where i (iota) is an imaginary unit such that i = – 1, we get
i 2 x 2 i3 x 3 i 4 x 4 i5 x 5 i 6 x 6 i 7 x 7 x 2 ix 3 x 4 ix 5 x 6 ix 7
eix  1  ix         1  ix       
2! 3! 4! 5! 6! 7! 2! 3! 4! 5! 6! 7!
 x2 x4 x6   x 3
x 5
x 7 
 1       i x     
 2! 4! 6!   3! 5! 7! 
   
Using the first two series, we get: eix = cos x + i sin x (1)
This identity was established by the great mathematician Euler and therefore is known as
“Euler’s Identity” or “Euler’s Formula”. Replacing x by –x in (1), we get
eix  cos x  isin x (2)

-:23:-
CHAPTER ONE FUNCTIONS AND THEIR APPLICATIONS
NOTE: sin (–x) = – sin x and cos (–x) = cos x
eix  eix
Adding (1) and (2), we get: eix  eix  2cos x  cos x 
2
eix  eix
Subtracting (2) from (1), we get: eix  eix  2isin x  sin x 
2i
Since, the pair (cos x, sin x) satisfies the equation x2 + y2 = 1 of a circle, therefore these functions
are called circular functions. Now by definition,

tan x 
sin x

eix  e ix 2i

  eix  e ix
, cot x 
1

i eix  e ix
,
 
cos x eix  e ix 2 i    eix  e ix 
tan x eix  e ix
1 2 1 2i
sec x   , csc x  
ix
cos x e  e  ix sin x e  e ix
ix

Hence, we have ‘six circular functions’ given below:


eix  e ix eix  e ix eix  e ix
sin x  ,cos x  , tan x  ,
2i 2 i  eix  eix 
cot x 

i eix  e ix  ,sec x  2
, csc x 
2i

ix ix
ix
e e e e ix
e  e ix
ix

(ii) Hyperbolic functions


In the following figures, there are shown the graphs of the exponential functions ex and e–x,
respectively.
e x  e x e x  e x
The graphs of the curve y  and that of y  are shown below:
2 2
Hyperbolic function y = cosh(x)
12 Hyperbolic function y = sinh(x)
15
11
12
10

9 9

8
6
7

6 3
Y - axis
Y - axis

5 0
4
-3
3

2 -6

1
-9
0
-12
-1

-2 -15
-3 -2.5 -2 -1.5 -1 -0.5 0 0.5 1 1.5 2 2.5 3 -3 -2.5 -2 -1.5 -1 -0.5 0 0.5 1 1.5 2 2.5 3
X - aixs X - aixs

It is found that these two functions have properties which in many respects are similar to those of
y = cos x and y = sin x. It can be shown that they bear a similar relation to the hyperbola that the
trigonometric or circular functions do to the circle. The function y  (e x  e  x ) / 2 is called the

hyperbolic cosine, and y  (e x  e x ) / 2 is called the hyperbolic sine. These are abbreviated as

e x  e x e x  e x
cosh x and sinh x, that is, cosh x  and sinh x  .
2 2
They describe the motions of waves in elastic solids, the shapes of hanging electric power lines,
and the temperature distributions in metal cooling fins.

-:24:-
CHAPTER ONE FUNCTIONS AND THEIR APPLICATIONS

tanh x 
sinh x


e ex x
 2  ex  ex , coth x  1  ex  ex ,
cosh x 
ex  e x  2 ex  e x tanh x e x  e  x

1 2 1 2
sec hx   , csc hx  
x
cosh x e  e  x sinh x e  e  x
x

It may be noted that the curve of cosh x is an important one. It is called the catenary, and is the
curve formed by a uniform flexible chain that hangs freely with its ends fixed. These functions
can be expressed in the form of series, which are derived from the series for ex. Since,
x 2 x3 x 2 x3
ex  1  x 
  and e x  1  x    and
2! 3! 2! 3!
Hence by addition and subtraction: Flexible wire with fixed ends

2 4 3 5
x x x x
cosh x  1    and sinh x  x    CATENARY
2! 4! 3! 5!
The following formulae on hyperbolic functions may be noted.
(1) cosh 2 x  sinh 2 x  1
2 2
2
 ex  e x
2
  ex  e x 
Pr oof : cosh x  sinh x     
 2   2 
   
e2x  2  e 2x  e2x  2  e 2x 4
  1
4 4
(2) 1  tanh 2 x  sec h 2 x
cosh 2 x sinh 2 x
Pr oof : Dividing first equation by cosh 2 x, we get 
cosh 2 x cosh 2 x
1
 or 1  tanh 2 x  sec h 2 x
2
cosh x
(3) coth 2 x  1  csc h 2 x
cosh 2 x sinh 2 x
Pr oof : Dividing first equation by sinh 2 x, we get 
sinh 2 x sinh 2 x
1
 or coth 2 x  1  csc h 2 x
2
sinh x
Vertical line test
The definition of a function says that for every x there is one y. The graphical interpretation of
this idea is considered in the vertical line test. If a vertical line (VL) intersects a curve or a graph
of a function in two or more points, then the graph of the given curve does not represent a
function. After all in that case, there would be two or more y values corresponding to a particular
value of x. See the following figures.
VL Y VL
Y

Not a function Not a function

-:25:-
CHAPTER ONE FUNCTIONS AND THEIR APPLICATIONS
VL VL
Y Y

A function A function
PHYSICAL APPLICATIONS OF FUNCTIONS
There is perhaps no field or area where functions are not used. In real life, where there is a
relation between two variables, the application of function is must. These applications are found
in social and natural sciences, engineering and technology. Applications of functions are also
known as mathematical modeling of functions.
Case Studies (Applied Problems and Simple Mathematical Modeling)
Example 01: Workers at a fast food restaurant earn $5 per hour for the first 40 hours in a week
and then $7.5 per hour for additional hours. Let x be the number of hours worked in a week, write
a two piece function W that describes a worker’s pay.
Solution: If a worker works for 0 up to 40 hours, he will be paid $5 per hour. Hence his pay is a
function of x (hours), which is
W(x)  5x for 0  x  40
When x is greater than 40, the worker makes $5 per hour for 40 hours ($200 total) plus $7.5 per
hour for each hour above the 40 hours. The extra hours will be (x – 40) so the earning would be
{200 + 7.5(x – 40)} dollars. After simplification, we have
W(x)  7.5x  100 for x  40
Thus, the two – piece function W is given by the formula
5x, for 0  x  40
Wx  
7.5x  100, for x  40
Example 02: A passenger train travels continuously for 10 hours. For the first 4 hours, the train
travels at an average speed of 70 miles per hour. The remainder of the trip is at night, and the
train goes an average of 58 miles per hour. Using t for time (in hours) and A for average speed (in
miles per hour), write the two piece function A(t) that describes the train’s average speed during
the 10 hours ride.
Solution: For t between 0 and 4, the train travels 70 miles per hour. Hence,
A  t   70 t for 0  t  4
When 4 < t ≤ 10 the train travels 58 miles per hour. The extra time will be (t – 4) hours, so
average speed can be expressed as follows in the form of two valued function:
70 t, for 0  t  4
At  
58 t, for 4  t  10
Example 03: The monthly charge for water in a small town is given by

18, if 0  x  20
f x  
18  0.1 x  20  , if x  20

where x is in hundreds of gallons and f is in dollars. Find the monthly charge for each of the
following usages. (i) 30 gallons (ii) 3000 gallons (iii) 4000 gallons
Solution: (i) Since x is in hundreds of gallons, hence 1 unit of gallon will be 1/100 = 0.01.

-:26:-
CHAPTER ONE FUNCTIONS AND THEIR APPLICATIONS
Thus 30 gallons will be equivalent to 0.30 units. Now, according to the given condition
f(t) = $18
(ii) Since x is in hundreds of gallons, 3000 gallons will be equivalent to 3000 100  30 of units.
Now, according to given domain: f  30  18  0.130  20  $19

(iii) Since x is in hundreds of gallons, 4000 gallons will be equivalent to 4000 100  40 of units.
Now, according to given domain: f  40  18  0.1 40  20  $20
NOTE: Gallon is a unit of liquid measure equal to 4.546 liters, used in United Kingdom and
Canada.
Example 04: A rectangular fence is to be constructed so that its length is 3x + 2 and its width is x
meters. If P is the function that gives perimeter, determine P(x).
Solution: Perimeter is the sum of the sides of any polygon. 3x + 2
Now for rectangle as shown in the figure, perimeter is x x
P(x) = 2(3x + 2) + 2x = 8x + 4 meters.
Example 05: A colony of bacteria is placed into a growth inhabiting 3x + 2
.environment Number of bacteria present at any time t (in hours) is given by
n(t) = 1000 + 20t + t2
Find: n(0), n(1) and n(10). Interpret your results.
Solution: (i) n(0) = 1000 + 0 + 0 = 1000. This means that at the beginning (t = 0) the number of
bacterial in the colony is 1000.
(i) n(1) = 1000 + 20 + 1 = 1021, this shows that after one hour number of bacteria is 1021.
(ii) n(10) = 1000 + 20 (10) + (10)2 = 1000 + 200 + 100 = 1300. This means that number of
bacteria in the colony after 10 hours is 1300.

WORKSHEET 01
1. What do you know about calculus and its applications in different fields of sciences?
2. How many branches are there of calculus and what do you know about them?
3. What are the contributions of Sir Isaac Newton and Gottfried Wilhelm Leibniz in the
development of calculus?
4. Mention briefly about the uses of calculus.
5. How can you learn calculus by taking different steps?
6. Draw the graphs of the following functions. Also mention the domain and range of these
functions. (a) y = 2x + 7 (b) y = 2x2 + 1 (c) y = (x2 + 1)/(x – 1)
7. Solve each of the following inequalities:
1
8. If f (x)  x 2  1 and g(x)  , show that fog ≠ gof.
4  x2
9. A patient paid $300 per day for a semiprivate hospital room and $1500 for an appendectomy
operation. Express the total amount for an appendectomy as a function of number of days of
hospital confinement.
10. In some cities you can rent a car for $18 per day and $ 0.20 per miles.
a. Find the cost of renting the car for one day and driving 200 miles.
-:27:-
CHAPTER ONE FUNCTIONS AND THEIR APPLICATIONS
b. If the car is rented for one day then express the total rental expenses as a function of the
number x miles driven.
11. Suppose the longer side of a rectangle has twice the length of shorter side, and if x is length of
shorter side, express the perimeter of the rectangle as a function of x.
12. The monthly charge (in dollars) for x kilowatt hours (KWH) of electricity used by a customer
is given by the following function:
7.52  0.1079x, 0x5
19.22  0.1079x, 5  x  750

Cx  
20.795  0.1058x, 750  x  1500
131.345  0.0321x, x  1500

Find the monthly charge for the following usages.


(a) 5 KWH (b) 6 KWH (c) 3000 KWH.
13. The pressure P of a certain gas is related to volume V according to: P = 100/V
(a) Is 0 in the domain of this function?
(b) What are P(100) and P(50)?
(c) As volume decreases, what happens to pressure?
15. The total cost of producing a product is given by
C(x) = 300 x + 0.1 x2 + 1200 dollars where, x represents the number of units produced.
a. What is the total cost of producing 10 units?
b. What is the average cost per unit when 10 units are produced?
16. A cigar box distributor’s revenue is R(x) = 1.35 x dollars, where x is the number of boxes
sold.
a. How much revenue is obtained from selling 5 boxes?
b. How much revenue is obtained from the sale of 5th box?
c. How much revenue is obtained from the sale of 8th box?
17. It costs a TV manufacturer C(x) = 0.1 x2 + 150x +1000 dollars to produce x TV sets. The
revenue from the sale of x TV sets is R(x) = 280x dollars.
a. Determine the profit function.
b. What is the profit on the manufacture and sale of 50 TV sets?
18. The cost on producing x radios is C(x) = 0.4x2 + 7x + 95 $. The revenue received is
R(x) = 40x $. What is the profit function? Find P(24) and P(25). What is the profit on the sale of
25th radio?
19. A psychologist needs volunteers for an experiment. She offers to pay $8 per hour for
volunteer who works up to 5 hours. Those who work more than 5 hours are paid $10 per hour for
additional hour.
(a) Write down the function that represents the volunteer’s pay V(x), where x represents the hours
worked. (b) Also find V(5), V(10) and V(15).

-:28:-
CHAPTER ONE FUNCTIONS AND THEIR APPLICATIONS
20. If p = 0.01x + 19 dollars is the price of one jacket and each jacket is sold for $80, determine
the profit function and then find P(2) and (50). What is BEP?
21. Sketch the graph of following piecewise functions.

2 for x  0  x for x  0
(a) f  x    (b) f  x    2
 x for x  0  x for x  0

 x for x  1 | x | for x  0
(c) f  x    (d) f  x   
 x for x  1 2x for x  0
22. A motorbike was purchased 2 years ago for Rs. 50, 000 is now worth Rs. 25, 000. Find the
linear relationship between its value (y) and the life (x) in years.
(a) How many years from the time of purchase will it be before motorbike is worth Rs. 35, 000.
(b) What would be its cost after 10 years?
23. A house was purchased 5 years ago for Rs. 5 million is now worth Rs. 8 million. Find the
linear relationship between its value (y) and the life (x) in years.
(a) How many years from the time of purchase will it be before house is worth Rs. 6 millions.
(b) What would be its cost after 8 years?
24. State which of the following functions is even or odd.
(a) f(x) = x4 + x2 – 1 (b) g(x) = x3 + sin x + 1 (c) h(x) = x3 – x (d) k(x) = |x| + cos x – 2
25. What is the domain and range of the following functions?
(i) f(x) = 1/(x2 – 1) (ii) g(x) = (x + 1)/(2x – 1)

(iii) h(x) = 1/ x 2  9 (iv) k(x) = 1/ 16  x 2

(v) m(x) = 1/ x 2  16 (vi) n(x) = 16  x 2


(vii) p(x) = x 2  16 (viii) q(x) = x 2  16 / x 2  9

-:29:-
CHAPTER TWO LIMIT AND CONTINUITY OF FUNCTIONS

CHAPTER
TWO LIMIT AND
CONTINUITY OF
FUNCTIONS
THE CONCEPT OF LIMIT
The concept of limit of a function is one of the fundamental ideas that distinguish CALCULUS
from other branches of mathematics.
To understand the concept of limit, let us consider a circular disc with area of one unit. If we
remove half of the disc, area of remaining part is 1/2. If we further remove half of the remaining
disc, the area of the left over part is 1/4 or (1/2)2. By further removing half of the remaining disc,
the area of left over part will be 1/8 or 1/23. If this process is continued, that is, at every step we
remove half of the remaining disc; the area of left over part at the nth stage will be 1/2n. Thus, we
observe that there is always some portion left over, however small it might be, and the process
will continue indefinitely. The area of the left over part gets smaller and smaller and ultimately it
will decrease to zero. We describe this fact by saying that limit of the left over area is zero. More
precisely, we say that the limit of sequence of numbers
1 1 1 1
1, , , , , , is zero.
2 22 23 2n
x 2  25
Before giving informal definition of limit, consider the function, f  x   .
x 5
This function is defined for all vales of x except 5. For If we substitute x = 5, we would get
25  25 0
f (5)   , a meaningless quantity. One could perhaps say here that why don’t we cancel
55 0
(x – 5) first and then put x = 5 to get f(5) equal to 10. There is a lapse in this argument as (x – 5)
is zero when x = 5 and cancellation of zero factor is not allowed. Consequently, we cannot
determine the value of f(x) at x = 5. But we do not leave the problem here. Instead, we try to
evaluate the value of f(x) when x is very near to 5 and this will finally lead us to a value that
would almost be the value of f(5). Thus we can evaluate f(x) at x = 4.99 or x = 5.1. The technique
is quite simple. Cancel (x-5) first (this step is perfectly legitimate as x is not equal to 5); then
substitute the value of x. For example, f(4.99) = 4.99 + 5 = 9.99 and f(5.01) = 5. 01+ 5 = 10.01.
We now write down some of the values given to x and the corresponding values acquired by f(x)
in the following tables.
Table 01: x approaches 5 from left
x-value 4 4.7 4.9 4.99 4.999
Value of f(x) 9 9.7 9.9 9.99 9.999

-:30:-
CHAPTER TWO LIMIT AND CONTINUITY OF FUNCTIONS
Table 02: x approaches 5 from right:
x-value 6 5.7 5.35 5.01 5.001
Value of f(x) 11 10.7 10.35 10.01 10.001

From the above tables, we note that, as x gets closer to 5, f(x) gets closer to 10. In such a case, we
x 2  25
say that limit of f(x), as x approaches 5, is 10 and we write: Limf  x   Lim  10
x 5 x5 x  5
This example leads to an informal definition of limit.
Definitions of Limits
Let a function be defined for all values of x except possibly for x = a where `a` is a real number.
The function f is said to have the limit L (a real number) as x approaches `a`.
If the value of f(x) can be made as close to L as we please by taking x sufficiently close to but not
equal to `a`, in such case, we write: Limf (x)  L
xa
Note that x may be close to ‘a’ both from left and right of ‘a’. Sometimes, Limf (x)  L can be
xa
evaluated by calculating f(a). This holds, for example, whenever f(x) is an algebraic combination
of polynomials and trigonometric functions for which f(a) is defined. For example,
(a) Lim x  3 (b) Lim  5x  3  10  3  7 (c) Lim 1  cos x   1  1  2
x 3 x 2 x 0
3x  4 6  4 2
(d) Lim  
x 2 x  5 2  5 3
Formal Definition of Limit
Let `a` be any real number and let f be a function from R to R which is defined for all x near to
`a` with the possible exception of the point x = a. The function f is said to have a limit L (where L
is real) as x approaches `a` if for every  > 0, there exists a positive real number δ (usually
depends on ε) such that: |f(x) – f(a)| <  wherever 0 < |x – a| < δ.
In this case, we write: Limf (x)  L
xa
Example 01: Prove, by definition that, Lim 2x  6
x 3
Solution: Here f(x) = 2x and f(a) = f(3) = 6 and a = 3. Thus by definition,
|f(x) – f(a)| = |f(x) – f(3)| = |2x -6| = 2 |x – 3|.
Now, let |x – 3| < δ then |f(x) – f(3)| = 2 δ = 
Clearly, if  > 0 then δ > 0.
This proves that: Lim 2x  6
x 3
Rules for Finding the Limits of Function (Direct Method)
The following rules of the limits may be applied directly hence are known as “Direct Method of
Finding the Limits”. These rules are:
(1) Limit of a Constant Function: Lim k  k .
x a
For example, (i) Lim 4  4 (ii) Lim ln 3  ln 3 (iii) Lim ( / 2)  (  / 2)
x 0 x 2 x 1

-:31:-
CHAPTER TWO LIMIT AND CONTINUITY OF FUNCTIONS
(iv) Lim sin( / 2)  sin( / 2)  1
x 0

(2) Limit of Power Function: Lim x n  a n .


x a

For example, (i) Lim x 3  23  8 (ii) Lim x 3/2  13/2  1


x 2 x 1
(3) Limit of Scalar Multiplication with a Function: Lim k f (x)  k f (a) .
x a
3 3
For example, (i) Lim 4x  4.2  4(8)  32 (ii) Lim (x 3 ) / 5  13 / 5  1 / 5
x 2 x 1
(4) Limit of Sum of Functions: Lim [f (x)  g(x)]  Lim f (x)  Lim g(x)
x a x a x a

For example, Lim (3x  2x  1)  3 Lim x  2 Lim x  Lim1  3(2)2  2(2)  1  15


2 2
x 2 x 2 x 2 x 2
(5) Limit of Product of Functions: Lim [f (x)g(x)]  Lim f (x) Lim g(x)
x a x a x a

x3 x3 x3



For example, (i) Lim (x 2  2)(x  1)  Lim(x 2  2) Lim(x  1)  32  2 (3  1)  (11)(2)  22 
(ii) Lim sin x cos x  Lim sin x Lim cos x  (sin )(cos )  0(1)  0
x  x  x 
(6) Limit of Quotient of Functions: Lim [f (x) / g(x)]  Lim f (x) / Lim g(x)
x a x a x a

x3 x3 x3



For example, (i) Lim (x 2  3) / (x  1)  Lim(x 2  3) / Lim(x  1)  32  3 / (3  1)  (12) / (2)  6 
(ii) Lim (x 2  1) / (x 2  2)  Lim (x 2  1) / Lim  (1  1) / (1  2)  2 / 3
x 1 x 1 x 1
Lim f (x) Lim (x 1)
(7) Lim cf (x)  c x  a ; for example, Lim 3(x 1)  3x  2  3(21)  33  27
x a x 2
Lim g(x)
(8) Lim f (x)g(x)  Lim f (x) x  a
x a x a
Lim (3x 1)
2 (3x 1) 2 x 2
For example: Lim (x )  Lim (x )  45  1024
x 2 x 2

(9) Lim ln(f (x))  ln Lim f (x) ; for example Lim ln (x 2  1)  ln Lim (x 2  1)  ln 8
x a x a x 3 x 3

Lim f (x) Lim (2x 2 1)


(2x 2 1)
(10) Lim e f (x)
e x a
; for example Lime e x 1
 e21  e
x a x 1

(11) Lim f (x) ; for example, Lim x 2  2  Lim(x 2  2)  3


x a x 1 x 1
The following theorem will enable us to calculate a many of the limit problems.
It is called the Sandwiching Theorem because it refers to a function `f` whose values are
sandwiched between the values of two functions g and h that have the same limit L at the point
`a`. The function f being trapped between the values of functions g and h, must also approach L.
The Sandwich Theorem: Suppose that g(x) ≤ f(x) ≤ h(x) for all x in some open interval
containing `a`, except possibly at x = a. Also suppose that
Lim g  x   Lim h  x   L then Lim f  x   L
x a x a x a

-:32:-
CHAPTER TWO LIMIT AND CONTINUITY OF FUNCTIONS
x2 x2
Example 02: Given that 1   f x 1 for all x ≠ 0 find Lim f (x)
4 4 x 0

 x2   x2 
Solution: Since Lim g  x   Lim 1    1  0  1 and Lim h  x   Lim 1   1 0 1
x 0 x  0  4  x 0 x  0  4 

Using the sandwich theorem, we get Lim f (x)


x 0

One–Sided Limits (Let hand and Right hand limits)


To have a limit L as x approaches `a`, a function f must be defined on both sides of `a`, and its
values f(x) must approach L as x approaches `a` from either side. Because of this, ordinary limits
are sometimes called two – sided limits.
It is possible for a function to approach a limiting value as x approaches ‘a’ from only one side,
either from the right or from the left. In this case we say that f has a one – sided (either right hand
or left hand) limit at `a`. The function f(x) = x/|x| graphed in following figure has limit 1 as x
approaches zero from the right, and limit –1 as x approaches zero from the left.
Let f(x) be defined on an interval (a, b) where a < b. If f(x)
approaches arbitrarily close to L as x approaches `a` from
within the interval (a, b), then we say that f has right-hand
limit L at `a` and we write: Lim f  x   L
x a 
Similarly, let f(x) be defined on an interval (c, a) where
c < a. If f(x) approaches arbitrarily close to M as x
approaches ‘a’ from within the interval (c, a), then we say that f has left–hand limit M at ‘a’, and
we write: Lim f  x   L
x a 
Example 03: Let f(x) = (|x| – 5) then Lim f (x)  Lim | x | 5   Lim  x  5   5  5  0
x  5 x  5 x 5

Also, Lim f (x)  Lim | x | 5   Lim  x  5   5  5  0 . Since, L.H.L = R.H.L so,


x  5 x  5 x  5

Limf  x   0 where, f(x) = | x | – 5.


x 5
Example 04: Let f (x)  x/ | x | . Then, Lim f (x)  1 / 1  1 and Lim f (x)  1 / 1  1. Since,
x 0 x 0
left and right limits are not equal hence, Limf (x)  Lim x/ | x | does not exist.
x0 x0
It may be noted that:
(i) The left hand limit of a function f(x) is shown in two different ways; viz, f(a – 0) or
Lim f (x) or Lim f (x) . Similarly, the right hand limit may be shown as f(a + 0) or
x a  0 x a 
Lim f (x) or Lim f (x) .
x a  0 x a 
(ii) If f(a – 0) = f(a + 0) = L; then Limf (x) exits and is equal to L. This means that if the right
xa
and left limits of a function exit and are equal to L then Limf (x)  L.
x a

-:33:-
CHAPTER TWO LIMIT AND CONTINUITY OF FUNCTIONS
Example 05: Determine Lim x
x 0

Solution: At first glance this limit may look simple, since you could easily believe that the limit
is zero. However, the limit is not zero. Although it is true that Lim x It is also true that
x  0

Lim x does not exist. Why? Because approaching from the left in this instance means using
x  0

numbers less than zero (negative numbers) for x, and x is not defined for such numbers. It may
be noted that a function must be defined for all values of x through which it approaches `a` or else
the limit does not exist. The following figure shows the graph of function f(x) = x which is
defined only for x ≥ 0.
f (x)  x

A function defined only for x ≥ 0

Our Conclusion: Lim x  0 but Lim x does not exist. Hence, Lim x does not exist.
x  0 x  0 x 0

Example 06: Determine Lim f (x) and Lim f (x)


x 0 x 0


x 2 , x  0
for the function defined by f (x)  
x  3, x  0

Solution: In this two – piece function, the value of f(x) is computed as x2 when x ≤ 0 and as x + 3
when x > 0. Thus: Lim f (x)  Lim (x  3)  3 and Lim f (x)  Lim (x 2 )  0
x 0 x 0 x 0 x 0
Since the limits are different, we conclude that Limf (x) does not exist.
x0
Example 07: Determine Lim f (x) and Lim f (x) for the function defined by
x 1 x 1

2x  1, x 1
f x  
4  x, x 1
Solution: Here Lim f (x)  Lim (4  x)  3 and Lim f (x)  Lim (2x  1)  3
x 1 x 1 x 1 x 1

Since the right – hand and left–hand limits are same, 3

that is; 3, therefore, we conclude that limit of given is 3.

4  x2
Example 08: Evaluate the following limit Lim
x 20 6  5x  x 2
Solution: We observe that the given function is one
-1/2 o 1 4
valued function. To evaluate the limit of such function,
we put x = a – h = 2 – h. This gives h = 2 – x. Now as x tends to 2, h will tend to zero. Thus,
.

-:34:-
CHAPTER TWO LIMIT AND CONTINUITY OF FUNCTIONS

4  x2 4  (2  h) 2 4  4  4h  h 2
Lim  Lim  Lim
x 20 6  5x  x 2 h 0 6  5(2  h)  (2  h) 2 h 0 6  10  5h  4  4h  h 2

4h  h 2 h(4  h) (4  h) (4  0) 4
 Lim  Lim Lim   2
h 0 h  h2 h 0 h(1  h) h 0 (1  h) (1  0) 1
Example 09: If f(x) = (x2 – 4)/(x – 2) find f(2 – 0) and f( 2 + 0). Hence find Limf (x)
x2

Solution: Be definition, f (2  0)  Lim f (x)  Lim [(x 2  4) / (x  2)]


x 2  0 x 2  0
Putting x = 2 – h then h = 2 – x. As x tends to 2 then h tends to 0. Thus,
(x 2  4) (2  x)2  4 4  4h  h 2  4
f (2  0)  Lim f (x)  Lim  Lim  Lim
x20 x20 (x  2) h0 (2  h)  2 h0 2h 2
h(h  4) h4 04
 Lim  Lim  4
h0 h h0 1 1
(x 2  4)
Similarly, to compute: f (2  0)  Lim f (x)  Lim
x20 x20 (x  2)
We put x = 2 + h then h = x - 2. As x tends to 2 then h tends to 0. Thus,
(x 2  4) (2  x)2  4 4  4h  h 2  4
f (2  0)  Lim f (x)  Lim  Lim  Lim
x 2  0 x 2 0 (x  2) h 0 (2  h)  2 h 0 2h 2
h(h  4) h4
 Lim  Lim 4
h 0 h h 0 1
Since f(2 – 0) = f( 2 + 0) = 4, we deduce that Lim f (x)  4 .
x 2

 x 2  1, x2
Example 10: Let f  x    . Find Limf (x)
 x  7, x2 x2

x 2 x 2
 
Solution: Lim f  x   Lim x 2  1  3 and Lim f  x   Lim
x 2 x 2
 
x 7  9 3

Sin ce, Lim f (x)  Lim f (x) thus, Lim f (x)  3 .


x 2 x 2  x 2

 2
x , x  1
Example 11: If f (x) is defined as: f  x    , find Lim f (x)
 3
 x 1
 x , x 1
Since function is two-valued function, hence to evaluate the required limit, we shall consider Left
and Right hand limits at x = 1.
Now, Lim f (x)  Lim x 2  1 . Also Lim f (x)  Lim x3  1 Since both left and right limits at
x 10 x 1 x 10 x 1
x = 1 are equal, we conclude that Lim f (x)  1 .
x 1

x  2, x  1
Example 12: If f (x) is defined as f (x)   2 , find a if the Lim f (x) exists .
ax , x  1 x1
Since function is two-valued function, hence to evaluate the required limit, we shall consider:

-:35:-
CHAPTER TWO LIMIT AND CONTINUITY OF FUNCTIONS
Left and Right hand limits at x = – 1: Now, Lim f (x)  Lim (x  2)  1 .
x10 x1

Right Hand Limit: Lim f (x)  Lim (ax 2 )  a


x 10 x 1
Since Lim f (x) exists, both left and right limits at x = – 1 must be equal. This gives a = 1.
x1
| 1  x | 1
Example 13: If f(x) is defined as f (x)  find Lim f (x) .
x x 0
Consider f(0 – 0). Put x = 0 – h then h = - x. As x tends to 0 then h also tends to zero. Thus,
| 1  h | 1 | (1  h) | 1 (1  h)  1 h
f (0  0)  Lim  Lim  Lim  Lim  1 .
h 0 | h | h 0 | h | h 0 h h 0 h

Now consider f(0 – 0). Put x = 0 + h then h = x. As x tends to 0 then h also tends to 0. Thus,
| 1  h | 1 | (1  h) | 1 (1  h)  1 h
f (0  0)  Lim  Lim  Lim  Lim  1
h 0 |h| h 0 |h| h 0 h h 0 h

Since f (0-0) ≠ f (0 + 0), hence Lim f (x) does not exist.


x 0
Limit by Indirect Method
In this section, we shall learn various techniques of finding the limits of given functions. These
methods are:
(1) Limit by Factorization
Given the function f(x)/g(x), we factorize both of them and apply the limit we get the solution of
the problem. It may be noted that if we apply any one of the direct methods we obtain an
indeterminate form either 0/0 or ∞/∞.
x2  4
Example 14: Evaluate Lim
x 2 x  2

Solution: Apply the limit directly we get an indeterminate form 0/0. Now, factorizing, we get
x2  4 (x  2)(x  2) (x  2)
Lim  Lim  Lim 224
x 2 x  2 x 2 (x  2) x 2 1

x3  8
Example 15: Evaluate Lim
x 2 x2  4
Solution: Apply the limit directly we get an indeterminate form 0/0. Now, factorizing, we get
x3  8 x3  23 (x  2)(x 2  2x  22 ) (x 2  2x  4) 12
Lim  Lim  Lim  Lim  3
x 2 x2  4 x 2 x2  4 x 2 (x  2)(x  2) x 2 (x  2) 4
Thus, limit of a given function is 3 when x approaches to 2.
x2  9
Example 16: Evaluate Lim
x3 x3  27
Solution: Apply the limit directly we get an indeterminate form 0/0. Now, factorizing, we get
x2  9 (x  3)(x  3) (x  3) 6 2
Lim  Lim  Lim   .
3 2 2
x3 x  27 x3 (x  3)(x  3x  9) x3 (x  3x  9) 27 9

-:36:-
CHAPTER TWO LIMIT AND CONTINUITY OF FUNCTIONS
x 4  16
Example 17: Evaluate Lim
x 2 x3  8
Solution: Apply the limit directly we get an indeterminate form 0/0. Now, factorizing, we get
x 4  16 (x 2  4)(x 2  4) (x  2)(x  2)(x 2  4) (x  2)(x 2  4)
Lim  Lim  Lim  Lim
x  2 x3  8 x  2 (x  2)(x 2  2x  4) x   2 (x  2)(x 2  2x  4) x  2 (x 2  2x  4)
 4(8) / (4  4  4)  (32 / 12)  8 / 3.

x2  x  6
Example 18: Evaluate Lim
x 2 x 2  3x  10
Solution: Apply the limit directly we get an indeterminate form 0/0. Now, factorizing, we get
x2  x  6 (x  3)(x  2) (x  3) 5
Lim  Lim  Lim 
x 2 2
x  3x  10 x  2 (x  5)(x  2) x  2 (x  5) 7

Limit by Formula
xn  an
(i) Lim  na n 1
x a x  a

Proof: Putting x = a + h. Now, as x approaches a then h tends to zero. Thus,


xn  an (a  h)n  a n a 2  na n 1h  ...  h n  a n na n 1h  ...  h n
Lim  Lim  Lim  Lim
x a x  a x a a h a h 0 h h 0 h

h(na n 1  ...  h n 1 )
 Lim  Lim (na n 1  ...  h n 1)  na n 1 (By Binomial Theorem)
h 0 h h0

xm  am xn  an x a xm  am x a 1 m
(ii) Lim  Lim   Lim  Lim  ma m   a mn
x a xn  an x a x  a n
x a n x a x  a n
x a x  a n
na n n

x 4  81
Example 19: Evaluate Lim
x3 x  3

Solution: Apply the limit directly we get an indeterminate form 0/0. Now, using formula (i), we
x 4  81 x 4  34
get Lim  Lim  4.341  4.33  4(27)  108.
x3 x  3 x3 x  3

x 6  64
Example 20: Evaluate Lim
x 2 x  2

Solution: Apply the limit directly we get an indeterminate form 0/0. Now, using formula (i), we
x 6  64 x 6  26
get Lim  Lim  6.261  6.25  6(32)  192.
x 2 x  2 x 2 x  2

x 4  16
Example 21: Evaluate Lim
x 2 x2  4
Solution: Apply the limit directly we get an indeterminate form 0/0. Now, using formula (ii), we
x 4  16 x 4  24
4
get: Lim  Lim  .242  2.22  2(4)  8.
2 2 2
x 2 x 4 x 2 x  2 2

-:37:-
CHAPTER TWO LIMIT AND CONTINUITY OF FUNCTIONS
x10  1024
Example 22: Evaluate Lim
x 4  16
x 2
Solution: Apply the limit directly we get an indeterminate form 0/0. Now, using formula (ii), we
x10  1024 x10  210 10 104 5 6 5
get Lim  Lim  .2  .2  (64)  5(32)  160.
4 4 4
x 2 x  16 x 2 x 2 4 2 2
Some Useful Limits
In this section we shall develop some important limits that will help us to use hem in an important
applications in calculus.
sin x
I. Evaluate Lim
x 0 x
Proof: We are familiar with geometrical proof of the above limit. Here, we produce its proof
once again to make you fully understand it.
Consider a circle centered at C with radius r. Let AB be an arc that subtends an angle of θ
measure in radians. Join AB by a chord and draw a tangent line at point A and produce CB to
meet this tangent line at point T. Also draw a perpendicular from B on AC. This is shown in the
adjacent figure. Now, from the figure, we see that:
Area of Δ ABC < Area of sector ABC < Area of Triangle CAT.
Now, area of Δ ABC = 0.5 (base) (height) = 0.5 |AC| |BM|.
From the triangle, CMB, sin θ = |BM|/|CB| giving |BM| = |CB| sin θ.
2
Thus, area of Δ ABC = 0.5 |AC| |CB| sin θ = 0.5 r . r sin θ = 0.5 r sin θ.
Now, area of sector ABC = 0.5 r2 θ and finally
Area of Δ CAT = 0.5 |AC| |AT|.
From the triangle CAT, tan θ = |AT|/|AC| Or |AT| = |AC| tan θ.
2
Thus, area of Δ CAT = 0.5 |AC| |AC| tan θ = 0.5 r tan θ.
Thus, 0.5 r2 sin θ < 0.5 r2 θ < 0.5 r2 tan θ. Dividing by 0.5 r2 sin θ, we get
1 < (θ/sin θ) < (tan θ)/(sin θ) or 1 < (θ/sin θ) < cos θ. Now taking limit θ → 0, we get
 
Lim 1  Lim  Lim cos   1  Lim  1.
 0 0 sin   0 0 sin 
 sin 
By sandwiching theorem, Lim = 1 or Lim . This proves the result.
0 sin  0 
Practically, we can also prove this limit as under. Place your calculator at “RADIAN” mode
before you compute the following values.
. x 1 0.5 0.1 0.001 0.000,000,1
. sin x 0.8414 0.4794 0.0998 0.00099999 1
. (sin x) /x 0.8414 0.9588 0.9983 0.99999 1

You may observe that as x approaches to zero, (sin x)/x approaches the value 1. Since 1 is a finite
number, we say that it is a limit of the function (sin x)/x as x tends to zero is 1. Thus, we have:
sin x
Lim 1.
x 0 x

-:38:-
CHAPTER TWO LIMIT AND CONTINUITY OF FUNCTIONS
x
 1
II. Evaluate Lim 1  
x   x
Solution: Using binomial theorem, we obtain
1 x  x  1 1 x  x  1 x  2  1
x
 1  
Lim 1    Lim 1  x      
x   x x   x 2! x2 3! x3 
 x 2 1  1 x  x 3 1  1 x 1  2 x    1  1 x  1  1 x 1  2 x  
 Lim 1  1      Lim 1  1    
x  
 2!x 2 3!x 3  x  
2! 3! 
Applying the limit on the right hand side, we get:
x
 1 1 1  c 
Lim 1    1  1     Note : Lim  0
x   x 2! 3!  x  x 
The sum of this well-known infinite series is approximately equal to 2.71828. This sum always
x
 1
lies between 2 and 3 and is denoted by e. Thus, Lim 1    e.
x   x
In the above limit, if we put y=1/x then y will approach to 0 as x approaches to  . Thus,
x
 1
Lim 1    e  Lim 1  y 
1/y
x   x y0

ax 1
III. Evaluate Lim a > 0.
x 0 x
Solution: In fact, if we place the value of x = 0 in the above function we get once 0/0 which is an
indeterminate expression. To solve this limit problem, we proceed as under:
x x
Let, a – 1 = y then a = 1 + y. Taking ln on each side, we get
ln 1  y 
ln a x  ln 1  y  or x ln a  ln 1  y  or x 
ln a
Moreover, as x tends to 0, y also tends to 0. Hence,
ax 1 y ln a ln a ln a
Lim  Lim  Lim  Lim  Lim
y0 ln 1  y  y0 ln 1  y  y0 1
ln 1  y  y0 ln 1  y 
x 0 x 1y

ln a y y
 1  ax 1  1   
  ln a.  NOTE : Lim 1  y   e 
1y
 ln a Lim   Lim  ln a 
 y0 ln 1  y    ln e 
1 y x 0 x y0
 
Example 23: Evaluate the following limits
25x  1 25x  16 x sin 3x
Lim 1  3x 
1/x
(i) Lim (ii) Lim (iii) Lim (iv)
x
x 0 16 1 x 0 x x 0 sin 5x x 0

25x  1  25x  1 x  25x  1 x 25x  1 x


Solution: (i) Lim  Lim     Lim  Lim  Lim  Lim
x 0 16  1 x 0  x
x
16  1  x 0 x
x x
x 0 16  1 x 0 x x
x 0 16  1

25x  1  1  ln 25 ln 5
2
2ln 5 ln 5
Applying the limit, we get: Lim  ln 25      
x
x 0 16  1  ln16  ln16 ln 4 2 2ln 4 ln 4

-:39:-
CHAPTER TWO LIMIT AND CONTINUITY OF FUNCTIONS

(ii) Lim
25x  16x
 Lim
25x  1  16x  1
 Lim
25x  1  16x  1    
x 0 x x 0 x x 0 x

 Lim 
 
 25x  1

 
16x  1   25x  1  16x  1 
25x  16x   25 
  Lim  Lim  Lim  ln 25  ln16  ln  
x 0
 x x  x 0 x x 0 x x 0 x  16 
 
sin 3x  sin 3x 3x 5x  sin 3x 3 5x 3 3
(iii) Lim  Lim      Lim .Lim .Lim  1. .1 
x 0 sin 5x x 0  3x 5x sin 5x  x0 3x x0 5 x0 sin 5x 5 5
3
 Lim 1  3x 
1/3x 
(iv) Lim 1  3x   e  e3
1/x 3
x 0 x 0  

ln  x  h   ln h sin  x  h   sin h
Example 24: Evaluate (i) Lim (ii) Lim
x 0 x x 0 x
ln  x  h   ln h 1 1 xh 1  x
Solution:  i  Lim
x 0 x
 Lim
x 0 x
ln  x  h   ln h  Lim ln    Lim ln 1  
x 0 x  h  x 0 x  h

1x  h x 1 h  h x 1 h
 x   x    x 
 Lim ln 1    Lim ln 1     ln Lim 1   
x 0  h x 0   h
 x 0 
  h 
  
Applying the limit, we get
ln  x  h   ln h 1 1
NOTE : Lim 1  x 
1x
Lim  ln e1 h  ln e    e and ln e  1
x 0 x h h x 0

     
(ii) Since, sin   sin   2cos   sin  
 2   2 
xhh xhh  x  2h  x
2cos   sin   cos   sin  
sin  x  h   sinh  2   2   Lim  2  2
Lim  Lim
x0 x x0 x x0 x
2
sin(x / 2)  x  2h 
 Lim .Lim cos    1.cos h  cos h
x 0 (x / 2) x 0  2 
Example 25: Evaluate the following limits
 1 3   1 3   1  x  x2  3 
i  Lim  
3
 Lim     Lim  
x 1 1  x 1  x  x 1 1  x (1  x)(1  x  x )  x 1  (1  x)(1  x  x ) 
2 2

 x2  x  2   (x  2)(x  1) 
 Lim    Lim  
x 1  (1  x)(1  x  x 2 )  x 1  (1  x)(1  x  x 2 ) 

 (x  2)(x  1)   (x  2)  3
 Lim    Lim    1
x 1  (x  1)(1  x  x 2 )  x 1  (1  x  x 2 )  3

-:40:-
CHAPTER TWO LIMIT AND CONTINUITY OF FUNCTIONS
cos ec x  cot x   1 cos x  1 1  cos x 
 ii  Lim    Lim      Lim  
x 0  x  x 0  sin x sin x  x x 0  x sin x 
1  cos x  1  cos x   1  cos 2 x 
 Lim     Lim  
x 0  x sin x  1  cos x  x 0  x sin x(1  cos x) 

 sin 2 x   sin x   1  1
 Lim    Lim    Lim    1  1 / 2
x 0  x sin x(1  cos x)  x 0  x  x 0  (1  cos x)  2

Limits by Formula Continued


There are some important limits where we can apply them directly for solving a limit problem.
They are
sin x sin px sin px p 1  cos px p 2
1. Lim 1 2. Lim p 3. Lim  4. Lim 
x 0 x x 0 x x  0 sin qx q x  0 1  cos qx q 2

ex  1 emx  1 ax 1 a x  bx a
5. Lim 1 6. Lim m 7. Lim  ln a 8. Lim  ln  
x 0 x x 0 x x 0 x x 0 x b
ax 1 ln a ln(1  x) x tan x
9. Lim  10. Lim 1 11. Lim 1 12. Lim 1
x 0 bx 1 ln b x 0 x x  0 sin x x 0 x

sin 2 px sin 1 x tan 1 x sin(x  a)


13. Lim  p2 14. Lim 1 15. Lim 1 16. Lim 1
x 0 x 2 x 0 x x 0 x x  a (x  a)

tan(x  a)
17. Lim 1
x  a (x  a)

sin 5x sin 6x 6 1  cos5x 52 25 e7x  1


Examples: Lim  5, Lim   2, Lim   , Lim 7
x 0 x x  0 sin 3x 3 x  0 1  cos7x 72 49 x  0 x

3x  2x 3 3x  1 ln 3
Lim  ln   , Lim 
x 0 x  2  x 0 2x  1 ln 2
Limit at Infinity
Sometimes we are concerned with the behavior of a function f as the magnitude of the variable
say x, increases without bound (that is; the magnitude becomes infinitely large). The limits
studied in such instances are called limits at infinity and are written as
Lim f (x) or Lim f (x)
x x
The notation x  can be read, “as x increases without bound” or “as x tends toward infinity.”
Similarly, x    is read, “as x decreases without bound” or “as x tends toward minus infinity.
(The English mathematician John Wallis (1616 – 1703) was the first to use the symbol  for
infinity.)
Consider the function f(x) = 1/x. If x becomes larger, the closer 1/x gets to zero.
The table that follows illustrates this statement. The graph of f(x) = 1/x also shows this; that is, as
x becomes larger, 1/x gets closer to 0. We say that the limit of
function f(x) = 1/x approaches to zero as x increases without bound.
.

This is also shown in the graph.

-:41:-
CHAPTER TWO LIMIT AND CONTINUITY OF FUNCTIONS
x 2  10  100  1000
y= 1/x  0.5  0.1  0.01  0.001

This is written as Lim(1/ x)  0 .


x
In a similar manner, we can determine that Lim (1 / x)  0
x 

(Notice that 1/x is never equal to zero).

The above limits merely describe the behavior of 1/x as the magnitude of x increases without
bound. They describe the tendency of 1/x toward zero as x tends toward infinity. Consider next
n
the function defined by: f(x) = 1/x , n being a positive integer.
Comparing 1/x and1/xn and noting that the magnitude of xn is larger than the magnitude of x
when x is approaching infinity or minus infinity. It follows that:
1 1
Lim  0 and Lim  0, (n  Z )
x  x n x  x n

If the function is f(x) = c/xn where c is a finite constant, then the limit as x increases or decreases
without bound will still be zero. Thus, Lim c / x n  0 .
x 

3x3  8x  1
Example 26: Evaluate Lim
x  4x 3  3x 2  16
Solution: We notice that if the value of x is placed ∞ directly, we get an expression of the form
∞/∞. This form of expression is known as “Indeterminate Form.” (We shall discuss such forms in
more detail in the coming chapters). In order to solve this problem, we notice that the functions in
the numerator and denominator are polynomials and the limit is taken at infinity. In such cases,
one should take common, the term with highest power of x, both from the numerator and
denominator. Doing this, we get

Lim
3x 3  8x  1
 Lim
   Lim
 3  8 / x 2  1 / x3 
x3 3  8 / x 2  1 / x3
Lim
3  0  0 3

3 2

x  4x  3x  16 x  x 3 4  3 / x  16 / x 3
  
x  x 4  3 / x  16 / x 3 x   4  0  0  4

Example 27: Evaluate Lim (x 3  2x 2  1)


x 

Solution: Here we see that there is an algebraic expression in the numerator only. To solve such
limit problem, we substitute x = 1/y so that y = 1/x Now if x tends to infinity, y will tend to zero.
 1   1  2y  y3  1  0  0 1
x 
 
Lim x 3  2x 2  1  Lim  
2
y0  y3 y 2
 1  Lim 
 x  
  y3


 0
 
0
This shows that the above limit does not exist.
NOTE: The idea of dividing each term by a power of x is used only for limits at infinity. It will
not help in the evaluation of other kinds of limits.
Infinite Limit at Finite Point
When considering Limf (x), it may happen that it increases without bound (that is, becomes
x a
infinite) as x approaches ‘a’.

-:42:-
CHAPTER TWO LIMIT AND CONTINUITY OF FUNCTIONS
In such instances, the limit does not exist in the usual sense. However, this situation can be
described by writing Limf (x)  .
xa
Such a limit is called an “infinite limit.” A study of the graph shown in the following figure
should help you to understand this kind of limit.
Lim(1/ x 2 )   and Lim(1/ x 2 )  0
x 0 x 
X 1 0.5 0.1 0.01 0.001
1/x2 1 4 100 10,000 1,000,000

Next, consider the graph of f(x) which suggests two


interesting one–sided limits. The graph is shown below:
.

Lim (1 / x)   and Lim (1 / x)  


x 0 x 0
Numerically, we can interpret this as follows:
If the number 1 is divided by positive numbers that get closer
and closer to 0, the results obtained will get larger and larger. In fact, for any positive number c:
.

Lim (c / x)    c  0 
x0
If c is a negative constant, then the result will be negative: Lim (c / x)    c  0
x0
Further Examples: Evaluate the following limits
csc x  cot x 1 1 cos x  1  cos x 
(i) Lim  Lim     Lim  
x 0 x x 0 x  sin x sin x  x 0  x sinx 

1  cos x  1  cos x   1  cos 2 x   sin 2 x 


 Lim  
    Lim    Lim  
x 0  x sinx   1  cos x  x 0  x sin x 1  cos x   x 0  x sin x 1  cos x  

 sin x   sin x   1  1 1 1
 Lim    Lim   Lim    1.  
x 0  x 1  cos x   x 0  x  x 0  1  cos x   1  cos 0 1  1 2
sinpx  sin px px qx  sin px px qx p p
 ii  Lim  Lim      Lim .Lim .Lim  1. .1 
x 0 sin qx x 0  px qx sin qx  x 0 px x 0 qx x 0 sin qx q q
a 2/3  b 2/3
 iii  Lim . Putting a  b  h then a  b  h. If a  b then h  0. Thus,
a b ab
a 2/3  b 2/3  b  h   b2/3 2/3
b 2/3 1  h / b 
2/3
 b 2/3
Lim  Lim  Lim
a b ab a b h a b h
Applying binomial theorem, we get
 2  h  1 2  2  h  2 
b 2/3 1     .   1   ...  1  2  1  1 2  1  h 
3  b  2! 3  3  b  b 2/3 h     .   2  ...
 
2/3 2/3
a b  3  b  2! 3  3  b 
Lim  Lim  Lim
a b ab h 0 h h 0 h
2
2 1 3 2
 b  0  0  ...  b1/3 .
3 3
n(n  1) 2 n(n  1)(n  2) 3
NOTE : Infinite binomial expansion is 1  x   1  nx 
n
x  x  ...
2! 3!
-:43:-
CHAPTER TWO LIMIT AND CONTINUITY OF FUNCTIONS
tan  sin x 
 iv  Lim : Putting y  sinx then y  0 if x  . Thus,
x  sin x
tan  sin x  tan y sin y 1 1 1
Lim  Lim  Lim  Lim  1.  1
x  sin x y 0 y y0 y y0 cos y cos 0 1

 v  Lim x sin 
1 1
  Lim x.Lim sin    0  M  0.
x 0  
x x  0 x  0 x
1
Here we have used the fact that sin   is bounded function because for any value
x
1
of x, the function sin   is bounded between  1 and  1.
x
right hand limit s are not equal hence the limit of given function does not exist.
 vi  Lim sin(1 / x) : Consider right hand limit for sin(1 / x) when x tends to zero
x 0
from the right. Putting x  0  h then if x  0 then h  0. Thus,
Lim sin(1 / x)  Lim sin(1 / h)  A (say)
x 0 h 0
Now con sider the left hand limit for sin(1 / x) when x tends to zero from left.
Putting x  0  h then if x  0 then h  0. Thus,
Lim sin(1 / x)  Limsin(1 / h)   Limsin(1 / h)  A
x 0 h 0 h 0
Since left hand limit and right hand limits are not equal hence, Lim sin(1 / x) does not exist.
x 0

 1  1
2 x 2 1  2  x 1 2
x 1  x  x  1 0 1
 vii  Lim  Lim  Lim
x  x  1 x   1 x   1  1 0
x 1   x 1  
 x  x
 2 x  x  1  2  x  x  1 x  2   2 3 
x 2
 2
 viii  Lim 1    Lim 1  x        
x   x x   x 2!  x  3!  x
 
 x 2 1  1 x  .22 x 3 1  1 x 1  2 x  .23 
 Lim 1  2    
x   2 3
 2!x 3!x 
 22. 1  1 x  23. 1  1 x 1  2 x    22 23  2
 Lim 1  2      1  2    e
x   2! 3!  2! 3! 
  
 1 x  x  1  1  x  x  1 x  2   1 3 
x 2
 1
 ix  Lim 1    Lim 1  x        
x   x x   x 2!  x  3!  x
 
 x 2 1  1 x  x 3 1  1 x 1  2 x  
 Lim 1  1    
x   2 3
 2!x 3!x 

 Lim 1  1 
1  1 x   1  1 x 1  2 x     1 1  1
  1  1    e
x   2! 3!   2! 3! 
1 1
x  1  x   x   1 x  x   1  x 
 x  Lim 
x  1
  Lim      Lim       Lim 1     e  e 1
x   1  x  x   x   x   x x   x   x 
     

-:44:-
CHAPTER TWO LIMIT AND CONTINUITY OF FUNCTIONS
ax 1
 xi  Lim ,  a  1
x  x
Putting a  1  h, we get
x  x  1 h 2
1  xh   ...  1
ax 1 1  h   1 x
2!
Lim  Lim  Lim
x  x x  x x  x
x  h   x  1 h / 2! ...
2
 Lim    h     1 h  ...  .
x  x 2!
Thus limit does not exist.
   
 3 1   3 1  1  3 1
 xii  Lim  3    Lim   Lim 
x 1  x  1 x  1  x 1   x  1 x 2  x  1
  
x  1  x 1 x  1  x 2  x  1 1 
    
 Lim
2
1 3  x  x 1 
 Lim 

1  2  x2  x 
  Lim
1  x 2  x  2 
 
x 1 (x  1) x2  x  1 x 1 (x  1)  x 2  x  1  x 1 (x  1)  x 2  x  1 

1  (x  2) (x  1)  (1  2) 3
 Lim      1.
x 1 (x  1)  x2  x  1  (1  1  1) 3
 
 x2 x2   1 1  2  (x  3)  (x  1) 
 
xiii Lim 

   Lim x 2 

   Lim x  
x   x  1 x  3  x   x  1 x  3  x   (x  1) (x  3) 

2 2x 2 2 x2 2
 Lim x 2 .  Lim  Lim  2
x  (x  1) (x  3) x  x  4 x  3 x  x 1  4 / x  3 / x
2 2 2

1 0  0

 2 2
x  x a 
 xiv  Lim  x  x 2  a 2   Lim  x  x 2  a 2    
x    x     x  x2  a2 
 
 

 Lim
x  x 2 2
 a2   Lim a2

a2
0
x   2 2 x   2 2 
x  x a  x  x a 
   

x2 1  1 / x2   Lim
 xv  Lim
x 
x
x2  1
3/2
x  x
 Lim
x  3/2  
x. 1  1 / x 2   . 1  0   .

Thus limit of given function does not exit.

 xvi  Lim
5x 3  3x 2  1
 Lim

x3 5  3 / x  1 / x3 
x  x 4  x3 x  x 4 1  1 / x 

 Lim
 5  3 / x  1 / x3  5  0  0 5
  0
x  x 1  1 / x   1  0  

x1/3  1  1  1 1  1 2
1/3
 xvii  Lim     
x 1 x  1 1  1 0 0
Thus the limit of given function does not exist.
-:45:-
CHAPTER TWO LIMIT AND CONTINUITY OF FUNCTIONS
 x2 x2   x 2 (3  x)  x 2 (1  x)   3x 2  x 3  x 2  x 3 ) 
 xviii  Lim     Lim    Lim  
x   (1  x) (3  x)  x   (1  x)(3  x)  x   (1  x)(3  x) 
 2x 2   2x 2  2
 Lim    Lim   2
x   x  4x  3  x   x (1  4 / x  3 / x )  1  0  0
2 2 2

 1 1 
 xix Lim   
x 3  x  3 | x  3 | 

Putting x  3  h and u sin g the fact that as x  3 then h  0, we get


 1 1   1 1   1 1 
Lim     Lim     Lim      NOTE : |  h | h 
x 3  x  3 | x  3 |  h 0  3  h  3 | 3  h  3 |  h 0  h |  h | 
 1 1  2  2
 Lim     Lim     . Thus, limit does not exist.
h  0   h h  h 0  h  0
 x 2  2x  8 
 xx  Lim 



x 2  x  4 
2

Putting x  2  h and u sin g the fact that as x  2 then h  0, we get

 x 2  2x  8    2  h 2  2  2  h   8   2 
Lim    Lim    Lim  4  4h  h  4  4h  8 
 
x 2  x  4  h 0   h 0  
2
 2  h 2  4  4  4h  h 2  4 
 h 2  8  0  8 8
 Lim     . Thus limit does not exist.
h 0  4h  h 2  0  0 0
 xxi  Lim x  x   Lim x. Lim  x   1.5  1.5  1.5 1  1.5
x 1.5 x 1.5 x 1.5
 xxii  Lim x  x   Lim x. Lim  x   1.5  1.5   1.5  2   3
x 1.5 x 1.5 x 1.5
 NOTE : See the functions f (x)   x  and f (x)   x  defined in chapter 2.
 x  3, if x  2
 xxiii  Find the value of cons tan t 'k ' if Lim f (x) exists where f (x)   3
x 0  kx , if x  2
Consider f  2  0   Lim f (x)  Lim (x  3)  2  3  1
x 2 x 2

Also f  2  0   Lim f (x)  Lim kx 2  k( 2) 2  4k


x 2 x 2
Since Lim f (x) exists hence f ( 2  0)  f ( 2  0) or 4k  1 giving k  1 / 4.
x 0
Limit by Rationalization
Sometimes, we are given the problem of a limit where we get an indeterminate form 0/0 when we
get an irrational function either in the numerator or in the denominator. Thus, upon
rationalization, and applying the limit, we get the solution.
This is shown in the following example.

Example 28: Evaluate Lim  4  x 2  2  / x


x 0  
Solution: Apply the limit directly, we get an indeterminate form 0/0. Now, rationalizing, we get

-:46:-
CHAPTER TWO LIMIT AND CONTINUITY OF FUNCTIONS

4  x2  2 4  x2  2 4  x2  2 4  x2  4 x 2
Lim  Lim   Lim  Lim
x 0 x x 0 x 4  x 2  2 x  0 x[ 4  x 2  2] x  0 x[ 4  x 2  2]
x 0
 Lim   0.
x 0 2
4x 2 22

Continuous and Discontinuous Functions


In this section we shall discuss a very important concept of calculus known as “Continuity”.
Definition: A function f(x) is said to be continuous at a point x = a if the following three
conditions hold:
i  f  a  is defined Means a  Dom f  In other words,f (a) is not inf inite.
 ii  Lim f  x  exists Means Lim f  x   Lim f  x  , that is f (a  0)  f (a  0)
x a x a  x a 
 iii  Lim f  x   f  a  Means Lim f  x   Lim f  x   f  a 
x a x a  x a 
For example, the function f  x   3x  5 is continuous at x  2 because
Lim f  x   Lim f  x   f  2   6  5  11.
x 2 x 2
If one or more of these conditions are not satisfied, we say that the function is discontinuous at
x = a. Continuous functions can be thought of as those whose graphs can be drawn without lifting
the pencil. Following figures show graphs of functions that are discontinuous at x = a.

a a

NOTE: First type of discontinuity is known as “JUMP DISCOTINUITY” and the second kind of
discontinuity is known as “SINGULARITY”.
Some types of functions are continuous at every number in their domain. Limits of such functions
can always be determined by the substitution approach. Polynomial functions are continuous at
every real number. Rational functions are continuous at every real number, except at numbers for
which the denominator is zero.
Example 01: Is the function f(x) = (x2 – 1)/(x2 + 1) continuous at x = 1?
Solution: Using the three conditions (i) f (1)  0 (ii) Lim f (x)  0 (iii) Lim (1)  f (1)  0
x 1 x 1
Hence, f (x) is continuous at x = 1.
Example 02: Is the function f(x) = (x2 – 1)/(x + 1)
continuous at x = –1?
Solution: Consider, Lim f (x)  Lim (x 2  1) / (x  1)
x1 x1

 Lim  x  1 x  1 /  x  1  Lim  x  1  2.


x 1 x 1
Now, f (1)  0 / 0 which is undefined quantity.

-:47:-
CHAPTER TWO LIMIT AND CONTINUITY OF FUNCTIONS
Thus, Lim f (x)  f (1) so given function is discontinuous at x = –1.
x1
This is shown in the above figure. It may be noted that such point of discontinuity is known
“missing point discontinuity”.
Example 03: Is the following function continuous?
0, x  0
f (x)  
 x, x  0
Solution: The graph of the function is shown here.
If the function has a point where it is discontinuous, the trouble spot is at x = 0. Since the function
is defined differently for x < 0 and x ≥ 0, we must evaluate limits when x approaches zero from
.

the left and from the right. Now,


consider Lim f (x)  Lim (0)  0, Lim f (x)  Lim (x)  0
x 0  x 0  x 0 x 0
Thus, Lim f (x)  0. Also f (0)  0. Since, Lim f (x)  f (0)  0
x 0 x 0
hence, given function is continuous at x  0

Example 04: Discuss the continuity of f(x) at x = 2 where f(x) is defined by

x  4,  1  x  2

f (x)   2
x  6, 2  x  5

Solution: Consider, Limf (x)  Lim(x  4)  2, Lim f (x)  Lim(x 2  6)  2  Thus, Lim f (x)  2
x 2
x 2 x 2 x 2 x 2

Also f (2)  2. Since, Limf (x)  f (2)  2 hence, given function is continuous at x  2.
x2
Example 05: Examine the continuity of given function at x = 0.
 x sin(1 / x), x  0
f x  
0, x0
Solution: Here f(0) = 0 (given)
Now, Lim f  x   Lim  x sin (1 / x)   Lim(x)  Lim sin(1 / x)   0.M  0
x 0 x 0 x 0 x 0
Since, Lim f (x)  f (0)  0 hence, given function is continuous at x  0.
x 0
NOTE : We have assumed that Lim sin(1 / x)   M sin ce, sin(1 / x) is bounded.
x 0
Example 06: Examine the continuity of f at x = 0 where
 e1/x  1
 , if x  0
f  x    e1/x  1
0, if x  0

Solution: Here f(0) = 0 (given)

Now, Lim f  x   Lim 


 e1 x  1 
  Lim 

 e1 x 1  e 1 x
   Lim  1  1 e1 x 
x 0 x 0  1x 
 e  1  x 0  e 
 1 x 1  e1 x
  x0  1  1 e1 x 
-:48:-
CHAPTER TWO LIMIT AND CONTINUITY OF FUNCTIONS
Applying the limit, we get
1 0  1 
Limf  x    1, 1x
since Lime   and Lim 1 x  0 
x 0 1 0  x 0 x 0 e 
Since, Lim f  x   f  0  hence,f  x  is discontinuous at x  0
x 0
Example 07: Is f (x)  x  continuous at x = 1.5?
Solution: Consider, f (1.5)  1.5  1
Also, Lim f (x)(x)  Lim  x   1, Lim f  x   Lim  x   1
x 1.5  x 1.5 x 1.5 
x 1.5
Since Lim f (x)  Lim f (x)  1 thus, Lim f (x)  1
x 1.5 x 1.5 x 1.5
Since Lim f (x)  Lim f (x)  1 thus, Lim f (x)  1
x 1.5 x 1.5 x 1.5

Now, Lim f (x)  f (1.5)  1 hence f (x) is continuous at x = 1.5.


x1.5
NOTE: The bracket function (the least integer function) is continuous at all real numbers except
the points where x is an integer.
 x3  8

Example 09: The function f(x) is defined as: f (x)   x  2 , if x  2
8, if x  2

Is f (x) continuous at x = 2?
x3  8 (x  2)(x 2  2x  4)
Solution: Consider Lim f (x)   Lim f (x)   Lim(x 2  2x  4)  12
x 2 x  2 x 2 x2 x 2
Now f (2) = 8. This shows that Limf (x)  f (2) . Hence given function is not continuous at x = 2.
x2
  DEFINITION OF CONTINUITY
A function f(x) is said to be continuous at x = a if for given  0 there exists a real number
  0 such that: | f(x) – f(a) | <  whenever | x – a| < 
We shall use the above definition of continuity to see how it is applied to see whether or not a
given function is continuous or not.
Example 10: Using  definition, show that f(x) = 2 x – 3 is continuous at x = 4.
Solution: Here f(x) = 2x – 3, thus f(a) = f(4) = 8 – 3 = 5. Now
| f(x) – f(a) | = | 2 x – 3 – 5 | = | 2 x – 8 | = 2 | x – 4 |.
Taking |x–a |=|x–4|< ,
This implies that | f(x) – f(a) | = | f(x) – f(4) | <  , where  = 2  .
This proves that given function is continuous at x = 4.
Example 11: Use  definition, to show that f(x) = x 2 – 3 is continuous at x = 2.
Solution: Here f(x) = x2 – 3, thus f(a) = f(2) = 4 – 3 = 1. Now
|f(x) – f(a) | = | x2 – 3 – 1 | = | x2 – 4 | = | (x – 2) (x + 2) | = | x – 2| | x + 2 |.
Taking |x–a |= |x–2|< ,
This implies that | f(x) – f(a) | = | f(x) – f(4) | <  , where  =  | x + 2|.

-:49:-
CHAPTER TWO LIMIT AND CONTINUITY OF FUNCTIONS
This proves that given function is continuous at x = 2.
Note: Since x is finite, hence ( x + 2 ) is also finite.
CASE STUDY PROBLEMS
In this section we shall present the applications of limit and continuity that will help the readers to
understand the areas where such concepts are helpful.
Example 01: Suppose that profit from the sale of x units of microchips is P(x) = 2000 –
(300/x) dollars for x ≥ 1. What will be the maximum profit if the sale increases indefinitely?
Solution: Clearly, we are asked to find the limit of P(x)
as x tends to infinity. Now
 300 
Lim P  x   Lim  2000    2000  0  2000
x  x   x 
Interpretation:
The result reveals that no matter how big sale of microchips
is made, the profit will never exceed $2000.
.

The graph of above function is depicted in the adjacent figure.


Example 02: The height of tree grows according to law h(t) = (9.5 t 0 2)/(t + 1) where h is in
feet and `t` is the time in years. Find the height of the tree after (a) one year (b) 2 years (c)
10 years and (d) 30 years. What will be maximum height of the tree as the time passes
indefinitely?
Solution: (a) Putting t = 1, we get: h = 7.5/3 = 2.5 feet
(b) Putting t = 2, we get: h = 17/3 = 5.67 feet
(c) Putting t = 10, we get: h = 93/11 = 8.45
(d) Putting t = 30, we get: h = 9.2
(e) If t increases indefinitely, that as t tends to ∞, we get
 9.5 t  2   t(9.5  2 / t)   9.5  0 
Lim h  t   Lim    Lim     9.5
t  t   t  1  t   t(1  1 / t)   1  0 
This means that maximum height of the tree will reach 9.5 feet.
Example03: Number of degrees `d` in each interior angle of a regular polygon of `n` sides is
180n  360
d
n
(a) Determine the number of degrees in each interior angle of a regular hexagon (n = 6)
(b) The number of degrees d in each interior angle of a regular polygon gets larger as n
gets larger.
180(6)  360
Solution: (a) Using the above formula and putting n = 6, we get: d   120o.
6
This means angle measure of regular hexagon is 120o.
(b) As n gets larger and larger means n tends towards infinity, that is;
 360 
n 180  
 180n  360   n   360 
Lim  d   Lim    Lim  Lim 180    180
n  n   n  n  n n   n 
This shows that the number of degrees tend towards 180o as n gets larger and larger.
-:50:-
CHAPTER TWO LIMIT AND CONTINUITY OF FUNCTIONS
Example 04: An ant trap is used to eliminate an ant colony. The number of ants living t
 2040  5t, if 0  t  24
hours after the trap is put out is given by: n(t)  
 2880  40t, if 24  t  72
(a) How many ants were originally present in the colony?
(b) How many ants are left after 72 hours?
(c) Is the function continuous at t = 24 hours? Explain.
Solution: (a) The number of ants originally present in the colony was: n(0) = 2040 – 5(0) = 2040
.

(b) The number of ants left after 72 hours is: n(72) = 2880 – 2880 = 0
.

The function will be continuous at t = 24 hours if Lim n(t)  Lim n(t)  n(24)
t 24 t 24

Now, Lim n  t   Lim  2040  5t   2040  120  1920,


t 24 t 24

Also, Lim n  t   Lim  2880  40t   2880  960  1920 


t 24 t 24
Finally, n(24) = 2880 – 40 (24) = 2880 – 960 = 1920.
We observe that Lim n(t)  Lim n(t)  n(24) this implies that the given function is
n 24 n 24

continuous at t = 24 hours. In other words, at the 24th hour, the elimination process of ants is
continued.
Example 05: A woman with a temperature of 103.7o F is given medicine that will reduce her
temperature. The medicine takes one hour before it begins to work, and after that (that is,
for t ≥ 1) her temperature at t hours will be
5.1t  4.5
T(t)  103.7  degrees
t
(a) What is her temperature at t = 1 hour, t = 3 hours and t = 10 hours?
(b) Eventually, to what temperature is her body reduced?
Solution: (a) The temperature of the woman at t = 1 hour will be
T(1)  103.7  5.1(1)  4.5 / 1  103.7  0.6  103.1 degrees.

The temperature at t = 3 hours will be: T(3)  103.7   5.1(3)  4.5 / 3  103.7  3.6  100.1o

The temperature at t = 10 hours is: T(10)  103.7  5.1(10)  4.5 /10  103.7  4.65  99.05o
(b) The temperature of the woman will be reduced if t gets larger and larger, that is; t   .
 4.5 
t  5.1  
 5.1t  4.5   t 
Lim T(t)  Lim 103.7    103.7  Lim
t  t   t  t  t
 c 
 103.7  5.1  98.6 degrees.  using nLim  0
  x 
This shows that as time is prolonged the temperature of the woman will be reduced to 98.6o F.
Example 06: The flow of current is given as under:

-:51:-
CHAPTER TWO LIMIT AND CONTINUITY OF FUNCTIONS
 t 2  1, if 0  t  1

I  t    2t, if 1  t  2
 3
 t  4, if t  2
where t is the time in seconds. Is the current flow continuous at t = 1 and t = 2? What will be
the value of current at t = 5 seconds?
Solution: To see the continuity of I(t) at t = 1, we have to show that Lim I(t)  1 .
t 1

Now consider I(1  0)  Lim I(t)  Lim( t 2  1)  1  1  2


t 1 t 1

Similarly, I(1  0)  Lim I(t)  Lim(2t)  2.1  2 . Also, I(1 – 0) = I(1 + 0) = 2 thus LimI(t)  2 .
t 1 t 1 t 1

Now what about I(1)? From the definition of function I(t) we see that current at t = 1 is not
defined. Thus, although left and right hand limits of I(t) are equal but I(1) is not defined hence,
the flow of current is not continuous at t = 1 second.
To see the continuity of the current at t = 2, we have: I  2  0   Lim I  t   Lim  2t   2.2  4
t 2 t 1

 
Similarly, I  2  0   Lim I  t   Lim t 3  4  8  4  4 .
t 2 t 2

Since, I(2 – 0) = I(2 + 0) = 4 thus LimI(t)  4 . Now what about I(2)? From the definition of
t 2
function I(t) we see that I(2) = 2(2) = 4. Thus, left and right hand limits of I(t) at t = 2 and I(2) are
all equal and finite. Thus, the flow of current is continuous at t = 2 second.
Example 07: A colony of bacteria is introduced to a growth – inhibiting environment and
grows at time t according to the formula:
 t  4, if  6  t  2

B  t    t, if  2  t  2
 t  4, if 2  t  6

Is the growth of bacteria continuous at t = 2?
Solution: The given function will be continuous at t = 2, if Lim B  t   Lim B  t   B  2 
t 2 t 2
Now, Lim B  t   Lim  t  4   2, Lim B  t   Lim  t   2 
t 2 t 2 t 2 t 2
Since, Lim B  t   Lim B  t   Therefore, the given function is not continuous at t = 2.
t 2 t 2

WORKSHEET 02
1. Evaluate the following limits:
x 4  3x x 1 | 1  x | 1 3 x
 a  xlim
 x  x  9
5 2
 b  lim
x 1 0
 c  lim
x 0
 d  lim
x 30 | x  3 |
x 1
2 |x|
2. Suppose the cost C of removing p percent of the particulate pollution from the smokestacks of
7300p
an industry plant is given by: C(p)  , find lim C  p 
100  p p
3. The monthly charge (in dollars) for x kilowatt hours (KWH) of electricity used by a
commercial customer is given by the following function:
-:52:-
CHAPTER TWO LIMIT AND CONTINUITY OF FUNCTIONS
7.52  0.1079x, 0x5
19.22  0.1079x, 5  x  750

Cx  
20.795  0.1058x, 750  x  1500
131.345  0.0321x, x  1500
a  lim
x 15000
C x   b lim C  x  .
x 5
4. The monthly charge for water in a small town is given by:
18, 0  x  20
f x   . Find lim f  x  .
18  0.1 x  20 , x  20 x 20

1400x  250
5. The profit from the sale of x units is: P  x   dollars for x  1
x
What is the limit of profit as the quantity sold increases without bound?
6. The amount of a drug that remains in a person’s bloodstream t hours after being injected is
given by: f(t) = 0.15t/(1 + t2). Find
lim f (x) if it exists. Explain the result you obtain.
x 

 (x 3  27) / (x  3), x 3
7. Function f(x) is defined as: f (x)  

9 / 2, x 3
Is f (x) continuous at x = 3?
x  2 0  x 1

8. Is the following function continuous? f (x)   x 1 x  2
x  5 2x3

9. Examine the continuity of following function at x = 0.
 e1/x  sin 7x
 x0  x0
(a) f (x)   e1/x  1 (b) f (x)   sin 6x
0 x0 
6 / 7 x0


1  4x 
1/x
x0
(c) f (x)   .

 e 4
x  0
10. Use   definition, show that f(x) = x – 3 is continuous at x = 4.
3

11. In 1986, Tax Reform Act created the following tax schedule for a single filers in 1988.
Tax Rate Tax Income
15% 0 - $17850
28% Above $17850
This means that the tax T is a function of income x as follows:
0.15x, 0  x  17850
T x  
2677.50  0.28  x  17850 , x  17850
Is T(x) continuous?
12. Residential customers in a small town have their monthly charge f(x) for x hundred gallons of
18, 0  x  20
water given by: f x  
18  0.1 x  20 , x  20
Is f(x) continuous?

-:53:-
CHAPTER TWO LIMIT AND CONTINUITY OF FUNCTIONS
13. Suppose that cost C of removing p percent of particulate pollution from the exhaust gases at
8100p
an industrial site is given by C(p)  . Describe any discontinuity for C (p).
100  p
14. The monthly charge (in dollars) for x kilowatt hours (KWH) of electricity used by a
commercial customer is given by the following function:
7.52  0.1079x, 0x5
19.22  0.1079x, 5  x  750

Cx  
20.795  0.1058x, 750  x  1500
131.345  0.0321x, x  1500
a. What will be the charges for using the 5 and 750 KWH of electricity?
b. Is the function continuous at x = 5 and x = 15?
15. Suppose the size of a population of bacteria in time t is given by:
t, when 0  1  1 2

P  t   1, when t=1 2
1  t, when 1 2  t  1

a. How many bacteria were originally present?
b. Is the function continuous at t = 1/2 hours?
16. The amount of a drug that remains in a person’s bloodstream t hours after being injected is
t  2, when 0  t  1

given by: f  t   t, when 1  t < 2
t +5, when 2  t  3
a. What was the amount of drug at the beginning?
b. Is the function continuous at t = 2 and t = 3 hours?
17. Suppose the cost of obtaining water that contains p% impurities is given by:
120, 000
  1200, if 0  p  10
C  p   p
10800, if 10  p  12

a. What is the cost of obtaining water that contains 12% impurities?
b. What is the cost of obtaining water that contains 5% impurities?
c. Is the function continuous at p = 10%?
18. The velocity V(t) m/sec of a particle is given as:
t, when t  0

V  t   t 2 , when 0 < t  4
2t  4, when t  4

a. What was the velocity at the beginning?
b. Is the velocity of the particle continuous at t = 0 and t = 4?
19. Suppose that the cost to remove x percent of the pollutants in a lake is given by:
 900,000
 , if 0  x  20
C  x    100  x
11250, if x  20
a. What is the cost to remove 10% of the pollutants from the lake?
b. Is the function continuous at x = 20%?

-:54:-
CHAPTER THREE DERIVATIVES

CHAPTER
THREE DERIVATIVES

INTRODUCTION
Before proceeding to a formal definition of derivative, consider some background information on
the nature of the derivative. Simply stated, the derivative is a rate of change. Three examples are
given next to offer some preliminary insight into the notion of rate of change:
 Consider an outbreak of the flu. A function specifies the number of people sick with flu
at any particular time. The derivative of the function indicates the rate at which illness
due to flu is spreading at any particular time.
 Suppose that a function gives the average cost per unit of producing x units. The
derivative of the function gives information about when the average cost per unit is
increasing and when it is decreasing.
 A function may describe the motion of a rocket, giving the distance traveled for any time
t. The derivative of this function is the rate of change of distance with respect to time –
the velocity. Using the derivative, we can determine the velocity at any instant desired.
Derivatives are also used to find the slope of a tangent line to curve y = f(x) at any point. For
2
example, what would be the slope of the tangent line to the curve say y = x + 1 at x = 2?
Major techniques of calculus are Differentiation and Integration or anti-derivative. The part of
calculus that is associated with differentiation is called differential calculus and the part of
calculus that involves integration is known as integral calculus. The main objectives of
differential calculus are to establish the measure of the changes in a function with mathematical
accuracy. “Differentiation is a process of finding the rate at which one variable quantity changes
with respect to another.” For example, fashion varies with time.
Increment of a Function
Literally the word increment means an increase; but in MATHEMATICS increment means small
change in the value of a variable. Increment of a variable may be positive or negative. The
increment in the variable x is denoted by x or x or h. Thus,
x = increment in the variable x
y = increment in the variable y.
Procedure to Find the Derivative of a Function
Assuming that x is an initial value of the independent variable, in which case y will denote the
initial value of the function, so that we have the relation
y = f(x)
. (1)
Let there be an increment /change x in x and the corresponding change in y be  y .
Then equation (1) becomes, y  y  f  x  x  (2)
Subtract (1) from (2), we get y  f  x  x   f  x  (3)
Dividing both sides of (3) by x to get the quotient

-:55:-
CHAPTER THREE DERIVATIVES
y f  x  x   f  x 
 (4)
x x
This ratio is called the incremental ratio or the average rate of change of f(x) over the interval ,
 x, x  x  Now we find the limit of this quotient as x  0, that is;
y f  x  x   f  x 
Lim  Lim (5)
x 0 x x 0 x
This limit, when it exists, is denoted by f   x  and is called the “Derivative” or Differential
Coefficient of y = f(x) with respect to x. This is also called the Instantaneous Rate of Change or
Slope of the curve y = f(x) at x. The method described above for finding the derivative of the
function f(x) is called First principle or ab-initio method. It is also known as derivative by
definition. Thus, (5) can be written as
y f  x  x   f  x 
f   x  = Lim  Lim
x 0 x x0 x
Other symbols used for the derivative of y = f(x) are: y`, Dy, dy / dx.
dy
Note: The notation is the originally by Leibniz.
dx
Example 01: Find the derivatives of the following functions by definition.
3 n
(i) y = x (ii) y = 1/x (iii) y = x
3
Solution: (i) Given y=x (1)
Let x be an increment in the value of x and  y the corresponding increment in the value of y
y  y   x  x 
3
so that (2)
Subtracting (1) from (2), we get
y   x  x   x3  x3  3x 2  x   3x  x    x   x3  3x 2  x   3x  x    x 
3 2 3 2 3

y
 3x 2  3xx   x 
2
Dividing both sides by x we get:
x
Taking the limit x  0, to obtain: Lim
y
x 0 x x 0
 2

 Lim 3x 2  3xx   x   3x 2 .

Since the limit exists (finite), hence, dy / dx  3x 2


(ii) We have y = 1/x (1)
Let x be an increment in the value of x and  y be the corresponding increment in the value of
1
y so that: y  y  (2)
 x  x 
1 1 x   x  x  x
Subtracting (1) from (2), we get: y    
 x  x  x x  x  x  x  x  x 
Dividing both sides by x and taking limit x  0, to obtain
y dy  1  1
Lim   Lim   2
x 0 x dx x 0  x  x  x   x
dy 1
Hence,  2
dx x
n
(iii) Given that y = x (1)
Let x be an increment in the value of x and  y be the corresponding increment in the value of
n
y so that: y  y   x  x  (2)

-:56:-
CHAPTER THREE DERIVATIVES
Subtracting (1) from (2), we get
n(n 1)
y   x  x   xn  xn  nxn1  x  
n 2 n
x  x   ...   x   xn
2!
Dividing both sides by x, and taking the limit x  0 to obtain
 n  1  n(n  1) x  x   ...   x n  1 
y  nx 
Lim  Lim x  2!  n x n  1  0  0  ...  0

x0 x x0  x 
 
dy dy
Since the limit exists (finite) hence,  nx n 1 . Thus if y = xn then  nx n 1 .
dx dx
One Sided Derivative
Recall that the function y = f(x) is said to be “Derivable” or “Differentiable” at x if the limit
y f  x  x   f  x 
Lim  Lim exists.
x 0 x x 0 x
Equivalently, we say that a function is derivable if the one – sided limits
f x  h  f x f x  h  f x
Lim and Lim
h 0  h h 0 h
exist and are equal. (Note: h = x ). If these limits exist, they are respectively denoted by Lf   x 

and Rf   x   If Lf   x  and Rf   x  exist but are not equal, that is; Lf   x   Rf   x  , we say
that the function f(x) possesses left hand and right hand derivatives but it is not derivable or
differentiable at x.
Derivative at a Point
It may be noted that if the derivative of f(x) is required at some point say at x = a, where a  Df .
dy
The derivative is denoted by: f   a  or
dx x  a
f a  h   f a 
Thus by definition, f   a   Lim 
h 0 h
Writing a + h = x in the above equation, we have
f  x   f a 
f   a   Lim  Note : x  a as h  0
x a
x a
This is another way to represent the derivative of f(x) at x = a.
NOTE: If a function f(x) is differentiable, it must be continuous. But the converse is not true, that
is, if a function is continuous it may not be differentiable.
Example 02: Let f: R → R be defined by f(x) = | x |.
Discuss the continuity and differentiability of f at x = 0.
Solution: Continuity at x = 0:
i  f  0  0  0
 ii  Lim f  x   Lim x  Lim   x   0,
x 0  x 0 x 0
Lim f  x   Lim x  Lim  x   0. Now, Lim f  x   Lim f  x   Lim f  x   0
x 0 x 0 x 0 x 0 x 0 x 0

 iii  Lim f  x   f  0   0 
x 0
-:57:-
CHAPTER THREE DERIVATIVES
Hence, f(x) = | x | is continuous at x = 0.
Differentiability at x = 0
f  x   f a 
By definition: f   a   Lim  Lf '(a)  Rf '(a)
x a x a
f  x   f  0 x 0  x   Lim
Since x = 0, we have: Lf   0   Lim  Lim  Lim  1  1
x 0 x 0 x 0 x x 0 x x 0
f  x   f  0 x 0 x 
Rf   0   Lim  Lim  Lim Lim 1  1
x 0 x 0 x 0 x x 0 x x 0

Since, Lf   0  Rf   0 , therefore, f  x   x is not differentiable at x = 0 although it is


continuous there at.
 x, 0  x 1
Example 03: Let f  x   
2x  1, 1  x  2
Discuss the continuity and differentiability of f at x = 1.
Solution: Continuity at x = 1:
 i  f 1  1 Graph of f(x)

 ii  f (11 )  Lim f  x   Lim  x   1,f (1 ) Lim f  x   Lim  2x  1  1 


x 1 x 1 x 1 x 1
Since, Lim f  x   Lim f  x   Limf  x   1 
x1 x1 x1
 iii  Limf  x   f 1  1  Hence, f(x) is continuous at x = 1.
x 1
f  x   f a 
Differentiability at x = 1: By definition, f   a   Lim 
x a x a

Putting x = 1, we have: Lf  1  Lim


   Lim x  1  Lim 1  1
f  x   f 1

x 1 x 1 x 1 x  1 x 1

Rf  1  Lim
f  x   f 1   Lim  2x  1  1  Lim  2x  2  Lim 2  x  1  2 
x 1 x 1 x 1 x 1 x 1  x  1 x 1  x  1
Since, Lf  1  Rf  1 , therefore, f(x) is not differentiable at x = 1.
Example 04: Find the values of a and b so that the function f is continuous and differentiable at

x3 , x 1
x = 1, where f x  
ax  b, x  1

Solution: Continuity at x = 1:
 i  f 1  a 1  b  a  b
 ii 
x 1 x 1
 
f (1) Lim f  x   Lim x 3  1, f (1) Lim f  x   Lim  ax  b   a  b
x 1 x 1
For a function to be continuous, we must have
a  b 1 (1)
Differentiability at x = 1:
f  x   f (1 ) 3
x  (1)
3 2
(x  1)(x  x  1)
Lf  1  Lim  Lim  Lim  Lim (x 2  x  1)  3,
x 1 x 1 x 1 x 1 x 1  x  1 x 1

-:58:-
CHAPTER THREE DERIVATIVES
f (x)  f (1 ) (ax  b)  (a  b) (ax  a) a(x  1)
Rf  1  Lim  Lim  Lim  Lim a
x1  x  1 x1  x  1 x1  x  1 x1 (x  1)

For a function to be derivable, we must have: Rf  1  Lf  1 this implies that a  3 
Substituting it into (1), we get: 3 + b = 1 or b = – 2
Hence, for a = 3 and b = –2, the given function is continuous as well as derivable.
Derivative as a Slope
In the rectangular coordinates system, the rate at which y – coordinate of a straight-line changes
with respect to the x – coordinate, is known as the slope of the line. Thus the slope is the rate of
change of y with respect to x.
change in y  coordinates y2  y1 y
Slope  m     
change in x  coordinates x 2  x1 x
The slope of a line is a number that specifies the change in y compared with the change in x in
going from point to point on the line. For any particular line, the slope is constant for the entire
line. For example, y = f(x) = 3x + 2 has the slope +3 because y increases by three units for every
unit increase in x. Let us see this:
x: –2 –1 0 1 2
y: –4 –1 2 5 8
Now take any two values of x and the corresponding values of y, say x = –1 and 1. The
corresponding values of y are –1 and 5. By definition:
change in y – coordinates y2  y1 y 5  (1) 6
Slope  m       3
change in x – coordinates x 2  x1 x 1  (1) 2
Now let us take other two values of x say x = 1 and 2. The corresponding values of y are 5 and 8.
y  y 85
Using the formula, we get: Slope  m   2 1  3
x 2  x1 2  1
This is same as before. Thus no matter what two values of x and the corresponding values of y are
considered, the slope of a line always remains constant.
But what happens when the function under consideration is not linear? In order to consider the
rate of change in such instances, we might choose to extend the notion of slope to curves other
than straight lines. Consider, for example, the graph of the function f(x) defined by f(x) = x2
shown in the following figure.
How we define the slope of a curve that is not a straight line?
The answer is very simple! The slope of a curve at any point
is defined as the slope of the line tangent to the curve at that
point. Tangent line is that which touches the curve but does
not intersect it. Such a line has the same steepness as the
curve at that point. For example, the slope of the parabola
f(x) = x2 at (1, 1) is the slope of the line tangent to the curve
at(1, 1).
From the above figure, we see that the slope of the curve (or tangent line) at (1, 1) is 1/2.
perp 1
For the above curve, we see that: tan    2
base 1 2

-:59:-
CHAPTER THREE DERIVATIVES
Definition: A line that intersects the curve at the two distinct points is called the secant line.
Geometrical Meaning of Derivative
Let f(x) be a differentiable function whose graph is shown as under. Let P(x, y) and
Q  x  x, y y  be two distinct points near to each other. If  be the angle that the secant line
PQ makes with the x – axis, then
QS  y  y   y y
tan     (1)
PS  x  x   x x
This can also be expressed as
f (x  x)  f (x)
tan   msec  msec means the slope of the secant line.
x
y
Thus, msec  , slope of the secant line APQ 
x
As x gets smaller, the secant line comes
closer and closer tangent line at P.
As x approaches zero, the secant line approaches
the tangent line. This means that the slope of the
the tangent line is the limit of the slope of the secant
line as x approaches zero. Hence taking limit
as x  0, equation (1) becomes:
y dy
Lim m  Lim  tan   mtan 
QP sec x0 x dx
Thus, the derivative of the function f at the point P represents the slope of the tangent line to the
curve y = f(x) at a point x.
Physical Meaning of Derivative
At the beginning of the chapter, we illustrated different problems of rate of change of some
different quantities with respect to time. This rate of change is known as the derivative. There are
enormous applications of derivatives in physical problems where the rate of change is under
consideration. Let us elaborate this in some more detail. Consider a particle moving along a
straight line. At any instant t, let s be its distance from a fixed point on the line.
s
O P

Then the distance s is a function of time t, which we write as s = f (t). During the interval [t 1, t2],
it travels a distance f (t2) - f (t1). The ratio f (t 2 )  f (t1 ) gives the average velocity/speed during
t 2  t1
the interval [t1, t2]. Now the particle may be moving faster at some point and slower at others.
Given any point to the velocity at to can be approximated by the average velocity in a small
interval containing to. Therefore,
f  t   f  t0 
f   t 0   Lim
t t 0 t  t0
may reasonably be called the “instantaneous velocity” at t0 or the rate of change of s with respect
to t at the instant t = to. Thus, the velocity v at the instant t is

-:60:-
CHAPTER THREE DERIVATIVES
s f  t  t   f  t  ds
v  Lim  Lim   f t  
t 0 t t 0 t dt
It is called the instantaneous velocity at time t.
Example 05: A body moves horizontally and its position (in feet) at time t (seconds) is
3 2
s = t – 6t + 9t. Find the body’s acceleration at time its velocity is zero.
3 2
Solution: We have s(t) = t – 6t + 9t. Differentiating with respect to t, we get
d
v  s  t  
dt
d 3
dt
 
t  6t 2  9t  3t 2  12t  9 

Again differentiating, a  dv d

dt dt

3t 2  12t  9  6t  12 (1)

Velocity of the body will be zero if


3t 2  12t  9  0 or t 2  4t  3  0 or t 2  t  3t  3  0 or  t  1 t  3   0 or t  1,3 
2
Thus, acceleration of the body at t = 1, a = 6(1) – 12 = – 6 ft/sec .
2
And the acceleration of the body at t = 3, a = 6(3) – 12 = 6 ft/sec
RULES OF DERIVATIVES
In the beginning, we obtained derivatives of functions by using its definition, namely
f  x  x   f  x 
f   x   Lim 
x
x 0
Because using the definition of the derivative can be a lengthy and difficult procedure, rules of
differentiation have been developed.
Functions such as f(x) = 2 and f(x) = 5 are called constant functions because they have the same
value for every x. The graph of any constant function f(x) = c is a horizontal straight line parallel
to x–axis. The slope of any such line is zero, which means that the derivative f `(x) is zero. Thus,
the derivative of a constant function is zero.
1. Constant Rule: The derivative of a constant function is zero, that is;
d
 c   0, c  R 
dx
d d
For example,  5  0,  10   0 and so on. O
dx dx
2. Power Rule: If f(x) = x where n  R , then f   x   nx n 1 .
n

For example:
d 2
dx
 
x  2x 21  2x,
dx
 
d 1 3 1 1 31 1 2 3
x  x
3
 x
3

1
3x 2 3
, and

    x   1 x   1 x 2  
d  1  d 1 11 1
dx  x  dx x2
3. Coefficient Rule: If f(x) = c u(x), c being a real constant then f `(x) = x u`(x).
For example:
d
dx
 
5x 3  5 
d 3
dx
   
x  5 3x 2  15x 2 

4. Sum Rule: If f(x) = u(x) + v(x) then f `(x) = u`(x) + v`(x).


For example:
d
dx

9x 2  3  d
dx
  d
9x 2   3  9  2x   0  18x
dx
5. Difference Rule: If f(x) = u(x) – v(x) then f `(x) = u`(x) – v`(x)
-:61:-
CHAPTER THREE DERIVATIVES

For example:
d
dx

4x 3  5 
d
dx
 d
 
4x 3   5   4 3x 2  0  12x 2 
dx
 
6. The Product Rule: If f(x) = u(x) . v(x) then f `(x) = u(x) v`(x) + v(x) u`(x)
For example:

dx

d 2 13
x x  x2            
d 13
dx
x  x1 3 
d 2
dx
x  x2 
 1 
2
 3x 3  x
13 1
 2x   x 4 3  2x 4 3
3
 
7. The Quotient Rule: If f(x) = u(x)/v(x) where u and v are differentiable functions of x with
v  x   0, then f   x    v  x   u  x   u  x   v  x   /  v  x 
2

For example:

  
 
d  x 2  4x   x  5 2x  4   x  4x 1 2x 2  4x  10x  20  x 2  4x x 2  10x  20
2
 
dx  x  5   x  5 2  x  5 2  x  5 2
8. The Chain Rule: The chain rule is defined in the following way:
dy dy du
If y = f(u) and u = g(x) then,  
dx du dx
1 x dy
Example 06:For example: Let y  , find using chain rule.
1 x dx
1 x
, then y  u   u 
12
Solution: Let u 
1 x
dy d  1/2  1 1/2 1 1 1 1 x
  u  u    
du du   2 2 u 1 x 2 1 x
2
1 x
du d  1  x  1  x 1  1  x  1 1  x  1  x 2
Now,     
dx dx  1  x  1  x  2
1  x  2
1  x 2
dy dy du 1 1  x 2 1
U sin g chain rule formula,      
dx du dx 2 1  x 1  x  2
1  x 1  x 
32

9. The General Power Rule: A direct result of the Chain Rule is a very useful rule, called the
n
general power rule, that is; if y = u where u is a differentiable function of x, then
dy du
 nu n 1 
dx dx

For example, if y  1  x , then


1 x

dy d  1  x  1  1  x 1 2 d  1  x  1  1  x 1 2 1  x 1  1  x  1


           
dx dx  1  x  2 1 x  dx  1  x  2  1  x  1  x 2
12 12
1 1 x  1 x 1 x 1 1 x  2 1
         
2 1 x  1  x  2 2 1 x  1  x  2
1  x 1  x 
32

NOTE: Students are well aware of the proofs of these rules which they have studies in their
intermediate book. Here we have simply reproduced them for the sake of revision.

-:62:-
CHAPTER THREE DERIVATIVES
Example 07: Find the derivative of the following functions
 2  5x 2
 i  y  2x3  5x 2  8  
 ii  y  1  3x 2 1  5x  iii  y 
x3  1
Solution:
(i)
dy d

dx dx

2x 3  5x 2  8 
dx
d
2x 3  
   
dx
d d
 
5x 2  8   2 3x 2  5  2x   0  6x 2  10x
dx
(ii)
dy d 

dx dx    
1  3x 2 1  5x    1  3x 2

d
dx
 d
1  5x   1  5x  1  3x 2
dx
 
 1  3x 2   5   1  5x  0  6x   5 1  3x 2   6x 1  5x 
dy
Or,  5  15x 2  6x  30x 2  45x 2  6x  5
dx

(iii)
dy d   2  5x  
 
2

 
x 3  1  2  2  5x  5     2  5x  3x 2
2
 
dx dx  x 3  1 
 
2
  x3  1

 
10  2  5x  x 3  1  3x 2  2  5x 
2
 2  5x  10  x 3  1  3x 2  2  5x  
 
   x3  1
2 2
x3  1

 2  5x  10x 3  10  6x 2  15x 3   2  5x   5x 3  6x 2  10 


 
 x  1  x3  1
3 2 2

 

dy   2  5x  5x  6x  10
3 2


 
dx 2
x3  1
Derivatives of Trigonometric Functions
In this section we shall derive the formulae for the derivatives of trigonometric functions.
1. Find the derivative of y = sin x by ab–initio method
Solution: Let x be changed to x  x and consequently y becomes y  y, so that
y  y  sin  x  x  or y  sin  x  x   y or y  sin  x  x   sin x
uv uv
Using the formula sin u  sin v  2cos sin , we get
2 2
x  x  x x  x  x 2x  x x  x  x
y  2cos sin or y  2cos sin  2cos  x   sin
2 2 2 2  2  2
Dividing both sides by x , we get
 x  x  x  x x
2cos  x   sin cos  x   sin sin
y  2  2 y  2  2 y  x  2
     cos  x   x
x x x  x x  2 
2 2
Taking limit as x  0, we get
 x  x
  sin  sin
y x  2 dy   x  2
Lim  Lim cos  x   x    Lim cos  x   Lim x
x 0 x x 0   2   dx x  0  2  x  0
 2  2
dy
Applying the limit, we get  cos x
dx
-:63:-
CHAPTER THREE DERIVATIVES
Similarly, you can find the derivatives of remaining five trigonometric functions. The following
table will help you to memorize the derivatives of the trigonometric functions.
. y dy/dx y dy/dx
. sinx cos x cos x –sin x
. tan x 2
sec x cot x –csc x
. sec x sec x tan x csc x – csc x cot x
NOTE: Students are advised to find the derivatives of the remaining trigonometric functions by
ab-initio method or they may refer to Intermediate book for this.
Example 08: Find the derivatives of the following functions
sin x sin x
i y  ii  y  tan  2x  3   iii  y  x2  a2  iv  y
1  cos x sin x
dy d  sin x  1  cos x  cos x    sin x  0  sin x 
Solution: (i)    (division rule)
dx dx  1  cos x  1  cos x 2
dy cos x  cos 2 x  sin 2 x cos x
OR,     cos 2 x  sin 2 x  1
dx 1  cos x  2
1  cos x  
2 

dy d d
(ii)   tan  2x  3   sec 2  2x  3  2x  3   sec 2  2x  3  2   2sec 2  2x  3 

dx dx dx

 
1/2
(iii) Given that y  x 2  a 2  x 2  a 2 . Differentiating w.r.t x, we get

   
dy 1 2 1/2 d 1 x
 x  a2 . x2  a2   2x  0  
dx 2 dx 2 x2  a2 x2  a2
sin x
(iv) Given that y  . Differentiating w.r.t x, we get
sin x

dy
sin x
d
dx
 
sin x  sin x.
d
dx

sin x 

sin x 
dx 2

1
sin x.
2
d
 sin x 1/2 .  sin x   sin x.cos x.
dx
d
dx
 x

sin 2 x
1  cos x 1 
  sin x.  sin x.cos x. 
sin 2 x  2 sin x 2 x
dy 1  x sin x.cos x  sin x cos x 
Thus,   
dx 2sin 2 x  x. sin x 
Derivatives of Inverse Trigonometric Functions
-1
Let y = sin x then x = sin y. Differentiating both sides w. r. t y, we get:
dx
dy
 cos y  1  cos y or
dy

1
dx cos y

1
2
1  sin y

1
1 x 2

d
dx
sin 1 x 
1
1 x 2

,x 1 
Similarly, you can find the derivatives of remaining five inverse trigonometric functions. Students
are advised to find the derivatives of remaining inverse trigonometric functions. They are also
advised to refer to their Intermediate book.
The following table shows the derivatives of all trigonometric functions.
-:64:-
CHAPTER THREE DERIVATIVES
y dy / dx y dy / dx
1 1
sin 1 x ,x 1 cos 1 x
1  x2 1  x2
1 1
tan 1 x cot 1 x 
2
1 x 1  x2
1 1
sec 1 x csc 1 x 
2
x x 1 x x2 1
Example 09: Find the derivative of the following functions
 2x   x sin a 
i y  cot 1    ii  y  tan 1  
 1  x2   1  x cos a 

Solution:  i 
dy d  1  2x  
 cot 
dx dx   
 1  x 2 
1
 2x 
2

d  2x   d
 2  
dx  1  x   dx

cot 1 x  1 

1  x2 
1  
 1  x2 

1  x 2   2    2x  2x   1  x 2 
2
1 2  x 2  4x 2
   
1  x 2   4x 2 1  x 2  1  x 2 
2 2
1  2x 2  x 4  4x 2 2

1  x 2 
2

dy   2  3x    2  3x 
2 2
or  
dx 1  2x 2  x 4 (1  x 2 ) 2
 x sin a 
(ii) Given that y  tan 1   . Differentiating w.r.t x, we get
 1  x cosa 
dy 1 d  x sin a 
 .  
dx  x sin a 
2 dx  1  x cos a 
1  
 1  x cos a 
d d
1  x cos a  2 1  x cos a  .  x sin a   x sin a 1  x cos a 
 . dx dx
1  x cos a 2   x sin a 2 1  x cos a  2


1
.
1  x cos a  .sina  x sin a.   cos a 
2 2 2 2
1  2x cos a  x cos a  x sin a 1
sin a  x sin a cos a  x sin a cos a sin a
 

1  2x cos a  x 2 sin 2 a  cos 2 a 1  2x cos a  x
2

 2x   2x 
Example 10: Differentiate tan 1  with respect to sin 1 
2 
1 x   1  x2 
 2x   2x 
Solution: Let u  tan 1   ; v  sin 1   . It is required to find du / dv.
 1  x2   1  x2 
 2 tan  
Putting x  tan     tan 1 x. Thus, u  tan 1  2 
 tan 1  tan 2   2  2 tan 1 x
 1  tan  

-:65:-
CHAPTER THREE DERIVATIVES
Differentiating with respect to x, we get
du

2
dx 1  x 2


d
 1
since dx tan x  
1 

1  x2 
(1)

 2 tan  
Similarly, v  sin 1  1 1
  sin  sin 2   2  2 tan x
 1  tan 2  
Differentiating with respect to x, we get
dv

2
dx 1  x 2


d
 1
since dx tan x  
1 

1  x2 
(2)

du du dv 2 2
From (1) and (2), we have     1
2
dv dx dx 1  x 1  x2
 2x  
1 1  x
2
Example 11: Differentiate tan 1   with respect to cos  
 1  x2   1  x2 
 

 2x  
1 1  x
2
Solution: Let u  tan 1   ; v  cos  
 1  x2   2
1 x 
It is required to find du/dx. Putting x  tan  so that   tan 1 x  Now,
 2 tan  
u  tan 1  1 1
  tan  tan 2   2  2 tan x
 1  tan 2  
Differentiating with respect to x, we get
du

2
dx 1  x 2


d

1
since dx tan x 
1 

1  x2 
 (1)

 1  tan 2  
Similarly, v  cos 1    cos1  cos 2   2  2 tan 1 x
 1  tan 2  
 
Differentiating with respect to x, we get
dv

2
dx 1  x 2


d
 1
since dx tan x  
1 

1  x2 
(2)

du du dv 2 2
From (1) and (2), we have:     1
dv dx dx 1  x 2 1  x 2
Derivatives of Exponential and Logarithmic Functions
In this section we shall find the derivatives of:
(1) loga x (2) ln x (3) ax, a > 0 (4) ex
dy 1 1
1. If y  loga x,  x  0, a  1 , then  loga e 
dx x x ln a
xh
log a  
dy f x  h  f x log  x  h   log a x  x 
Proof: By definition,  Lim  Lim a  Lim
dx h 0 h h 0 h h 0 h
x
1  h 1x  h 1  h h
 Lim loga 1    Lim loga 1    Limloga 1  
h 0 h  x  h 0 x h  x  x h 0  x
-:66:-
CHAPTER THREE DERIVATIVES
 x  x 
dy 1   h h  1   h h 
  log a  Lim 1     log a e  NOTE :Lim 1    e  (1)
dx x h 0  x x h 0  x
   
If we want to convert the base of the log function from a to the base e, then following formula is
logc b
used: loga b 
logc a
log e
e  ln e 1
Now, loga e     Note: ln x  log e x and log e e  1
loge a ln a ln a
d 1 1
Thus, equation (1) becomes:  loga x   loga e 
dx x x ln a
2. If we put a = e in (1) then y  log e x  y  ln x so that
dy d 1 1
  ln x     NOTE : ln e  1
dx dx x ln e x
dy
3. If y  a x , then  a x ln a
dx
Proof: Given that f  x   y  a x  By definition,

dy f x  h  f x a x h  a x
 Lim  Lim
dx h 0 h h 0 h
dy   a h  1   ah 1    ah 1  
 Lim a x     a x Lim    a x ln a  Note : Lim    ln a 
dx h 0   h   
h 0  h 
  h 0  h  
 
dy
Thus, if y  a x , then  a x ln a
dx
dy
4. If f  x   y  ex , then  ex
dx

Proof: Using the result of (3) if we put a = e, we obtain:


d x
dx
 
e  ex  ln e  ex

An Alternative Method
x 2 x3 x 4
(i) Let y  e x  1  x     ... This is the series expansion of ex.
2! 3! 4!
Differentiating both sides w . r . t x, we get,
dy 2x 3x 2 4x3 x2 x3
 0 1    ...  1  x    ...
dx 2.1 3.2.1 4.3.2.1 2.1 3.2.1
dy x 2 x3 x 4 dy
1 x     ...  e x . Thus if y = ex, then  ex
dx 2! 3! 4! dx
y
(ii) y = ln x = loge x. Taking the antilog on both sides, we get: e = x.
Differentiating both sides w. r. t y, we get
dx y dy 1 1 dy 1
 e or   . Thus if y  ln x or 
dy dx e y x dx x
(iii) Let y = ax. Taking log on both sides, we get:
ln y = ln ax or ln y = x ln a or x = ln y / ln a
-:67:-
CHAPTER THREE DERIVATIVES
Differentiate both sides w . r . t y, we get
dx 1 dy dy
 or  yln a  a x ln a . Thus if y = ax,  a x ln a .
dy yln a dx dx
y
(iv) Let y = loga x. Taking the antilog on both sides, we get a = x. Differentiating both sides
y dx dy 1 1 dy 1
w . r . t y, we get: a ln a     . Thus if y  log x then,  
dy y
dx a ln a x ln a a dx x ln a
The following table summarizes the above formulae.
y . dy/dx y . dy/dx
x x
e e ln x 1/x
x x
a a ln a loga x ln a/x
.

Example 12: Find the derivatives of the following functions.


 1  x2  2 2  ex  1 x 
 i  y  ln    ii  y  32x  x  iii  y  e2x  x  iv  y  ln    v  y  ln  
2  1  ex 
1 x    1 x 
dy d   1  x 2   1 d  1  x2  d 1
Solution: (i)  ln       , using  ln x  
dx dx   1  x 2    1  x 2  dx  1  x 2  dx x
   
 1  x2 
 



  2
 

dy  1  x 2   1  x  2x   1  x  2x   2x  2x 3  2x  2x 3
2
4x
  
 2 
dx  1  x  

1  x2
2
  

2
1 x 1 x 2 1  x4  
(ii)
dy d  2x 2  x  2x 2  x d
 3
dx dx  

3 
dx

2x 2  x  ln 3, using
dx

d x
a  a x  ln a  
2 2
 32x  x  4x  1 ln 3   4x  1 32x  x ln 3

(iii)
dy d  2x 2  x  2x 2  x d
 e
dx dx  

e 
dx
2x 2  x  
2 2
 e2x  x  4x  11   4x  1 e 2x  x
 ex 
(iv) Given that y  ln    
  ln e x  ln 1  e x  x.ln e  ln 1  e x  x  ln 1  e x
 1  ex 
 
  
Differentiating w.r.t x, we get
dy
dx
1
1 e
1
x 0  ex  1  ex  ex
1 e x

1
1  ex
1 x 
(v) y  ln     
  ln 1  x  ln 1  x . Differentiating w.r.t x, we get
1 x 
dy

1
dx 1  x dx
.
d

1 x   1
1  x dx
.
d

1 x 
1 
1 x 
 
0
1 
 
1 
2 x  1 x 
 0
1 

2 x
 
1  1 1  1  1 x 1 x 
    
2 x 1  x 1  x  2 x  1  x 1  x 
    
dy 2 1
  
dx 2 x 1  x  x 1  x 
-:68:-
CHAPTER THREE DERIVATIVES
Derivatives of Hyperbolic and Inverse Hyperbolic Functions
We are familiar with the following hyperbolic functions:
e x  e x e x  e x e x  e x
sinh x  , cosh x  , tanh x  ,
2 2 e x  e x
e x  e x 2 2
coth x  , sec hx  , csc hx  
x
e e  x x
e e  x e  e x
x
Now, we find the derivative of these functions. Let’s differentiate the first function:
d  e x  e x  1 d x e x  e x  
d
dx
 sinh x   
dx  2

 2 dx


e e x 1 x
 e e
2
 
x

2
 cosh x

d
Therefore,  sinh x   cosh x
dx
NOTE: Students are advised to find the derivatives of the remaining hyperbolic functions. It may
be noted that the derivative of tanh x for example may be computed by letting
y = tanh x = sinh x/cosh x and then using the quotient law of derivatives. Same technique is used
for the other hyperbolic functions.
The complete list is given in the following table.
y . dy/dx y
. dy/dx
sinh x cosh x coth x –cosech2 x
cosh x sinh x sech x –sech x tanh x
tanh x sech 2 x cosech x – cosech x coth x
Derivatives of Inverse Hyperbolic Functions

Show that:
d
dx

sinh 1 x 
1

x2  1
Let, y  sinh 1 x or x  sinh y . Differentiating with respect to y, we get
dx dy 1 1 1
 cosh y or   
dy dx cosh y 1  sinh 2 y 1  x2

Hence,
d
dx
 
sinh 1 x 
1
2
x 1
The derivatives of other inverse hyperbolic functions may be found in a similar way. The list of
derivatives of inverse hyperbolic functions is given under. The readers are advised to solve them
taking the help of their teachers.
y dy / dx y dy / dx
1 1
sinh 1 x coth 1 x
x2  1 1  x2
1 1
cosh 1 x sech 1 x
2
x 1 x 1  x2
1 1
tanh 1 x csch 1 x
2
1 x x x2 1
Example 13: Differentiate the following functions
i  f  x   x a x sinh x ii  f  x   sinh 1  tanh x 
Solution: (i) We have, f  x   x a sinh x . Differentiating, we get
x

-:69:-
CHAPTER THREE DERIVATIVES
d
dx
f  x   
d
dx

xa x sinh x  xa x
d
dx
 d d
 
 sinh x   x a x sinh x   x  a x sinh x
dx dx
 xa x cosh x  xa x  ln a  sinh x  a x sinh x
d d  1 d
(ii)  f  x   
 sinh 1  tanh x     tanh x 
dx dx  
tanh 2 x  1 dx
 

sec h 2 x
since

d
sinh 1 x 

1 d
 
and  tanh x   sec h 2 x 
tanh 2 x  1 dx x2 1 dx 
Implicit Differentiation
Up until now, every function of x that we have differentiated has been given in an explicit
manner, such as : y  x 2  3x  2, and f  x   x3  6
In other words, the variable y has been written explicitly in terms of the other variable x. But in
practice very few functions occur explicitly. A function that is not explicit is known as an
“Implicit Function”. For example, the equations
3x 2  4y 2  2x  y  5, y3  x 3  xy  3, sin xy  e y  8  0
provide relationships between x and y in which y cannot be expressed explicitly in terms of x.
Moreover, it can be observed that for one value of x, there correspond many values of y. Thus
above equations are in facts not functions. Since such equations possess a relationship between
two variables, hence they are sometimes called implicit functions.
The process used to determine the derivative is called “implicit differentiation”. The following
procedure will be employed to determine dy/dx by implicit differentiation.
If the equation involve implicit function, then
i. Differentiate both sides (all terms) of the equation with respect to x.
ii. Collect all terms containing dy/dx on one side and remaining terms on the other side.
iii. Factor out dy/dx from all terms in the equation.
iv. Solve for dy/dx upon dividing both sides by the coefficient of dy/dx.
Example 14: Differentiate the following equation with respect to x
ysin 1 x  x tan 1 y  1
Solution: Differentiating both sides with respect to x, we get
 d

1

1  dy    d 1

1  dx   d
 y dx sin x  sin x.  dx     x dx tan y  tan y  dx    dx 1
     

  1   dy     1   dy  
y   sin 1 x      x      tan 1 y. 1   0

  1  x
2 
  dx     1  y 2   dx  
y  dy  x  dy 
 sin 1 x     tan 1 y  0
  2  dx 
1 x 2 dx 1  y

 1
 sin x 
x   dy 
  tan 1
y 
y
or   
 1  y 2 sin 1 x  x  2 1
  dy   1  x tan y  y
    dx 
 1  y 2   dx  1  x2
 1  y2 1  x2
 

dy   
1  y 2  1  x 2 tan 1 y  y 

Hence, 
dx
 
1  x 2  1  y 2 sin 1 x  x 
 
xy
Example 15: Find dy/dx if x 2  y2 
xy
3 2 2 3
Solution: Cross multiplying and simplifying, we get; x + xy – x y – y = x + y.

-:70:-
CHAPTER THREE DERIVATIVES
Differentiating both sides w.r.t x, we get:
2 2 2 2
3x + x.2y.y` + y .1 – x .y` - 2xy – 3y .y` = 1 – y`
2 2 2 2
Or (2xy – x – 3y + 1).y` = (1 – 3x – y + 2xy)
2 2 2 2
Or dy/dx = (1 – 3x – y + 2xy)/ (2xy – x – 3y + 1)
-1
Example 16: Find dy/dx given that x + sin y = xy
Solution: Differentiating both sides with respect to x, we get
d
dx
d
dx
  d
 x   sin 1 y   xy 
dx
  1  x 1  y 2 
1 dy dy 1 dy  . dy  y  1
1 .  x.  y.1 or   x  y  1 or 
1  y 2 dx dx  1  y2  dx  1  y2  dx
   
dy 1  y 2  y  1
or 
dx 1  x 1  y2
Logarithmic Differentiation
v
If f(x) = u , where both u and v are functions of x, the derivative of f(x) can be obtained by taking
natural logarithm of both the sides and then differentiating. This process of finding the derivative
is called “logarithmic differentiation.”
Example 17: Find the derivative of the function y = xcos x
cos x
Solution: We have y = x . Taking ln on both sides, we get
ln y  ln x cos x or ln y  cos x.ln x
Differentiating both sides with respect to x, we get
1 dy 1 cos x
 cos x    ln x   sin x    sin x ln x
y dx x x
dy  cos x   cos x 
or  y  sin x ln x   x cos x   sin x ln x 
dx  x   x 
x1/x
Example 18: Find dy/dx if y  sin 1 x  
x1/x

Solution: Given that y  sin 1 x  . Let, u = x
1/x
then y = (sin x) .
-1 u

-1 u -1
Taking ln on both sides, we get: ln y = ln (sin x) = u ln (sin x).
Now, differentiating both sides w.r.t x, we get
1 dy d du 1 d du
 u ln (sin 1 x)  ln (sin 1 x).  u sin 1 x  ln (sin 1 x).
y dx dx dx  1
sin x dx dx
dy  1 1 du 
 y u .  ln (sin 1 x)  (1)
dx  sin 1 x dx 
 1  x2
ln x
Now, u  x1/x . Taking ln on both sides, we get : ln u  ln(x1/x )  . Differentiating w.r.t x
x
1 du x(1 / x)  ln x(1) du  1  ln x 
.    u  . Put this in equation (1), we obtain
2
u dx x dx  x2 

-:71:-
CHAPTER THREE DERIVATIVES

dy  1 1  1  ln x    1 (1  ln x) ln(sin 1 x) 
 y u .  ln (sin 1 x)u    uy   
 sin 1 x 2   1 2 
dx  1  x 2  x    1  x 2
sin x x 
x1/x  (1  ln x) ln(sin 1 x) 

dy
dx

 x1/x sin 1  

 2
1
1

x 2
.

 1 x sin x 
x x
Example 19: Differentiate (sin x) with respect to (cos x)
Solution: Let u   sin x  and v   cos x  . It is required to find du/dv.
x x

First we take, u   sin x   Taking ln on both sides, we get: ln u  x ln  sin x 


x

Differentiating both sides with respect to x, we get


1 du  1  du
 x   cos x   ln  sin x 1 or  u  x cot x  ln  sin x 
u dx  sin x  dx
du
  sin x   x cot x  ln  sin x 
x
(1)
dx
Now we take, v   cos x   Taking ln, we get: ln v  x ln  cos x 
x

Differentiating both sides with respect to x, we get


1 dv  1  dv
 x    sin x   ln  cos x 1 or  x tan x  ln  cos x 
 v 
v dx  cos x  dx
dv
  cos x  
 x tan x  ln  cos x 
x
(2)
dx
Since du / dx   du / dv    dv / dx  , then from (1) and (2), we have

du  sin x   x cot x  ln  sin x 


x

dv  cos x  x  x tan x  ln  cos x  
 
x
Example 20: Find dy/dx if y = x
ln y  ln x x or ln y  x ln x
x
Solution: We have y = x . Taking ln on both sides, we get:
Differentiating both sides with respect to x, we get
1 dy 1 dy
 x    ln x  x   1  x ln x or  y 1  x ln x   x x 1  x ln x 
y dx x dx

Example 21: Find dy/dx if y  x  x  x  ...

Solution: Since y  x  x  x  ...  x  y . Squaring both sides, we get


y2  x  y. Differentiating both sides, we have
2y2
dy
 1
dy
or
dy
 
2y2  1  1 giving
dy

1
dx dx dx dx 2y2  1  
Example 22: Find dy/dx if x y  exy
Solution: Given x y  ex y . Taking ln on both sides, we get

   
ln x y  ln e x  y or y ln x   x  y  ln e  x  y  ln e  1
Differentiating both sides, we have
1 dy dy dy y dy xy dy xy
y  ln x.  1  or  ln x  1  1  or  ln x  1  or 
x dx dx dx x dx x dx x  ln x  1

-:72:-
CHAPTER THREE DERIVATIVES
Example 23: Find dy/dx if x y  y x  c
Solution: Given x y  y x  c. Let u  x y anf v  y x . Thus, u  v  c
Differentiating both sides w.r.t x, we get
du dv
 0 (1)
dx dx
Now, u  x y then ln u  ln x y  y ln x. Now differentiating w.r.t x, we get
1 du 1 du y  y 
 y.  ln xy` or  u   ln x.y`  x y   y`ln x  (2)
u dx x dx  x   x 
Now, v  y x then lnv  lny x  x lny. Now differentiating w.r.t x, we get
1 dv 1 dv x  x 
 x. .y` lny.1 or  v  .y` lny   y x  .y` lny  (3)
v dx y dx y  y 
Putting the values of du / dx and dv / dx, in equation (1), we get
y  x 
x y   y`ln x   y x  .y` lny   0
 x   y 
or yx y  x  y`.x y ln x  xy x 1y` y x ln x  0
or  x y ln x  xy x 1  .y`  yx y  x  y x ln x   0
   
 yx y  x  y x ln x 
or
dy
  
dx  x y ln x  xy x 1 
 
ln x
Example 24: Find dy/dx if y = x .
Solution: Taking ln on both sides, we get
 
ln y  ln x ln x  ln x.ln x   ln x  . Differentiating w.r.t x, we get
2

1 dy d 1 2ln x dy  2ln x  ln x  2ln x 


 2ln x.  ln x   2ln x.  or  y x  
y dx d x x dx  x   x 
x2
 1
Example 25: Find dy/dx if y  1  
 x
Solution: Taking ln on both sides, we get
x2
 x 1
  x ln  x  1  ln x  . Differentiating w.r.t x, we get
2
ln y  ln 
 x 
1 dy  1 1
 x2.    ln  x  1  ln x  .2x
y dx  x 1 x 
2
dy  2  x  x 1   x  1   1 
x  x2  x  1 
  y  x .   2x.ln     1     2x.ln  
dx   x(x  1)   x   x   x(x  1)  x  

Example 26: Find dy/dx if y  xa x sinh x


Solution: Taking ln on both sides, we get

 
ln y  ln x.a x .sinh x  ln x  lna x  ln(sinh x)  ln x  x lna  ln(sinh x).
Differentiating w.r.t x, we get

-:73:-
CHAPTER THREE DERIVATIVES
1 dy 1 1 d 1 1
  ln a.1  .  sinh x    ln a  .coshx
y dx x sinh x dx x sinh x
dy 1  1 
  y   ln a  coth x   xa x sinh x   ln a  coth x 
dx  x   x 
Example 27: Differentiate the following functions w.r.t x
 1  2x 
(i) tan 1  
 2x 
 1  2x 
Solution: Let y  tan 1   . Differentiating w.r.t x, we get
 2x 
dy 1 d  1  2x  2  x  2  x  2   1  2x  1
2
  
2 dx  2  x 
dx
1 
 1  2x   2  x   1  2x 
2 2
 2  x
2

 2x 
4  2x  1  2x 5 1
  
2
4  4x  x  1  4x  4x 2
5 x 1 x 1
2 2
 
(ii) sin 1 ex
Solution: Let y = sin 1 ex . Differentiating w.r.t x, we get
dy

1 d x
e 
ex
 
 
dx 2 dx
1  e2x
1  ex

(iii)  sin 1 x 2 

 

Solution: Let y  sin 1 x 2 . Differentiating w.r.t x, we get

       
dy 1 d 1 1 d 2
  sin 1 x 2  sin 1 x 2   sin 1 x 2  x
 
dx dx 2 2 dx
1 x

 
1 2x
  sin 1 x 2 .
1  x4
 x2  1 
(iv) f  
 x2 1 
 
 x2  1 
Solution: Let y  f   . Differentiating w.r.t x, we get
 x2 1 
 

dy  x2  1  d  x2 1 
 f      f  2 
 2 2


 x 2  1  d  x  1  2x   x  1  2x     
 x 2  1  dx  x 2  1   x  1  dx  
 
dx 2
       x2 1 
 
 
 x2  1   x2 1  x2 1  4x  x2 1 
 f    2x     f   
 x2 1  2  2 
   
2
   x2 1  x2 1  x 1 
 
-:74:-
CHAPTER THREE DERIVATIVES

(v) cos1 1  x 2
Solution: Let y  cos 1 1  x 2 . Differentiating w.r.t x, we get

 
dy 1 d  2 1 1 1/2
   1 x    . 1  x2  2x 
dx 2 dx   1 1  x2 2
1   1  x 2 
 
x 1
 
2
x 1 x 1  x2

(vi) sin 1 cot 1 (ln x) 
 
Solution: Let y  sin 1 cot 1 (ln x) . Differentiating w.r.t x, we get

  d
dy

1

d

cot 1 (ln x)   1

1    lnx 
2  1   ln x 2  dx
   
dx 2 dx
1  cot 1 (ln x) 1
1  cot (ln x)  

1  1  1 1
    
2  1   ln x 2   x 
   
2
1  cot 1 (ln x)   x 1   ln x   1  cot 1 (ln x)
2
 
 
(vii) exp csc1 x 1 

   
Solution: Let y  exp csc1 x 1  . Differentiating w.r.t x, we get

dy
dx
 exp csc1 x 1  
  dx
d
    
csc1 x 1  exp csc1 x 1  .
  1 2
x
1
x 1
 
d 1
dx
x  
 
csc1 x 1
 exp csc1 x 1  
     1  e
.   
1  x2  x 
2
x 2
1  x2
Example 28: For the following implicit equations, find dy/dx.
(i) x  y  c
Solution: Differentiating both sides w.r.t x, we get
d d d 1 1 dy 1 dy 1 dy y
x y c   0    
dx dx dx 2 x 2 y dx 2 y dx 2 x dx x
(ii) x  y  c
Solution: Differentiating both sides w.r.t x, we get
d d d 1 1 dy 1 dy 1 dy y
x y c   0    
dx dx dx 2 x 2 y dx 2 y dx 2 x dx x

 
3
(iii) x 2  y2 y
Solution: Differentiating both sides w.r.t x, we get
d x 2  y2  
 2 3
 2 2
   
d 2 d 2
x y  .y  3 x  y 2
.  y  3 x 2  y 2 .  2x  2y.y   y
dx dx dx

   

   
2 2 2 2
 6x x 2  y 2  6y x 2  y 2 y  y  6y x 2  y 2  1 y  6x x 2  y 2
 

-:75:-
CHAPTER THREE DERIVATIVES

   
2 2
dy  6x x 2  y 2 6x x 2  y 2
  
dx 

2
6y x  y
2 2
 
 1



 2
1  6y x  y
2 2


 
Parametric Differentiation
If y = f(t) and x = g(t) then such equations are known as parametric equation and t is called
parameter. In such case, the derivative of y w.r.t x is found as follows:
dy dy dx
  (1)
dx dt dt
Example 01: Find dy/dx in each of the following case.
(i) y = a(1 – cos t), x = a(1 – sin t)
Solution: Given equations are parametric equations hence,
Given equations are parametric equations hence,
dy dy dx d d a  0  sin t  2sin(t / 2).cos(t/ 2) t
   a 1  cos t   a  t  sin t     tan  
dx dt dt dt dt a 1  cos t  2
2sin (t / 2)  2

3at 2 3at
(ii) y  ,x
2
1 t 1  t2
Solution: Given equations are parametric equations hence,

dy dy dx d 3at 2
   
d 3at
     
 1  t 2 .6at  3at 2  2t    1  t 2 .3a  3at  2t  
   
dx dt dt dt 1  t 2 dt 1  t2
1  t2
2
  1  t2
2
   
1  t 2 
2
3 3
6at  6at  6at 6at 6at 2t
    
3a 1  t  1  t 
1  t  2 2 2 2 2
3a  3at  6at 3a  3at

CASE STUDY PROBLEMS


In this section, we shall study some case study problems taken from diverse areas of physical,
social sciences, economics etc.
Rates of Change
Applications that follow from interpreting the derivatives as a rate of change are presented in this
section. To set the stage for the rate of change interpretation, we begin with three examples of
average rate of change.
Example 01: If John walks 7 miles in 2 hours, what is his average speed?
Solution: Average speed is defined as the change in distance Δs divided by the change in time Δt.
Average speed = s / t
s 7miles
In this instance, = =3.5miles per hours. Hence, his average speed is 3.5miles per
t 2hours
hours.
Example 02: From 1986 to1991, M/s Dunhill’s annual revenue increased from $861,000,000 to
$1,359,000,000. What was the average change in revenue per year during this time?
Solution: The average change in revenue per year is the change in revenue ΔR divided by the
change in years Δt.

-:76:-
CHAPTER THREE DERIVATIVES
R 1,359,000,000  861,000,000 $498,000,000
   $99,600,000 per year.
t 1991  1986 5 years
It is observed that from 1986 to 1991, Amtrak’s annual revenue increased at the average rate of
$99,600,000 per year.
2
Example 03: What is the average rate of change of y = x from x = 1 to x = 5?
Solution: The average rate of change is the change in y divided by the change in x, namely:
y  5   1
2 2
25  1 24
    6.
x 5 1 4 4
Example 04: A ball dropped from the top of a cliff will fall such that the distance it has traveled
2
after t seconds is s(t) = –16t feet.
(a) What is the average velocity for the first 3 seconds? (b) How fast is the ball traveling at 3
seconds?
Solution: (a) The average velocity for the first 3 seconds will be
 2  2
s s  3  s  0   16  3    16  0   144
=    48
t 30 3 3
Thus, the average velocity is 48 feet per second. The minus sign indicates that the ball is traveling
in a downward direction.
(b) The velocity at 3 seconds is an instantaneous velocity. It is the velocity at a specific time,
when t=3. So we need to evaluate ds/dt at t =3. Since, s = -16t2:

Therefore, v 
ds d
dt dt

16t 2  16
dt

d 2
 
t  32t feet per second.

ds
At t =3,  32  3  96 feet per second.
dt
This shows that the ball is traveling 96 feet per second (downward) after 3 seconds.
3 2
Example 04: Let y = 5x – x + 8x + 1. Determine the value of the derivative of y when x = 4.
3 2
Solution: Given that y = 5x – x + 8x + 1. Differentiate w.r.t x, we get
dy d

dx dx
 
5x3  x 2  8x  1  15x 2  2x  8

dy
 15  4   2  4   8  240
2
Putting x = 4 we obtain:
dx
Example 05: The driver of an experimental racing car begins a test run. During the first 6
1
seconds, the distance s (in feet) of the car from the starting point is: s = 14t 2 - t 3 , 0  t  6
3
where t is the number of seconds the car has been moving. What is the velocity of the car after
4 seconds have passed?
Solution: The velocity at any particular time is the value of ds/dt at that time.
Thus we seek ds/dt for t=4. Now,
1 ds d  1 
s  14t 2  t 3  v   14t 2  t 3   28t  t 2 .
3 dt dt  3 
ds d  2 1 3 
 14t  t   28  4    4   96.
2
Specifically, for t = 4, v
dt dt  3 
Hence, the velocity of the car after 4 seconds is 96 feet per second.
-:77:-
CHAPTER THREE DERIVATIVES
Example 06: In 1980 the population of one of small town in Sindh, was 355,000. In 1990 it was
328,000. What was the average rate of decrease in the population per year between 1980 and
1990?
Solution: The average rate of change of decrease in the population is the change in the population
P divided by the change in years t , namely

P 328,000  355,000
  2700.
t 1990  1980

It means that per year 2700 buffaloes are decreased.


Example 07: In 6 years a corporation’s annual profit increased from $10,000 to $130,600. What
was the average rate of increase in profit per year during this 6-year period?
Solution: Let P be the annual profit of the corporation. The average rate of increase in profit per
P 130,600  10,000 120,600
year during 6-year period will be:    20100.
t 6 6
Thus, it is $20100 per year.
Example 08: Assume that a hawk dives from a height of 300 feet and that its distance from the
ground at t seconds is s=300-16t2 feet. What is the hawk’s average velocity during the first 4
seconds?
Solution: The hawk’s average velocity will be the change in the distance s divided by the
change in time Δt .
 2  2
s s  4   s  0  300  16  4    300  16  0   44  300 256
     64
t 40 4 4 4
Thus, the average velocity of the hawk is 64 feet per second. The minus sign indicates that the
hawk is diving in downward direction.
Example 09: A toy rocket is shot straight up from the ground and travels so that its distance from
the ground after t seconds is s = 200t-16t2 feet. What is the velocity of the rocket after 2 seconds
have passed?
ds d
Solution: The velocity v is given by: v  
dt dt
 
200t  16t 2  200  32t .

After 2 seconds the velocity of the rocket v = 200  322  136 feet per second.
Example 10: A brick comes loose from near the top of a building and falls such that its distance s
2
(in feet) from the street (after t seconds) is given by s = 150 – 1 6t . How fast is the brick falling
after 3 seconds have passed?
Solution: The velocity after t seconds have been passed is:

v 
ds d
dt dt
  ds
150  16t 2  32t. Now, at t=3, we have v   32  3  96.
dt
Thus, the brick is falling with velocity of 96 feet per second (downward) after 3 seconds.
Example 11: A racing car begins a short test run and travels according to s= 8t 2 + (t 3 / 3) , where s
is the distance traveled in feet and t is the time in seconds. What is the velocity of the car after 3
seconds have passed?
ds d  2 1 3  1
Solution: The velocity of the car is: v    8t  t   16t  3   t 2  16t  t 2 .
dt dt  3   3
-:78:-
CHAPTER THREE DERIVATIVES
 ds 
 16  3   3  57 feet per second.
2
Now, after 3 seconds it is: v t 3   
  t 3
dt
Example 12: A colony of 1000 bacteria is introduced to a growth-inhibiting environment and
2
grows according to the formula n = 1000 + 20t + t , where n is the number of bacteria present at
any time t (t is measured in hours).
(a) According to the formula, how many bacteria are present at the beginning?
(b) What is the rate of growth of the bacteria at any time t?
(c) What is the rate of growth after 3 hours?
(d) How many bacteria are present after 3 hours?
Solution: (a) The number of bacteria present at the beginning is:
n  1000  20  0    0   1000.
2

(b) The rate of growth of the bacteria at any time t will be:

v
dn d

dt dt
 
1000  20t  t 2  20  2t.

(c) The rate of growth after 3 hours will be


dn
v  20  2t  20  2  3  26, bacteria per hour.
dt
(d) The number of bacteria present after 3 hours is:
n  1000  20  3   3  1000  60  9  1069.
2

Example 13: Suppose that in June a chain of stores had combined daily sales of ice cream cones
2
given by s = – 0.01x + 0.48x + 50, where s is the number of hundreds of ice cream cones sold
and x is the day of the month.
(a) How many ice cream cones were sold by the chain on June 3?
(b) At what rate were sales changing on June 10?
(c) At what rate were sales changing on June 28?
(d) On what day was the rate of change of sales equal to 10 cones per day?
Solution: (a) On June 3, the number of ice cream cones sold is:
s  [0.01 3  0.48  3  50] 100  5135
2

(b) Differentiate s w.r.t x, we get:


ds/dx = – 0.02x + 0.48. (1)
Put x = 10, and multiplying by 100, we obtain,
ds/dx = 0.28 (100) = 28.
th
This shows that on 10 June the rate of increase in the sale of ice cream cones is 28.
(c) Putting x = 28 in equation (1) and multiplying by 100, we get
ds/dx = – 0.08 (100) = – 8.
th
This shows that on 28 June, the rate of sale of ice cream cone decreases by 8.
(d) Putting ds/dx equal to 10 in equation (1) and dividing the right hand side by 100, we get:
10 = (– 0.02x + 0.48)/100 or x = 4.76 ~ 5
th
This shows that on 5 June the rate of change of sale of sale will be 10 cones per day.
Example 14: Given L = 12 t , find the rate of change of L with respect to t when t = 4.
Solution: The rate of change of L with respect to t will be:

-:79:-
CHAPTER THREE DERIVATIVES
d 1 
dL d
=
dt dt
12 
t = 12
dt  t 
d 1/2
= 12  t 
dt
 1  3/2
= 12     t 
 2 
=  6 / t 3/2

 dL 
when t = 4,    6 / (4)3/2  6 / 23  6 / 8  3 / 4.
 dt  t  4
Example 15: Determine the rate of change of temperature in degrees Fahrenheit (F) w.r.t C when
the relationship between F and C is F = 1.8 C + 32.
Solution: The rate of change of the temperature in degrees Fahrenheit (F) is
dF d
 1.8C  32  1.8
dC dC
Example 16: The volume V of a spherical balloon with radius r is V = (4 / 3) πr 2 . When, air is
blown into the balloon, both the radius and the volume of the balloon increase. Determine the rate
of the change of the volume with respect to the radius when the radius is 10 centimeters.
Solution: The rate of the change of the volume with respect to the radius will be;
dV d  4 3  4 dr 3 4
  r     (3.14)(3r 2 )  12.56r 2
dr dr  3  3 dr 3
3
When r = 10cm, cm .
Example 17: A dynamite blast blows a heavy rock straight up with a launch velocity of 160 ft/sec
2
(about 109 mph). It reaches a height of s = 160t – 16t feet after t seconds.
(a) How high does the rock go?
(b) What is the velocity and speed of the rock when
it is 256 feet above the ground on the way up; on the way down?
(c) What is the acceleration of the rock at any time t during its
flight (after the blast)?
(d) When does the rock hit the ground again?
Solution: (a) In the coordinate system we have chosen s as height from the ground up, so the
velocity is positive on the way up and negative on the way down. The time when the rock is at the
highest point during the flight its velocity is zero. Therefore to find the maximum height, all we
need to do is to find t when v = 0 and evaluate s at this time. Now at any time t, the velocity is:
ds d
v 
dt dt
 
160t  16t 2  160  32t feet/sec.

Equate this to zero, we get: 160  32t  0 or t  5sec.


This means that rock reaches at maximum height:
smax  s  5  160  5  16  5  400 feet.
2

(b) To find the rock’s velocity at 256 feet on the way up and on the way down, we find the two
values of t for which s  t   160t  16t 2  256  Let’s solve this equation,

160t  16t 2  256 or  16t 2  160t  256  0 or t 2  10t  16  0 giving t  2 or t  8.


The two values of t show that after the blast rock is at the height of 256 feet after 2 seconds on the
way up and also 8 seconds after the blast on the way back to the ground. The rock’s velocities at
these times are: v  2   160  32  2   96 ft/sec and v 8   160  32 8   96 ft/sec.
Thus at both instants, the rock’s speed is 96 ft/sec.

-:80:-
CHAPTER THREE DERIVATIVES
(c) At any time during its flight following the explosion, the rock’s acceleration is,
dv d
a  160  32t   32 ft/sec2
dt dt
2
Thus, the acceleration of the rock is – 32 ft/sec . Minus sign indicates that the direction is towards
the ground.
(d) When the rock hits the ground, the height is zero, that is; s = 0. So,
160t  16t 2  0 or t  0 or t  10
This shows that, the rock will hit the ground after 10 seconds,
Example 18: If two resistors of R1 and R2 ohms are connected in parallel in an electric circuit to
make an R ohm resistor, the value of R can be found from the equation
1 / R  (1 / R1 )  (1 / R 2 )
If R1 is decreasing at the rate of 1 ohm/sec, and R2 is increasing at the rate of 0.5 ohm/sec,
at what rate is R changing when R1 = 75 ohms and R2 = 75 ohms?
1 1 1
Solution: Given = + (1)
R R1 R 2
dR1 dR 2
 1 ohm/sec,  0.5 ohm/sec , R1  75 ohms , R 2  50 ohms
dt dt
1 R 2  R1 R1R 2
From (1), we have  or R  (2)
R R1R 2 R1  R 2
Differentiating (2) with respect to t, we get
d d
dR d  R1R 2   1
R  R 2   R1R 2   R1R 2  R1  R 2 
  dt dt

dt dt  R1  R 2   R1  R 2  2

 R1  R 2   R1
dR 2 dR   dR dR 2 
 R 2 1   R1R 2  1 
 dt dt   dt dt 

 R1  R 2 2
Substituting the values of R1 = 75 and R2 = 50, we get
dR  50  75 75  0.5   50  1   75  50  1  0.5 
  0.02 ohms / sec
dt  50  752
Hence R is changing at the rate of 0.02 ohms/sec.
Related Rates
In physical and social sciences many variables occur which are functions of time. Problems
involving rates of change of such variables with respect to time are called “Related rates
problems” Methods of solutions of such problems are illustrated by following examples.
Example 01: A 30 feet ladder is leaning against a building.
Suppose the ladder is sliding down the wall in such a way that
the bottom of the ladder is moving away from the wall at the
.

rate of 2 feet per second. At what rate is the top of the ladder
.

is sliding down the wall when the top of the ladder is 24 feet
.

above the ground?


.

-:81:-
CHAPTER THREE DERIVATIVES
Solution: Let AB be the ladder leaning against the wall OB and |OA| = x and |OB| = y. We are
given that at a certain instant of time y = 24 feet and dx/dt = 2 ft/sec and we have to find dy/dx at
.

this instant. From the right-angled triangle OAB, we have


. . x2 + y2 = 302 or x2 + 242 = 302 (1)
2
This gives x = 900 – 576 = 324 or x = 18 ft. Differentiating (1) with respect to time, we get
dx dy d dx dy
 2y   30  or x  y  0
2
2x
dt dt dt dt dt
Substituting the values, we get
dy dy 36
18 2   24  0 or    1.5
dt dt 24
The minus sign shows that the ladder is sliding down so the distance y decreases.
Thus, top of the ladder is sliding down at the rate of 1.5 feet/sec.
Example 02: A cylindrical Can with radius 6 inches and height 20 inches is
completely filled with water. Suddenly, it is punctured at the bottom, after which
.

the water pours out at the rate of 12 cubic inches per second. How fast is the water
.

level falling?
2
Solution: The volume of cylinder is given by given V = π r h. Here, r = 6 inches, h = 20 inches,
3
dV/dt = 12 inch /sec. we have to compute dh/dt = ? Differentiating V with respect to time t, we
dV dh dh dh 12
get:  36 or 12  36  3.14  or   0.11 inches/sec.
dt dt dt dt 113.04
Thus, the water level is falling (approximately) at the rate of 0.11 inches/sec.0000000
Example 03: If a tumor is approximately spherical in shape, its volume is approximately
4
V = πr3 . The radius of a tumor is growing in an animal is increasing at a rate of 1.25
3
millimeters per month. Determine how fast the volume of the tumor is increasing when the radius
is 10 millimeters?
dr dV
Solution: Given that  1.25 mm / month, r =10 mm, we have to find
dt dt
Now, V  (4 / 3)r 3 (1)
Differentiating (1) with respect to t, we get
dV 4
  3r 2
dt 3
 dr
dt
 4  3.14 100 1.25 or
dV
dt
 1570 mm3 /month

MARGINAL ANALYSIS
The managers of a manufacturing operation are naturally concerned about the total cost of
maintaining a particular level of production. In other words, they want to know the cost C(x) of
producing x units. Furthermore, when a particular level of production is being maintained, it is
important to know the cost of producing one additional unit. For example, if there are already 100
TV sets, what will it cost to make one more- the 101thTV? Such information assists management
in marketing decisions about production.
The rate of change interpretation of the derivative leads to a calculus application here. If C(x) is
the total cost of producing x units, then C`(x) is the rate of change of the total cost and gives the
approximate cost of producing one additional unit. C`(x) is called the marginal cost.
Similarly, if P(x) and R(x) are the profit and revenue then P`(x) and R`(x) are known as marginal
profit and marginal revenue respectively.
-:82:-
CHAPTER THREE DERIVATIVES
2
Example 01: Suppose the cost of producing x units is C(x) = 100 + 30x – x dollars
(for 0 ≤ x ≤ 12). Determine the marginal cost when x = 9.
Solution: The marginal cost is C`(x) = 30 – 2x.
For x = 9, we have: C`(9) = 30 – 18 = 12
This means that after 9 units are produced, the cost of producing the next unit (the tenth unit) will
be approximately $12.
It may be noted that exact cost of producing the tenth unit can be computed as

C 10 - C  9  = 100 + 30 10  - 10 


2
- 100 + 30 9  - 9 
2
= 300 - 289 = $11
   
Example 02: A furniture manufacturer determines that the marginal cost for making tables is
always increasing. The company decides to stop table production when the marginal cost reaches
2
$110. Assuming the cost function for table is: C(x) = 0.01x + 80x + 100. How many tables will
the company make before it halts table production?
2
Solution: Since the cost is given by C(x) = 0.01x + 80x + 100, it follows that the marginal cost
is C`(x) = 0.02x + 80.
Now it is given that marginal cost is $110. Hence, 110 = 0.2x + 80 or x = 1500
This means that before further production of table is stopped, the furniture manufacturer must
produce 1500 tables in order to get marginal cost equal to $110.
2
Example 03: The cost of producing x deep-tread radial tires is C(x) = 4000 + 70x -0.01x dollars,
2
and the revenue from the sale of x tires is R(x) = 105x – 0.02x dollars.
(a) Determine marginal cost (b) Determine marginal revenue
(c) Determine MR(50) and tell what it means (d)Determine the marginal profit.
(d)For what value of x is the marginal cost equal to the marginal revenue, and what is the
marginal profit in that instance?
Solution: (a) The marginal cost MC = C`(x0 = 70 – 0.02x dollars.
(b) The marginal revenue MR = R`(x) = 105 – 0.04x dollars.
(c) From (b) it follows that MR(50) = 105 – 0.05(50) = $103.
This means that once 50 tires have been sold, the revenue to be obtained from the sale of the next
st
tire (the 51 ) is approximately $103.
(d) The profit function is given by:
2 2 2
P(x) = R(x) – C(x) = (105x – 0.02x ) – (4000 + 70x – 0.01x ) = – 0.01x + 35x – 4000
Differentiating yields the marginal profit: MP(x) = P`(x) = -0.02x + 35 dollars.
(e) The marginal cost is 70 – 0.02x and marginal revenue is 105 – 0.04x. If they are equal then,
70 – 0.02x = 105 – 0.04x. This gives x = 1750
The marginal profit is then P`(x) = – 0.02(1750) + 35 = 0 giving x = 1750. This should not be
particularly surprising; because if marginal profit is zero then C`(x) = R`(x).
2
Example 04: If the revenue function for a product is R(x) = 60x /(2x+1) find the marginal
revenue.
Solution: The marginal revenue can be found in the following way:
d  60x 2   2x +1120x  - 60x  2  240x 2 +120x -120x 2 120x 2 -120x 120x  x -1
2
R x  =  = = = =
dx  2x +1   2x +12  2x +12  2x +12  2x +12
Example 05: If the total revenue function is given by R(x) = 60x and the total cost function is
2
given by C(x) = 200 + 10x + 0.1x , what is the marginal profit at x = 10?
2
Solution: We have R(x) = 60 x and C(x) 200 + 10x + 0.1x . The profit function will be

-:83:-
CHAPTER THREE DERIVATIVES
2 2
P(x) = R(x) – C(x) = 60 x – 200 + 10x + 0.1x = 60 x – 200 – 10x – 0.1x .
Now the marginal profit is: P(x) =
dP
dx
 
= 200  50x  0.1x 2 = 50x  0.2x

When x = 10, P`(x) = P`(10) = 50 – 0.2(10) = 50 – 2 = 48


Example 06: If the total revenue function for a commodity is R = 40x - 0.02x 2 , with x
representing the number of units.
(a) Find the marginal revenue function.
(b) At what level of production will marginal revenue be 0?
Solution: (a) The marginal revenue function will be:
R  x  =
d
dx
R x =
d
dx
 
40x - 0.02x 2 = 40 - 0.04x.
(b) The level of production at which the marginal revenue will be zero can be found as:
40 – 0.04x =0 this implies that x = 1000. Hence, 1000 number of units must be produced in order
to have zero marginal revenue. This means that there will be no increase in the revenue if 1000
units are produced, that is, revenue will remain constant.
2
Example 07: Suppose the cost of producing x units is given by C(x = 200 + 15x – 0.5 x ) dollars,
for 0 < x < 120.
(a) Determine the marginal cost when x = 7 units.
th
(b) Determine the exact cost of the 8 unit.
(c) What is meaning of MC(7)?
Solution: (a) The marginal cost is: MC(x) = C`(x) = 15 – x
When x=7, MC(7) = 15 – 7 = 8
Hence, when 7 units are produced, the marginal cost will be $ 8.
th
(b) The exact cost of the 8 unit is given by:
2 2
C(8) – C(7) = [200 + 15(8) – 0.5(8 )] – [200 + 15(7) – 0.5(7 )] = $ 7.5
th
(c) MC(7) means that once 7 units are produced, the cost of producing the next unit (the 8 ) is
approximately $8.
2
Example 08: Let the revenue function for a stereo system is R(x) = 300 x – x , where x denotes
the number of units sold,
(a) What is the marginal revenue if 50 units are sold?
(b) What is the marginal revenue if 100 units are sold?
(c) What is the marginal revenue if 150 units are sold?
(d) What is happening to revenue when 150 units are sold?
Solution: (a) MR(x) = R`(x) = 300 – 2x.
When x = 50, MR(50) = 300 – 2(50) = 300 – 100 = $ 200. Thus, the marginal revenue is $200 if
50 units are sold.
(b) When x =100, MR(100) = R`(100) = 300 – 2(100) = 300 – 200 = $100.
(c) When x = 150, MR(150) = R`(150) = 300 – 2(150) = 300 – 300 = $0.
(d) From part (c) we see that after the sale of 150 stereos the revenue will decrease.
Example 09: If the total revenue function for a toy is R(x) = 2x, and the total cost function is
2
C(x) = 100 + 0.2x + x dollars,
(a) What is the marginal profit function?
(b) What is the marginal profit if 10 units are produced and sold?

-:84:-
CHAPTER THREE DERIVATIVES
2
Solution: (a) The profit function is given by: P(x) = R(x) – C(x) = 2x – (100 + 0.2x + x)
2
= x – 0.2x – 100.
2
Thus, the profit function is, P(x) = x – 0.2x – 100
Now, the marginal profit is, MP = P`(x) = 1 – 0.4x.
(b) When x = 10, MP(10) = P`(10) = 1 – 0.4(10) = 1 – 4 = – 3
The negative sign indicates that rate of change of profit (MP) is decreasing at the rate of $3 when
10 items are sold.
Example 10: Find the marginal revenue function for the total revenue function R(x) = 16x –
2
0.02x and find the marginal revenue for this product at x = 40?
Solution: The marginal revenue function is MR(x) = M `(x) = 16 – 0.04x,
When x = 40, the marginal revenue function will be, MR(40) = 16 – 0.04(40) = 14.4 dollars.
st
This means that the sale of 41 item will increase the total revenue approximately by $14.40.
HIGHER DERIVATIVES
So far we have studied the first derivative of a function. Since the derivative of a function is itself
a function, so we can a find the derivative of the derivative of a function. The derivative of a first
derivative is called a second derivative. We can find the second derivative of a function f by
differentiating it twice.
Let y = f(x) be a differentiable function on an open interval (a, b). If we apply the definition of
derivative to f`(x) the resulting limit (if it exists) will be the second derivative of y = f(x) and is
f (x  h)  f (x)
denoted by y``(x) = f ``(x). Thus, y  f   x   Lim
h 0 h
Continuing in this way, we can evaluate the third, fourth and higher derivatives of f whenever
they exist. The successive derivatives of y = f(x) are denoted as follows:
dy d2 y
First : y = f  (x) = Dy = Second : y = f  (x) = D 2 y =
dx dx 2
d3 y iv d4y
Third : y = f  (x) = D3 y = Fourth : yiv = f 4
(x) = D y =
dx 3 dx 4
n 1 n
d y d y
 n -1th : y(n 1) = f (n 1) (x) = D n 1y = n 1
n th : y (n) = f (n) (x) = D n y =
dx dx n
Example 01: If y = arc tanx, show that (1  x 2 )y  2xy  0
-1
Solution: Given that y = tan x. Differentiating it twice, we get
1 (1  x 2 )(0)  (1)(2x) 2x
y  and y  =
2 2 2
1 x (1  x ) (1  x 2 )2
Substituting these values into the equation (1  x 2 )y  2xy  0 we get
2x 1 2x 2x
(1  x 2 ).  2x.  0 or  000
2 2 2 2
(1  x ) 1 x 1 x 1  x2
th
Standard n Derivatives
th m
1. Find the n derivative of y = x
m
Let y = x . Then its successive derivatives are
m–1 m–2 m–3
. y` = m x , y `` = m(m – 1) x , y``` = m(m – 1)(m – 2) x .

-:85:-
CHAPTER THREE DERIVATIVES
(n) m–n
Continuing this process, we get: y = m(m – 1)(m – 2) … [m – (n – 1)] x = m(m – 1)(m – 2)
m – n
… (m – n + 1) x Multiplying and dividing by (m – n) …. 3.2.1, and using the fact that
m(m – 1)(m – 2)….3.2.1 = m! and (m – n)…3.2.1 = (m – n)!, we obtain:
m(m  1)(m  2)...(m  n  1)(m  n)...3.2.1 mn m!
y(n)  x  x mn , where n  m (1)
(m  n)....3.2.1  x 2 (m  n)!
n! n! n!
If n = m, then: y(n)  x n n  .x 0  .1  n! , (constant)
(n  n)! 0! 1
If n > m, then all successive derivatives greater than m are all zero.
6 (5) (6) (7)
Example 02: Let y = x find y , y and y .
(5) 6–5
Solution: Using the formula (1), we have: y = [6!/(6 – 5)!] x = (6!/1) x = 720 x
(6) 6–6
y = [6!/(6 – 6)!] x = (6!/0!) x = (720/1) x = 720 x
(7) th 6
Finally, y = 0 because power of x is 6 so the 7 derivative of x is zero. (7 > 6).
th m
2. Find the n derivative of (ax + b)
m
Let y = (ax + b) . Then its successive derivatives are
m –1. m–1
. y` = m (ax + b) a, = ma (ax + b) ,y `` = m(m – 1)a2 xm – 2 , y```= m(m–1)(m–2) a3 xm – 3
(n) n m–n
Continuing this process, we get: y = m(m – 1)(m – 2) … [m – (n – 1)] a x
n m–n
= m(m – 1)(m – 2) … (m – n + 1) a x
Multiplying and dividing by (m – n) …. 3.2.1, and using m(m – 1)(m – 2)….3.2.1 = m! and
(m – n)…3.2.1 = (m – n)!, we obtain:
m(m  1)(m  2)...(m  n  1)(m  n)...3.2.1 n mn m!
y(n)  a x  a n x mn , where n  m. (2)
(m  n)....3.2.1  x 2 (m  n)!
n! n! n!
If n = m, then: y(n)  a n x n n  .a n x 0  a n .1  n! a n (a constant.)
(n  n)! 0! 1
If n > m, then all successive derivatives greater than m are all zero.
7 (5) (6) (8)
Example 03: Let y = (3x + 4) , then find y , y , y(7) and y .
(5) 7–5 5 2
Solution: y = [7!/(7 – 5)!] (3x + 4) × 3 = 612360(3x + 4)
(6) 7–6 6
y = [7!/(7 – 6)!] (3x + 4) × 3 = 3674160(3x + 4)
(7) 7–7 7
y = [7!/(7 – 7)!] (3x + 4) × 3 = 11022480
(7) (8)
Since y is constant hence, y = 0.
th
3. Find the n derivative of 1/(ax + b)
1
=  ax + b 
-1
Let y =
ax + b
Substituting m = – 1 , in (2), we get
(1)n n! a n
y(n)  (1)(2)(3)...(n)(ax  b) 1n a n  (1) n n! (ax  b) (n 1) a n  (3)
(ax  b)n 1
Example 04: If y = 1/(5x – 3) then find y 
4

(1)4 4! 54 15, 000


Solution: Using equation (3), we have: y(4)  
41
(5x  3) (5x  3)5
th
4. Find the n derivative of ln(ax + b)
Let y = ln(ax + b) Differentiating n times, we get
-:86:-
CHAPTER THREE DERIVATIVES
a
y   a(ax  b)1, y  a(1)(ax  b) 2 .a  (1)a 2 (ax  b) 2
ax  b
(1)2 2! a 3 (1)n 1 n! a n
y  (1)(2)a 2 .a(ax  b) 3  . Continuing this way, we obtain : y(n)  (4)
(ax  b)3 (ax  b) n
Example 05: Find the nth derivative of y = 1/ (a 2  x 2 )
1 1
Solution: We have y= = (i)
2
a x 2 (a  x)(a  x)
Decomposing it into partial fractions, we get
1 A B
=  (ii)
(a  x)(a  x) a  x a  x
Multiplying it throughout by (a – x)(a + x), we get
1 = A(a + x) + B(a + x) (iii)
Put, a – x = 0 or x = a in (3), we get: 1 = A(a – a) + B(a + a) = 2aB giving B = 1/2a
Now putting a + x = 0 or x = – a in (3), we get: 1 = A(a + a) + B(a – a) = 2aA giving A = 1/2a
1 1 1 1  1 1 
Thus, (2) becomes: =  or y =   
(a  x)(a  x) 2a(a  x) 2a(a  x) 2a  x  a x  a 
Now, differentiating both sides n-times and using formula (3) as shown above, we get
1 1 1 1  1 1 
=  or y =   
(a  x)(a  x) 2a(a  x) 2a(a  x) 2a  x  a x  a 

dn y 1  d n  1  d n  1   1  ( 1) n n! ( 1) n n! 
=       =   
dx n 2a  dx n  x + a  dx n  x  a   2a   a + x n+1 (x  a) n 1 
 

n!(1)n 1 1 
   
n 1
 (x  a)
2a (x  a) n 1n+1 
(3)
Example 06: Let y = ln(2x + 3) then find y
2
Solution: y  ln(2x  3)  y   2(2x  3)1, y  2(1)(2x  3) 2 .2  (1)22 (2x  3) 2
2x  3
( 1) 2 2! 23 16
y  ( 1)(2)22 (2x  3) 3 .2  
3
(2x  3) (2x  3)3
( 1) 2 2! 23 16
Method  II :U sin g the formula in (4) taking n  3, we get : y(3)  
3
(2x  3) (2x  3)3
th ax
5. Find the n derivative of e
ax
Let y = e . Differentiating successively w.r.t x, we get
y = a eax , y = a 2eax , y = a 3eax ,... , y (n) = a n eax (5)
2x (7)
Example 07: If y = e then find y
(7) 7 2x 2x
Solution: Using formula (5), we have y = 2 .e = 128e .
6. To find the n th derivative of sin(ax + b)
Let y = sin(ax + b). Differentiating successively, we get
 π   π 
y = cos  ax + b  × a = asin  ax + b +  since cos θ = sin  θ +  
 2   2 

-:87:-
CHAPTER THREE DERIVATIVES
 π  π π  π
y = a × cos  ax + b +  × a = a 2sin  ax + b + +  = a 2sin  ax + b + 2 
 2  2 2  2
 π  π π  π
y = a 2 × cos  ax + b + 2  × a = a 3sin  ax + b + 2 +  = a 3sin  ax + b + 3 
 2  2 2  2
..........................................
dn y  π
y(n) = = a n sin  ax + b + n  .
dx n  2
dn y  π
Similarly, if y  cos(ax  b) then y  =
n
= a n cos  ax + b + n 
dx n  2
th ax
7. Find the n derivative of e sin(bx + c)
ax
Let y = e sin(bx + c). Differentiating w.r.t x, we get
y = eax cos(bx + c)× b + sin(bx + c)×eax ×a or y = eax a sin(bx + c) + bcos(bx + c) (1)
2 2 2 2 2 2
Putting, a = r cos θ and b = r sin θ so that a + b = r (cos θ + sin θ) = r  r  a 2  b2
-1
Also (b/a) = (r cos θ/r sin θ) = tan θ giving θ = tan (b/a).
Substituting theses in (1), we get
y  eax  r cos  sin  bx  c   r sin  cos  bx  c  = r eax sin  bx + c  cos θ + cos  bx + c  sin θ 

Or y = r eax sin  bx + c + θ  since sin  α + β  = sin α cosβ + cos α sin β 

Thus, y has been obtained from y by multiplying it by r = a 2 + b2 and increasing the angle by
θ = tan (b/a). Differentiation y w.r.t x successively and using the above process, we get
-1

 b
  
y = r 2eax sin  bx + c + tan 1  = a 2 + b 2 e ax sin  bx + c + 2tan 1 
 a 
b
a
 b
   b
3 2 ax
y = r 3eax sin  bx + c + tan 1  = a 2 + b 2 e sin  bx + c + 3tan 1 
 a  a
...............................
 b

y  = r n eax sin  bx + c + tan 1  = a 2 + b 2

 b
n 2 ax
e sin  bx + c + n tan 1 
n
 a  a



 b
n 2 ax
Similarly, if y  eax cos(bx  c) then y (n)  a 2 + b 2 e cos  bx + c + n tan 1 
 a
Leibniz’s Theorem
th
Leibniz’s theorem helps us to find the n derivative of the product of two functions of x.
th
Statement: If y = uv, where u and v are functions of x, having derivatives of n order, then
y n = n C0 u n v + n C1u n-1v1 + n C2 u n-2 v 2 + n C3u n-3v3 + ... + n Cr u n-r v r + ... + n C n-1u1v n-1 + n C n uv n
where suffices of u and v denote the number of times they are differentiated and n Cr denotes the

number of combinations of n with r where n Cr is given by: n Cr  n!/ [r!(n  r)!]


Using this formula we have following results:
nC = n! n! n! n  n-1! n! n  n-1 n-2 ! n  n-1
0 = =1, n C1 = = =n, n C2 = = = ,
0! n-0 ! n! 1! n-1!  n-1! 2! n-2 ! 2 n-2 ! 2
n(n 1)(n  2)(n 3)! n(n 1)(n 2) n C = n! = n!=1
nC   , .... n
3 3!(n 3)! 6 n! n-n ! n!

-:88:-
CHAPTER THREE DERIVATIVES
Proof: By “Mathematical Induction”, put n = 1, we get
y1 = u1 v + u v1
Put n = 2, we have y2 = u2 v + 2 u1 v1 + u v2
Thus theorem is true for n = 1, 2. Suppose now that the theorem is true for n = k. Then
yk = k C0u k v + k C1u k-1v1 + k C2u k-2 v2 + k C3u k-3v3 +...+ k Ck-1u1vk-1 + k Ck uvk
Differentiating both sides of the above equation, we obtain
y k+1 = k C0 (u k+1v + u k v1 ) + k C1 (u k v1 + u k-1v 2 ) + ... k C k-1 (u 2 v k-1 + u1v k ) + k C k (u1v k + uv k+1)
= k C0 u k+1v + [ k C0 + k C1 ]u k v1 + [ k C1 + k C2 ]u k-1v 2 + ... + [ k Ck-1 + k Ck ]u1v k + k Ck uv k+1
But we know that for all n  N, n Cr  n Cr 1  n 1Cr 1 . Using this result, we get
y k+1 = k+1C0 u k+1v + k+1C1u k v1 + k+1C2 u k-1v 2 + ... + k+1Ck u1v k + k+1Ck+1uv k+1

Replacing k + 1 by n, we get
y n = n C0 u n v + n C1u n-1v1 + n C2 u n-2 v 2 + ... + n C r u n-r v r + ... + n C n-1u1v n-1 + n C n uv n
Hence the result is true for all positive integer n and theorem is proved.
NOTE: It may be noted that the term n Cr u n-r v r is a general term of the binomial expansion of
n
(u + v) .
th 3 ax
Example 01: Find the n derivative of the function y = x e using Leibniz’s Theorem.
Solution: Let u = eax , u n = a n eax , u n-1 = a n-1eax , u n-2 = a n-2eax , u n-3 = a n-3eax
Again let v = x 3 , v1 = 3x 2 , v 2 = 6x, v3 = 6, v 4 = 0
Using Leibniz’s theorem, we have: y n = n C0 u n v + n C1u n-1v1 + n C2 u n-2 v2 + n C3u n-3 v3
Substituting all the values, we get

yn = (1)(a n eax )(x 3 ) + (n)(a n 1eax )(3x 2 ) +


n(n  1) n 2 ax
2

(a e )(6x) a n-2eax  6x  
n(n  1)(n  2) n 3 ax
 (a e )(6)  yn = eax a n x 3 + 3na n-1x 2 + 3n  n -1 a n-2 x + n  n -1 n - 2  a n-3 
6  
Example 02: If y = a cos lnx + bsin lnx  , then show that x 2 yn+2 + (2n +1) xy n+1 + (n 2  1) y n = 0
Solution: Given that y = a cos  lnx  + bsin  lnx  then differentiating it, we get
1  1  bcos(lnx) a sin(lnx)
y1 =  a sin  lnx    + bcos  lnx    =   xy1 = bcos(lnx)  a sin(lnx)
x x x x
Again differentiating, we get
1 1 1
xy2  y1(1)  bsin(ln x)    a cos(ln x)     a cos(ln x)  bsin(ln x)
x x x
x 2 y 2 + xy1 + y = 0 (1)
Differentiating (1) n times using Leibniz’s theorem, we get
 n C0 yn+2 x 2 + n C1yn+1 (2x) + n C0 y n+2 (2)  +  n C0 y n+1x + n C1y n (1)  + y n = 0
   
x 2 y n  2  2nxy n 1  n(n  1)y n  xy n 1  ny n  y n  0

-:89:-
CHAPTER THREE DERIVATIVES
x 2 y n+2 + (2n +1) xy n+1 + n 2 y n  ny n + ny n + y n = 0  x 2 y n+2 + (2n +1) xy n+1 + (n 2  1)y n = 0
k
Example 03: Use Leibniz’s theorem to show that if y =  x + 1+ x 2  , then show that
 
(1  x 2 )y n  2  (2n  1)x y n 1  (n 2  k 2 ) y n  0
k
Solution: Given that y =  x + 1+ x 2  , differentiating it, we get
 
k 1  1  k 1  
 

y1 = k  x + 1+ x 2  1+ 1 1+ x 2  2x   = k  x + 1+ x 2  1+
x
2 
   2     
  1+ x 2 
k-1  2  k
= k  x + 1+ x 2   x + 1+ x  or 1+ x 2 y1 = k  x + 1+ x 2   ky
   1+ x 2   
 


Squaring both sides, we get 1+ x 2  y1  = k 2 y 2  2

Again differentiating, we get


(1  x 2 )(2y1 )y 2 + (y12 )(2x)  k 2 (2y)(y1 ) or (1  x 2 )y 2 + xy1 = k 2 y (1)
Differentiating equation (1) n times using Leibniz’s theorem, we get
 n C0 y n+2 (1  x 2 ) + n C1y n+1 (2x) + n C 2 y n (2)  +  n C 0 y n+1(x) + n C1y n (1)  = k 2 y n
   
(1  x 2 )y n+2 + 2n x y n+1 + n(n  1)  n  1 y n + xy n+1 + ny n  k 2 y n = 0
(1  x 2 )y n+2 + (2n +1)xy n+1 + n 2 y n  ny n + ny n  k 2 y n = 0
(1  x 2 )y n+2 + (2n +1)x y n+1 + (n 2  k 2 )y n = 0

WORKSHEET 03
1. Differentiate the following functions:
2
i  f  x  
1 2 1
2
1  1
x  x  , ii  f  x    x   , iii  f  x   x 2  3 2x 2  1
7 4  x
  
iv  f  x    
x  2x  1 x 2  x  1 ,  v  f  x  
x 1
x 1

2. If f(x) = 6x2 – 5x + 3, find f `(0). For what value of x is f `(x) = 0


3. Find y` where:
1 x 
(a) y = xsin x (b) y  ln 
1 x 

 1  2x 
(c) y  tan 1 
 2x 
 
(d) y  ln sin 1 ex 
1  cos x
1  cosh x
   y

4 
1 2 1 2
(e) y  sin x (f) y  cos x (g) 1  cos x (h) y 
1  cosh x
 x 1 
(i) y  log10 
 x 
 (j) y  cos 1 1  x 2 
(k) y  sec1  sinh x  (l) y  cosh 1 1  x 2 
 
3
(m) x  y  2  0 (n) xy2  2xy  x  1 (o) x 3  y3  3axy  0 (p) y  x 2  y2

-:90:-
CHAPTER THREE DERIVATIVES
1/2
y  1  cos x 
(q) tan 1    yx  1
2
(r) y  tan 1  
x  1  cos x 
(t) NOTE: See the change in the problem
 1 x 

(u) y  sin 1  ln x   ln tan 1 x 
(v) y  sin 2  cot 1
   H int : Put x  cos 
1  x 

(w) x = a(t – sin t) and y = a(1 – cos t) (x) x = 3 cos t – cos 3t and y = 3 sin t – sin 3t
2 2
(y) x = a[cos t + ln tan (t/2)] and y = a sin t (z) x  tan 1 and y  sin 1
1 2
1  2
4. Differentiate logarithmically:
x 1  2x 
2/3
(a) y  (b) y  x x ex sin  ln x 
 2  2x   3  4x 
3/4 4/3

1
sec1   x  2 2
(c) y  e x (d) y 
 x  1  x 2  3
2

(e) y = xx (f) y = (sin x)x


(g) y = xsin 2x (h) y = (sin x)cos x
cos x sin x
(i) y = (sin x) + (cos x) (j) y = xsinx + (sinx)x
5. If sin y = x sin(a - y) prove that y` = sin2(a - y)/sin a [The question may be corrected]
6. If x2 + xy + 3y2 = 1, prove that (x + 6y)3 y`` + 22 = 0.
7. Differentiate with respect to x each of the following:
 i  f  x   eax cos  b arctan x   ii  f  x   ln  arcsin e x  iii  f  x   ln  tanh 2x 
 iv  f  x   arccos  1  x   v f  x   cosh
2 1
1  x 
2

dy
8. Find in each of the following:
dx
 i  x3  y3  3axy  0  ii  y  arcsin  ln x   ln  arctan x 
 iii  arctan  x  y   arcsin  e y  x   iv  yarcsin x  x arctan y  1
9. Find dy/dx in each of the following:
1
i  y   tan x cot x   cot x tan x ii  y   x  x iii  y  x ln x iv  y  x sin y
10. If y = eax sin bx, prove that y`` – 2a y` + (a2 + b2) y = 0
11. The driver of an experimental racing car begins a test run. During the first 6 seconds, the
distance s (in feet) of the car from the starting point is
s  14t 2  t 3 / 3 0  t  6
where t is the number of seconds the car has been moving. What is the velocity of the car after 4
seconds have passed?
12. At time t, the position of a body moving along the s – axis is s = t3 – 6t2 + 9t m.
(a) Find the body’s acceleration each time the velocity is zero. (b) Find the body’s speed each
time the acceleration is zero. (c) Find the total distance traveled by the body from t = 0 to t = 2.
13. A bicycle manufacturer estimates that it can price its bicycles at p = 140 – 0.02x2 dollars
each, where x is the number sold. The cost of producing x bicycles is 900 – 0.01x2 dollars.
Determine the marginal profit when 20 bicycles are sold.

-:91:-
CHAPTER THREE DERIVATIVES
14. The pollution in a lake is being reduced over a 5–year period. The amount of pollutants
90t
(in pounds) is given by: A  110  , 0  t  5 , where t is the time in years.
t 1
(a) Determine the amount of pollution in the lake at the beginning, after 2 years, and after 5 years.
(b) Determine the rate at which the pollution in the lake is changing at t = 2 years.
(c) What is the meaning of the given sign obtained in part (b)?
15. On Earth, in the absence of air, a rock at a velocity of 24m/sec (about 86km/h) would reach a
height of s = 24t – 4.9t2 meters in t seconds.
(a) Find the rock’s velocity and acceleration at time t.
(b) How long would it take the rock to reach its highest point?
(c) How high would the rock go?
(d) How long would it take the rock to reach half its maximum height?
(e) How long would the rock be aloft (in the air)?
16. A 45–caliber bullet fired straight up from the surface of the moon would reach a height of
2
s = 832t – 2.6t feet after t seconds. On Earth, in the absence of air, its height would be
2
s = 832t – 16t feet after t seconds. How long will the bullet be aloft in each case? How high
would the bullet go?
17. Suppose an oil spill has taken the form of a circular region and its area is increasing at the rate
of 100 square meters per hour. At what rate is the radius of the region increasing when the radius
is 200 meters?
18. Consider a spherical balloon with volume V  4r3 / 3 that is being inflated by helium at the
rate of 4 cubic feet per minute. At what rate is the radius increasing when the radius is 2 feet?
19. A rocket is launched straight up. There is an observation station 7 miles from the launch site.
At what rate is the distance between the rocket and the station increasing when the rocket is 24
miles high and traveling at 200 miles per hour?
20. A helicopter rises vertically at the rate of 10 feet per second. There is a maker M 80 feet from
where the helicopter lifts off. At what rate is the distance between the helicopter and the marker
changing when the helicopter is 192 feet high?
21. A ball is thrown straight up from the ground and travels such that its distance from the ground
at any time t is s  16t 2  80t feet. Find its acceleration at any time.
22. Find the nth derivative of following:
x x
(a)  a  y  2  b  y  2 2  c  y  cos2 x  d  y  sin 2x cos 3x
2x  3x  1 x a
(e) y = ex cos2 x (f) y = ln x2 (g) y = ex sin 4x cos 6x
3 n–1
(h) y = x ln x (i) y = x ln x (j) y = ex ln x
(k) y = e4x sin(2x + 3) (l) y = ln (2x + 3)
23. If y = sin(a sin-1x) prove that (1 – x2)yn+2 = (2n + 1) x yn+1 + (n2 – a2) yn.
  n 2
 n 1

24. If y  e m arcsin x , show that 1  x 2 y   2n  1 xy   n 2  m2 y   0
n

d n  ln x   1 n! 
n
1 1 1
25. Show that: n   n 1  ln x  1      
dx  x  x  2 3 n
th 4 2x
26. Use Leibniz’s theorem to find the n derivative of y = x e


27. If y = cos(m ln x), show that x 2 yn  2   2n  1 xyn 1  m2  n 2 yn  0 
 
28. If y  sin 1 x / 1  x 2 then show that 1  x 2 yn  2   2n  3 xyn 1   n  1 yn  0
2

  
29. If y = sinh(m sinh-1x), prove that 1  x 2 yn  2   2n  1 xyn 1  n 2  m2 yn  0 
  
30. If y1/m + y-1/m = 2x, prove that x 2  1 yn 2   2n  1 xyn 1  n 2  m2 yn  0 
-:92:-
CHAPTER FOUR PARTIAL DERIVATIVES

CHAPTER
FOUR PARTIAL
DERIVATIVES
INTRODUCTION
In the previous sections we dealt with the calculus of functions of one variable and saw what
powerful tools differentiation could be in assisting with analysis and design in engineering,
natural and social sciences and marginal analysis situations. In many engineering and other
applications, however, the system or phenomenon we wish to model mathematically depends on
more than one variable. For example, the area A of a rectangular metal plate of with x and
breadth y is given by: A=xy
Since the variables x and y are independent of one another, we say that the dependent variable
A (area) is a function of two independent variables x (width) and y (breadth). We express this by
writing A = f(x, y) or simply A(x, y). As another example, the life L of an aircraft wing may be
2
modeled by: L=kAv ρ
where k is a constant, A is the area of the wing, v is the aircraft speed and ρ is the air density.
This is an example of a function of three independent variables: A, v and ρ. In this case we write
L = f(A, v, ρ) or simply L(A, v, ρ).
More generally, a variable z may be a function of n independent variables x1, x2, x3,.. xn, which
we express as: z = f(x1, x2, x3,.. xn) (1)
As for functions f(x) of one variable, discussed in the previous chapters, the function of
n-variables given in (1) has an associated domain in n-dimensional space, a range and a rule that
assigns each n-tuple of real numbers (x1, x2, x3,.. xn) in the n-dimensional domain with a real
number z in the range. Again the rule is frequently expressed in terms of mathematical formulae.
Furthermore, a domain may be a restricted region in n-dimensional space; for example, if:
τ = τ(x, y, z)
represents the temperature of a heated rigid body at a point in the body having coordinates
(x, y, z) then the functional relationship has no meaning at points outside the body, so the domain
is the set of all points (x, y, z) within the body.
In this section we are concerned primarily with extending the concept of differentiation to
functions of more than one variable. As results can be adequately illustrated using functions of
two or three independent variables, we shall restrict our attention to these.
Geometrical Representation
A function z = f(x, y) (2)
of two variables may be viewed geometrically in one of two ways. One is to draw the level
curves, which are curves in the two-dimensional (x, y) plane on which the function takes constant
values; that is, the level curves are determined by
f(x, y) = c
-:93:-
CHAPTER FOUR PARTIAL DERIVATIVES
2 2
For example, the level curves for the function z = x + y
are concentric circles of radius √c, where c is the value of z on the level curve (see fig. 1).
Alternatively, in the particular case of a function of two variables, the function (2) may be viewed
as a surface in three dimensional space, the surface being obtained by plotting the points
corresponding to (x, y, z), with z = f(x, y), using the rectangular Cartesian axes. Such a surface
2 2
may be built up from the level curves as illustrated in figure 2 for the function z = x + y .

Fig. 1 Fig. 2
2 2 2 2
In the particular example z = x + y it is relatively easy to draw the surface z = x + y .
In general, however, plotting such surfaces is not easy. There is now widely available computer
software that helps to plot the graphs in 3-dimensions.
Analogously to level curves for functions of two variables, we have level surfaces for functions of
three variables w = f(x, y, z)
These are the surfaces on which the function takes constant values, and so are determined by
f(x, y, z) = constant
2 2 2
Figure 3 shows the level surface for w = x + y + z .
Partial Derivatives
2
Consider a function of two variables z = f(x, y) with Df  R .
If x is changed to x + ∆x and y remains constant, then the change Fig. 3
∆z in z is given by: ∆z = f(x +∆x, y) – f(x, y)
Dividing both sides by ∆x and taking the limit ∆x tends to zero, we have
Δz f  x + Δx, y   f  x, y 
Lim = Lim
Δx 0 Δx Δx 0 Δx
If the limit on right side exists, it is called the partial derivative of z w.r.t x and is usually denoted
z z
by or f x . The symbol " " is called “dawa”. Hence is read as “dawa z over dawa x”. It is
x x
also read as “partial z over partial x”.
Similarly, the partial derivative of z = f(x, y) w.r.t y is
Δz f  x, y + Δy   f  x, y 
Lim = Lim
Δy0 Δy Δy0 Δy
provided the limit on the right side exists.
The calculation of partial derivatives of a given function z = f(x, y) is quit simple. To obtain f x,
we find the derivative of f w.r.t x holding y constant. Thus, if
2 2 z z
z = x + y then  2x  0  2x and  0  2y  2y
x y
2 2 z z
and if z = x y then  2xy2 and  2x 2 y
x y

-:94:-
CHAPTER FOUR PARTIAL DERIVATIVES
Geometrical Meaning of Partial Derivative
3
Suppose z = f(x, y) is a function of two variables. We know that z represents a surface in R . Let
y hold a constant value say “c”. Then we are considering those points of the surface z = f(x, y) for
which y = c. Geometrically, this means we are considering those points of the surface where the
surface z = f(x, y) and the plane y = c intersect each other which in fact is a curve
(see the figure 4). Surface z = f(x, y) P (x, c, f(x, c))
On this curve z changes with x while y remained constant.
Therefore, ∂z/∂x represents a slope of the tangent to this curve z = f(x, c)
z = f(x, c) at the point P. This is shown in figure 4.
Similarly, ∂z/∂y is the slope of the curve z = f (d, y) at the point
(d, y, f(d, y)). Fig. 4
Partial Derivatives of Higher Orders
The partial derivatives fx and fy may possess derivatives where f is a function of two variables x
and y. In such cases, we may define the second order partial derivatives as follows:
  z   2 z  2f   z   2 z  2f
   2  2  f xx ,    2  2  f yy
x  x  x x y  y  y y
  z   2 z  2f   z   2 z  2f
    f yx and     f xy
y  x  yx yx x  y  xy xy
The partial derivatives fxy and fyx are called mixed derivatives. In general they are not equal,
 2f  2f
however, if they both exist, then:  or f xy  f yx
yx xy
2 2 2 2 2 2
For example, let z = f(x, y)= x y , then fx = 2xy , fy = 2x y, fxx = 2y , fyy = 2x , fxy = 4xy,
fyx = 4xy. You may observe that:
. fxy = fyx
-1
Example 01: Show that fxy = fyx, if f(x, y) = sin (x/y)
Solution: Differentiating f partially w.r.t x, we get
f 1  x y 1 1
     (i)
x 1  (x / y)2 x  y  y2  x 2 y y2  x 2
Differentiating (i) partially w.r.t y now, we get
 2f  1  y
  (y 2  x 2 ) 1/2  1 / 2(y 2  x 2 ) 3/2 (2y  0)  (ii)
yx y y 2  x 2 y (y 2  x 2 )3/2
Now differentiating f partially w.r.t y, we get
f 1  x y x x
      (iii)
y 1  (x / y)2 y  y  y2  x 2 y
2
y y2  x 2
Differentiating (iii) w.r.t x, we obtain
 2f  x 1 x x  y
    2   (iv)
xy x y2  x 2 y y2  x 2 y  (y  x 2 )3/2  (y2  x 2 )3/2
From (ii) and (iv) we see that: fxy = fyx
x y
Example 02: If f(x, y) = e sin y + e cos x, show that f satisfies Laplace equation
fxx + fyy = 0.
x y
Solution: Differentiating f w.r.t x, we get: fx = e sin y – e sin x

-:95:-
CHAPTER FOUR PARTIAL DERIVATIVES
x y
Differentiating again w.r.t x, we have: fxx = e sin y – e cos x (i)
x y
Now differentiating f w.r.t y, we get: fy = e cos y + e cos x
x y
Differentiating again w.r.t y, we obtain fyy = – e sin y + e cos x (ii)
x y x y
Adding (i) and (ii) we get: fxx + fyy = e sin y – e cos x – e sin y + e cos x = 0
y x  2f x 2  y2
Example 03: If f (x, y)  x 2 tan 1    y 2 tan 1   ,show that 
x y xy x 2  y2
Solution: Differentiating f w.r.t y, we get
 
 
f 2 1  y  2 1  x 1  x  
x  y    tan   .2y 
y   y   y  x    x  y  y 
2 2
y 
1       1   
  x     y 
 
x2 1  2 y2  x  1  x   x3 xy 2 x
x 2
 y    tan   .2y     2y tan 1  
2  2
 x 2  y2  x 
     y2  x 
2 2
2
  y  y  2
 y
  
y x

y x

x(x 2  y 2 ) x x


  2y tan 1    x  2y tan 1  
 y2  x 
2
y  y

Now differentiating w.r.t x now, we get


 2f 1  x 2y3 1 y 2  x 2  2y 2 x 2  y 2
 1  2y      
 
1 .
xy  x  x  y 
2
y2  x 2 y y2  x 2 x 2  y2
1  y 
 
x 2  y2
Example 04: If f (x, y)  ,prove that (f x  f y )2  4(1  f x  f y )
xy
Solution: Differentiating f w.r.t x, we get,
(x  y).2x  (x 2  y 2 ).1 2x 2  2xy  x 2  y 2 x 2  2xy  y 2
fx    . Similarly, by symmetry
(x  y) 2 (x  y) 2 (x  y) 2
2
y 2  2xy  x 2  (x 2  2xy  y 2 )  (y 2  2xy  x 2 
2
fy  . Adding both results, we get : (f x  f y )   
(x  y) 2  (x  y) 2 
2 2 2
 2(x 2  y 2 )   ( x  y)(x  y)   (x  y) 
   4   4  (1)
 (x  y) 
2
 (x  y)
2
  (x  y) 
. Now,
 x 2  2xy  y 2 y 2  2xy  x 2 
4(1  f x  f y )  4 1   
 (x  y) 2 (x  y) 2 
2
 x 2  2xy  y 2  x 2  2xy  y 2  y 2  2xy  x 2  x 2  2xy  y 2  (x  y) 
 4 4  4  (2)
 (x  y) 2  (x  y) 2  (x  y) 
From (1) and (2) the required result follows.
Example 05: Suppose that the revenue from the sale of x Model-A stereo speakers and y Model-
2 2
B stereo speakers is given by R(x, y) = 100 x – 0.3x – 0.02y dollars. Determine the rate at

-:96:-
CHAPTER FOUR PARTIAL DERIVATIVES
which revenue will change with respect to the change in the number of model-A speakers sold,
when 50 Model-A speakers and 40 Model-B speakers have been sold?
R
Solution: The rate of change we seek is  50,40 
x
2 2
Since, R(x, y) = 100 x – 0.3x – 0.02y (1)
Differentiating (1) partially with respect to x, we get
R
 100  0.06x
x
R
At (50, 40):  50,40  100  0.06 50  97
x
This means that the additional revenue that will be obtained from the sale of the next Model-A
speakers is approximately $97, assuming that 50 Model-A speakers and 40 Model-B speakers
have been sold.
Differentiability
Let z = f(x, y) be a function of two variables. The differential of z is defines as:
z z
dz  dx  dy
x y
In case if z = f(x, y) is constant then dz = 0. In this case
z z dy f
dz  dx  dy  0 or f x dx  f y dy  0 or  x (1)
x y dx fy
Formula (1) is used to find the ordinary derivative if a function is given in implicit form.
3 2 3
Example 01: Find dy/dx if x + x y + y = 0
3 2 3 2 2 2
Solution: Here f(x, y) = x + x y + y OR fx = 3x + 2xy and fy = x + 3y
dy f 3x 2  2xy
This implies that  x 
dx fy x 2  3y 2

WORKSHEET 04
1. Evaluate the following limits:
3xy xy 3xy xy  2x
(i) lim 2 (ii) lim (iii) lim (iv) lim
x 0 x  y 2 x 0 x  5y x 1 x y
2 2 x 1 xy  2y
y 2 y0 y y1
2x  y  
2. If f  x, y   show that lim  lim f  x, y   lim  lim f  x, y 
2x  y x 0  y0  y0  x 0 
3. Show that the function f(x, y) defined as

2x  y,
2
 x, y   1,2 
f  x, y    is not continuous at (x, y) = (1, 2).

0 ,  x, y   1,2 
 y  2xy 
4. Show that the function (a) f (x, y)  tan 1   and (b) f (x, y)  tan 1  2 2 
 satisfies the
x x y 
 2f  2f
Laplace equation  0.
x 2 y2

-:97:-
CHAPTER FOUR PARTIAL DERIVATIVES
 2f  2f
5. Show that  where:
xy yx
(a) f(x, y) = ln (ex + ey) (b) f(x, y) = ln (x2 + y2) – ln(xy)
(c) f(x, y) = x sin xy + y cos xy
x y 2
6. If Z = ln(e + e ), show that Zxx . Zyy = (Zxy)
7. If u = rm and r2 = x2 + y2 + z2, show that uxx + uyy + uzz = m (m + 1) rm – 2
8. If ux + vy = 0 and u/x + v/y = 1, show that ux – vy = [(x + y )/(y – x )]
2 2 2 2

3u 3u
9. If u = xy, prove that 
x 2y yxx
10. Find dy/dx in each of the following case. Verify the results by using the formula
dy/dx = –fx/fy.
2 2 4 3 2 2
(a) y + x y + ax = 0 (b) x + x + xy + sin y = 0

-:98:-
CHAPTER FIVE MEAN VALUE THEOREMS AND INDETERMINATE FORMS

CHAPTER
FIVE MEAN VALUE THEOREMS
AND
INDETERMINATE FORMS
In this chapter we shall discuss some important theorems of fundamental importance in calculus.
These theorems help to study the behavior of various functions.
ROLLE’S THEOREM
Statement: Let a function f(x) be such that
(i) It is continuous in the closed interval [a, b]
(ii) It is derivable in the open interval (a, b)
(iii) The value of the function f(x) at x = a and x = b is same, that is, f(a) = f(b)
Then there exists at least one value ‘c’ inside the open interval (a, b) such that f `(c) = 0.
Proof: Since given function f(x) is continuous on [a, b] hence it is bounded therein and attains its
bound. Let M and m are the upper and lower bounds of f on [a, b]. There occur two cases:
Case I: When M = m, that is, the upper and lower bounds are equal. In this case the function f(x)
is constant (see figure 1) and so its derivative at every point in (a, b) is zero, that is, f `(x) = 0 for
f(x) = k → f `(x) = 0
x  (a, b). k

Hence the theorem is true in this case.


If M ≠ m, then at least one of them will be different
from f(a) and f(b). This is shown in the figure 2.
. a fig. 1 b
M

fig. 2
m
f(a) f(b) f(a ) f(b)

a c b a c b
f(a) = f(b) ≠ M f(a) = f(b) ≠ m
Suppose M ≠ f(a) = f(b) (1)
Now let the point where the upper bound M occurs be c that is, f(c) = M. Then from (1) `c` must
be different from a and b. This implies that `c` lies inside the interval [a, b], that is c  (a, b). Take
h as a positive real number such that c – h and c + h both lie in the interval (a, b). Then,
f(c – h) ≤ f(c) and f(c + h) ≤ f(c)
f (c  h)  f (c)
Or, 0 (2)
h
f (c  h)  f (c) f (c  (h))  f (c)
and  0
h h
Multiplying both sides by – 1, we get

-:99:-
CHAPTER FIVE MEAN VALUE THEOREMS AND INDETERMINATE FORMS
f (c  (h))  f (c)
0 (3)
h
Taking limit h tends to 0, we obtain respectively from (2) and (3)
f ` (c ) ≤ 0 and f `(c) ≥ 0
This implies that f ` (c) = 0. This proves the theorem.
Geometrical Interpretation of Rolle’s Theorem
Since f(a) = f(b) the ends of the graph of f(x) are at the same horizontal level. Since f(x) is
continuous, the graph of f(x) is either a horizontal line or a smooth curve joining the points
 a,f (a)  and  b,f (b)  

O O
a b a b
In the first case the slope of a function is zero for all x. In the latter case, the graph must have
turning points hence the tangent at these points must be horizontal.
Remarks: Under the conditions stated, Roll’s theorem guarantees the existence of at least one
point `c` in (a, b) such that f `(c) = 0. There may be more than one such point the theorem states
nothing about it.
Geometrical Interpretation (Another Method)
There is strong geometric evidence that between any two points where a differentiable curve
crosses the x– axis, there is a point on the curve where the tangent is horizontal. A 300–year old
theorem of Michel Rolle (1652 – 1719) assures that this is indeed the case. This is depicted in the
following graphs.

f `(c) = 0
. f `(c1) = 0 f `(c3) = 0
Fig (i) Fig (ii)

f   c2   0

0 a c b 0 a c1 c2 c3 b
Roll’s theorem states that a differentiable curve has at least one horizontal tangent between two
points a and b if f(a) = f(b). There may be just one point (fig. (i)), or it may have more than one
(fig. (ii)).
Physical Meaning of Rolle’s Theorem
Let a stone be thrown from the ground into the air. Suppose the height of the stone after time t be
s = f(t). Surely the stone will hit the ground after some time T. Thus clearly f(0) = f(T) = 0. The
function s = f(t) satisfies the conditions of Roll’s theorem on the interval [0, T] due to continuous
motion. Hence at certain time t o   0,T  , the velocity of the stone is zero, that is; f `(to) = 0. We

know that indeed it happens.

-:100:-
CHAPTER FIVE MEAN VALUE THEOREMS AND INDETERMINATE FORMS
Important Points to Remember:
(i) Rolle’s Theorem fails to hold for a function that does not satisfy even one of the three
conditions given above.
(ii) Every polynomial is a continuous function P(x) for every real value of x.
(iii) sin x,cos x,ex are continuous functions for all values of x.
(iv) log x is a continuous function for all x  0 
(v) If f and g are both continuous on the closed interval [a, b] then f  g and f.g are also
continuous on [a, b]. The function f/g is also continuous on [a, b], provided g(x)  0
for any x  a,b 
(vi) If f is derivable for every point in a given interval, then it must be continuous in this
interval, that is; Derivability  Continuity 
2
Example 01: Verify Rolle’s Theorem for the function f(x) = x – 6x + 8 in [2, 4].
Solution: Here a = 2 and b = 4.
Since f(x) is a polynomial, therefore, it is continuous on [2, 4]
f   x   2x  6 exists in the open interval (2, 4)
f (2)  4  12  8  0,f (4)  16  24  8  0  f (2)  f (4)  0
2
Hence, f(x) = x – 6x + 8 satisfies all three conditions of Rolle’s Theorem. So there must exist at
least one number `c` between 2 and 4 such that f   c   0  Now,
f (x)  2x  6  f (c)  2c  6  0  c  3  This is a point in the open interval (2, 4) and thus, the
theorem is verified.
2/3
Example 02: Verify Roll’s theorem for the function: f(x) = 1 – x on [–1, 1].
Solution: If we plot the given function between –1 and + 1, then there will be no break or jump in
the graph of the function. Therefore, f is continuous on [-1, 1]. Also f(–1) = f(1) = 0.
2
f   x   0  2x 1 3 / 3   . Observe that the derivative of given function does not exist at
3x1/3
x = 0 where x  0  (1, 1) . This means the last condition of Rolle’s Theorem is not satisfied.
This means the Rolle’s theorem is not valid for the given function and the interval [– 1, 1].
Example 03: Verify Roll’s theorem for the function: f  x  = x  x + 3 ex 2 on [  3, 0].

Solution: Since the polynomial x  x  3  x 2  3x and the exponential functions both are
continuous and derivable every where hence their product is also continuous and derivable every
where. It implies that f (x) is continuous in [  3, 0].

   1 1

Now, f   x   x 2  3x e x 2     e x 2  2x  3   2x  3 e x 2  x 2  3x e  x 2
 2 2

1
2 
1
2
  1
 
 2  2x  3  x 2  3x  e x 2  x  6  x 2 ex 2   x 2  x  6 ex/2 . This is finite in the
 2
given interval so f `(x) is differentiable in the interval (– 3, 0).
Now, f  a   f  3  3 3  3 e3 2  0,f  b   f  0   0  0  3 e0  0  f  a   f  b   0 
Thus, f satisfies all three conditions of Roll’s theorem. So, there must exist at least one number c
in (– 3, 0) such that f `(c) = 0, that is;

-:101:-
CHAPTER FIVE MEAN VALUE THEOREMS AND INDETERMINATE FORMS
1
 (c2  c  6)ec 2  0  c2  c  6  0  c  2 or c  3. [Note: e is not zero]
-c/2
2
Only c = – 2 belongs to the interval (– 3, 0). Hence Rolle’s Theorem is valid when c = – 2.
Example 04: Verify the Rolle’s Theorem and find c (if possible) for the following functions
where interval for each function is also given.
(i) f  x   x 2  3x  2 on 1, 2 : Given function f  x   x 2  3x  2 is a polynomial,
therefore, it is continuous and derivable on 1, 2. Also, f (1)  1  3  2  0 and
f (2)  4  6  2  0 so f (1)  f (2). Hence, all three conditions of Rolle 's theorem
are satisfied. Thus, there must be a point 'c ' in the int erval (1, 2) such that f `(c)  0.
Now, f `(x)  2x  3  f `(c)  2c  3  0  c  3 / 2  (1, 2).This verifies Roll's theorem.
(ii) f (x)  sin 2 x on 0,  : Given function f (x)  sin 2 x is continuous and derivable
on  0, . Also, f (0)  sin 2 0  0 and f ( )  sin 2   0 so f (0)  f ( ).
Hence, all three conditions of Rolle 's theorem are satisfied. Thus, there must be a
po int 'c ' in the int erval (0, ) such that f `(c)  0. Now, f `(x)  2sin x cos x  sin 2x
 f `(c)  sin 2c  0  2c  sin 1 (0)    c   / 2  (0, ).This verifies Roll 's theorem.
3 3
(iii) f (x)  1  x 3/4 on[ 1, 1] : Given function f (x)  1  x 3/4  f `(x)  x 1/4  1/4 .
4 4x
f `(0)   (undefined). Thus one of three conditions of Roll 's theorem is not satisfied.
Thus Roll 's theorem fails for the function f (x)  1  x 3/4 on (1,1).
1  x2
(iv) Given function f  x   on [ 1, 1] is continuous and derivable on ( 1, 1)
1  x2
being the division of two polynomials and the deno min ator is not zero at any point
of the int erval ( 1, 1). Also, f ( 1)  0  f (1). Hence, all three conditions of Rolle 's
theorem are satisfied. Thus, there must be a point 'c ' in the int erval ( 1, 1) such that
(1  x 2 )( 2x)  (1  x 2 )(2x) 2x( 1  x 2  1  x 2 ) 4x
f `(c)  0. Now, f `(x)   
2 2 2 2
(1  x ) (1  x ) (1  x 2 ) 2
4c
 f `(c)   0  c  0  ( 1, 1).This verifies Roll's theorem.
(1  c 2 ) 2
MEAN VALUE THEOREM
Statement: If a function f is such that
(i) It is continuous in the closed interval [a, b]
(ii) It is derivable in the open interval (a, b)
then there exists at least one value `c` in the open interval (a, b) such that
f (b)  f (a)
 f (c)
ba
Proof: Define a function G(x) by G(x) = A x + f(x) ,where A as some constant to be
determined such that G(a) = G(b).
Since Ax is a continuous as well as derivable function for each real x, hence G(x) is also
continuous and derivable function. Thus G(x) must satisfy all three conditions of Rolle’s
Theorem including G`(c) = 0. Now

-:102:-
CHAPTER FIVE MEAN VALUE THEOREMS AND INDETERMINATE FORMS
G`(x) = A + f `(x) or G`(c) = A + f `(c) or A + f `(c) = 0 or A = –f `(c)
Now, G(a) = G(b) which implies that A a + f(a) = A b + f(b) or A = [f(a) – f(b)]/(b – a). Thus,
above last equation becomes: – f `(c) = [f(a) – f(b)]/(b – a) or
f (b)  f (a)
 f (c)
ba
This verifies the “Mean Value Theorem”.
Geometrical Meaning of Mean Value Theorem
Let A and B be points on the graph of the function y = f(x) corresponding to x = a and x = b.
Therefore, the coordinates of the points A and B are (a, f(a)) and (b, f(b)) respectively.
difference of ordinates f (b)  f (a)
Now, Slope of chord AB   (1)
difference of abscissae ba
Since f is continuous in the interval [a, b] hence, its graph is a continuous curve from A to B.
Moreover, f(x) is derivable in the interval (a, b) therefore it possesses a unique tangent at every
point between A and B. It is evident from the figures shown below that there is at least one point
P between A and B, where the tangent is parallel to chord AB. If c be the abscissa of this point,
then slope of tangent thereat is f `(x) Hence,

P Q B
B
A A
f(a) f(b) f(a) P R f(b)

0 a b 0 a b
Now we know that when two lines are parallel there slopes are equal. Thus, using (1), we obtain
f (b)  f (a)
 f (c)
ba
This establishes the geometrical proof of “Mean Value Theorem”.
Example 01: Verify Mean Value Theorem for the following function and find `c` if possible.
f(x) = (x – 1) (x – 2) (x – 3) in [0, 4].
.

2 3 2
Solution: Given f(x) = (x – 1) (x – 2) (x – 3) = (x – 3x + 2) (x – 3) = x – 6x + 11x – 6
Since f(x) is a polynomial, therefore it is continuous in the interval [0, 4] and derivable on (0, 4).
Now, f `(x) = 3x2 – 12 x + 11
Therefore, by the Mean Value Theorem, we have
f (b)  f (a) f (4)  f (0) (64  96  44  6)  (6)
f (c)   3c2  12c  11   3c2  12c  11 
ba 40 4
Simplifying, we obtain: 3c2  12c  11  3  3c2  12c  8  0 
Using quadratic formula, we get: c  2  1.155, 2  1.155 or c  3.155, 0.845
We observe that both values of c are admissible as both lie in the interval (0, 4).
This verifies Mean Value Theorem.
Example 02: Find c (if possible) of the Mean Value Theorem for the following
functions:

-:103:-
CHAPTER FIVE MEAN VALUE THEOREMS AND INDETERMINATE FORMS
(i) The function f (x)  x 3  3x 2  3x  2 on [1, 2]:
Given function is a polynomial function hence, it is continuous and derivable as well.
Now, f `(x)  3x 2  6x  3  f `(c)  3c 2  6c  3. Also f (2)  4 and f (1)  3. Thus, by MVT,
f (b)  f (a) 43 6  12
f `(c)   3c 2  6c  3   1  3c 2  6c  2  0  c   c  1.58 or 0.423
ba 2 1 6
The only value of c that lies in the int erval (1, 2) is c  1.58. This verifies MVT.
(ii) The function f (x)  x 2  7x  12 on [3, 4]:
Given function is a polynomial function so it is continuous as well as derivable on (3, 4).
Now, f `(x)  2x  7  f `(c)  2c  7. Also f (4)  0 and f (3)  0. Thus, by MVT,
f (b)  f (a) 00
f `(c)   2c  7   0  2c  7  0  c  7 / 2  3.5.
ba 43
This value of c lies in the int erval (3, 4). This verifies MVT.
(iii) The function f  x   x(x  2)3 on [0, 2]:
Given function being a polynomial function so it is continuous and derivable.
Thus there exists a point 'c' in (0, 2) such that f `(c)  f (b)  f (a) / (b  a). Now,
f `(x)  (x  2)3 .1  x.3(x  2) 2  (x  2) 2 [x  2  3x]  2(x  2) 2 (2x  1)
 f `(c)  2(c  2) 2 (2c  1). Also f (2)  0 and f (0)  0. Thus, by MVT,
f (b)  f (a) 00
f `(c)   2(c  2) 2 (2c  1)   0  2(c  2) 2 (2c  1)  0  c  2, 1 / 2.
ba 20
The only value of 'c' that lies in the int erval (0, 2) is c  1 / 2. This verifies MVT.
 11 13 
(iv) The function f (x)  x 3  3x  1 on   ,  :
 7 7
Given function is continuous and derivable on the given int erval being a polynomial.
f (b)  f (a)
Thus there exists a po int 'c ' in the given int erval such that f `(c)  . Now,
ba
 13   11 
f `(x)  3x 2  3  f `(c)  3c 2  3. Also f    0.17 and f    0.17. Thus, by MVT,
7  7 
f (b)  f (a) 0.17  0.17
f `(c)   3c 2  3   0  3c 2  3  0  c  1.
ba  (13 / 7)  ( 11/ 7) 
Both values of 'c ' lie in the given int erval. This verifies MVT.
(v) The function f  x   x  2 on  2, 4  is continuous and derivable.
f (b)  f (a)
Thus there exists a po int 'c ' in the given int erval such that f `(c)  . Now,
ba
1 1
f `(x)   f `(c)  . Also f  4   2  1.414 and f  2   0. Thus, by MVT,
2 x2 2 c2
f (b)  f (a) 1 1.414  0 1 1
f `(c)     0.707   1.414  c  2   0.707
ba 2 c2 42 c2 1.414
 c  2  0.5 or c  2.5. This value of 'c ' lies in the given int erval (2, 4). This verifies MVT.
INFINITE SERIES
A question that frequently arises in both engineering and mathematical problem-solving is the
behaviour of a solution when one (or more) parameters in the problem statement are changed.

-:104:-
CHAPTER FIVE MEAN VALUE THEOREMS AND INDETERMINATE FORMS
This occurs in sensitivity analysis when we examine solutions for their dependence on errors in
the original data.
One of the mathematical tools for such analysis is Taylor’s Theorem. In this section we shall
develop the theorem and then use it to solve problems.
Maclaurin’s Series
Suppose the function f(x) is represented within a certain interval (including x = 0) by a power
series of the form: f (x)  a 0  a1x  a 2 x 2  a 3x 3  a 4 x 4   a n 1x n 1 
where a 's are constants and f(0) = a0.
Now differentiating the power series term by term, w.r.t x, we get
f (x)  1a1  2a 2 x  3a 3x 2  4a 4 x 3  , f (x)  0  1.2a 2  2.3a 3x  4  3a 4 x 2 
f (x)  2.3a 3  2.3.4a 4 x  , .... , f (n 1) (x)  (n  1)! a n 1  n! a n x 
Assuming that derivatives of all orders exist at x = 0, then
f (0) f (0)
f (0)  a1  a1  f (0),f (0)  2! a 2  a 2  ,f (0)  3! a 3  a 3  ,
2! 3!
f (n 1) (0)
f (n 1) (0)  (n  1)! a n 1  a n 1  , Thus, the power series becomes
(n  1)!

f (0) 2 f (0) 3 f (n 1) (0) n 1  xk k


f (x)  f (0)  f (0)x  x  x   x    f (0)
2! 3! (n  1)! k  0 k!
This is known as Maclaurin series.
It should be noted that a function cannot be represented by a Maclaurin series unless the function
and all its derivatives exist at x = 0. Maclaurin series is useful in computing the value of a
function only when x is small (near to zero).
x
Example 01: Find the Maclaurin series of the following functions: (i) sin x (ii) e
Solution: (i) Let f(x) = sin x. Differentiating successively with respect to x, we get
f (x)  cos x, f (x)   sin x, f (x)   cos x, f (4) (x)  sin x,
f (5) (x)  cos x, f (6) (x)   sin x, f (7) (x)   cos x, f (8) (x)  sin x,
Substituting x = 0, we get: f (0)  0,f (0)  1,f (0)  0,f  (0)   1,f (4) (0)  0,

f (5) (0)  1,f (6) (0)  0,f (7) (0)  1,f (8) (0)  0,
The Maclaurin’s series is
f (0) f (0) f (4) (0)
f (x)  f (0)  xf (0)  x 2  x3  x4 
2! 3! 4!
x2 x3 x4 x5 x6 x7 x8
 sin x  0  x(1)  (0)  ( 1)   (0)  (1)  (0)  ( 1)  (0) 
2! 3! 4! 5! 6! 7! 8!
x3 x5 x7
sin x  x    
3! 5! 7!
(ii) Let f  x   e x
Differentiating successively with respect to x, we get
f (x)  ex ,f (x)  ex ,f (x)  ex ,f (4) (x)  ex ,... at x  0, we have

-:105:-
CHAPTER FIVE MEAN VALUE THEOREMS AND INDETERMINATE FORMS
At x  0, we get:

f (0)  e0  1, f (0)  e0  1,f (0)  e0  1,f (0)  e0  1,f (4) (0)  e0  1, ...
x2 x3 x4
Now, the Malaren’s series is: f (x)  f (0)  xf (0)  f (0)  f (0)  f (4) (0) 
2! 3! 4!
2 3 4
x x x
Substituting the above values, we have: e x  1  x    
2! 3! 4!
1 1 1
Example 02: Prove that, tan 1x = x  x3 + x5  x 7 + ... . Hence find the value of π 
3 5 7
1
Solution: Let f (x)  tan 1 x,f (0)  tan 1 (0)  0, f (x)   f (0)  1
1  x2
f (x)  (1  x 2 ) 1  f (x)  1(1  x 2 ) 2 (2x)  f (0)  0,
f (x)  2 (1  x 2 ) 2 (1)  x(2)(1  x 2 ) 3 (2x)   2 (1  x 2 ) 2  4x 2 (1  x 2 ) 3 
   
f (x)  8x 2 (1  x 2 ) 3  2(1  x 2 ) 2 , f (0)  2,

  
f (4) (x)  8 x 2  3 (1  x 2 ) 4 (2x)  (1  x 2 ) 3 (2x)  2 2(1  x 2 ) 3 (2x) 
 
 8 6x 3 (1  x 2 ) 4  2x(1  x 2 ) 3  8x(1  x 2 ) 3

 48x 3 (1  x 2 ) 4  16x(1  x 2 ) 3  8x(1  x 2 ) 3


f (4) (x)  24x(1  x 2 ) 3  48x 3 (1  x 2 ) 4 , f (4) (0)  0,
x2 x3 x4
Now the Maclaurin’s series is: f (x)  f (0)  xf (0)  f (0)  f (0)  f (4) (0) 
2! 3! 4!
x2 x3 x4 x5
tan 1 x  0  x(1)  (0)  (2)  (0)  (24) 
2! 3! 4! 5!
x3 x5 x7
tan 1 x  x    
3 5 7
1 1 1 
Putting x = 1 in the above expansion, we get: tan 1 1  1     . But tan 1 (1) 
3 5 7 4
 1 1 1  1 1 1 
Thus, 1       4 1      
4 3 5 7  3 5 7 
Hence the value of π can be obtained to any degree of approximation using the above result.
It may be noted that humanely it is not possible to take infinite number of terms of this series if a
value of given function is to be calculated at x = 0. If we take only finite number of terms of the
Maclaurin series there must occur, some error. This error part of the series is known as
“Remainder”. The remainder of the Maclaurin series is given by:
x n 1  n 1
Rn  f  x  , where 0    1
(n  1)!
Thus if a function f(x) is expressed in an infinite series at x = 0, then it can be expressed as:
f(x) = Pn(x) + Rn(x)

-:106:-
CHAPTER FIVE MEAN VALUE THEOREMS AND INDETERMINATE FORMS
x2 x3 xn n
Here, Pn (x)  f (0)  f (0)x  f (0)  f (0)   f (0)
2! 3! n!
This is a polynomial of degree n and Rn(x) is a remainder after (n + 1) terms.
The remainder Rn(x) helps us:
(i) To compute the number of terms of the series we must take if the error is known.
(ii) To compute the error if the number of terms of a series are known.
x
Example 03: Find the approximate value of e by using Maclaurin series expansion of f(x) = e if
only 10 terms of the series are taken. Show the validity of your results by taking the value of e to
27 decimal places from the window calculator.
x
Solution: We know that series expansion of e is:
x 2 x3 x 4
ex  1  x     (1)
2! 3! 4!
x n 1  n 1
The remainder is: Rn  f  x  , where 0    1
(n  1)!
x (n + 1) x (n + 1) θx
Now, f(x) = e or f (x) = e or f (θx) = e
x n 1  x 
Hence, Rn  e , where 0    1
(n  1)!
n+1 θx
Since x = 1, and 0 < θ < 1, hence x e ~ 1.
Thus Rn ~ 1/(n + 1)!.
Now if n = 10, then Rn ~ 1/11! = 0.00000002. This shows that if 10 terms of the series (1) are
taken and if we take x = 1 to find the value of e, there will be an error of about 0.00000002.
Now take 10 terms of the series (1) and putting x = 1, we get
1 1 1 1 1 1 1 1
e  1  1         ~ 2.718281526 (2)
2! 3! 4! 5! 6! 7! 8! 9!
Now the value of `e` to 27 decimal places (from the window calculator) is
e = 2.718284590452353602874713527 (3)
This agrees with the above result in (2) to 5 decimal places.
Although our result should have been accurate to 7 decimal places as is clear from the value of
Rn, the difference between the values of e in equations (2) and (3) is due to the approximate value
θx
of e which we have assumed to be equal to 1 whereas it isn’t.
Taylor’s Series
Unless a function is defined at x = 0 and all of its derivatives also exist at x = 0, the function
5/3
cannot be represented by a Maclaurin series. Functions such as x , ln x, cot x and csc x cannot
be represented by a Maclaurin series as these functions are not defined at x = 0.
Let a  R be different from zero and suppose that f(x) is a function which is represented within a
certain interval by a power series of the form shown below in which c’s are constants and
f(a) = c0. Differentiating term by term with respect to x, we get
n 1
f  x   c0  c1  x  a   c 2  x  a   c3  x  a   c 4  x  a    c n 1  x  a 
2 3 4

f   x   c1  2c 2  x  a   3c3  x  a   4c 4  x  a  
2 3

-:107:-
CHAPTER FIVE MEAN VALUE THEOREMS AND INDETERMINATE FORMS

f   x   2!c 2  3!c3  x  a   4  3c 4  x  a  
2

f   x   3!c3  4!c 4  x  a  

f
n 1
 x    n  1!cn 1  n!cn  x  a  
f   a  f   a 
f   a   c1  c1  f   a  , f   a   2!c 2  c 2  , f   a   3!c3  c3  ,
2! 3!
Since derivatives of all orders exist at x = a, we get
 n 1 a
 
f    a    n  1!cn 1  cn 1 
n 1 f
,
 n  1!
Thus, the power series becomes
f
n 1
f   a  f   a  a 
f  x   f  a   f   a  x  a   x  a 2
 x  a 3
   x  a n 1 
2! 3!  n  1!
This is called the ‘Taylor’s series.’ Also called the expansion or development of f(x) in powers of
(x – a). The remainder is
(x  a)n 1  n 1
Rn  f  a  x  , where 0    1
(n  1)!
Another useful form of Taylor’s series can be obtained if we replace x by a+ h that is;
h2 h3 h n 1  n 1
f  a  h   f  a   hf   a   f   a   f   a    f a  
2! 3!  n  1!
Note that both forms of Taylor’s expansion reduce to Maclaurin’s expansion if a = 0. Thus
Maclaurin’s series becomes a special case of Taylor’s series.
Example 01: Obtain the Taylor’s development of the following function up to 5 terms at the
given point: f(x) = ln x at a = 1.
1
Solution: We have, f  x   ln x  f  a   f 1  ln 1  0, f   x    f   a   f  1  1 1  1,
x
1 2
f   x     f   a   f  1  1 1  1, f   x    f   a   f  1  2 1  1,
2
x x3
6
f   x   f    a   f   1  6 1  6,
4 4 4
4
x
Taylor’s series is:
f   a  f   a  f   a 
4
f  x   f  a   f   a  x  a    x  a 2   x  a 3   x  a 4 
2! 3! 4!
Substituting a = 1, we get
f  1 f  1 f   1
4
f  x   f 1  f  1 x  1   x  12   x  13   x  14 
2! 3! 4!
 x  12  x  13  x  14  x  15
ln x   x  1     
2 3 4 5
Example 02: Establish the Taylor’s series for the function f(x) = log (x + 1) at x = 2. Work up to
4 terms.
Solution: Let f  x   log x  f  a   f 1  ln1  0,

-:108:-
CHAPTER FIVE MEAN VALUE THEOREMS AND INDETERMINATE FORMS
1 1
f  x    f   a   f  1  1, f   x     f   a   f  1  1,
x x2

 f   a   f  1  2, f    x     f    a   f   1  6


2 6
f   x  
4 4 4
3 4
x x
Taylor’s series is:
f   a  f   a  f   a 
4
f  x   f  a   f   a  x  a    x  a 2   x  a 3   x  a 4 
2! 3! 4!
 x  12  x  13  x  14
log x   x  1    
2 3 4
x 2 x3 x 4
Replacing x by x + 1, we get: log  x  1  x    
2 3 4
1 1 1
At x = 1, log  2   1    
2 3 4
Example 03: Apply Taylor’s theorem to prove that
 h2 h3 
i a x+h = a x 1 + h loga +  loga 2 +  loga 3 + ...
 2! 3! 
h h2 h3
 ii  log  x + h  = log x +  + + ...
x 2x 2 3x 3
h2 h3
 iii  ln sin  x + h  = ln sinx + h cotx - csc 2 x + cotx csc 2 x - ...
2 3
Solution: (i) Let f  x  h   a x h . Putting h = 0 we have f  x   a x

f   x   a x  log a,f   x   a x  log a  ,f   x   a x  log a  ,


2 3

h2 h3
Hence, f  x  h   a x h  f  x   hf   x   f   x   f   x  
2! 3!
h2 x 2 h
3
 a x  h  a x  ha x  log a   a  log a   a x  log a  
3
2! 3!
 h2 2 h
3 
Or f  x  h   a x  h  a x 1  h  log a    log a    log a 3  
 2! 3! 
(ii) Let f  x  h   log  x  h  . Putting h = 0 we have f(x) = log x
1 1 2
f  x   ,f   x    ,f   x   , Hence,
2
x x x3
h2 h3
log  x  h   f  x  h   f  x   hf   x   f   x   f   x  
2! 3!
2
1 h  1  h3  2 
 f  x  h   log x  h     2   6  3  
x 2  x  x 
h h2 h3
This gives log  x  h   f  x  h   log x    
x 2x 2 3x 3
(iii) Let f  x  h   ln sin  x  h  Putting h = 0 we have f(x) = ln (sin x).
-:109:-
CHAPTER FIVE MEAN VALUE THEOREMS AND INDETERMINATE FORMS
1
f  x    cos x   cot x,f   x    csc2 x,f   x   2csc x   csc x cot x   2cot x csc2 x,
sin x
h2 h3
Hence; ln sin  x  h   f  x  h   f  x   hf   x   f   x   f   x  
2! 3!

 ln sin x  h cot x 
h2
2

 csc 2 x 
h3
6
 
2cot x csc 2 x  
h2 h3
Or ln sin  x  h   f  x  h   ln sin x  h cot x 
csc 2 x  cot x csc 2 x 
2 3
L’HOPITAL’S RULE AND INDETERMINATE FORMS
In the late seventeenth century, John Bernoulli discovered a rule for calculating limits of fractions
whose numerators and denominators both approached zero. The rule is known today as
“L’Hopital’s Rule”, after Guillaume Francois Antoine de L’Hopital (1661 – 1704).
If the functions f(x) and g(x) are both zero at x = a, then: Lim f  x  g  x  cannot be found by
xa
substituting x = a. The substitution produces 0/0 which is a meaningless expression known as an
indeterminate form. The L’Hopital’s Rule enabled us to evaluate such limits that lead to
indeterminate forms. There are many indeterminate forms, for example: 0/0, ∞/∞, 0 × ∞, ∞ – ∞,
0 ∞ 0
0 , 1 , ∞ . All these forms are called indeterminate quantities because they are undefined,
indefinite, unfixed, imprecise, uncertain, and vague.
Indeterminate Form (0/0)
If f and g are two functions of x which can be expanded by Taylor’s series and if f(a) = g(a) = 0
f x f  x 
then: Lim  Lim l
x a g  x  x a g  x 

provided the latter limit exists, whether finite or infinite. In case, if the last limit exists, the same
is the limit of the first function. In case, it is infinite, we say that limit of given function does not
exist for a particular value of x.
Proof: By Taylor series
f
n 1
f   a  f   a  a 
f  x   f  a   f   a  x  a    x  a 2   x  a 3    x  a n 1  (i)
2! 3!  
n  1 !
If we put x – a = h or x = a + h, then (i) becomes
h2 h3 h n 1  n 1
f  a  h   f  a   hf   a  
f   a   f   a    f a   (ii)
2! 3!  n  1!
If we assume that h is so small that its square and higher powers are neglected, we get
f  a  h  ~ f  a   hf   a 
But f(a) = 0, hence f  a  h  ~ hf   a  (iii)
Similarly, g  a  h  ~ hg  a  (iv)
Dividing (iii) by (iv) and taking limit h tends to zero or x tends to a (because x = a + h), we get
f a  h  f (x) hf   x  f  x 
Lim  Lim  Lim  Lim
h 0 g  a  h  x a g(x) x a hg  x  x a g  x 
This proves the result.
NOTE: In case if f `(a) = 0 = g`(a), we continue applying the L’Hopital Rule till we get some
definite value.

-:110:-
CHAPTER FIVE MEAN VALUE THEOREMS AND INDETERMINATE FORMS
Example 01: Evaluate the following limits:
2
x  sinx ex  1
  i  Lim  ii  Lim
x 0 x 2 x 0 cosx  1
x  sin x 0
Solution: (i) Lim f  x   Lim 
x 0 x 0 x2 0
1  cos x 0
Using L’Hopital’s Rule, we get: Lim f  x   Lim 
x 0 x0 2x 0
sin x 1
Again using L’Hopital’s Rule, we get: Lim f  x   Lim  Lim  sin x   0
x 0 x 0 2 2 x 0
2
ex  1 0
(ii) Lim f  x   Lim 
x 0 x 0 cos x  1 0
2
2xe x 0
Using L’Hopital’s Rule, we get: Lim f  x   Lim 
x 0 x 0  sin x 0
Again using L’Hopital’s Rule, we get
  2 2 
2  x  2xe x   e x 1  2 2
    4x 2e x  2e x 02
Lim f  x   Lim  Lim   2
x 0 x 0  cos x x 0  cos x 1
Indeterminate Form ∞/∞
If Limf  x    and Limg  x   , then Limf (x) / g(x)  Limf (x) / g(x)
x a x a x a x a
Note: While evaluating Lim f  x  g  x  when it is of the form  /  care must be taken to
x a
change over to the form 0/0 as soon as it is conveniently possible, otherwise the process of
differentiating the numerator and denominator would never terminate.
ln  sin3x 
Example 02: Evaluate Lim
x 0 ln  sinx 
ln  sin 3x  ln 0  
Solution: Lim f  x   Lim    . Applying the L’Hopital’s Rule, we
x 0 x 0 ln  sin x  ln 0  
1
 cos3x  3 sin x cos3x 0
get: Lim f  x   Lim sin 3x  3Lim 
x 0 x 0 1 x 0 sin 3x cos x 0
 cos x 
sin x

Again using L’Hopital’s Rule, we get: Lim f  x   3Lim


sin x   sin 3x  3  cos3x  cos x
x 0 x 0 sin 3x   sin x   cos x  cos3x  3

cos3x cos x  3sin 3x sin x 1


 3Lim  3   1
x 0 3cos3x cos x  sin 3x sin x 3
Indeterminate Form 0 × ∞
Let Lim f  x   0, Lim g  x     We write
x a x a
gx  f x 0 
f xgx   form  or f  x   g  x    form 
1 f x   1 g x 0 
Thus, 0 is changed into the form  /  or 0/0 and then L’Hopital’s Rule is applied to
evaluate such limits.

-:111:-
CHAPTER FIVE MEAN VALUE THEOREMS AND INDETERMINATE FORMS
 πx 
Example 03: Evaluate Lim 1- x  tan  
x 1  2 
 x 
Solution: Limf  x   Lim 1  x  tan    0  
x 1 x 1  2 
 Lim 1  x  / cot(x / 2)  0 / 0
x 1
Using L’Hopital’s Rule, we get
1 2   x    2  2
Limf  x   Lim    Lim sin 2       1 
 x     x 1   2     
 csc2     
x 1 x 1
  
2 2
Indeterminate Form ∞ – ∞
To evaluate Lim f  x   g  x  , when Limf  x     Limg  x  , we have
x a x a x a
1 1

 1 1   g   x
x f 0 
Lim f  x   g  x   Lim     Lim 1  form 
x a 1 f  x  1 g  x   x a
x a 

1 0 
f x gx
Now, we can apply the L’Hopital’s Rule to evaluate the given limit.
 x 1 
Example 04: Evaluate the limit Lim   
x 1  x  1 lnx 
 x 1 
Solution: Limf  x   Lim      
x 1 x 1  x  1 ln x 
 x ln x   x  1  0
 Lim    
x 1   x  1 ln x  0
Using L’Hopital’s Rule, we get
1
x    ln x  1
x ln x x ln x 0
Lim f  x   Lim  Lim  Lim  
 x  1    ln x 1 x 1  
x 1 x 1 1 x  1  x ln x x 1  x  1  x ln x 0
 x x
Again using L’Hopital’s Rule, we get
1
x    ln x
x 1  ln x 1
Lim f  x   Lim  Lim 
x 1 x 1 1 x 1 2  ln x 2
1  x    ln x 1
x
0 ∞ 0
Indeterminate Forms 0 , 1 , ∞
g x  
To evaluate Lim f  x   , when
x a 
(i) Lim f  x   0  Lim g  x  (ii) Lim f  x   1, Lim g  x   
xa xa xa xa
(iii) Lim f  x   , Lim g  x   0
xa xa
g x 
Here we let y  f  x  , then ln y  g  x  ln f  x  (1)
(i) If Lim f  x   0  Lim g  x  , then (1) is of the form 0
xa xa
(ii) If Lim f  x   1, Lim g  x   , then (1) is of the form  0 
xa xa

-:112:-
CHAPTER FIVE MEAN VALUE THEOREMS AND INDETERMINATE FORMS
(iii) If Lim f  x   , Lim g  x   0, then (1) is of the form 0
xa xa
Thus, all the limits can be evaluated from the form 0   
Example 05: Evaluate the following:
tanx
1
i  Lim  tanx 
sin2x
ii  Lim 1+ sinx 
cotx
iii  Lim  
x 0 x 0 x 0  x 

Solution: (i) Let y   tan x   Lim y  Lim  tan x 


sin 2x sin 2x
(00 )
x 0 x 0

Taking ln on both sides, we get: ln  Lim y   Limsin 2x ln  tan x  0  


 x 0  x0
ln  tan x  
 Lim   . Using L’Hopital’s Rule, we get
x 0 csc 2x 
cos x  1 
1
ln  Lim y   Lim tan x
sec2 x 
 Lim

sin x  cos2 x 
 x 0  x 0  csc 2x cot 2x  2  x 0 2csc 2x cot 2x
1 sin 2x tan 2x
 Lim  Lim  Lim   tan 2x   0
x 0 2sin x cos x csc 2x cot 2x x 0  sin 2x x 0 ,
or Lim  tan x 
sin 2x
 e0  1 
x 0
(ii) Let y  1  sin x   Lim y  Lim 1  sin x  (1 )
cot x cot x
x 0 x 0
Taking ln on both sides, we get
ln 1  sin x  0
ln  Lim y   Lim cot x ln 1  sin x     0   ln  Lim y   Lim  
 x 0  x 0  x 0  x 0 tan x 0
Using L’Hopital’s Rule, we get
1
 cos x  cos3 x
ln  Lim y   Lim 1  sin x
cos x
 Lim  Lim 1
 x 0  x 0 sec2 x x 0 sec 2 x 1  sin x  x 0 1  sin x

Thus, Lim 1  sin x 


cot x
 e1  e
x 0
tan x tan x
1 1
(iii) Let y     Lim y  Lim   (0 ) . Taking ln on both sides, we get
x x 0 x 0  x 
  1  
ln  Lim y   Lim  tan x ln     0     ln  Lim y   Lim
ln(1 / x)
 
 x 0  x 0   x   x 0  x 0 cot x 


x 1 / x 2   Lim 1x sin 2 x  sin x 
 Lim  x  
2
 Lim  Lim    0 1  0
x 0  csc2 x x 0 csc 2 x x 0 x x 0  x 
tan x
1
Thus, Lim    e0  1
x 0  x 
APPLICATIONS OF LIMITS
In this section we shall show a very useful application of the limits known as “Asymptotes”. It
may be noted that asymptotes help us to draw the graph of a function. The asymptotic behaviour
of any curve is very much important in engineering or natural sciences.
Asymptotes
Asymptotes can be defined formally using the idea of limit in CALCULUS as:
-:113:-
CHAPTER FIVE MEAN VALUE THEOREMS AND INDETERMINATE FORMS
Definition: A straight line L is called an asymptote for a curve C if the distance between L and C
approaches zero as the distance moved along L from some fixed point on L tends to infinity.
There are three types of asymptotes (i) Horizontal asymptote (ii) Vertical asymptote and
(iii) Oblique asymptote. We now discuss how to find these asymptotes.
i. Horizontal Asymptote
A line y = b is called a horizontal asymptote of the curve y = f(x) if
Lim f  x   b or Lim f  x   b
x  x 

ii. Vertical Asymptote


A line x = a is a vertical asymptote of the curve y = f(x) if
Lim f  x    or Lim f  x   
xa  xa 
Note: Horizontal asymptote is parallel to the x–axis and vertical asymptote is parallel to the y–axis.
Example 06: Find the horizontal and vertical asymptotes of the curves defined by the following
1 1
equations:  i  f  x  =  ii  f  x  = x + x=1

x 1 x
Solution: (i) Horizontal asymptote
1 1
Lim f  x   Lim  0 or Lim f  x   Lim 0 y=0
x  x  x  1 x  x  x  1
It implies that y = 0 (x-axis) is a horizontal asymptote for the given f.
Vertical asymptote
1 1
Lim f  x   Lim   or Lim f  x   Lim  
x 1 x 1 x  1 x 1 x 1 x  1
It implies that x = 1 is a vertical asymptote for the given function f(x).
(ii) Horizontal asymptote
 1
Lim f  x   Lim  x     y=x
x  x   x
Thus there is no horizontal asymptote for the given curve.
Vertical asymptote
 1
Limf  x   Lim  x     . Thus x = 0 (y–axis) is a vertical asymptote for the given curve.
x 0 x 0  x
(iii) Oblique asymptote
An asymptote that is neither parallel to x–axis nor parallel to y–axis is said to be an oblique
asymptote. The equation of such asymptote is of the form y = m x + c.
It may be noted that the graph of a rational function has an oblique asymptote if numerator is of
one degree greater than that of the denominator. Consider a rational function defined by the
equation f  x   (x 2  1) / x (1)
Here degree of the numerator is one greater than the degree of the denominator. If possible let
f(x) = mx + c be an oblique asymptote of the curve (1). Then as y-coordinate of the curve (1) and
the line f(x) = mx + c coincide as x  This means that as x 
x2  1
mx  c   mx 2  cx  x 2  1   m  1 x 2  cx  1  0 (2)
x
c 1
Dividing by x2, we get  m  1   2  0
x x
-:114:-
CHAPTER FIVE MEAN VALUE THEOREMS AND INDETERMINATE FORMS
Since x  so we have m – 1 = 0 or m = 1. Put this value of m in equation (2), we get
cx  1  0  c  1 / x 
Since x  we get c = 0. Substituting the values of m and c in f(x) = mx + c, we get
f(x) = x, which is the required oblique asymptote of the curve (1).
Note: f  x   (x 2  1) / x  x  (1/ x), which is the same curve as given in the above example
where it was mentioned that f(x) = x is an oblique asymptote of the given curve.
Another Method to Find an Oblique Asymptote
If the degree of the numerator of a rational function f(x) /g(x) is one more than the degree of the
f x hx
denominator, we can write   x   
gx gx
Here,   x  is the oblique asymptote of the graph of the function f  x  g  x  

Example 07: Find the oblique asymptote of f  x  = (x 2  4) / (x  1)

x2  4 3
Solution: f  x    f  x    x  1  
x 1 x 1
2
[Students are advised to divide (x – 4) by (x – 1) and see the result]. It implies that the oblique
asymptote for the given curve is   x   x  1

Example 08: Find all possible asymptotes to the curve y = (x 2  3) / x


 x2  3  x  x  (3 x) 
Solution: (Horizontal asymptote) Lim y  Lim    Lim 
x  x   x  x  x
Thus, there is no horizontal asymptote for the given function.
 x2  3 
(Vertical asymptote) Lim y  Lim      x  0 is a vertical asymptote for the
x 0 x 0  x 

given curve.
(Oblique asymptote)
x2  3 3
y  y  x   y  x is an oblique asymptote for the given function.
x x
NOTE: When the equation of the curve is given in the implicit form f(x, y) = 0, then there is
another method to find the horizontal and vertical asymptote for a given function that is explained
in the following example.
Example 08: Find the horizontal and vertical asymptotes for the following curves:

i  x3  x 2 y  x  6 = 0  ii   x  y  (x 2  y2 )  10  x  y  x 2 +12y2 + 2x + y = 0
2

Solution: Horizontal asymptote


Here the term of the highest power of x is x3. Putting its coefficient equal to zero, that is;
1 = 0 (This is not possible)
It shows that the given curve does not have any horizontal asymptote.
Vertical asymptote
Here the term of the highest power of y is y, put its coefficient equal to zero, that is;
x 2  0  x  0 . Thus, x = 0 is the vertical asymptote for the given curve.
(ii) Simplifying the given equation
-:115:-
CHAPTER FIVE MEAN VALUE THEOREMS AND INDETERMINATE FORMS

 x  y 2  x 2 + y 2   10  x  y  x 2 +12y 2 + 2x + y = 0
  
or x 2  2xy  y 2 x 2  y 2  10x 3  10x 2 y  12y 2  2x  y  0

or x 4  2x 3 y  x 2 y 2  x 2 y 2  2xy3  y 4  10x 3  10x 2 y  12y 2  2x  y  0

   
or x 4  y 4   2y  10  x 3  2y 2  10y x 2  2y3  2 x  12y 2  y  0
Horizontal asymptote
4
Here the term of the highest power of x is x , put its coefficient equal to zero, that is;
1 = 0 (which is not possible)
It shows that the given curve does not have any horizontal asymptote.
Vertical asymptote
Here the term of the highest power of y is y4, put its coefficient equal to zero, that is;
1 = 0 (which is not possible)
It shows that the given curve does not have any vertical asymptote.

WORKSHEET 05
1. Verify the Rolle’s Theorem and find c (if possible) for the following functions where interval
for each function is also given.

(i) f  x   x 2  3x  2; 1, 2 (ii) f (x)  sin 2 x; 0,  (iii) f (x)  1  x 3/4 ;  1,1
x
1  x2 
(iv) f  x   ;  1,1 (v) f  x   x  x  3  e  3,0 (vi) f  x   2   x  1 ; 0, 2 
2; 32

1 x 2

(vii) f (x)  x 2  6x  8;  2, 4  (viii) f  x   x 3  4x;  2, 2  (ix) f  x   8x  x 2 ;  0,8


(x) f  x   x 2 ;  1,1 (xi) f  x   sin x;  ,  (xii) f  x   e x ;  0, 

(xiii) f  x   sin x / e x ; 0,  (xiv) f  x   4  x 2 ;  2, 2  (xv) f  x   tan x;  0, 


(xvi) f  x   x 2  7x  12; 3, 4 (xvi) f  x   x  x  2  ; 0, 2  (xvii) f  x   sec x;  0, 2
3

(xviii) f  x   x 3  3x 2  3x  2; 1, 2
2. Verify Mean Value Theorem on the given interval and find ‘c’ where possible.
(i) f  x   x 3  3x 2  3x  2; 1, 2 (ii) f  x   x 2  7x  12; 3, 4 
 11 13 
(iii) f  x   x  x  2  ; 0, 2  (iv) f  x   x 3  3x  1;   , 
3

 7 7
(v) f  x   x  2;  2, 4 (vi) f  x   x 3  5x 2  4x  2; 1,3
(vii) f  x   x 2 3 ;  1,1 (viii) f  x   f  x   x  x  1 x  2  ; 0,1 / 2 

 ix  f  x    x  1 x  2  x  3 ; 0, 4  x  f  x   x 2  4;  2, 4
(xi) f (x)  log x; 1,e  (xii) f  x   e x ; 0,1

(xiii) f  x   x 3  5x 2  3x; 1,3 (xiv) f  x   x 2  9; 3, 4 


3. Find the Mc Loren’s series of the following functions:
 i  f  x   sin x  ii  f  x   cos x  iii  f  x   sec x
(iv) f(x) = tan x (v) f  x   ln 1  x  (vi) f  x   esin x

-:116:-
CHAPTER FIVE MEAN VALUE THEOREMS AND INDETERMINATE FORMS
4. Apply Taylor’s Theorem to prove that:
 h2 h3 
i  a x  h  a x 1  h ln a   ln a 2   ln a 3  
 2! 3! 
 h 2 h3  1  h h 2 h3 
 ii  e x  h  e x 1  h      iii 
1
 1   2  3  
 2! 3!  x  h x  x x x 
h h2 h3
iv  ln  x  h   ln x   2 3
x 2x 3x
h2 h3
 v  ln sin  x  h   ln sin x  h cot x  csc 2 x  cot x csc 2 x 
2 3
h2 h3
 vi  ln cos  x  h   ln cos x  h tan x sec 2 x  sec 2 x tan x 
2 3
h x h2
 vii  sin 1  x  h   sin 1 x    
 
3 2 2!
1  x2 1  x2
2
h xh
 viii  tan 1  x  h   tan 1 x   
1 x
1  x 
2 2 2

h h2 2x 2  1
 ix  sec1  x  h   sec1 x    
x x2 1 
2! x 2 x 2  1 3 2

x tan  x  h   tan x  h sec 2 x  h 2 sec 2 x tan x 
h3
3

sec 2 x 1  3tan 2 x  
5. Prove the following:
x 4  256 x 4  256 x 2  3x 1
 a  lim  256  b  lim  32  c  lim 
x 4 x  4 x 4 2
x  16 x 3 2
x 9 2
x 2 x
e e xe ex  1 1
 d  lim  e2 (e) lim  1 (f ) lim 
x 2 x  2 x 0 1  e x x 0 tan 2x 2
ln(2  x) cos x  1 1 e 2x  e 2x
(g) lim 1 (h) lim  (i) lim 4
x 1 x  1 x 0 cos 2x  1 4 x 0 sin x
8x  2 x 1 2 tan 1 x  x ln sec 2x
 j lim  ln 2  k  lim 1
1  l  lim 4
x 0 4x 2 x 0 2x  sin x x 0
ln sec x
ln cos x 1 cos 2x  cos x 3 ln x
 m  lim 2   n lim
2
  o  lim 0
x 0 x 2 x 0 sin x 2 x  x

csc 6x 1 5x  2ln x x4  x2
p lim  q  lim 5 r  lim 0
x
 csc 2x 3 x  x  3ln x x  ex  1
2

s lim
x 0 
ln cot x
csc2 x
0 t lim
x 
e x  3x 3
4e  2x x 2

1
4
u 
x 0
 
lim e x  1 cot x  1
e
1
 v lim x 2 e x  0 w lim x csc x  1 x lim csc x ln x  
x  x 0 x 1 

 y lim e  tan x sec 2 x  0


x  /2

 z  lim x  sin 1 x csc3 x  
x 0
1
6

-:117:-
CHAPTER FIVE MEAN VALUE THEOREMS AND INDETERMINATE FORMS

 a` lim 
x 2  x 2
4
4

1 

x  2

1
4
 b  1
lim  
1 
0
x 0  x sin x 
 c  
lim sec3 x  tan 3 x  
x  /2

 ln x 1 
 d  lim 
x 
 e` lim  2  
1 1 4 2 1
    f ` lim   0
x 1  ln x x 1  2 x 0  x 1  cos x  3 x   x x

 i` lim  e x  3x 
1/x
 g` lim x x  1  h` lim  cos x 1/x  1  e4
x 0 x 0 x 0

 
ex
 j` lim 1  e x  e 1  k` lim  sin x  cos x 
tan x
 e 1  l` lim  tan x 
cos x
1
x  x  /2 x  /2
x x
2x
 n` lim 1    e
1
 m` lim  x  tan
2 e 2/ 
 o` lim 0
x 1 x   x x  3x

 p` lim
e3/x
0  q` lim
ex 
ex 1  ex
lim
1 e  lim
: H int
 x

x 0 x 2 x 0 (1  x)(1  x) x 0 (1  x) x 0 (1  x)
6. Determine the horizontal and vertical asymptotes for the following functions:
NOTE: It may be noted that any curve may have at most two asymptotes. They may be
(i) Horizontal and vertical asymptotes (ii) Horizontal and oblique asymptotes (iii) Vertical
and oblique asymptotes
 i  y   x  2 2 / x 2  ii  x 2 y 2  12  x  3 (iii) 2xy  x 2  3
(iv) x 2 (x  y) 2  a 2 (x 2  y 2 )  a 2 xy (v) x 2 y  xy 2  xy  y 2  3x  0
(vi) (x  y) 2 (x 2  y 2 )  10(x  y)x 2  12y 2  2x  y  0

-:118:-
CHAPTER SIX APPLICATION OF DERIVATIVES

CHAPTER
SIX APPLICATIONS OF
DERIVATIVES

In this chapter, we shall present some important applications of the derivatives that help the
students to understand in detail how these applications work in engineering and technical
problems.
TANGENT AND NORMAL
Definition: A straight line that touches the curve y = f(x) at
only one point, say at P, is called tangent to the curve y = f(x) at P.
Equation of the tangent line can be is derived as shown below:
Let P(x1, y1) be any point on the curve y = f(x).
Suppose it is required to find the equation of the tangent line to
the curve y = f(x) at the point P(x1, y1). We assume that f is
derivable at P(x1, y1), so that f `(x) exists at P(x1, y1). Now, the
slope of the tangent at P(x1, y1) is the value of dy/dx at this point. Let this value be denoted by m.
Now the tangent at the point P is a line through P(x1, y1) having slope m.
Therefore, equation of the tangent at P(x1, y1) is:
y  y1  m  x  x1   using point-slope form 
 dy 
y  y1 =    x  x1  (1)
 dx   x1, y1 
where (x, y) is any point on the tangent line. Hence, equation (1) is called the equation of tangent
line to the curve y = f(x) at the point (x1, y1).
Steps to find the equation of the tangent at a point
1. Find dy/dx from the equation of the curve. This gives the slope of the tangent at the general
point (x, y).
2. Find the dy/dx at the given point (x1, y1). This gives the slope of the tangent at (x1, y1).
 dy 
 x  x1  , where  
dy
3. Now, equation of the tangent is y  y1 =   denotes the
 dx  x1,y1   dx  (x1, y1)
slope found in step 2.
Definition: The normal to a curve at any point P(x1, y1) is the straight line through the point
P(x1, y1) perpendicular to the tangent to the curve at that point. Equation of the normal can be
derived as follows:
Let slope of the tangent to the curve at P(x1, y1) be m. Then slope of the normal to the curve at
P(x1, y1) will be – 1/m (Since the product of the slopes of two perpendicular lines is equal to –1).
Therefore, equation of the normal line to the curve at P(x1, y1) is:

-:119:-
CHAPTER SIX APPLICATION OF DERIVATIVES
1
y - y1 =   x  x1   using point - slope form Normal Line
m
1
y  y1 =   x  x1  (2) Tangent Line
 dy 
 dx 
( x1, y1)
where (x, y) is any point on the tangent line.
Hence, equation (2) is called the equation of a normal line to the curve y = f(x).
Example 01: Find the equation of tangent and normal to the curve xy = 10 at (1, 10).
Solution: We are given x.y = 10 (1)
and the point (x1, y1) = (1, 10). Differentiating (1) with respect to x, we get
.

y dy y
xy + y = 0 or xy = y or y =  or =
x dx x
dy 10
At (1, 10), m = =  = 10 = m
dx 1,10  1
Equation of tangent: y  y1 = m  x  x1 
Substituting the values, we get: y – 10 = –10(x – 1) or 10x + y – 20 = 0. This is the equation of
required tangent line for the given curve.
Equation of normal: y – y1 = – m  x – x1 
1
Substituting the values, we get: y – 10 = –  x – 1 or x – 100 y – 101 = 0
–10
This is the equation of the required normal line for the given curve.
Example 02: Find the equations of tangent and normal to the curve x3 + xy2 – ay2 = 0 at x = a/2.
Solution: putting x = a/2 in the given equation, we get
(a/2)3 + (a/2) y2 – ay2 = 0 this gives, y = ± a/2.
Hence, points where the tangent and normal are to be found are (a/2, a/2) and (a/2, –a/2).
Now from the given equation, we have f(x, y) = x + xy2 – ay2.
3

Or,
dy
dx
 
= – f x / f y = – (3x 2 + y2 ) / (2xy – 2ay)

dy 3(a / 2)2 + (a / 2)2


At (a/2, a/2): m= =– = 2.
dx 2(a / 2)(a / 2) – 2a(a / 2)
Hence equation of tangent line is: (y – a/2) = 2 (x – a/2) or 4x – 2y = a.
The equation of normal is: (y – a/2) = –(1/2) (x –a/2) or 2x + 4y = 3a
Similarly the equations of tangent and normal at (a/2, –a/2) are: 4x + 2y = 2 and 2x – 4y = 3a
respectively.
Example 03: Find the equations of tangent and normal to the curve y = 3x2 + 5x touching at y – axis.
Solution: We are given the curve y = 3x2 + 5x (1)
Since the curve touches the y-axis, hence at that point x = 0. Substituting x = 0 into (1), we get:
y = 3  0  + 5 0  = 0. Thus the point where the tangent and normal are to be found is
2

 x1, y1  =  0,0 . Now we will find the equation of tangent and normal at (0, 0). Differentiating
dy
equation (1) with respect to x, we get: = 6x + 5 = m . At (0, 0),
dx
m 0,0 = 6  0  + 5 = 5

-:120:-
CHAPTER SIX APPLICATION OF DERIVATIVES
Equation of tangent: y  y1 = m  x  x1 
Substituting the values, we get: y – 0 = 5(x – 0) or 5x – y = 0, this is the required equation of the
tangent line for the given curve.
1
Equation of normal: y  y1 =  (x  x1)
m
1
Substituting the values, we get: y  0 =   x  0 + 5y = 0, which is the required equation of the
5
normal line for the given curve.
Example 04: Find the points where the tangent is parallel to the x–axis and where it is parallel to
the y–axis for the following curve x3 + y3 = a3.
Solution: We are given
x3 + y3 = a3 (1)
Differentiating 1 with respect to x, we get
dy 2 dy 2 dy dy x2
2
3x + 3y 2 2
= 0 or x + y = 0 or y = x or
2
= 2 (2)
dx dx dx dx y
dy
If the tangent is parallel to x–axis, we have: =0
dx
x2
Then (2) becomes: 0 = - 2 or x 2 = 0 or x = 0
y
Substituting x = 0 into (1), we get
0 + y3 = a 3 or y3 = a 3 or y = a
Thus, the tangent is parallel to x–axis at (0, a).
dy
For tangent to be parallel to y–axis or perpendicular to x–axis: =
dx
x2 x2
Then (2) becomes:  =  or y2 =  =0  y=0
y2 
Substituting y = 0 in (1), we get
x3 + 0 = a 3 or x3 = a 3 or x = a
Thus, the tangent is parallel to y–axis at (a, 0).
Example 05: Find the angle of intersection between the curves y2 = 4 ax and x2 = 4 ay at the point
other than (0, 0).
Solution: We know that the angle between two curves is the angle between their tangent lines at
the point of their intersection. Now the point of intersection of the given curves can be found by
solving their equations simultaneously.
Given: y2 = 4 ax (1)
2
and x = 4 ay (2)
2
From (2), y = x /4a. Substituting this in (1) and simplify, we get x = 4a.
Putting this in (2) gives y = 4a. Hence the point where two curves intersect is (4a, 4a).
Now differentiating (1) w.r.t x, we get: dy/dx = 2a/y (3)
Similarly, differentiating (2) w.r.t x, we get
dy/dx = x/2a (4)
At (4a, 4a): Form (3), m1 = 1/2 and from (4), m2 = 2
-:121:-
CHAPTER SIX APPLICATION OF DERIVATIVES
If θ is the angle between two tangent lines, then
m  m1 2  1/ 2 3/ 2 3
tanθ = 2 = = = or θ = tan 1(0 / 0.75)  370
1+ m1m2 1+ 2(1/ 2) 2 4
CURVATURE AND RADIUS OF CURVATURE
Definition: The rate at which the tangent to a curve turns is called curvature of the curve. It is
denoted by a Greek letter κ (kappa). Let P and Q be two neighboring points on a curve AB. Let
arc AP = s and arc AQ = s + δ s, so that arc PQ = δ s. Let A be a fixed point on the curve from
where the arcs are measured.
Let the tangents at P and Q make angles ψ (psi) and ψ + δψ B

respectively with x–axis. Then the ratio δψ / δs, the average y = f(x)
Q
.change of direction of the tangent to the curve y = f(x).
That is; δψ / δs is the average curvature over the arc PQ. A s P
s
Now the limit of the average curvature when Q → P
   
. is called the “curvature of the curve at the point P.”
Thus, the curvature  (kappa) at the point P is
δψ δψ dψ
Lim = Lim =
QP δs δs0 δs ds
Radius of curvature
The reciprocal of the curvature of the curve at P (provided this curvature is not zero) is called the
Radius of the Curvature of the curve at P and is usually denoted by a Greek alphabet ρ  Rho  .
1 ds
Thus, ρ= =  Note : Radius is always positive
κ dψ
Note:
i. The curvature of a straight line is always zero. Hence the radius of the curvature of
the straight line is infinite (not defined).
ii. The radius of curvature of the circle is simply its radius.
We now write the formulae to compute the radius of curvature when the equations of the curves
are given in different forms.
(1) Explicit equation: If the equation of the curve is given in the explicit form y = f(x), then
32
1+ f (x)2 
 
ρ= 
f (x)

(2)Implicit equation: If the equation of the curve is given in the implicit form, f(x, y) = 0, then
32
(f x ) 2 + (f y ) 2 
ρ=  
f xx (f y )  f x f y f xy + f yy (f x ) 2
2

(3) Parametric equations: If the equation of the curve is given in parametric form, that is;
x = f  t  , y = g  t  , then
32
f (t)2 + g(t)2 
ρ=  
f (t)× g(t) - g(t)× f (t)
-:122:-
CHAPTER SIX APPLICATION OF DERIVATIVES
(4) Polar equation of a curve: If the equation of the curve is given in polar form f(r, θ), then
32
 2  dr 2 
r +   
  dθ  
ρ=
2
 dr  d 2r
r2 + 2   - r × 2
 dθ  dθ
Example 01: Find the radius of curvature of the curve f(x) = x2 – 5x – 6 at the point (4, – 10).
Solution: The equation of the curve is given in explicit form, that is;
f(x) = x2 – 5x – 6 (1)
Differentiating (1) with respect to x, we get
f (x) = 2x - 5 . At (4, – 10), f   x  = 2(4)  5 = 3
 4,10
Differentiating with respect to x again, we get f (x)  2 
32
1+ f   x 2  1+  32 
32

Now, ρ = 
   10 3 2
=  =  15.8
f   x  2 2

Hence, the radius of curvature of given curve at the given point is 15.8 units approximately.
Example 02: Find the radius of curvature of the curve y = 2x 3  6x 2 +11x at the point (1, 7).
Solution: Equation of the curve is given in explicit form, that is;
f(x) = 2x 3  6x 2 +11x (1)
Differentiating (1) with respect to x, we get
f (x) = 6x 2  12x +11 . At (1, 7), f (x) (1,7) = 6(1)2  12(1) +11 = 5

Differentiating again with respect to x, we get


f (x)  12x  12 .At (1, 7), f (x) (1, 7)  12(1)  12  0 

32
1+ f (x)2 
 
Now radius of curvature in explicit form is: ρ = 
f (x)

Since f ``(x) = 0, hence ρ is undefined. It implies that radius of curvature of the given curve at the
indicated point does not exist.
Example 03: Find the radius of curvature at any point of the curve x = a cos t, y = a sin t
Solution: Equation of the curve is given in parametric form. Now, differentiating w.r.t t, we get:
f   t  = -a sin t, g  t  = a cos t, f   t  = -a cos t, g  t  = -a sin t
32
 f   t  2 + g  t  2 
    
Since, ρ= 
f   t  × g  t  - g  t  × f   t 
Substituting the values, we get

-:123:-
CHAPTER SIX APPLICATION OF DERIVATIVES
32 32
 -asint 2 +  acost 2  a 2sin 2 t + a 2cos2 t 
  (a 2 )3/2
ρ= =  = =a
 -asint  -asint  -  acost  -acost  a 2sin 2 t + a 2cos2 t a2

[NOTE: sin2t + cos2t =1]


Example 04: For the cycloid x = a  t +sint  , y = a 1- cost  , prove that the radius of curvature

is given by ρ = 4a cos  t 2  .

Solution: The given equations x = f(t) = a 1+ sint  and y = g(t) = a 1- cost  parametric
equations. Hence differentiating twice with respect to t, we get
f   t  = a 1+ cost  ,f   t  = -asint,g  t  = asint,g  t  = acost
32
 f   t  2 + g  t  2 
    
Since, ρ =  so substituting the values, we get
f   t  × g  t  - g  t  × f   t 

 
32 32
a 2 1+ cost 2 + a 2sin 2 t  a 2 1+ 2cost + cos 2 t + sin 2 t 
ρ=   =  
a 1+ cost  acost  - asint  -asint  2 2 2 2
a cost + a cos t + a sin t 2

a 3  2 + 2cost  23 2 a 3 1+ cost 


32 32
ρ=
a 2 1+ cost 
=
a 2 1+ cost 
= 2 2a 1+ cost = 2 2a  
2cos t 2 = 4acos  t 2 

a  π
Example 05: Find the radius of curvature of the curve r = at the point  a,  .
1+ cosθ  2
a
Solution: The equation of the curve is given in polar form, that is; r = (1)
1+ cosθ
Differentiating (1) with respect to θ , we get
dr d  a  d a sinθ
 = a 1+ cosθ  =
-1
=  (2)
dθ dθ  1+ cosθ  dθ 1+ cosθ 2
 π  dr asin  π 2   π π 
At  a,  , = = a,  Note : sin = 1,cos = 0 
 2  dθ  π
 a, 

1+ cos π 2  2
 2 2 
 2

Differentiating (2) with respect to  , we get

d2r d  sinθ   1 + cosθ 2  cosθ   sinθ  2 1 + cosθ  sinθ  


=a  =a 
dθ  1 + cosθ 2    
dθ 2 
4
   1 + cosθ
π π π
acos 1 + cos  + 2asin 2
d r acosθ 1 + cosθ  + 2asin θ
2 2 2
d r 2 2 2
 2 = . Thus, 2 = = 2a.
dθ 1 + cosθ 3
dθ  a, π 
 
 π 
3

 2 1 + cos 
 2
32
 2  dr 2 
r +   
  dθ  
Now, ρ=
2
 dr  d2r
r2 + 2   - r × 2
 dθ  dθ
-:124:-
CHAPTER SIX APPLICATION OF DERIVATIVES
Substituting the values, we get

 2a 2 
32 32
 a  2 +  a  2 
  2 2a 3
ρ= = = = 2 2a units.
 a  + 2  a  - a  2a 
2 2
a2 a2

Example 06: Prove that the radius of curvature at the point (2a, 2a) on the curve x2y = a(x2 + y2)
is 2a.
Solution: Given function is given in implicit form. Here f(x, y) = = ax2 + a y2 - x2y. Thus
fx = 2ax – 2xy, fy = 2ay – x2, fxx = 2a – 2y, fyy = 2a, fxy = -2x.
At (2a, 2a): fx = 2a (2a) – 2(2a)(2a) = – 4a ,
2
fy = 2a(2a) – (2a)2 = 0,
fxx = 2a – 2(2a) = – 2a, fyy = 2a, fxy = –2(2a) = – 4a.
32


 xf  2
+ f
23 2
 
y 


 
 -4a

2 2 2
+  0 
 64a 6
ρ= = = = 2a
   
2 2
f xx f y - 2f x f y f xy + f yy  f x 
2
-2a  0  - 2(-4a 2 )(0)(-4a) + (2a) -4a 2
2 32a 5

Circle of Curvature
Let y = f(x) have a continuous derivative in a neighborhood of x = a and a non-zero second
derivative f ``(x) at x = a. Then both the curvature k and the radius of curvature ρ at the point
(a, f(a)) of curve C defined by y = f(x) exit and are non-zero. If f ``(x) > 0, C is concave up and if
f ``(a) < 0 then the curve C is concave down at (a, f(a)). The center of curvature of C at (a, f(a)) is
the point Q on the normal to C at (a, f(a)) on the concave side of C whose distance from (a, f(a))
is ρ. The circle of curvature of C at (a, f(a)) is the circle with center Q and radius ρ. This circle is
also called the Osculating circle of the curve C at (a, f(a)). The osculating circle has the same
tangent as the curve at (a, f(a)).

Concave up Concave down

Definition: Let y = f(x) be twice derivable function with nonzero curvature at a point P(x, y).
Then the coordinates (h, k) of the center of curvature at point P are given by:
1+ (y )2 1+ (y )2
h = x  y , k = y+
y y
Example 07: Find the center of the curvature of the curve defined by y = x3 at the point (1, 1).
Also find radius of curvature and equation of Osculating circle.
Solution: Since y = x3 or y` = 3x2 and y`` = 6x.
At (1, 1), y ` = 3 and y`` = 6. Thus coordinates of the center of curvature are:
1+ (y`)2 1+ 9 1+ (y`)2 1+ 9 8
h = x - y` =1– 3 = –4 , k=y = 1+ =
y`` 6 y`` 6 3
Thus center of the curvature is: (–4, 8/3).
32
1+ f   x  2 
    3/2
1+ (3)2 
Radius of curvature is ρ = 
5 10
=  =
f   x   6  3

The equation of osculating circle is:


-:125:-
CHAPTER SIX APPLICATION OF DERIVATIVES
(x – h)2 + (y – k)2 = ρ2
2  2
2  8 5 10  2 2
Or, (x + 4) +  y –  =   Or 3x + 3y + 24x – 16y – 14 = 0
 3  3 
MAXIMA AND MINIMA of A FUNCTION of ONE VARIABLE
Applications of calculus to business, science, and industry are widespread. Our examples and
exercises have been chosen to provide a feeling for how the derivatives can be used to solve real
problems when the situation can be represented by a function.
Given a few guidelines, you will be able to look at the graph of a function and see where the
function is increasing, where it is decreasing and, where it is maximum or minimum. For
example, given a profit function curve, we have to observe when the profit is maximum/minimum
or the profit is increasing or decreasing. Similarly, if we are given temperature curve against the
time, you will be able to look at the graph and see when a metal is being heated or cooled.
Learning how to interpret graphs will be an important experience, one filled with practical
applications. A study of how derivatives apply to graphs will also enable you to make many of
the same determinations without the benefit of a graph.
Increasing and Decreasing Functions
A function is said to be increasing when its graph rises
as it goes from left to right. A function is decreasing when
its graph falls as it goes from left to right.
The increasing or decreasing concept can be “A graph with increasing and
associated with the slope of the tangent line. decreasing function”
The slope of the tangent line to a curve will be positive
when the curve is rising and negative when it is falling. + ve slope – ve slope
This is shown in the adjacent figure.
Since f   x  is the slope of the tangent line,

it follows that if f   x  > 0, then function f is then f is decreasing.


Increasing/ Decreasing
1. At a point x = a at which f is defined:
(a) If f   a  > 0, then f is increasing at x = a
(b) If f   a  < 0, then f is decreasing at x = a
2. On an interval where f is defined
(a) If f   a  > 0, for all x in an interval, then f is increasing on the interval.
(b) If f   a  < 0, for all x in an interval, then f is decreasing on the interval.
Example 01: Consider the function defined by f(x) = x2 – 8x + 7. Find the interval on which the
function is and the interval on which it is decreasing.
Solution: Since, f(x) = x2 – 8 x + 7 (1)
Differentiating (1) with respect to x, we get: For f (x) to be increasing:
2x – 8 > 0 or 2x > 8 or x > 4
Thus f(x) is increasing in the interval (4, ∞). For f(x) to be decreasing:
2x – 8 < 0 or 2x < 8 or x < 4. Thus f(x) is decreasing in the interval (– ∞, 4)

-:126:-
CHAPTER SIX APPLICATION OF DERIVATIVES
Example 02: Let T(x) = –3x + 60 x + 70 be the temperature after x seconds of a metal tray
2

undergoing a chemical finishing process. Determine when the metal being cooled and heated.
Solution: The metal is being cooled when the temperature T(x) is decreasing. We know that a
function is decreasing when its derivative is less than zero, that is. Thus T(x) is decreasing when
T `(x) < 0.
Now, T(x) = –3x 2 + 60x + 70  T(x) = – 6x + 60
Hence, 6x  60  0  6x  60  x  10
Thus, the metal is being cooled after 10 seconds.
You may observe that the temperature during first 10 seconds of metal tray is increasing, that is;
T `(x) > 0 for 0 < x < 10 and then it starts cooling after 10 seconds.
Example 02: Suppose that P(x) = – 0.01 x2 + 60 x – 500 is the profit from the manufacture and
sale of x telephones. Is the profit increasing or decreasing when 100 phones have been sold?
Solution: Since, P(x) = – 0.01x2 + 60x – 500 (1)
Differentiating (1) with respect to x, we get: P(x) = – 0.02x + 60
When x = 100, we have : P 100  = – 0.02 100  + 60 = 58
Since, P(100) > 0, this means that profit is increasing when 100 phone sets have been sold.
Relative Maxima and Minima
The absolute maximum value of a function is the largest possible value of the function. The
absolute maximum value of a particular function may or may not be the same as the relative
maximum value. Consider the function graphed in the figure 1 below, defined for x in the interval
[2, 8]. The relative maximum value of the function is 5, but the absolute maximum value of the
function is 9. Consider next the function graphed in figure 2, defined for x in the interval [–3, 6].
The relative maximum value of the function is 4, and this is also the absolute maximum value of
the function.
(i) The absolute maximum function value occurs either where there is a relative maximum or at an
endpoint of the interval.
(ii) The absolute minimum function value occurs either where there is a relative minimum or at an
end point of the interval. Similar drawings and reasoning can be used to present the absolute
minimum versus relative minimum.

5 4

2 8 -3 6
Fig. 1 Fig. 2

Maximum and Minimum values of function y = f(x)


A function f(x) is said to have a relative maximum value f(a) at x = a if f(x) increases before x = a
and decreases after x = a.
A function f(x) is said to have a relative minimum value f(a) at x = a if f(x) decreases before x = a
and increases after x = a.

-:127:-
CHAPTER SIX APPLICATION OF DERIVATIVES
Note:
(1) Maximum and minimum values are also called extreme values or turning values or stationary
values.
(2) The points where a function has a maximum or minimum value are called turning points or
stationary points.
(3) The values at which f `(x) = 0 are called stationary values or critical values.
(4) A point that is neither maximum nor minimum is called point of inflexion or saddle point.
Conditions for maximum and minimum values of a function
1. First Derivative Test
(a) f(x) has a maximum value at x = a if f(x) increases
Before x = a, and decreases as x increases beyond a.
Thus, when x is slightly less than a, y increases f  (0) = 0

and therefore f  (x) is positive. When x is slightly f  (x) > 0 f  (x) < 0

greater than a, y decreases and therefore f  (x) is


negative. Therefore f  (x) changes sign from O a

positive to negative as x passes through the value a. Hence we have the following two conditions
for y = f(x) to have a maximum value f(a) at x = a.
(i) f  (x) = 0 at x = a.
(ii) f  (x) changes sign from positive to negative as x passes f `(x) < 0 f `(x) > 0

through the value a.


(b) f(x) has a minimum value at x = a if f(x) f `(x) = 0
decreases before x = a and increases as x increases beyond a.
Thus, when x is slightly less than a, y decreases and therefore o a
f  (x) is negative. When x is slightly greater than a, y increases and therefore
f  (x) is positive. Therefore f  (x) changes sign from negative to through the value a. Hence we
have the following two conditions for y = f(x) to have a minimum value f(a) at x = a.
(i) f  (x) = 0 at x = a.
(ii) f  (x) changes sign from negative to positive as x passes through the value a.
Second Derivative Test
(a) As observed in the first derivative test, f(x) has maximum value at x = a if f `(x) changes sign
from positive to negative at x = a. But f `(x) is itself a function of x which changes sign from
positive to negative, therefore, it decreases at x = a and hence its derivative f ``(x) is negative at
x = a. Hence a function y = f(x) has a maximum value at x = a if
(i) f `(x) at x = a (ii) f ``(x) is negative at x = a
(b) As has been seen in the first derivative test, f(x) has minimum value at x = a if f `(x) changes
sign from negative to positive at x = a. But f `(x) is itself a function of x which changes sign from
negative to positive, therefore, it increases at x = a and hence its derivative f ``(x) is positive at
x = a. Hence a function y = f(x) has a minimum value at x = a if
(i) f `(x) = 0 at x = a (ii) f ``(x) is positive at x = a
Definition: The values of y = f(x) at the points where f `(x) = 0 are called stationary values of the
function.
Notes: (1) Maximum and minimum values are all stationary values, but the converse is not true,

-:128:-
CHAPTER SIX APPLICATION OF DERIVATIVES
that is; a stationary value need not be a maximum or minimum value, because the curve may have
a point of inflexion at x = a.
(2) f `(x) = 0 at x = a implies that the tangent to the curve y = f(x) at x = a is parallel to the
x – axis. Therefore at stationary points the tangent is parallel to x – axis.
Working Rules for Finding the Maximum and Minimum Values of y = f(x)
First method:
(i) Find f `(x) and equate it to zero. Solve this equation for real values of x. Let these
values be a, b, c …
(ii) Find f ``(x). Put x = a, b, c … in f ``(x) turn by turn
If f ``(x) is negative at x = a, then f(x) is maximum at x = a and the corresponding maximum
value of f(x) is f(a). If f ``(x) is positive at x = a, then f(x) is minimum at x = a and the
corresponding minimum value of f(x) is f(a). Similarly for the points x = a, b, c..
(iii) If f  (x) = 0 at x = a but f  (x)  0 at x = a, then x = a is a point of inflexion.
Second method:
Sometimes, the process of finding f ``(x) becomes tedious. In such cases, first derivative test
should be preferred.
(i) Find f `(x) and equate it to zero. Solve this equation for real values of x. Let these
values be a, b, c…
(ii) Consider the value x = a. Study the signs of f `(x) for values of x slightly less than a
and slightly greater than a.
(iii) If f `(x) changes sign from positive to negative then f(x) is maximum at x = a. If
f `(x) changes sign from negative to positive then f(x) is minimum at x = a.
(iv) If f `(x) does not change sign, then x = a is a point of inflexion. Similarly for the
points x = b, c, …
Example 03: Determine the maximum and minimum values and point of inflection (if any) of the
function f(x) = x5 – 5x4 + 5x3 – 1.
Solution: Given f(x) = x5 – 5x4 + 5x3 – 1 (1)
Differentiating (1) with respect to x, we get
f `(x) = 5x – 20x + 15x
4 3 2
(2)
Putting f `(x) = 0, we get 5x – 20x + 15x = 0 or 5x (x – 4x + 3) = 5x (x – 1)(x – 3) = 0
4 3 2 2 2 2

Thus, the stationary values are x = 0, 1, and 3.


Differentiating (2) with respect to x, we get
f  (x)  20x 3  60x 2  30x (3)
For x  0, f  (0)  0 . Therefore, x = 0 gives neither maximum nor minimum value.
Also, f  (x)  60x 2  120x  30 (4)
For x  0, f  (0)  30  0, therefore, x  0 is a point of inflexion.
For x  1, f  (1)  20(1)3  60(1) 22  30(1)  10  0, therefore, f  x  is maximum at x = 1

Maximum value at x  1 is: f (1)  (1)5  5(1)4  5(1)3  1  0


For x = 3, f  (3)  20(3)3  60(3) 2  30(3)  90  0, therefore, f(x) is minimum at x = 3
Minimum value at x = 3 is: f (3)  (3)5  5(3)4  5(3)3  1  28

-:129:-
CHAPTER SIX APPLICATION OF DERIVATIVES
APPLICATIONS OF MAXIMA AND MINIMA
Example 01: MUET, Jamshoro advertises for a short course as per ISO requirements. The profit
from this course is P(x) = – 0.02x2 + 120 x + 100 rupees, where x is an amount spent on
advertisement. Find the amount to be spent on advertisement in order to maximize the profit.
Also find the maximum profit.
Solution: We have P(x) = – 0.02x2 + 120 x + 100 (1)
Differentiating 1 with respect to x, we get: P (x)  0.04x  120
Now P (x)  0  0.04x  120  0  x  3000
Since, P (x)  0.04  P  3000   0.04  0  P(x) is max imum when x  3000.
Thus, the maximum amount to be spent on the advertisement in order to maximize the profit is
Rs.3000 and the maximum profit is: P  3000   0.02  3000   120  3000   100  1,80,100 
2

Example 02: A farmer has 1000 meters of barbed wire which he is to fence off three sides of a
rectangular field, the fourth side being bounded by a straight canal. How can the farmer enclose
the largest field?
Solution: Suppose the length of the side parallel to the canal is x meters and that of perpendicular
side is y meters. Then the area A = x . y (1)
According to given conditions, we have
x + 2y = 1000 or x = 1000 – 2y (2) y
Rectangular field
Thus equation (1) becomes
A = (1000 – 2y).y = 1000y – y
2
(3) x
Differentiating (3) with respect to y, we get
dA/dy = 1000 – 4y. Now, for maxima, dA/dy = 0 giving 1000 – 4y = 0 or y = 25. Also
d2A/dy2 = – 4 which is negative for all y. Thus, largest field can be covered if the side of the field
are taken y = 25 meters and x = 2y = 50 meters.
Example 03: A topless rectangular box with a square base is to have a volume of 1296 cubic cm.
The material for the base costs Rs.3 per square cm and the material for the sides cost Rs. 2 per
square cm. What dimensions should the box have to minimize its cost?
Solution: Let the length of the topless rectangular box be x cm, width be also x cm
(Since, the base is a square) and height be the y cm. Then open top
x  x  y  1296  using V  LWH
or, x2 y = 1296 (1)
Cost of material for the base = 3x2 (Rs)
Cost of material for the four sides = 2(4xy) = 8xy (Rs)
5
Total cost  C  3x  8xy 
 1296  10368
Using equation (1) we get: C  3x 2  8x   3x 2  (2)
2 
 x  x
This is the function to be minimized. Differentiating (2) with respect to x, we get
C  6x  10368 / x 2 .For min ima, C  0  6x  10368 / x 2  0 giving 6x 3  10368  0
This gives x = 12. Now, C  6  (20736 / x 3 )
20736
At x  12, we have, C (12)  6   6  12  8  0 
(12)3
-:130:-
CHAPTER SIX APPLICATION OF DERIVATIVES
Thus, C is minimum for x = 12. Substituting x = 12 into equation (1), we get y = 9.
Hence, the required dimensions of the box are: 12 cm, 12 cm and 9 cm.
Example 04: A manufacturer of storage bins plans to produce some open–top rectangular boxes
with square bases. The volume of each box is to be 100 cubic feet. Material for the base costs $8
per square foot, and material for the sides costs $5 per square foot. Determine the dimensions of
the box that will minimize the cost of materials.
Solution: Suppose x be the width of the bin. Since the base is square, the length must also be x.
No information is given about the height, so we will use h to represent it. A drawing is given
below:
The area of the base is x  x or x2 ft2 Thus, at $8 per square foot, the cost
of the material for the base is 8x2 dollars. The area of each side is xh, so the h
area of the four sides is 4xh  Since the cost of the material for the sides is
$5 per square foot, the cost of the material for all four sides is $5  4xh or
20 xh dollars. Thus, the total cost C for all the materials is of a bin is x
2
C = 8x + 20 xh dollars (1)
Since, V  x  x  h  V  x 2 h  100  x 2 h (V  100 ft 3 )  h  100 / x 2
Differentiating, (1) with respect x, we get: C  16x  (2000 / x 2 )
 100  2000
Now C becomes: C  8x 2  20x   C  8x 2  (2)
2 
x  x
2000
For critical numbers, we have: 16x   0  16x 3  2000  0  x 3  125  x  5
2
x
4000 4000
Now, C  16   At x  5, C (5)  16   48  0
3
x (5)3
Thus, cost C is minimum when base side x = 5 ft and h = 100/x2 = 100/25 = 4 ft.
Example 05: A farmer has 1600 feet of fencing to make a rectangular enclosure for his dogs.
What should be the dimensions of the enclosure if he wants the largest area?
Solution: Let x feet and y feet be the length and width of the rectangular pen respectively. Then
x feet

y feet

Perimeter  2  x  y   1600  2  x  y   y  800  x (1)

Also, Area  (x)(y)  x(800  x)  A  800x  x 2 (2)


Differentiating (2) with respect to x, we get dA / dx  800  2x
For critical numbers, we have: 800  2x  0  x  400
Also, d 2 A / dx 2  2and at x  400, A  2  0
Thus, area A is maximum when x = 400 and y = 400. Hence, the dimensions are x = 400 ft and
y = 400 ft. This means that area can be maximum if the require field is of square shape.
Example 06: A builder decides to fence in a rectangular area of 800 square feet behind his
warehouse, using the wall of the building as one of the four sides (see figure). What is the least
amount of fencing necessary for the other three sides?
Solution: Let x feet and y feet be the width
-:131:-
CHAPTER SIX APPLICATION OF DERIVATIVES
and length of rectangular area respectively.
Then length of fencing
L = y + x + x = y + 2x WALL
or, y = L – 2x (1)
Also, Area = (x)(y) (2)
Or 800 = x(L – 2x)
Simplifying, we get L = 2x + (800)/x (3)
dL 800
Differentiating (3) with respect to x, we get: 2
dx x2
For critical numbers, we get
800
2  0  x 2  400  x  20  ve sign is not admissible 
x2
d2L 1600 d2L 1600 1
Also,   At x = 20,   0
dx 2
x 3
dx 2
 20 
3 5

Thus, L is minimum when x = 400. From (2), we have: A  xy  800  20y  y  40


Hence, the dimensions of field are: 40 ft by 20 ft. Also, total fencing required is
40 + 2(20) = 80 ft
Example 07: A storage company wants to create a storage facility by walling in a rectangular
region containing 1728 square feet. It will also use walls to subdivide the region into five equal
storage compartments (see the figure).
y

What should be the width (x) and length (y) of the storage facility in order to use the least amount
of material for the walls?
Solution: Let x and y be the dimensions of rectangle then its area is: A = x y
It is given that the area of this region is A = 1728 ft2. Therefore, we have
.y = A/x = 1728/x (1)
Now, the Perimeter = 6x + 2y (since there are six walls)
 1728  3456
Using equation (1), we get P  6x  2y  P  6x  2    6x  (2)
 x  x
Differentiating (2) with respect to x, we get
3456
P(x)  6  (3)
x2
3456
For critical numbers, we set 6  0  x 2  576  x  24
2
x
We need to minimize the P. Therefore, P(x)  6912 / x 3 (4)
6912
Putting x = 24 we get: P  24    0.5  0
 24 3

Thus, P is minimum at x = 24. Now, from equation (1), we have: y  1728 / 2  72 .

-:132:-
CHAPTER SIX APPLICATION OF DERIVATIVES
Hence, the dimensions of the storage facility in order to use the minimum material for the walls
are: width = 24 feet and length = 72 feet.
Example 08: Determine the dimensions of a closed rectangular box with a square base if the
volume must be 1000 cubic centimeters and the area of the outside surface is to be as small as
possible.
Solution: Let x be the width of the box. Since the base is square,
the length must also be x. Let h be its width. So, h x
V  x  x  h  V  x 2h (1) x
1000
1000  x 2 h  h  (2)
x2
Also the surface area S  2lw  2lh  2xh  S  2(x)(x)  2(x)(h)  2(x)(h)

 
Usin g (2), we get : S  2 x 2  2xh  S  2  x 2  2x 

1000 
x2 

4000
 S  2x 2  (3)
x
dS 4000
Differentiating (3) with respect to x, we get = 4x 
dx x2
4000
For critical numbers, we set: 4x   0  4x 3  4000  x 3  1000  x  10
2
x
8000 8000
Also, S  4   S x 10  4 
 12  0 .
3
x 103
Thus, S is minimum at x = 10. From (2), we have h  1000 / 100  10
From (1), we can verify that V = (10)2 (10) = 1000. Hence, the dimensions of the closed
rectangular box are: Length  10 cm, Width  10 cm and Height  10 cm.
Example 09: A square piece of cardboard 40 centimeters by 40 centimeters is used to make an
open box as shown in the figure. A small square is cut from each corner of the cardboard, and
then the sides are folded up. Determine the size of the cut (x in the figure) that will lead to the box
of largest volume. (Note: At some point it may appear that there are two answers, but only one of
them will make sense given the size of the cardboard)
x 40 – 2x
x

Original cardboard Corners cut out Folded on dotted lines

Solution: It is given that the length and width of the original cardboard is 40 cm. If a small square
is cut from each corner of the cardboard (x in the figure), then length and width of the cardboard
will be  40  2x  cm. Thus, the volume of the box, that is; after folding, is
V = (40 – 2x)(40 – 2x) (x) = 1600x – 160x2 + 4x3 (x being the height) (1)
We need to maximize the volume V so , differentiating (1) with respect to x, we get
V  1600  320x  12x 2
For critical numbers, we have: V  0  1600  320x  12x 2  0  3x 2  80x  400  0
-:133:-
CHAPTER SIX APPLICATION OF DERIVATIVES
Using quadratic formula, we get: x = 20 and x = 20/3.
If we put x = 20 then 40 – 2x = 40 – 80 = – 40 which is the invalid length of the side of box after
cutting the x-portion. Thus, admissible value of x = 20/3.
Also, V  320  24x. Putting x  20 / 3, we get V  160  0.
So, the greatest volume of the box is V = 5481.5 cm2 20/3 cm is cut off from the four sides.
Example 10: A metal CAN is to be made in the form of a right circular cylinder that will contain
16π cubic inches of metal. What radius of the can will require the least amount of metal (see the
figure)? Note that there are three parts: a circular top, a circular bottom and the curved side.
V 16 16
Solution: The volume of cylinder is V  r 2 h  h  h h (1)
r 2 r 2 r2
 16 
Also, surface area: S  2r 2  2rh  S  2  r 2  r   . r
 r2 
(Here, we have used the value of h of equation (1).
 16 
S  2  r 2   (2) h
 r 
 16 
Differentiating (2) with respect to r, we get: S  2  2r  
 r2 
 16  16
For critical numbers, we set: 2  2r    0  2r   0  2r 3  16  r 3  8  r  2
2
 r  r2
 32   16 
We need to minimize the S. Therefore, S  2  2    S  4 1  
3
 r   r3 

 
When r  2, S  4 1  (16 / 23 )  12  0 .

This means when r = 2 inches, the least amount of metal is required for surface area and hence for
making the CAN.
Example 11: The output power P of a battery is given by P = VI – RI2. For what value of the
current I is the power maximum if the voltage V and resistance R in the circuit are 10 volts and
20 ohms respectively?
Solution: Since, P = VI – RI2 (1)
Using V = 10 and R = 20, we get P = 10 I – 20 I . Differentiating (1) with respect to I, we get
2

dP dP
 V  2RI   10  40I
dI dI
For critical numbers, we set: 10 – 40 I = 0 giving I = 1/4.
Also, Now, P  40 so at I  1 / 4, P  40  0 which is negative.
Thus, P is maximum when current I = 1/4 (A). The maximum power of the battery is then
P = 10 I – 20 I 2 = 10/4 – 20/16 = (40 – 20)/16 = 1.25 Watt.
DIFFERENTIALS AND THEIR APPLICATIONS
Let y  f (x) giving y  y  f (x  x)
Or y  f (x  x)  f (x) (1)
In (1), x denotes a small increment in the value of x. It can be positive or negative. Here,
y denotes the corresponding exact change in the value of y. For example,
y  f (x)  x 2  2
-:134:-
CHAPTER SIX APPLICATION OF DERIVATIVES
If x changes from 1 to 1.1, then corresponding exact change in the value of y is given by
y  f (1  0.1)  f (1)  f (1.1)  f (1)  (1.1) 2  2  (1) 2  2  0.21
y f (x  x)  f (x)
Now dividing (1) by x from both sides, we get: 
x x
Now let x  0, then we have:
y f (x  x)  f (x) dy
Lim  Lim  [The slope of the curve y  f (x)]
x 0 x x 0 x dx
dy
Since,  f (x)  dy  f (x)dx
dx
Thus, if y = f(x) is differentiable then the differential of x, written dx, is a small real number and
the differential of y, written dy, is the product of f (x) and dx Note that
dx = Δx and dy  Δy
Thus, dy gives the approximate change in the value of y. So differentials are useful to obtain
approximations. As we deal with approximations then the errors will also occur. Therefore, We
study two types of errors.
Δy
1. Relative or Average Error in y is , where y is the quantity to be measured and y is
y
the error in y
Δy
2. Percentage Error in y = ×100%
y
For example, if pressure P = 10 and P  0.01 , then: Relative error = 0.01/10 = 0.001
Percentage error  (0.01/ 10) 100%  0.001100%  0.1%
Example 01: The radius r of a circle increases from r = 10 m to 10.1 m. dr = 0.1
Estimate the increase in the circle’s area A by calculating dA. Compare
this with the true change ∆A
Solution: Since, A  r 2
r  10m
Using differentials, we get: dA  (2r)dr  dA  2 r dr
Substituting the values, we get: dA  2(10 m)(0.1m)  dA  2 m 2
The true change is A  (10.1)2  (10)2  (102.01  100)  2.01 m 2  2 m 2  0.01 m 2
Thus, the error in the approximation is 0.01 m 2 
Example 02: Let f(x) = 3x2 + 1
(a) Use differentials to find dy (the approximate change in y) when x changes from 5 to 5.01.
(b) Determine the exact change in y (that is, Δy ) when x changes from 5 to 5.01.
Solution: (a) Since f (x)  3x 2  1  f (x)  6x
Using dy  f (x) dx , we have: dy  6x dx
Substituting the values, we get: dy  6(5)(0.01)  0.3 . This is an approximate change in y.
(b) The exact change is
y  f (x  x)  f (x)  f (5  0.01)  f (5)  f (5.01)  f (5)  3(5.01) 2  1  3(5)2  1  0.3003
Note that differentials provide a good and quick approximation.

-:135:-
CHAPTER SIX APPLICATION OF DERIVATIVES
Example 03: The radius of a sphere is found by measurement to be 10.5cm with a possible error
of 0.1cm. Find the possible error in its surface area and volume.
Solution: Let r, S and V be the radius, surface area and volume of the given sphere respectively.
Then, S  4 r 2 (1)
and V  (4 / 3) r 3 (2)
Using differentials, (1) becomes: dS  4  2r  dr  8 r dr

Substituting the values, we get: dS  8(10.5 cm)(0.1 cm)  8.4 cm 2


Thus, the possible error in its surface area  8.4 cm 2  Now using differentials, (2) becomes
4
 
dV   3r 2 dr  4r 2dr
3
dV  4 10.5cm   0.1cm   44.1 cm3
2
Substituting the values, we get

Thus, the possible error in its volume  44.1 cm3


Example 04: A spherical balloon has radius 10units. Show that the percentage increase in its
volume is approximately 0.3units, if its radius increases 0.1 percent.
Solution: Let r and V be the radius and volume of the spherical balloon respectively. Then,
V  (4 / 3) r 3 (1)
Using differentials, (1) becomes: dV  (4 / 3)  (3r 2 )dr  4 r 2dr
Substituting the given values, we get
  1  
dV  4  3.14 10   0.01  10   0.01   dV = 12.56 unit
2
 using dr  0.1% of r  0.1
3

  100  
dV 12.56 37.68
Percentage error:  100%   100%   100%  0.3%
4
V
 3.14 10 3 12560
3
Hence it is shown that the percentage increase in the volume of the spherical balloon is 0.3% 
Example 05: Use differentials to approximate the change in volume of a sphere – shaped tumor
 4 3
 V = π r  when its radius increases from 1 to 1.1 cm.
 3 
Solution: Since V  (4 / 3) r 3 (1)
4
Using differentials, (1) becomes dV  (3r 2 )dr  4r 2dr
3
Substituting the values, we get: dV  4(3.14)(1)2 (0.1)  1.256 cm3
Thus, approximately the change in volume of the sphere – shaped tumor is 1.256 cm3.
Example 06: A fast food restaurant serves soft drinks in a cylindrical cup that has a radius of 1.5
inches. The volume of soft drink that this cup can hold is V = 2.25 π h cubic inch.
(using V = πr 2 h with r = 1.5) . Ordinarily, the restaurant fills up to a height of 8inches. Suppose
that instead the restaurant decides to fill the cup to a height of 7.5 inches only.
Use differentials to approximate the number of cubic inches of soft drink the restaurant saves on
each serving.

-:136:-
CHAPTER SIX APPLICATION OF DERIVATIVES
Solution: Since V  2.25 h (1)
Using differentials, (1) becomes: dV  2.25 dh
C
Substituting the given values, we get: dV  2.25(3.14)(0.5)   3.53 cubic inches. O
L
Here, negative sign shows the decrease in capacity of the cup. A
Hence, approximately the number of cubic inches of the soft drink
that the restaurant saves on each serving is 3.53 in2.
Example 07: A fast food restaurant serves soft drinks in a cylindrical cup that has a radius of 1.5
inches. The volume of soft drink that this cup can hold is given by V = πr 2h . Ordinarily the
restaurant fills up to a height of 7.8 inches. Suppose that instead the restaurant decides to fill the
cup to a height of 8inches. Use differentials to approximate the number of cubic inches of soft
drink the restaurant spend on each serving.
Solution: Since V  r 2h (1)
Now r = 1.5 therefore (1) becomes V  2.25 h (2)
Using differentials, (2) becomes dV  2.25  dh
Substituting the given values, we get: dV  2.25(3.14)(0.2)  1.413 in 3
Hence, approximately the number of cubic inches of the soft drink that the restaurant
.spends on each service is 1.413 in3.
Example 08: Leaking sand forms a conical pile in which the height is always twice h
.the radius (h = 2r). Consider the moment at which the radius is 9cm.
Use differentials to .determine the approximate change in the volume when the
.radius changes by 0.1 cm. r

Solution: V  (1 / 3) r 2 h (1)


It is given that the height is twice the radius which means that h = 2r. Thus,
1 2
V  r 2  2r   V  r3 (2)
3 3
2
 
Using differentials, (2) becomes: dV   3r 2 dr  2r 2dr
3
2 3
Substituting the values, we get: dV = 2(3.14)(9) (0.1) = 50.868 cm . Hence, the approximate
change in the volume of the conical pile is 50.868 cm3.

WORKSHEET 06
1. Find the equations of tangent and normal to the curves (a) x 2  xy  y 2  7 at the point (-1, 2)
(b) y(x – 2) (x – 3) – x + 7 = 0 at the point where it cuts the x-axis.
2. Find the two points where the curve x 2  xy  y 2  7 crosses the x – axis, and show that the
tangents to the curve at these points are parallel. What is the common slope of these tangents?
3. At what points of the curve y  2x 3  3x 2  2x  4  are the tangents parallel to the
line 10x  y  7  0 . Find the equation of the normal at each of these points. Also find the angle
between the two curves: x2 – y2 = a2 and x2 + y2 = √2 a2.
4. Find the angle between two curves x2 – y2 = a2 and x2 + y2 = √2a2
5. Show that parabolas y2 = 4ax and 2x2 = ay intersect at an angle tan-1(3/5).
-:137:-
CHAPTER SIX APPLICATION OF DERIVATIVES
6. Prove that curves x /a + y /b = 1 and x /c + y /d = 1 will cut orthogonally if a – b = c – d.
2 2 2 2 2 2 2 2

7. Find the lengths of tangent, normal, sub-tangent and sub-normal to the curve:
x = a(t + sin t), y = a(1 – cos t)
8. For the curve x = a cos θ, y = a sin3 θ, show that length of tangent is y cosec θ. Also find the
3

length of: sub-tangent, normal and sub-normal.


9. For the curve x = a(ln cot θ/2 – cos θ), y = a sin θ, find the lengths of sub-tangent and sub-
normal at the point θ = π/4.
10. Find the radius of curvature at the given point of each of the following curves:
(i) y  x 2  5x  6 at  4, 10 (ii) 9x 2  16y2  180 at  2,3
   3a 3a 
iii y  sec x at  ,2  (iv) x3  y3  3axy at  , 
3   2 2
11. Find the radius of curvature of the given curve at the specified point:
 1 
i  r  4sin 2 at  2,    ii  r  a sec2;  a,0  , a  0
 12 
12. Find the equation of the osculating circle to the curve y = ln x at the point (1, 0).
13. Find the equation of the osculating circle to the curve x 2 / 4  y 2 / 9  1 at the point (–2, 0).
14. A tennis ball thrown straight up is 16t 2  96t  7 feet above the ground after t seconds. How
high will the ball go?
15. The sum of two positive numbers is 100. If their product is maximum then what are the two
numbers? (Hint: Let one number be x. What expression represents the other number?)
16. A farmer has 1600 feet of fencing to make a rectangular pen for his hogs. What should be the
dimensions of the pen if he wants the largest area?
17. What is the smallest amount of fencing that can be used to enclose a rectangular garden
having an area of 900 square feet?
18. An open rectangular box (that is, a box with no top) with a capacity of
z
36,000 cubic inches is needed. If the box must be twice as long as it is
x
wide, what dimensions would require the least material? y
19. A builder plans to construct a gutter from a long sheet of metal by making two folds of equal
size (see the figure). The folds are made to create perpendicular sides.
x

Sheet of metal
x

The metal is 28 centimeters wide and 500 centimeters long. How much the long sides are turned
up for each side in order for the gutter to hold the most water?
20. Determine the dimensions of the smallest size (that is, smallest area) rectangular piece of
paper that satisfies all of the following conditions:
(a) You can print 50 square inches of material on it (the shaded area in the fig).
(b) There will be 2 – inch margins on the top and bottom.
(c) There will be 1 – inch margins on the sides.

-:138:-
CHAPTER SIX APPLICATION OF DERIVATIVES
21. A gardener wishes to fence in a rectangular area of 1728 square feet. He also wants to insert a
fence that will divide the area into two rectangular sub areas. The
drawing shows that some fencing costs $4 per foot2 and some costs
$2 per foot2. Find the dimensions that will minimize the cost of the
fencing.
22. A wire 50 centimeters long is cut into two pieces. One piece (call its length x) will be bent to
form a square. The other piece (of length 50 - x) will be bent to form a circle. How much wire
should be used for the square if the total area (square plus circle) is to be the smallest possible?

23. Test the curve y = x4 for point of inflexion.


24. Find a and b if the function f(x) = a/x + bx such that f(2) = 1 possesses extreme point at x = 2.
Is (2, 1) a point of minima or maxima?. Confirm it by drawing the graph.
5. Show that function f(x) = sinm x cosn x attains a maximum when x  tan 1  m/n 
26. The horse power developed by an aircraft traveling horizontally with velocity v feet per
second is given by H = aw2/v + bv, where a,b, w are constants. For what value of v is the horse
power H is maximum?
27. The velocity of waves of wave length λ on deep water is proportional to  / a   a /  ,
where ‘a’ is some constant. Prove that the velocity is minimum when λ = a.
2
28. In a submarine telegraph cable, the speed of signaling varies as x ln(1/x), where x is the ratio
of radius of the core to that of the covering. Show that greatest speed is attained when x is 1/ e .
29. The efficiency E of a screw jack is given by E = tan x/tan(x + a), where a is a constant. Find
x if this efficiency is to be maximum. Also find the maximum efficiency.
30. Show that all the rectangles of given area, the square has the least parameter.
31. Find the rectangle of greatest perimeter that can be inscribed in a circle of radius a.
32. After t hours, the number of bacteria in a laboratory culture id given by n = 6t2 + 200. Use
differentials to approximate the change in the number of bacteria when t changes from 5 hours to
5 hours and 3 minutes.
33. Use differentials to approximate the change in volume of a sphere – shaped tumor

 V  4 / 3r  when its radius increases from 1 to 1.1 centimeters.


3

34. Leaking sand forms a conical pile in which the height is always thrice the radius (h = 3r). Consider
the moment when the radius is 9cm. Use differentials to determine the approximate change in the
volume when the radius changes by 1%, and volume of the cone is given by V = πr2h/3.
35. A fast food restaurant serves soft drinks in a cylindrical cup that has a radius of 1.5 inches.
The volume of soft drink that this cup can hold is given by V  r 2 h . Ordinarily, the restaurant
fills up to a height of 7.8 inches. Suppose that instead the restaurant decides to fill the cup to a
height of 8inches. Use differentials to approximate the number of cubic inches of soft drink the
restaurant spend on each serving.

-:139:-
CHAPTER SEVEN CALCULUS OF SEVERAL VARIABLES

CHAPTER
SEVEN CALCULUS OF
SEVERAL VARIABLES

INTRODUCTION
The idea of several variables was introduced in Chapter 5. The concept of partial derivatives of
first and higher derivatives was also discussed.
In this chapter, we shall study further properties and some applications of partial derivatives such
as differentials, extreme values in two variables, tangent and normal planes and chain rule.
Homogeneous Functions
A function f(x, y) is defined as a homogeneous function of x and y of degree n if it can be
expressed in the form f(λx, λy) = λn f(x, y) where   0,n  Q.
To test whether a function is homogeneous or not
Write λx for x and λy for y and if f(λx, λy) = λ f(x, y) then function f(x, y) is homogeneous of
n

degree n. For instance,


1. Let f (x, y)  (x 3  y3 ) / (x  y) . Replacing x by λx,tx and y by λy, we get

f  x, y   

3 3
 3 x 3   3 y3  x  y 
3
 2
x3  y3
  2f  x, y  
x  y   x  y xy
Thus, function f(x, y) is a homogeneous function of degree 2.
x 3y
2. Let f  x, y     cos y / x  ln x  ln y 
y 4x
Replacing x by λx and y by λy, we get
x 3 y x 3y
f  x, y     cos y / x  ln x  ln y    cos y / x  ln   ln x  ln   ln y
y 4 x y 4x
x 3y
f  x, y     cos y / x  ln x  ln y  f  x, y   t 0f  x, y  
y 4x
Thus, f(x, y) is a homogeneous function of degree 0.
y x
3. Let f  x, y   
yx
Replacing x by λx and y by λy, we get

f  x, y  
y   x

 y y 

 1/2 y  y 
  1/2f (x, y) 

y  x (y  x) (y  x)
Thus, f  x, y  is a homogeneous function of degree –1/ 2.

-:140:-
CHAPTER SEVEN CALCULUS OF SEVERAL VARIABLES
x 3  y3
4. Let f  x, y   sin
xy
Replacing x by λx and y by λy, we get

f  x, y   sin
 3 x 3   3 y3
 sin 

 3 x 3  y3    3
  sin   2 x  y 
3

x   y    x  y   x  y 
  
x 3  y3
Since f  tx, ty  is not of the form  2 sin , therefore, f  x, y  is not a homogeneous
xy
function.
xy
5. Let f  x, y   ln x  ln y 
xy
st
1 Method: Replacing x by λx and y by λy, we get
x  y   x  y
f  x, y   ln x  ln y   ln   ln x  ln   ln y  . Thus,
x  y   x  y
xy
f  x, y   ln x  ln y   f  x, y   0f  x, y 
xy
Therefore, f(x, y) is a homogeneous function of degree 0.
2nd Method: If f (x, y) is a homogeneous function of degree n, it can be expressed as:
 y
f  x, y    n g  
x
x 3  y3
For example, consider the function f  x, y   which is a homogeneous function of degree
xy
x 3  y3 x 3 (1  (y / x)3 ) y  y  (1  (y / x) )
3
2. Now, f  x, y     x 2g   where g    .
xy x(1  y / x) x x (1  y / x)
Therefore, given function is homogeneous function of degree 2.
EULER’S THEOREM ON HOMOGENEOUS FUNCTIONS
Statement: If u = f(x, y) is a homogeneous function of degree n, in x and y, then
u u
x + y = nu
x y
Proof: Since u = f(x, y) is a homogeneous function of degree n in x and y, therefore it can be
 y
expressed as: u  xn f  
x
Differentiating partially with respect to x, we get
u  y  y   y y y  y 
 nx n 1  f    x n  f        nx n 1  f    x n  f       
x x  x  x  x  x  x   x2 
u  y  y
 nx n 1  f    x n 2 y  f   
x x x
Multiplying both sides by x, we get
u  y  y u y
x  nx n  f    x n 1y  f     x  nu  x n 1y  f    (1)
x x x x x
Now differentiating partially w.r.t y, we get

-:141:-
CHAPTER SEVEN CALCULUS OF SEVERAL VARIABLES
u y  y   y  y 1  y
  0   f    x n  f        x n  f        x n 1  f   
y  
x  
x y  
x    
x x x
u y
Multiplying both sides by y, we get: y  x n 1y  f    (2)
y x
Adding (1) and (2), we get
u u y y u u
x  y  nu  x n 1  f     x n 1y  f    . Therefore, x y  nu.
x y x x x y
Note: Euler’s Theorem can be extended to a homogeneous function of any number of variables.
Thus, if f  x1, x 2 , x3 , , x n  be a homogeneous function of n variables x1 , x 2 , x 3 , , x n of
f f f f
degree n, then: x1  x2  x3   xn  nf
x1 x 2 x3 x n
 x 3 + y3  u u
Example 01: If u = tan 1   , show that x + y = sin2u
 x-y  x y
 
 x 3  y3  x 3  y3
Solution: We have, u  tan 1    tan u 
 xy  xy
 
 y3 
x 3 1  3 
 x 
2 1 y x
3
x 3  y3  
Let z  tan u   x
xy  y 1 y x
x 1  
 x 
Thus z is a homogeneous function of degree 2 in x and y. By Euler’s theorem, we have
z z
x  y  2z (1)
x y
Substituting the value of z in (1), we get

x

y
  u 

 u 

x  tan u   y  tan u   2  tan u   x sec 2 u    y sec 2 u    2 tan u
 x   y 
 u u  u u sin u
sec2 u  x  y   2 tan u  x y 2  cos 2 u
 x y  x y cos u
u u u u
x  y  2sin u cos u  x  y  sin 2u  using sin 2  2sin  cos  
x y x y
 x 2 + y2  u u
Example 02: If u = ln   , prove that x + y = 1.
 x+y  x y
 
 x 2  y2  x 2  y2
Proof: We have, u  ln    eu 
 xy  xy
 
 y2 
x 2 1  2 
 
 x   x1 y x
2
x 2  y2
Let z  eu  
xy  y 1 y x
x 1  
 x
Thus z is a homogeneous function of degree 1 in x and y. From Euler’s theorem we have
z z
x y z (1)
x y

-:142:-
CHAPTER SEVEN CALCULUS OF SEVERAL VARIABLES
Substituting the value of z in (1), we get

x
 u
x
 
e y
 u
y
   
 u   u 
e  eu  x eu    y eu    eu
 x   y 
 
 u u  u u
or eu  x  y   eu  x y 1
 x y  x y
Example 03: If U = f  x, y  is a homogeneous function of degree 10, prove that
x 2f xx + 2xyf xy + y2f yy = 90f  x, y 
Solution: Since f is a homogeneous function of degree 10, we have
xf x  yf y  10f (1)
Differentiating (1) with respect to x, we get
xf xx  f x 1  yf yx  10f x  xf xx  f x  yf yx  10f x (2)
Differentiating (1) with respect to y, we get
xf xy  yf yy  f y 1  10f y  xf xy  yf yy  f y  10f y (3)
Multiplying (2) with x and (3) with y and then adding, assuming f xy  f yx , we get

   
x xf xx  f x  yf xy  y xf xy  yf yy  f y  10xf x  10yf y

Or, 2 2

x f xx  xf x  xyf xy  xyf xy  y f yy  yf y  10 xf x  yf y 
x 2f xx  2xyf xy  y2f yy  10f  10 10f   x 2f xx  2xyf xy  y2f yy  100f  10f
or x 2f xx  2xyf xy  y2f yy  90f
TOTAL DIFFERENTIALS
We have studied the concept of the differentials of a function of one variable. For y = f(x) the
differential was defined as dy  f (x) dx with dx  x and dy  y  The differential dy was
used to approximate  y for small values of dx  We estimated such things as the change in
revenue associated with small changes in advertising expenditures and the change in price that
would cause a small change in demand. The differential concept can be extended to functions of
two variables by the following definition.
Definition: Let z = f(x, y) The total differential dz is defined as
f f
dz  f x  x, y  dx  f y  x, y  dy OR dz   dx   dy
x y
where dx  x,dy  y and dz  z and z  f  x  x, y  y   f  x, y  
Example 01: A rectangular plate expands in such a way that its length changes from 10cm to 10.5
cm and its breadth changes from 5 to 5.3 cm. Find, the approximate change in its area.
Solution: Let x and y be the length and breadth of the rectangular
.plate respectively. According to the question, we have
x  10cm,dx  0.5cm, y  5cm and dy  0.3cm 
A A
Since, Area = A = x . y, using total differentials, we get: dA  dx  dy  dA  ydx  xdy
x y
Substituting the values, we get: dA  (5)(0.5)  (10)(0.3)  dA  5.5 cm2 . This is the change in
the required area.

-:143:-
CHAPTER SEVEN CALCULUS OF SEVERAL VARIABLES
Example 02: A manufacturer of paper drinking cups decides to make its standard cup slightly
1
smaller than before. The cups are conical and hold volume V  r 2 h where r is the radius of
3
the top and h is the height (see figure). The radius will be changed from 1.5 inches to 1.4 inches,
and the height will be changed from 4 inches to 3.9 inches.
Use differentials to approximate the reduction in volume that results from these changes.
Solution: We have r  1.5in, dr   0.1in, h  4in, dh  0.1in
1
Now, V  r 2 h , therefore, using differentials, we get r
3
V V 1 1 1
dV  dr  dh  dV    2rh  dr   r 2  dh   r  2hdr  rdh  h
r h 3 3 3
Substituting the given values, we get
1
dV   3.14 1.5 2  4  0.1  1.5  0.1  1.57  0.8  0.15  dV  1.4915
3
Thus, negative sign indicates the reduction in volume. Thus, approximately 1.4915in 3 volume
will be reduced.
Example 03: Approximate the change in the hypotenuse of a right triangle of legs 6 and 8 units,
when the shorter leg is increased by 1/2 units, the long leg is decreased by 1/4 units.
Solution: Let x and y be the length of shorter and long leg respectively. According to the
conditions, we have: x  6, dx  0.5, y  8 and y   0.25
Since, z2  x 2  y2  Pythagoras theorem (1)
z 2   6   8  z 2  100  z  10 units
2 2
B

Using total differentials (1) gives B` z


2z dz  2x dx  2y dy  z dz  x dx  y dy
Substituting the values, we get y=8 z`
10dz  6  0.5  8  0.25 or 10dz  3  2
or dz  1/ 10 giving dz  0.1units
Hence, the approximate change in the hypotenuse is 0.1unit. A x=6 C C`
[NOTE: ABC is the position of original right triangle and AB`C` is its position after the changes
occur in its two sides where the point A is fixed]
MAXIMA AND MINIMA OF A FUNCTION OF TWO VARIABLES
We began our search for extreme values of functions of one variable by finding critical numbers.
Now with functions of two variables, we will seek critical points (a,b) since we want the x and y
values needed to maximize or minimize a function z = f(x, y) The definition given next is an
extension of the definition of critical number in case of functions of a single variable.
Critical Point
Let z = f(x, y) be defined at point (a, b). Then, (a, b) is a critical point of f(x, y) if f x  a, b   0
and f y  a, b   0 
With functions of one variable, the derivative is the slope of the tangent line. With functions of
two variables, there are two tangent lines to be considered. The partial derivatives f x and f y are

the slopes of these two tangent lines. For (a, b) to be a critical point, we insist that f x  x, y  and

-:144:-
CHAPTER SEVEN CALCULUS OF SEVERAL VARIABLES
f y  x, y  both be zero at (a, b). There must be two horizontal lines at  a, b  one parallel to the
x-axis and the other one is parallel to the y-axis.
Example 01: Find the critical points of f  x, y  = 3x3 + y2 – 6x + 2y
Solution: First, we find the partial derivatives: f x  x, y   9x 2  36,f y  x, y   2y  10 
Next we solve the system of equations created by setting each partial derivative equal to zero. In
other words, solve the system: 9x2 = 36 and 2y – 10 = 0.
This gives: x = ± 2, y = 5. Thus critical points are (– 2, 5) and (2, 5).
Now we will study the rules to check that whether a critical point is maximum or minimum.
A critical point that is neither maximum nor minimum is called the saddle point for z = f(x, y).
There is a second derivative test to determine whether a critical point is associated with a
maximum, minimum or neither.
Second Partial Derivative Test
For z = f(x, y) if f x  a, b   0 and f y  a, b   0, then consider D = f xx . f yy  (f xy )2
to be evaluated at (a, b)
 i  If f xx < 0 and D > 0, the  a, b  Þ maximum.
 ii  If f xx > 0 and D > 0, then  a, b  Þ minimum.
 iii  If D < 0, the  a, b  is a saddle point.
 iv  If D = 0, then the test is inconclusive.
The expression D = f xx . f yy  (f xy )2 is called the discriminant of f. It is sometimes easier to
remember the determinant form:
f xx f xy
f xx .f yy  (f xy ) 2 =
f xy f yy
Note: As an aid in remembering the second partials test, consider that between D > 0 the test for
maximum or minimum comes down to whether f xx  0 (minimum) or f xx  0 (maximum),
which resembles the second derivative test for functions of one variable.
Example 02: Find the maximum and minimum values of each of the following functions, if there
are any: (i)f  x, y  = x 2 + xy  y2  2x  2y + 3 (ii)f  x, y  = x 2 + 3xy  y2 + 4y  6x +1
Solution: (i) We have f(x, y) = – x2 + xy – y2 – 2x – 2y + 3
Therefore, f x  x, y   2x  y  2, f y  x, y   x  2y  2
Now, 2x  y  2  0 (1) x  2y  2  0 (2)
Solving (1) and (2), we get: x = – 2, and y = – 2. Thus, (– 2, –2) is the only critical point for given
function. Now fxx = – 2, fyy = – 2 and fxy = 1. At (– 2, –2), fxx = – 2, fyy = – 2 and fxy = 1.
D = f xx .f yy  (f xy ) 2 = ( 2)( 2)  12 = 3. Thus, D > 0 and fxx = – 2 < 0. Therefore, (– 2, – 2) is
maximum point for this function. The maximum functional value is:
f(– 2, – 2) = –(– 2)2 + (–2)( –2) – (–2)2 –2(–2) – 2(–2) + 3 = –4 + 4 – 4 + 4 + 4 + 3 = 7
(ii) We have f(x, y) = x2 + 3xy – y2 + 4x – 6y + 1
Therefore, f x  x, y  = 2x + 3y  6, f y  x, y  = 3x  2y + 4
Now, critical values 2x  3y  6  0 (1) 3x  2y  4  0 (2)

-:145:-
CHAPTER SEVEN CALCULUS OF SEVERAL VARIABLES
Solving (1) and (2), we get: x = 0 and y = 2. Thus, (0, 2) is the only critical point for the given
function. Now, at (x, y) = (0, 2), fxx = 2, fyy = – 2 and fxy = 3, so that
D = fxx.fyy – fxy2 = (2)( –2) – (3)2 = – 4 – 9 = – 13 which is negative. Therefore, (0, 2) is a saddle
point of given function.
Example 03: A local company advertises on the radio and in the newspaper. Let x and y represent
the amounts (in thousands of dollars) spent on the radio and newspaper advertising, respectively.
The company’s profit based on this advertising has been determined to be (in thousands of
dollars) P(x, y) = –2x2 – xy – y2 + 8x + 9y + 10. How much money should the company spend on
each type of advertising in order to maximize the profit?
Solution: We have P(x, y) = – 2x2 – xy – y2 + 8x + 9y + 10 = 0 (1)
Differentiating partially equation (1) with respect x and y respectively, we get
Px   4x  y  8, Py  x  2y  9
Solving simul tan eously the equations :  4x  y  8  0 (i) and  x  2y  9  0 (ii)
we get, x = 1 and y = 4. Thus, (1, 4) is the only critical point for the given profit function.
Now, Pxx = – 4; Pyy = – 2; Pxy = – 1
Therefore, at (1, 4), D = (Pxx)(Pyy) – (Pxy)2 = (– 4)( –2) – (–1)2 = 8 – 1 = 7
Thus, D > 0 and Pxx = – 4 < 0, therefore (1, 4) is maximum point for the given function. This
means that the company should spend $1000 on the radio and $4000 on newspaper so as to
increase its profit. In this case, the profit will be
P(1, 4) = – 2(1) – (1)(4) – (4) + 8(1) + 9(4) + 10 = – 2 – 4 – 16 + 8 + 36 + 10 = $ 32, 000.
2 2

Example 04: Find the three positive numbers that satisfy both of the following conditions
(i) Their sum is 27.
(ii) The sum of the squares of the numbers is as small as possible.
Solution: Let x, y and z be the three required positive numbers. According to the first condition,
x  y  z  27 or z  27  x  y (1)
According to the second condition, we have
f (x, y)  x 2  y 2  z 2 (2)
Substituting the value of z from equation (1) into equation (2), we have
f (x, y)  x 2  y 2  (27  x  y) 2
Simplifying, we get f  x, y   2x 2  2y2  2xy  54x  54y  729 (3)
Differentiating partially the equation (3) with respect to x and y partially, we get
f x (x, y)  4x  2y  54, f y (x, y)  4y  2x  54
Solving the equation 4x + 2y – 54 = 0 and 2x + 4y – 54 = 0, we get x = 9 and y = 9.
Thus, (9, 9) is the only critical point for the function. To apply the second partials test, we must
evaluate D for the point (9, 9). Here, f xx = 4,f yy = 4 ,f xy = 2. Now,
D = (fxx)(fyy) – (fxy)2 = ( 4)(4) – (2)2 = 16 – 4 = 12.
Thus, D > 0 and fxx = 4 > 0. Therefore, (9, 9) is the minimum point for the given function.
Substituting x  y  9 into (1) we obtain z  27  9  9  9 .
Hence, those three required positive numbers are: (9, 9, 9).
-:146:-
CHAPTER SEVEN CALCULUS OF SEVERAL VARIABLES
Example 05: A rectangular cardboard box (with a top) is being made to contain a volume of 27
cubic feet. Find the dimensions that will minimize the amount of material used to make the box.
Solution: Let the base of the box be x cm by y cm and its height be z cm.
Then xyz  27  z  27 / xy (1)
The surface area S of the box is given by
 27 27   27 27 
S  2  xy  xz  yz   2  xy  x   y    2  xy   
 xy xy   y x 
We need to find the minimum value of S. Therefore
 27   27   54  108  54  108
Sx  2  y   , Sy  2  x   , Sxx  2    , Syy  2    , S  2.
 x2   2
 3
 3  y3  y3 xy
 y  x x  
 27  27  27 
For critical value, Sx  0  2  y   0y and S y  0  2  x  0
 x2  x2  y 2 

27 27 x4 x4 x4
x . Putting y  27 / x 2 , we get x   27   x 0
y2 (27 / x) 2 27 2 27 27
 27x  x 4  0 or x(27  x 3 )  0  x  0 or 27  x 3  0  x  0, 3.
But x  0 is not admissible, thus, x  3. This gives,
 108  108 
  2   12  0
2
y  27 / 32  27 / 9  3. Now, at (3, 3), D  (Sxx )(S yy )  (Sxy ) 2  
3  3 
 3  3 
Since, D > 0 and Sxx > 0, therefore S has a minimum value at x  y  3  Also,
z  27 / (xy)  27 / (3  3)  27 / 9  3
Hence, the required dimensions of the box are 3 feet by 3 feet by 3 feet.
Example 06: An open rectangular box is being made to contain a volume of 108 cubic feet. Find
the dimensions that will minimize the amount of material used to make the box.
Solution: Let the length, width and height of the open rectangular box be x, y
and z feet respectively. The surface area of the box is:
S  xy  2xz  2yz and V  xyz  108 ft 3  z  108 / (xy)
108 108 216 216
Thus, S  xy  2x   2y   xy   (1)
xy xy y x
We need to find the minimum value of S. Therefore
216 216 432 432
Sx  y  ,Sy  x  ,Sxx  ,Syy  and Sxy  1
2 2 3
x y x y3
216 216 216
For critical points, we set: Sx  0  y  2  0  y  2 and Sy  0  x  0
x x y2
Putting y = 216/x2 in the 2nd equation, we get
2
 216  x4 x4
x  216    x  216   x   216x  x 4  0  x(216  x 3 )  0
2 
 x   216  2 216

 x  0 or x 3  216. Now, x  0 is not admissible, thus x 3  216  63  x  6


Put this value of x in the equation of y, we get y  216 / 62  216 / 36  6

-:147:-
CHAPTER SEVEN CALCULUS OF SEVERAL VARIABLES
 432  432 
 1  3  0
2
Now at (6, 6); D = Sxx .Syy  Sxy 2  
3  3 
 6  6 
Since D > 0 and Sxx > 0 at (6, 6), therefore S has a minimum value at x = y = 6. Now,
z  108 / (6  6)  3 . Hence, the required dimensions of the box are 6 feet, 6 feet and 3 feet.

WORKSHEET 07
1. Verify the following using Euler’s Theorem:
x y u u
(i) If u  sin 1  tan 1 , prove that x y 0
y x x y
xy u u 1
(ii) If u  sin 1 , prove that x y  tan u
xy x y 2
x 2  y2 u u
(iii) If u  sin 1 , prove that x y  tan u
xy x y
 x 3  y3  u u
(iv) If u  sec1   , show that x y  2cot u
 xy  x y
 
xy u u 1
(v) If u  cos1 , prove that x y   cot u
xy x y 2

 
(vi) If u  ln x 2  xy  y 2 , prove that x
u
x
y
u
y
2

2. The area of a triangle is given by A  0.5 ab sin C . When a = 20 cm, b = 30 cm and


 C = 30o, find
(a) The rate of change of A with respect to a, when b and C are constant.
(b) The rate of change of A with respect to b, when a and C are constant.
(c) The rate of change of A with respect to C, when a and b are constant.
3. Determine whether or not the following functions satisfy the Laplace equation Zxx + Zyy = 0
(i) Z = ex. cosy (ii) Z = 0.5 ex + y (iii) Z = x2 – y2.


4. In a certain electrostatic field, the potential is u   x  1  y2   z  2 
2

2 1 2
Find the rate of
change of u in the positive x, y, and z directions, respectively, at the point (2, -2, -4).
5. If resistors of R1, R2 and R3 ohms are connected in parallel to make an R ohm resistor, the
value of R can be found from the equation:
1 1 1 1
   . Find the value of R / R 3 , where R1  20, R 2  35, and R 3  50 
R R1 R 2 R 3
6. A thin metal plate is being heated in such a way that the temperature T at any point (x, y) on
the surface of the plate is given by: T  x, y   350  x 2  y2 degrees,
where x and y are measured in centimeters. What is the temperature at the origin?
(a) On the basis of the function T, why will the temperature be greatest at the origin?
(b) Determine the rate of change of temperature with respect to distance at the point (10, 6)
assuming x can vary and y is held constant?
(c) Determine the rate of change of temperature with respect to distance at the point (10, 6)
assuming y can vary and x is held constant.
(d) Explain why the rates of change in parts (c) and (d) are negative.

-:148:-
CHAPTER SEVEN CALCULUS OF SEVERAL VARIABLES
7. Concern about body heat loss led to the development of a formula for measuring the surface
area S of a person’s body on the basis of the individual’s weight w in kilograms and height h in
centimeters: S w,h   0.0072w0.425h0.725
(a) Determine S(80, 178) (b) Find Sw (w, h) and Sh (w, h)
(c) Find Sw (w, h) and Sh (w, h) at (80,178) (d) Explain the meaning of Sw and Sh
8. Find the approximate change in the hypotenuse of a right triangle of legs 6 and 8 inches when
the shorter leg is extended by 1/4 inch and the longer leg is condensed by 1/8 inch.
9. The dimensions of a rectangular block of wood were found to be 10, 12, and 20 inches, with a
possible error of 0.05 in each of the measurements. Find (approximately) the greatest error in the
surface area of the block and the percentage error in the area caused by errors in the individual
measurements.
10. Two sides of a triangle were measured as 150 ft and 200 ft, and the included angle is of 60 
If the possible errors are 0.2ft in measuring the sides and1o = π/180 in the angle, what is the
greatest possible error in the computed area?
11. The altitude of a right circular cone is 15 inches and is increasing at 0.2 in/min. The radius of
the base is 10 inches and is decreasing at 0.3 in/min. How fast is the volume changing?
12. At a certain instant the radius of a right circular cylinder is 6 inches and is increasing at the
rate 0.2 in/sec while the altitude is 8 inches and is decreasing at the rate 0.4 in/sec. Find the time
rate of change (a) of the volume and (b) of the surface, at that instant.
13. Divide 120 into three non – negative parts such that the sum of their products taken two at a
time is a maximum.
14. Show that a rectangular parallelepiped of maximum volume V with constant surface area S is
a cube.
15. A rectangular box, open at the top, is to have a volume of 32 cubic centimeters. Find the
dimensions of the box requiring least material for its construction.
16. A manufacturer of aquarium wants to make a large rectangular box – shaped aquarium that
will hold 64 ft3 of water. If the material for the base costs $20 per square foot and the material for
the sides costs $10 per square foot, find the dimensions for which the cost of the materials will be
the least.
17. An open rectangular box is being made to contain a volume of 108 cubic feet. Find the
dimensions that will minimize the amount of material used to make the box.

-:149:-
CHAPTER EIGHT ANTIDERIVATIVES

CHAPTER
EIGHT INTIGRATION AND
IT’S TECHNIQUES

INTRODUCTION
At the beginning of this book you were told that there are two branches of calculus, that is;
Differential Calculus and Integral Calculus. In differential calculus we begin with a function f(x)
and obtain the derivative f   Interpreting f  as a rate of change of f(x) led to a variety of
applications. By contrast, there are situations in which we know the rate of change and seek the
function f(x). We need to be able to reverse the differentiation process in such cases. In other
words, in differential calculus we are given a function and we are required to find its derivative,
while in integral calculus we are required to find the function whose derivative is given. This
process is depicted as under:
Differentiation process
f (x) f (x) = F(x)
Anti derivative/Integration process
f (x) f `(x) = F(x)
Thus integration is also known as anti-derivative. For example, if
3 2 2 3
f(x) = x then f ` (x) = 3x , thus anti-derivative of (3x ) = x
d
Definition: If f(x) is a differentiable function such that F(x)  f (x), then F(x) is called an
dx
integral or anti – derivative of f(x) and is written as  f  x  dx  F  x  . The symbol dx indicates
that integration is performed with respect to x.
Integration: The process of finding the integral of a function is called integration.
Integrand: The function to be integrated is called the integrand.
For example, if  f (x)dx  F(x) then f(x) is the integrand.

Integral sign: The symbol `  ` is used to denote the “sign of integration or integral sign” and is
used to represent the process of integration. The mathematician Leibniz introduced this symbol.
Constant of Integration and Indefinite Integral
d d d d
Let F(x)  f (x), then F(x)  c  F(x)  c  F'(x)  0  f (x)
dx dx dx dx
Therefore,  f (x) dx  F(x)  c
The arbitrary constant ‘c’ is called the constant of integration. Since  f  x  dx  F  x   c
involves an arbitrary constant ‘c’ that is why it is called the indefinite integral of f(x) because the
value of ‘c’ is not definite.
-:150:-
CHAPTER EIGHT ANTIDERIVATIVES
2
Example 01: Find the anti – derivative of f(x) = 3x .
2
Solution: We need to determine the function F(x) that yields f(x) = 3x when differentiated. It
does not take long to realize that the derivative of x is 3x . So,  3x 2 dx  x 3
3 2

d 3 d 3 d 3
But it is also true that (x  1)  3x 2 , (x  4)  3x 2 , (x  2)  3x 2
dx dx dx
d 3
Generalizing, we see that (x  c)  3x 2 for any constant `c`.
dx
In view of this, we say that the  3x 2 dx  x 3  c
NOTE: If f(x) and g(x) are any functions of the variable x, then following properties are always
true.
(i)  c1f  x   c2g  x  dx  c1  f  x  dx  c2  g  x  dx (ii)  k dx  k x  c
provided the anti-derivatives of f(x) and g(x) exist. Here c1 and c2 being constants.
Integral of a constant:  k dx  k x  c , c being constant of integration.
METHODS OF INTEGRATION
There is no uniform technique to find integral of a given function. Several methods have been
developed to find the integrals. You learn these methods gradually. To start with there are four
major techniques.
1. Integration by formulas 2.Integration by substitution
3. Integration by parts 4. Integration by partial fraction
Integration by Formulas
The integral of a given function may be found through direct formulas.
Power Rule of integration
x n 1
 x n
dx   c,  n  1 and 1.dx  x  c
n 1
To integrate a power of x, increase the exponent by 1 and then divide by the new exponent.
x3
 x dx 
2
Example 01: c
3
3x3 2x 2
Example 02:  (3x 2  2x  4) dx  3 x 2 dx 2 x dx  41dx    4x  c
3 2
 x3  x 2  4x  c
Note 1: In the power rule, if x is replaced by x + a (a being a constant), then similar type of result
holds. Thus,
(x  a)n 1 (x  2)4
 (x  a) dx 
n
 c,  n  1 . For example,  (x  2)3 dx  c
n 1 4
Note 2: If x be replaced by ax + b (a and b being constants) on both sides of the power rule, the
standard form remains true. The result on the right side is divided by ‘a’ the coefficient of x. That is,
(a x  b)n 1
 (a x  b) dx 
n
 c,  n  1
n 1
(3x  2)4 (3x  2)4
For example,  (3x  2)3 dx  c c
3.4 12

-:151:-
CHAPTER EIGHT ANTIDERIVATIVES
Integrals of the form  [f (x)]n .f `(x) dx

d  [f (x)]n 1 
Since   c   [f (x)]n .f `(x), n  1
dx  n  1 

n 1
 F  x  
 f  x  f   x  dx  n  1  c, provided n  1
n
Therefore,
This is known as the general power rule of integration.
4
Example 03: Evaluate   x 2  x 
 
 2x  1 dx
d 2
Solution: Here the function x2 + x having power 4, together with derivative (x  x)  2x  1 ,
dx
5
x2  x 
so using above formula, we get:  x  x  2x  1 dx    c
4
 2 
  5
Example 04: Evaluate  (x 2  2x) 4 (x  1) dx
2
Solution: Here the derivative of x + x is not present in multiplication, but
d 2
(x  2x)  2x  2  2(x  1)
dx
We observe that the constant 2 is missing, so re-writing the above integral as
1 (x 2  2x)5
 (x  2x) (x  1) dx  2
2 4 2 4
(x  2x) (2x  2) dx dx  c
5
f (x)
Integrals of the form  dx  ln f (x)  c
f (x)
If the derivative of the denominator is present in numerator, then integral will be
ln (denominator) + c. You should keep in mind that
  f (x) n 1
  c, provided n  1
 f (x) f (x) dx   n  1
n

ln f (x)  c, provided n  1


  
2x  1
Example 05: Evaluate  2 dx
x x
2
Solution: Here derivative of x + x is 2x +1 is in the numerator, hence
2x  1
 x 2  x dx  ln(x  x)  c
2

x 1
Example 06: Evaluate  2 dx
x  2x
2
Solution: Here the derivative of x + 2x is; 2x + 2 is not in numerator, but if we multiply and
divided by 2, then, we get:  2
x 1
x  2x
1 2x  2
dx   2
2 x  2x
1
dx  ln x 2  2x  c
2
 
Integrals of Exponential functions
ax
(i) We know that therefore  e dx  e  c and  a dx 
x x
c x
2 ln a
 f x 
(ii) Since, d  ef  x   c   ef  x   f   x  and d  a  c   af  x   f  x 
dx   dx  ln a 
 

-:152:-
CHAPTER EIGHT ANTIDERIVATIVES
a f (x)
 e .f (x) dx  e  c  a .f (x) dx  c
f (x) f (x) f (x)
Therefore, and
ln a
e dx (ii)  esin x cos x dx
5x
Example 07: Evaluate (i)
Solution: (i)  e5x dx  c

e .f (x) dx  ef (x)  c , hence


f (x)
(ii) Here we observe that function is of the type

e .cos x dx  sin x  c
sin x

1
e tan x
Example 08: Evaluate  dx
1  x2
e .f (x) dx  ef (x)  c
f (x)
Solution: The integral is of the form
1
e tan x
 
1  d 1 1 
Therefore,  1  x2 dx  e tan x  c  dx tan x 


1  x2 
Integration with the use of trigonometric identities
The trigonometric identities are sometimes helpful for integrating certain functions. First you may
remember the following formulas:
 i   sin x dx  cos x  c  ii   cos x dx  sin x  c  iii   tan x dx  sec x  c
 iv   cot x dx  csc x  c  v   sec x dx  ln  sec x  tan x   c
 vi   csc x dx   ln  csc x  cot x   c
cos mx
It may be noted that  sin mx dx   c
m
Thus if any trigonometric function appears with multiple angle, then above six rules are
applicable with division by the multiple factor.
Example 09: Integrate the following functions
2 2 2 2
(i) sin x (ii) cos x (iii) sec x csc x

1  cos 2x  1 1 sin 2x 
Solution: (i)  sin 2 x dx   dx   1dx   cos 2x dx    x  c
2 2   2 2 
1  cos 2x  1 1 sin 2x 
(ii)  cos2 x dx   dx   1dx   cos 2x dx    x  c
2 2  2 2 

1  1  1
(iii)  sec2 x csc2 x dx   dx  4   dx   4  dx
2 2
cos x.sin x   2sin x cos x  2   sin 2x  2
 
  cot 2x 
 4 csc2 2x dx  4    c  2cot 2x  c
 2 
Integration by Substitution
Almost this technique is used when we observe that given function is the product or quotient of
two functions where the derivative of one function is present either in multiplication or in
quotient form.

-:153:-
CHAPTER EIGHT ANTIDERIVATIVES
Note: There are no hard and fast rules for making suitable substitutions. Experience is the best
channel in this matter. If after a substitution the new integrand becomes more complicated, then
either some other substitution should be tried or other methods need to be employed.
n
Example 10: Evaluate  (ax 2  2bx  c)  ax  b  dx
d
Solution: Here we observe that derivative (ax 2  2bx  c)  2  ax  b  is in multiplication
dx
form without constant 2, so let
dz dz
z  (ax 2  2bx  c) so that  2  ax  b  giving   ax  b  dx
dx 2
n n z n 1 (ax 2  2bx  c) n 1
Thus,  (ax 2  2bx  c)  ax  b  dx   z dz   c
2  n  1 2  n  1

Example 11: Evaluate  1  cos 2 x sin 2x dx


2
Solution: Here we see that derivative of 1 + cos x is – 2sin x cos x = – sin2x is present in the
main function, so let
dz
z  1  cos2 x so that  2sin x cos x   sin 2x giving  dz  sin 2x dx
dx
1/2 z3/2 2(1  cos2 x)3/2
Thus,  1  cos 2 x sin 2x dx   z ( dz)  
3/ 2

3
c

1
Example 12: Integrate
x ln x
dz 1 dx
Solution: Putting, z  ln x    dz 
dx x x
dx 1
Therefore,  x ln x   z dz  ln z  ln  ln x   c
Some well – known substitutions
The following substitutions are generally helpful to transform the integrand to an easier from.
If the integrand contains Make the substituion

a2  x2 x = a sin θ cos θ

a2  x2 x = a tan θ or x = a sinh θ

x2  a2 x = a sec θ or x = a cosh θ
2
ax  b ax + b = z
s
(ax  b) r/s ax + b = z

Note: If the integrand contains some linear expression of the variable, put that linear factor equal
to say z.If the integrand is a rational function of ex put z = ex. If the integrand contains a logarithm
of f(x) or an exponential function in which the index is f(x) put z = f(z).

-:154:-
CHAPTER EIGHT ANTIDERIVATIVES
dx
Example 13: Evaluate
 1+ x +1
Solution: Let x  1  z so that z 2  x  1  2zdz  dx and on substitution, we have
dx 2z z  1   1 
 1+ x +1
=
1+ z
dz = 2
1+ z
dz = 2 1   dz = 2  1dz  
 1+ z  
dz 
1+ z 

 1+
dx
x +1  
= 2  z  ln 1+ z   = 2  x +1  ln 1+ x +1  + c
 
dx
Example 14: Evaluate 
x2 + 4
dx dx x
Solution: Let I =  = = sinh 1 +c
x +4 2
x +2 2 2 2
Example 15: Evaluate the following integrals
(i)  tan x dx
sin x
 tan x dx   cos x dx. Putting z  cos x  dz   sin x dx. Thus,

 tan x dx    z dz   ln z  ln  z   ln  cos1 x   ln sec x   c


1 1

(ii)  cot x dx
cos x
 cot x dx   sin x dx. Putting z  sin x  dz  cos x dx. Thus,

1
 cot x dx   z dz  ln z  ln  sin x   c
(iii)  sec x dx
(sec x  tan x)
 sec x dx   sec x (sec x  tan x) dx
Putting z  secx  tan x  dz  (sec 2 x  secx tanx) dx
1
  sec x dx   dz  ln z  ln  sec x  tan x   c
z
(iii) Evaluate  csc x dx
Multiplying and dividing by (csc x  cot x), we obtain
(csc x  cot x)
 csc x dx   csc x (csc x  cot x) dx
Putting z  cscx  cot x  dz  (csc 2 x  csc x cotx) dx
1
  csc x dx    dz   ln z   ln  csc x  cot x   c
z
1 x
(iv)  1 x
dx

Multiplying & dividing by 1  x , we obtain


1 x 1 x dx x. dx 1 (1  x 2 ) 1/2 ( 2x)
 dx      sin 1 x 
2
dx   dx
1 x 1  x2 1  x2 1  x2 1  x2

-:155:-
CHAPTER EIGHT ANTIDERIVATIVES

 
2 1/2
1 x 1 1 1 x
 dx  sin x   sin 1 x  1  x 2  c
1 x 2 1/ 2
1
(v)  dx
k  mx  n
Putting z 2  mx  n  2z dz  m dx, we obtain

k
dx

2 z dz

1
 
2z
dz 
1
 
ln z 2  k  c
mx  n
 k
m kz 2 m 2 m
 z2
1

1  x  tan 1 x
(vi) dx
2

Putting z  tan 1 x  dz  1 / (1  x 2 ) dx
1 dz
 dx    ln z  ln (tan 1 x)  c
2 1
(1  x ) tan x z
sin x  cos x
(vii)  sin x  cos x dx
Putting z  sin x  cos x  dz  (cos x  sin x) dx
sin x  cos x dz
 dx    ln z  ln  sin x  cos x   c
sin x  cos x z
sin x
(viii)  x
dx

Putting z  x  dz  dx / 2 x  2 x dz  dx or 2z dz  dx
sin x z.sin z dz
 dx  2   2  sin z dz  2cos z  2cos x  c
x z
1
(ix)  ex  e x dx
1 ex
 e x  e x dx  
e 2x
1
dx. Putting z  e x  dz  e x dx


e ex
1
x
dx  
2
z 1
dz
 
 tan 1 z  tan 1 e x  c

e 2x
(x)  dx
ex  1
Solution : Putting z  e x  1  z 2  (e x  1)  e x  1  z 2  e x dx  2z dz
2z dz 2z dz
Now, dx   dx 
x
e 1  z2

1  z 2  2z.dz
2
 z3  2
 2  1  z 2  dz  2  z    z 3  z 2 
2x
e
 dx  
ex  1 z 1  z 2   3  3  

2 ex  1  2 ex  1 
 3  ex  1  2  ex   c
3   3  

-:156:-
CHAPTER EIGHT ANTIDERIVATIVES

x2  a2
(xi)  x4
dx

Putting x = a secz then x 2  a 2  a 2  sec 2 z  a tan z


Also, dx  a sec z tan z dz

x2  a2 a tan z.a sec z tan z dz 1 tan 2 z


 dx     dz
x4 a 4 sec 4 z a 2 sec3 z
1 sin 2 z 1
 cos2 z .cos  sin
3 2
 z dz  z.cos z dz
2
a a2
Putting u  sin z  du  cos z dz

x2  a2 1 1 u3 1
 x4  u du 
2
 dx  .  sin 3 z  c (1)
2 2 3 2
a a 3a
Now, x  a sec z  sec z  x / a   Hyp. / Base 

Thus, Perp  Hyp 2  Base 2  x 2  a 2

Thus, sin z  Perp. / Hyp  x 2  a 2 / a. Thus equation (1) becomes


3
1  x 2  a 2 
x2  a2
 
3/2
  1 x2  a2
 x4 dx 
3a 2  a  3a 5
c

1
(xii)  dx
2x 2  3x  4
Consider,

 3   3
2
3 3
2   3
2
23 
2x 2  3x  4  2  x 2  x  2   2  x 2  x        2   2  x    
   4 4 

2 2 4  16 
  
 3 23 23
Putting  x    z  dx  dz. Also putting  a 2 so that a  .
 4 16 4
1 1 1 1 z 1  4x  3 
 dx   dz  sinh 1    sinh 1  c
2x 2  3x  4 2 z2  a 2 2 a 2  23 
Integration by Parts
Consider the product rule of differentiation from integration point of view leads to another
method of integration called integration by parts. The derivative formula for a product of two
functions of x, called u and v, is given by

 u.vdx  u  vdx    u.  vdx   dx


d dw du
Derivation: Consider  u.w  = u. + w. (1)
dx dx dx
Let, dw / dx  v so that that w =  v dx . Thus equation (1) become
d du du
 u.w  = u.v+ w. or uv = d  uw   w
dx dx dx
du
Integrating both sides, we get :  uvdx  uw   .w dx
dx
-:157:-
CHAPTER EIGHT ANTIDERIVATIVES
Putting w   vdx, we obtain :  uvdx  u  vdx   u  vdx  dx
This formula is known as integration by parts. The integrand consists of two function u whose
derivative and v whose integral are known. The following few examples will help you to
understand how to choose u and v functions.

 x.e x  x.sin 2 x dx
x 2 2x
Example 15: Evaluate (i) dx (ii) .e dx (iii)

Solution: This integral cannot be evaluated by any of the formulas we have studied so far. It will
be evaluated using the formula of integration by parts. One part of the integral must be chosen as
u, and other one is v. Using the above given formula, we get

 
(i)  x.e x dx  xe x   1.e x dx  xe x  e x  c  e x  x  1  c Taking u  x and v  e x 
 
(ii)  x 2 .e x dx  x 2e x    2x.e x  dx  x 2e x  2  x.e x dx  x 2e x  2 e x  x  1   c
 
 e x  x 2  2x  2   c  Use (i) and taking u  x 2 and v  e x 
 
  cos 2x    cos 2x  x cos 2x sin 2x
(iii)  x.sin 2 x dx  x  2
  1. 
  2
 dx  
 2

4
c

NOTE: Taking u = x and v = sin 2x


Guidelines:
(i) If one factor of the product is a power of x, take it as the first function u provided the
integral of the second function is known.
(ii) If, the integral of the second function is not known such as the inverse circular
functions or a logarithmic function, in that case, take that function as u.
(iii) If the integral is a single inverse circular or logarithmic function, take that function as
u and “1” as the second function.
Example 16: Evaluate the following integrals
x  1  x ln x 
(i)  x 2 .tan 1 x dx (ii)  ln x dx (iii) e   dx (iv) e
ax
sin  bx  c  dx
 x 
x3 1 x3
Solution : (i)  x 2 .tan 1 x dx  tan 1 x.  . dx Taking u  tan 1 x, v  x 2 
3 1 x 2 3  

x 3 tan 1 x 1  x   x3 x 
   x   dx  NOTE : x 
3 3  1  x2   1  x2 1  x 2 

x 3 tan 1 x 1  x 2 ln 1  x
2
    c 
 NOTE :  x  dx 
ln 1  x 2   

3

3 2

2     1  x 2  2 
   
1
(ii)  ln x dx   ln x.1dx  ln x.(x)   x .x dx  NOTE : Taking u  ln x & v  1

 x ln x  1dx  x ln x  x  x  ln x  1  c

 1  x ln x  x1 x  x ln x  x1
e  dx   e   dx   e   dx   e   dx   e  ln x  dx
x x
(iii) 
 x  x  x  x
 e x .ln x   e x .ln x dx   e x .ln x dx  e x .ln x  c [NOTE : Integrat ing 1st by parts with]

-:158:-
CHAPTER EIGHT ANTIDERIVATIVES
 1  x ln x 
  ex   dx  e .ln x  c
x
[u  e x & v  1/ x and keep the 2nd int egral same]
 x 
(iv) Let I   eax sin  bx  c . Taking u  eax and v  sin(bx  c), we get
  cos(bx  c)  ax  cos(bx  c) 
 eax .     ae    dx
 b   b 
eax cos(bx  c) a ax
I   e cos  bx  c  dx
b b
Integrating by parts once again with u  eax & v  cos(bx  c), we obtain
eax cos(bx  c) a  eax sin(bx  c) 1 
I     aeax sin  bx  c  dx 
b b  b b 
eax cos(bx  c) aeax sin(bx  c) a2 eax cos(bx  c) aeax sin(bx  c)
   I I 
b b2 b2 b b2
 b 2  a 2  eax b cos(bx  c)  aeax sin(bx  c)
I 
 b2  b2
 
 I(a 2  b 2 )  eax [a sin(bx  c)  b cos(bx  c)]
eax
Thus, I   eax sin  bx  c  dx  a sin(bx  c)  b cos(bx  c)
(a 2  b 2 )
eax
Similarly,  eax cos  bx  c   a cos(bx  c)  bsin(bx  c)
(a 2  b 2 )

x n 1 tan 1 x 1 x n 1
Example 17: Show that  x n .tan 1 x dx 
n 1  1  x2
 dx
n 1

Hence evaluate  x 3 .tan 1 x dx


-1 n
Solution: Integrating by parts taking u = tan x and v = x , we obtain

1 x n 1 tan 1 x 1 x n 1
 x .tan x dx  n  1  1  x2
n
 dx (Pr oved)
n 1
Replacing n by 3, we have
x4 x4 1  2 1 
  
1 1
x 3 tan 1 x dx  tan 1 x  dx  x 4 tan 1 x   x 1   dx
2
4 4 1 x 4 4  1  x2 
x4 x4 1  x3 

1 1
 tan 1 x  dx  x 4 tan 1 x    x  tan 1 x   c
4 4 1  x2 4 4  3 
Integration of Rational Algebraic Functions
An expression of the type R(x) = N(x)/D(x) where both N(x) and D(x) are polynomials with real
coefficients is called a rational function. In this section we perform the integration of such
functions by the method of partial fractions and completing the squares.
In the fraction R(x) = N(x) /D(x), if the degree of the numerator is less than that of the
denominator, the fraction is called a proper fraction, otherwise the improper fraction.
NOTE: Every improper fraction can be reduced to a sum of a polynomial and a proper fraction. Thus,
-:159:-
CHAPTER EIGHT ANTIDERIVATIVES
N(x) R(x)
 Q(x) 
D(x) D(x)
Now, there are four cases of partial fractions.
CASE 1: When all factors of the denominator D(x) are linear and distinct.
2x  3
For example, . In this case, we write
(x  1)(x  1)(2x  3)
2x  3 A B C
= + +
 x  1 x +1 2x + 3  x  1 x +1 2x +3
CASE 2: When all the factors of the denominator D  x  are linear but some are repeated.

x2
For example, . In this case
(x  1)3 (x  1)
x2 A B C D
   
3
(x  1) (x  1) x  1 (x  1) 3
(x  1) 2 x 1
CASE 3: When D(x) have non – repeated irreducible quadratic factors.
1 A Bx  C
For example,  
(x  1)(x  4) 2 x 1 x2  4
CASE 4: When D(x) has repeated irreducible quadratic factors.
8x 2 8x 2 8x 2
= =
1- x 4 1+ x 2  1- x 2 1+ x 2 1+ x 2  2













 1- x 1+ x  1+ x 2 

In this case, 8x 2 A B Cx + D Ex + F
= + + +
1- x 4 1+ x 2  1- x 1+ x 1+ x 2 2
   1+ x 2 
    
 
NOTE: It may be noted that before we use the techniques of partial fraction, it has to be checked
out whether or not the degree of a polynomial in the numerator is less than the degree of
polynomial in the denominator. If yes, then we directly start the partial fraction technique. In case
it isn’t, we divide the numerator by the denominator till the degree of the polynomial in the
numerator becomes less than that of the polynomial in the denominator. After this process is
completed, we resolve the resultant fractions in to partial fractions.
All these cases are studied by means of few examples.
Example 01: Evaluate the following integrals
x 1 x2 1 x
(i)  dx (ii)  (x  2)(x  3)2 dx (iii)  (x  1)(x 2  4) dx
(x  2)(x  3)
Solution: (i) Here the degree of a polynomial in the numerator is less than the degree of
polynomial in the denominator, so by using partial fractions, we have
x +1 A B (1)
= +
( x  2)(x  3) x 2 x 3
Multiplying both sides of (1) by (x – 2) (x – 3), we get
x + 1 = A(x – 3) + B(x – 2)
. (2)

-:160:-
CHAPTER EIGHT ANTIDERIVATIVES
Now put x – 2 = 0 or x = 2 in (2), we get: 3 = A(2 – 3) giving A = – 3
Again iput x – 3 = 0 giving x = 3. Put this in (2), we get: 4 = B(3 – 2) giving B = 4
Substituting these values in (1), we have
x 1 3 4
  (3)
 x  2 x  3 x  2 x  3
Now integrating both sides of (3), we have
x 1
  x  2 x  3 dx  3 x  2  4 x  3  3ln  x  2  4ln  x  3  c 
dx dx

(ii) Here we observe that the denominator contains one linear factor and the other one the
repeated linear factor. Thus,
x2  1 A B C
   (1)
(x  2)(x  3) 2 x  2 (x  3) (x  3) 2
multiplying both sides by (x  2) (x  3) 2 , we get
x 2  1  A(x  3) 2  B(x  2)(x  3)  C(x  2) (2)
Putting x  2  0 or x  2 in (2), we get : 5  A
Putting x  3  0 or x  3 in (2), we get : 10  C
To find B, we re  write equation (2) to get
x 2  1  A(x 2  6x  9)  B(x 2  5x  6)  C(x  2)
Comparing the coefficients of x 2 , we obtain :1  A  B  B  1  A  1  5  4
Thus, equation (1) becomes after int egration
x2  1 1 1 1
 (x  2)(x  3)2 dx  5
(x  2)
dx  4 
(x  3)
dx 10 
(x  3) 2
dx

10
 5ln(x  2)  4ln(x  3)  10  (x  3) 2 dx  5ln(x  2)  4ln(x  3)  c
(x  3)
(iii) Here we observe that one factor in the denominator is linear and other one is quadratic, so by
partial factions we have
x A Bx  C
  (1)
2
(x  1)(x  4) x 1 x2  4
2
Multiplying both sides of (1) by (x – 1)(x + 4), we have
x  A  x 2  4    Bx  C  x 1 (2)
 
Substituting x = 1 in (2) we have: 1 = A(1 + 4) giving A = 1/5
Now to find B and C, we simplify (2) that is;
2 2 2
. x = Ax + 4A + Bx – Bx – C = (A + B) x + (C – B) x + (4A – C) (3)
2
Now comparing the coefficients of x , x and constants on both sides of (3), we get
A + B = 0, C – B = 1, 4A – C = 0. But A = 1/5, thus
1/5 + B = 0 giving B = – 1/5. Also C – (– 1/5) = 1 giving C = 4/5.
Substituting the values of A, B and C in (1), we have

-:161:-
CHAPTER EIGHT ANTIDERIVATIVES
x

1

 1 5 x  4 5  1  1  1  x  4
 x 1  x2  4  5  x 1 x2  4 5 x 1 5 x 2  4

Now integrating, we have


x
dx 
1

dx 1 x4
 
1 1
 
 x 
dx  ln(x  1)  ln x 2  4  2 tan 1   
 x  1  x2  4 5  x  1 5 x 2  4 5 2  2 

Example 02: Evaluate the following integrals:


x2 cosx
i   dx ii   dx
3
 x -1  x +1 1+sinx  2 +sinx 3+sinx 

Solution: (i) x2 x2 A B C D (1)


= = + + +
3
 x -1  x +1     
x -1 x -1 x -1 x +1 x -1 2
 x -1  x -1
3 x +1

3
Multiplying both sides of (1) with  x 1  x  1 , we have
2 3
x 2 = A  x -1  x +1 + B  x -1 x +1 + C  x +1 + D  x -1 (2)

Substituting x -1 = 0 or x = 1 into 2  , we have: 12 = C  2  or C =1 2

Substituting x 1  0  x  1 into (2), we have: 2 3


 1  D  2   D  1 8
Now to find the values of A and B, we rewrite the equation 2  in the following way:

x 2  A  x 1  x 2 1  B  x 2 1  C  x 1  D  x3  3x 2  3x 1


     

 1
 1 1 3 3
x3  A x3  x  x 2  1  Bx 2  B  x   x3  x 2  x 
2 2 8 8 8
1
8
 1  3  1 3  1 1
0x3  1x 2  0x  0x 0   A   x3   A  B   x 2   A    x   A  B    x 0
 8  8  2 8  2 8
Comparing the coefficients from both sides, we have
A 1/ 8  0  A  1/ 8 and  A  B  3 / 8  1  B  3 / 4
Hence, A  1/ 8,B  3 / 4,C  1/ 2 and D  1/ 8
Substituting all these v alues into equation (1) we have
x2 1 3 1 1
   
 x 1  x 1 8  x 1 4  x 1 2  x 1 8  x 1
3 2 3

Integrating both sides with respect to x, we have


x2 1 dx 3 dx 1 dx 1 dx
 3
 x -1  x +1
dx 
8 x -1
+
4  x 1

2 2   x -1

3 8 x 1 
1 2
1 3  x 1 1  x 1 1 1 x -1 3 1
 ln  x 1    ln  x  1 = ln - - +c
8 4 1 2 2 8 8 x +1 4  x -1 4 x -1 2
 
(ii) Let z  sin x  dz  cos x dx  Therefore,
cos x dz
I  1 sin x  2  sin x 3  sin x  dx   1+ z  2 + z 3+ z 
-:162:-
CHAPTER EIGHT ANTIDERIVATIVES
Now, 1 A B C (1)
= + +
1+ z  2 + z 3+ z  1+ z 2 + z 3+ z
Multiplying both sides of (1) with 1+ z  2 + z  3+ z  , we get

1 = A  2 + z  3+ z  + B 1+ z  3+ z  + C 1+ z  2 + z  (2)


Substituting 1 z  0  z  1 into (2), we get: 1 = A(1)(2) giving A = 1/2
Substituting 2 + z = 0 or z = – 2 into (2), we get
1 = B(–1) (1) giving B = –1
Substituting 3 + z = 0 or z = – 3 into (2), we get: 1 = C(–2)( –1) = 2C giving C = 1/2
Hence, A = 1/2, B = –1, C = 1/2
1 1 1 1
So, equation (1) becomes:    (3)
1 z  2  z  3  z  2 1 z  2  z 2 3  z 
Integrating both sides of (3) with respect to z, we get
dz 1 dz dz 1 dz 1 1
  
1  z 2  z  
3  z

2 1+ z 
-
2  z

2 3  z 2 
 ln 1 z   ln  2  z   ln  3  z 
2
Replacing z by sinx, we get
cosx 1 1
 1+sinx  2 +sinx 3+sinx  dx = 2 ln 1+sinx  -ln  2 +sinx  + 2 3+sinx  +c
Integration by Completing the Squares Method
If the rational algebraic function N(x)/D(x) is such that D(x) is quadratic function that do not have
real factors, the method of completing the squares is used to evaluate the integrations. The
method is well presented by the following two examples.
Example 03: Evaluate the following by “Completing the Squares Method”
x 1
 
x
(i) dx (ii) dx
2
x  4x  5 2
x  6x  7
x 1 2x
Solution: (i) Let I =
 x2 + 4x + 5 dx  2  x 2  4x  5 dx
1 2x  4  4 1 2x  4 1
 
2 x 2  4x  5
dx 
2 x 2  4x  5 
dx  2
2
x  4x  5
dx 
1 1
 ln(x 2  4x  5)  2
2 2
x  4x  5
dx

1 1
 ln (x 2  4x  5)  2  x2  4x  4 1 dx  ln (x 2  4x  5)  2  (x  2)2  5 dx
Let x + 2 = z or dx = dz. Thus,

tan 1
1 2 z
I  ln (x 2  4x  5)  2  2
dz  ln (x 2  4x  5)  c
z2   5 5 5

(x  2)
tan 1
2
 ln (x 2  4x  5)  c
5 5
x 1 1 2x  2 1 2x  6  4
(ii) Let I   x2  6x  7 dx  2  x 2  6x  7 dx  2  x 2  6x  7 dx

-:163:-
CHAPTER EIGHT ANTIDERIVATIVES
1 2x  6 1 1 1
  2
2 x  6x  7
dx  2
2
x  6x  7 2 
dx  ln(x 2  6x  7)  2
2
(x  6x  9)  9  7
dx 
1
 ln x 2  6x  7  2
(x  3)2 16
dx 
Let x - 3 = z or dx = dz. Thus,
1 1 z4
I  ln x 2  6x  7  2  z2  42 dz  ln
x 2  6x  7  2 ln
4 z4
c

1 x 3 4 1 x 7
 ln x 2  6x  7  ln  c  ln x 2  6x  7  ln c
2 x 3 4 2 x 1
1
Example 04: Evaluate
 x 4 +1 dx
Solution:
1 1 2 1 x 2 1  x 2 1 1 x 2 1 1 x 2 1
Let I 
x4 1

dx 
2 x 4 1
dx 
2 x 4 1
dx  
2 x 4 1
dx 
2 x 4 1
dx (1)  
2 2 2 2
Now consider, I  x  1 dx  x (1  1/ x ) dx  (1 1/ x ) dx
1 x4 1

x 2 (x 2  1/ x 2 )
 (x 2 1/ x 2 )

2 2 2 2 2 2 2
Let z = x – 1/x or dz = (1 + 1/x ) dx. Also z = x – 2 + 1/x or x + 1/x = z + 2.
(x 1/ x) 1 (x 2 1)
tan 1 tan 1 tan 1
1 1 1 z 1
Thus, I 
z2  2

dz 
2
dz    
 
1 2 2 2 2 2 2x
z2  2

x 2 1 x 2 (1 1/ x 2 ) (1 1/ x 2 )
Now consider, I 
2 x4 1
dx  
x 2 (x 2 1/ x 2 )
dx  
(x 2 1/ x 2 )
dx 
2 2 2 2 2 2 2
Let z = x + 1/x or dz = (1 – 1/x ) dx. Also z = x + 2 + (1/x ) or x + (1/x ) = z – 2. Thus,
1 1 1 z 2 1 (x 1/ x  2)
I 
2 
z2  2
dz 
2
dz   ln  ln
2 2 z  2 2 2 (x 1/ x  2)
z2  2  
1 (x 2  2x 1)
 ln
2 2 (x 2  2x 1)
Thus equation (1) becomes:

1 1 (x 2 1) (x 2  2x 1) 
tan 1
1 1 1
 x 4 1
dx  [I  I ]  
2 1 2 2 2
 2x
 ln
2 2 (x 2  2x 1) 
c

NOTE: The above example does not belong to “Completing the Squares Method”, nevertheless it
is an important type of integral. Students are advised to see the steps taken in solving this integral.
1 1 x a 1 1 1 x
The following formulae have been used here:
 x2  a 2 dx  2a ln x  a and  x2  a 2 dx  a tan a
Integration of Irrational Algebraic Functions
In this section, we shall study the solutions of integrals of the functions which are polynomials
and at least one of these polynomials appears under radical sign. In addition to this, the integrand
may contain the functions of the form a 2  x2 , a 2  x2 or x2  a 2 . In such cases, the

-:164:-
CHAPTER EIGHT ANTIDERIVATIVES
following substitutions will help in evaluation of the integrals that may contain one of these
functions. If the integrand contains:

a2  x2 substituting x  a sin 
a2  x2 " x  a tan  or x  a sinh 
x2  a2 " x  a sec  or x  a cosh 
Example 01: Evaluate the following integrals.

i  x
2 25- x 2 dx  ii  e
x 1- e2x dx

Solution: (i) Putting x  5sin   dx  5cos  d Then

 
I  x 2 25  x 2 dx  25sin 2  25  25sin 2  5cos   d

 
 625 sin 2  cos  1  sin 2  d  625 sin 2  cos 2  d
 1  cos 2  1  cos 2  625 
 625 
4 
2 
  d  1  cos 2  d

 2  2 
625   1  cos 4   625  1 1 

4 
 1d    2  d  
 

4  2 1d  2  cos 4 d 
625  1 1  625  1 
     sin 4      sin 4 
4  2 8  8  4 

Replacing  by sin (x/5) we get: I  625 sin 1  x   1 sin 4  sin 1 x    c


-1
8  5 4  5 
x x
(ii) Put z = e giving dz = e dx. Then
x 1  e2x dx  z 1  z 2  dz  
I
e  
 z 
  1  z 2 dz

 x 2 2 a 2 1  x  
1  z2  sin 1 z
z 1
I
2 2
 using


 a 2  x 2 dx 
2
a  x  sin  
2  a 

ex
1-e2x + sin-1  ex  +c
Replacing z by e we get:
x 1
I=
2 2
If the integrals are of the form:  dx
OR  Quadratic dx
Quadratic
the following tips can be used to evaluate such integrals. This technique is known as Completing
the Squares Method.
2
 Make the coefficient of x unity by taking its numerical coefficient outside the square
root sign.
 Complete the square in terms containing x by adding and subtracting the square of half
the coefficient of x.
 Use proper standard form.
Example 02: Evaluate
 3x 2 – 4x +1 dx

4 1
Solution: I 
 3x 2  4x  1 dx  3  x 2  x  dx
3 3
Completing the squares, we get
-:165:-
CHAPTER EIGHT ANTIDERIVATIVES
 2 2 2
 x 2  4 x   2     2   1 dx   2 1
I 3   3     
3  3
3   x    dx
 3 9
 

Substituting z  x  2  dz  dx and a 2  1  a  1 , we get


3 9 3
z a2 z
I= 3  z2 -a 2 dz = 3 
2

z2 -a 2 -
2
cosh-1  + c
a

 2 
Replacing z by x – 2/3, we get: I = 3  1  x - 2   x - 2  - 1 - 1 cosh -1 x - 2 3  + c
2
 
3   3  9 18 3 
 
Five Standard Cases
In this section, we shall study the five important forms of integrations.

 Linear
1
CASE 1: Integral of the form dx
Linear
Rule to evaluate: Put Linear  z

 Quadratic
CASE 2: Integral of the form 1
dx
Linear

Rule to evaluate: Put Linear  z

CASE 3: Integral of the form 1


dx ,
Linear Quadratic
Rule to evaluate: Put Linear  1/ z

  Pure Quadratic
CASE 4: Integral of the form x
dx
Pure Quadratic

Rule to evaluate: Put Pure Quadratic  z

  Pure Quadratic
CASE 5: Integral of the form 1
dx
Pure Quadratic

Rule to evaluate: Put x = 1/y and in the resulting integral put Pure Quadratic  z
Example 03:
Evaluate the following integrals
1 1 dx x dx

i 
 2x +3  x +5
dx  ii  
 x +1 x -1
2
dx  iii 
2 2
x x +1
 iv 
 2 
 x - 2x + 2  x -1

 

dx  1dx 
Solution: (i) Let I =
  form 
 2x +3 x +5 

Linear Linear 

Put z  x  5  z2  x  5  2zdz  dx  Then


2zdz 2dz 2 dz dz
I=  2  z2 -5  + 3  z  =  2z2 -10 + 3 = 2  z2 - 7 2 = 
 
2
    z2 - 72

-:166:-
CHAPTER EIGHT ANTIDERIVATIVES
1 z- 7 2 1 x +5 - 7 2
I= ln = ln +c
2 7 2 z+ 7 2 14 x +5 + 7 2

dx  dx 
(ii) Let I 
  x 1 x 2 1
form
  
Linear Quadratic 

Put x  1  1  x  1 1  dx   1 dz  Then
z z z2
 1 z 2  dz
1  1  
 
I  2
   dz 
 z2 
 1  2z  z2  z2
1 z   1z 1 1 1 z 
  z2
 1 z 2  dz
 dz 1 2
 
I  1 z2  1  2z

 1 2z 1 2   1 2z  dz
 
12
1 1  2z  2 x 1 2 x 1
I  1  2z  1    c
2 12 x 1 x 1 x 1

dx  dx 
(iii) Let I 
2 2
x x 1
 form
 Quadratic Quadratic

 
Put x  1  dx   1 dy  Then
y y2
 1 y2  dy  1 y2  dy
  ydy 1 2ydy
   
I  2 2
  1  y2
  1  y2

2  1  y2
1 y  1 y   1 1
 y2 
  y2
 (1 2)1 
 1  y2    n 1 
1  f   x  dx f  x  
I      
 using   
2 (1 2)  1   f  x 
n  n  1 
  
 
2
1 x2 1 x 2 1
I   1  y2   1       c
x x2 x

x dx  x dx 
(iv) Let I 
  x2  2x  2  x 1
form
  Quadratic 
Linear 
 

Put z  x 1  z2  x 1  2zdz  dx  Then


 z 2  1 2z dz 1  z 2  dz
     1  z2 1  1 z2
  2  
I   2   2
2



 2
z4  1
dz  2 1  z4  1 z2  z2
dz (1)
 z  1  2  z  1  2  z 
    

 
Put z  1  t  1 1  dz  dt into (1), we get
 
z  z2 

-:167:-
CHAPTER EIGHT ANTIDERIVATIVES
 t   z –1 z   2 
dt dt 2 -1 z – 1 
I=2  2
=2  2
= tan -1   = 2tan -1   = 2tan 
 2z 
t +2 t2 +  2 2  2  2   

 x –1–1  -1  x – 2  + c
I = 2tan -1   = 2tan  
 2 x – 1   2x – 2 

WORKSHEET 08
Evaluate the following integrals:
3
1. 
ax 2  bx  c
x2
dx 
2.  x ax  bx  c dx 2
  1
3.   x   dx
 x
3
 1 
  dx  1  1 
3
4.   x   dx 5.  a 2  x 2 6.   x   x 2  2  dx
 x  x  x 
x 3  3x 2  4 dx  2x  13
7.  dx 8.  9.  dx
x x 1  x  2 x 1
10.  sin 2x 1  cos 2x dx  e 
 e x ln a dx 12.  1  x  1  x dx
a ln x
11.

dx dx 1 x
13. x2  4
14.  4  x2
15. 
1 x
dx

sin(ln x) 2x  7 x 6
16.  dx 17.  dx 18.  dx
x x 2  7x  6 x 2  12x  7
19.  sin 5  cos3  d 20.  sec5  tan 3  d 21.  csc5  cot 3  d

22.  a 2  x 2 dx 23.  x 2  a 2 dx 24.  e cos x sin x dx

25. 
sin 2x
1  cos 2 x
dx 26. 
1
x
cosec x cot x dx 27.  3
sin x

 sin x  cos x dx
3

sin x
28.  dx 29.  x cosec 2 x dx 30.  x n ln x dx
2cos x  7 cos x
31.  csc3 x dx 32.  x sin 1 x dx 33.  e ax cos(bx  c) dx

sin x e x  e x dx
34.  dx 35.  ex  e x dx 36. 
4  5cos x 
x 1 x 
dx dx x 5
37. x x2  a2
38.   2x  32  1 39.  1  x2
dx

e x
cos e   dx
x

ln 1  x 2  dx 42.  x.cot 1 x dx
40.  x
41. 
x 2

3 x2
43.  x e dx 44.  sin  ln x  dx 45.  e x sec x 1  tan x  dx
sin  ln x  dx 2x  3
46.  dx 47.  2x 2  x  1 48.  dx
x 3
 x  1  2x  3
2

x dx
49.  dx 50.  1  3ex  2e2x
x  12x  35
2

-:168:-
CHAPTER EIGHT ANTIDERIVATIVES
3x  1 x dx x dx
51.  dx 52.   x  1 53.  x4  x2 1
 x  1  x  3
2
x 2
4 
1  x2 dx
54.  (1  x 2 )2 dx 55.  5  4x  x 2 dx 56. 
2x 2  3x  4
cos xdx dx (x  1) (x  2)
57.  58.  60.  dx
4sin x  4sin x  5
2 (x  1) x
2
x2  x  2
sin x x2 cos x
61.  1  cos x  2  cos x  dx 62. 
(1  x)(x  4x  1)
2
dx 63.  (cos x  sin x) dx
sin x tan x
64.  dx 65.  dx
(sin x  cos x) 1  cos x
66. A tourist accidentally drops his camera from the top of a cliff that is 576 feet above the water
below. Assume the acceleration due to gravity to be -32 feet per second per second.
(a) Determine the velocity v(t) of the camera at any time t during its fall.
(b) Determine s(t) the height of the camera above the water at any time t during its fall.
(c) How fast is the camera falling 4 seconds after it is dropped?
(d) How long will it take the camera to hit the water?
67. To test learning, a psychologist asks people to memorize a long sequence of digits. Assume
that the rate at which digits are being memorized is dy/dt = 5.4e-0.3t words per minute, where y is
the number of digits memorized and t is the time in minutes.
(a) Find y as a function of t, which gives the number of digits memorized after t minutes.
(b) How many digits will be memorized after 5 minutes?
68. The rate of change of the volume of a spherical balloon with respect to its radius is dV/dr =
4πr2. Use this fact and the fact that V = 0 when r = 0, determine the volume of the balloon when
its radius is 6 centimeters.
69. If the marginal cost when x units is produced is C(x) = 100 – 0.5x dollars and the overhead
cost is $40, what is the cost of producing 10 units?
70. Let R(x) be the revenue a company receives from the sale of x units of its product. If its
marginal revenue is R`(x) = 100 – 0.2x dollars per item, find the:
(a) Revenue function R(x) and (b) the revenue from the sale of 20 units. Assume there is no
revenue when zero units are sold.
71. A woman gets into her car and 71. A man starts his car and then drives it with a constant
acceleration of 22 feet per second per second.
(a) Determine the velocity function.
(b) Determine the distance function.
(c) How far does the car go in 6 seconds?
72. A colony of 200 bacteria is introduced to a growth-inhibiting environment and grows at the
rate of dn/dt = 30 + 2t, where n is the number of bacteria present at any time t (t is measured in
hours).
(a) Determine a function that gives the number of bacteria present at any time t.
(b) How many bacteria are present after 3 hours?
73. The weight of a mold is growing exponentially at the rate of dw/dt = e0.2t milligrams per hour.
How much will the mold weigh in 10 hours if it weighs 70 milligrams now?

-:169:-
CHAPTER EIGHT ANTIDERIVATIVES
74. The rate of change of the temperature T inside a furnace after x minutes is dT/dx = 2x + 15
(1 < x < 20) degree/min. Assume the temperature inside the furnace is 200F initially.
(a) Find the formula for the temperature at any time x.
(b) What is the temperature inside the furnace after 14 minutes?
75. After t hours of production, a coal mine is producing coal at the rate of
dP/dt = 30 + 2t – 0.03t2 tons per hour. Find a formula for the total output of the coal mine after t
hours of production. (Note: Coal production P = 0 at t = 0).
76. The rate at which atmospheric pressure P changes as the height x above sea level changes is
dP/dx = -3.087 e-0.21x measured in pounds per square inch and x is in miles. Determine P as a
function of x when at sea level, P is 14.7 pounds per square inch.
77. The rate of the change of the area of a circular region with respect to its radius is dA/dr = 2πr.
Use this fact, and the fact that A = 0 when r = 0, to determine the area of a circular region when
the radius is 4 centimeters.
78. The rate of change of the volume of a spherical balloon with respect to its radius is dV/dr =
4πr2. Use this fact, and the fact that V = 0 when r = 0, to determine volume of the balloon when
its radius is 6 centimeters.
79. Flu epidemic is spreading at the rate dn/dt = 180t – 6t2, where n is the number of people who
are sick with flu on any particular day t after the outbreak started.
(a) Determine an equation for n as a function of t. Assume no one has the flu at the
beginning (when t = 0).
(b) How many people have the flu the tenth day after the outbreak begins?
80. Plaque builds up on the inside walls of an artery reduces the diameter of the artery (and thus
reduces the blood flow). Suppose that an artery has a diameter of 0.4 centimeter and the length of
diameter (D) is decreasing at the rate of dD/dt = -0.03e-0.001t cm/year.
(a) Determine D as a function of t.
(b) What will the diameter of the artery be after 10 years?
81. A manufacturer estimates that marginal cost of a certain production process is given by
C(x)  e0.01x  3 x , where x units are produced. What does it cost to produce 10 units if the
cost of producing 4 units is $ 2000?

-:170:-
CHAPTER NINE DEFINITE INTEGRATION WITH APPLICATIONS

CHAPTER
NINE DEFINITE INTEGRATION
WITH APPLICATIONS

INTRODUCTION
As we prepare to introduce the definite integral, the need will arise for a compact way of writing
sums. Consider the sum of the integers from 1 through 50.
1 + 2 + 3 + … + 50
We shall use the capital Greek letter sigma () to specify a sum. Along with sigma, a letter such
as i, j or k is used as the index, or counter. The first and last values of the index are written on the
50
sigma as shown next. 1+2+3+...+50= k
k =1
This particular example of sigma notation, or summation notation, specifies a sum of numbers of
the form `k` where k begins at 1 and ends up to 50. Thus, it specifies the sum of integers from
1 through 50. Here is another sum written in sigma notation,
6

 2k 1  2 1  1  2  2 1  2 3 1  2  4 1  2 5 1  2  6 1  48
k 1

The index can also be used for subscripts. Consider the sum x1 + x2 + x3 + … + xn Using the
n
summation notation, this sum can be written as 
k 1
xk

 f  xi  assuming that f(x) = x , x1 = 0, x2 = 1, x3 = 2 and x4 = 3.


4
3
Example 01: Find the sum
i=1

         
4
Solution:  f x = f x + f x + f x + f x = f  0  + f 1 + f  2  + f 3  = 0 +1+8 + 27 = 36
i=1 i 1 2 3 4
Example 02: Use the summation notation to write each of the following expression in condensed
form  i  1+ 12 + 13 + 14 +...+ 100
1
 ii  12 + 32 + 43 + 54 +...+ 50
49

100 49

 1
 k +1 
1 1 1 1 1 2 3 4 49 k
Solution:  i  1+ + + + ... + =    ii  + + + + ... + =
2 3 4 100 i=1 i 2 3 4 5 5 k =1

Area and the Definite Integral


The study of geometry includes formulas for determining the area bounded by such geometric
figures as circles, triangles and rectangles (see fig 1). In this topic, we will develop the calculus
necessary to determine the area of other types of regions – regions bounded by various curves
(see fig 2).

-:171:-
CHAPTER NINE DEFINITE INTEGRATION WITH APPLICATIONS
l
r w

A  r 2 A  bh / 2 A  lw
Fig. 1 “Areas enclosed by geometric figures”

To begin, let us consider the area bounded by the graph of y = f(x) the x – axis, and the vertical
lines x = a and x = b. This is usually called simply the area under the
curve.
We will be considering the integral from x = a to x = b and we will
assume that the graph of y = f(x) is continuous, that is, it has no
breaks or gaps. We will also assume that f(x) ≥ 0 for all x between
a and b. The area we are considering is shown shaded in the
adjacent figure.
At this time we have no means of determining the exact value of the
shaded area. However, we can approximate the area by using
rectangles, because it is an easy matter to determine the area of any rectangle. In view of this, let
us create a rectangular approximation of the area under the curve.
We begin by dividing the interval from a to b into n equal subintervals. Since the whole interval
has width b - a it follows immediately that the width of each of the n subinterval is (b – a)/n. We
call this width x and is given by: x  (b  a) / n
One rectangle will be constructed for each subinterval, and the width of each rectangle will be
x .The length of each rectangle will be the distance between the x – axis and the graph,
measured vertically at the right end of each subinterval. This is illustrated in the figure given
below. Here n = 4 so there are four subintervals, each of width Δx If the area of each rectangle is
computed, and then all four areas are added, the result will be an approximation to the area under
the curve. The area of a rectangle is computed as length times the width. Here the width is Δx in
each instance. The length is the distance from x – axis to the graph. That distance is the value of
f(x) for the particular x. If we call the x values x1, x2, x3, and x4, then lengths of the rectangles are
f(x1), f(x2), f(x3) and f(x4). The area of the first rectangle is f(x1) Δx. Similarly, for the entire
region, we have: Area = [f(x1) + f(x2) + f(x3) + f(x4)] Δx
4
Using summation notation, we express the total area of the rectangles as: Area 
 f  x  x
i 1
i

-:172:-
CHAPTER NINE DEFINITE INTEGRATION WITH APPLICATIONS

This is a rough approximation to the area under the curve. The approximation can be improved by
using more rectangles. Compare the approximation shown by using four rectangles
(See above 1st figure). A better approximation obtained by using eight rectangles. (See the 2nd
figure above). The approximation to the area under the curve y = f(x) improves more as the
number of rectangles n increases further. We find the area of n rectangles to be
n
A  f (x1 )x  f (x 2 )x  f (x 3 )x  ...  f (x n )x   f (x i )x
i 1
The exact area between the graph of y = f(x) and the x – axis (on the interval fro a to b) is the
limit of this sum as the number of rectangles n approaches infinity. Thus, assuming the limit
n
exists, A  Lim  f (x i )x
n  i 1

Saying that n  is the same as saying that x  0 since the width of each rectangle gets
smaller and smaller as the number of rectangles increases. Thus, we can also write
n
A  Lim  f (xi )x
x 0 i 1

This limit is given a special name and notation. It is known as the definite integral of the function
f(x) from a to b.

The Definite Integral of f(x) from a to b


If f(x) is continuous on [a, b] then the definite integral of f(x)
from a to b is given by
b

 f  x  dx  F(x)
b
a
 F(b)  F(a)
a
assuming the limit exists. The interval [a, b] is divided into n
equal subintervals of width x given by (b – a)/n and xi is the
right most point in the ith interval.

 f  x  dx  F  x a  F  b   F  a 
The result b

is so important for the study of calculus that it is known as the Fundamental Theorem of
Calculus. It provides a key link between differential calculus and integral calculus.
-:173:-
CHAPTER NINE DEFINITE INTEGRATION WITH APPLICATIONS

The Fundamental Theorem of


Calculus
If f(x) is continuous on the interval [a, b] then
b


b
f  x  dx   F  x    F  b   F  a 
a
a
where F(x) is any anti-derivative of f(x).

Note: The numbers a and b are called limits of integration, b is the upper limit and a is the lower
limit.
2


Example 03: Evaluate x 2 dx using the Fundamental Theorem of Calculus
1
2 2
Solution: 2 1 3  1 3 2 1 3 3 7
 x dx   3 x 1  3 x 1  3 2 1  3  
1

Note: Because the Fundamental Theorem of Calculus says we can use any anti-derivative, we
will always use the anti-derivative with C = 0. However, if you did use some other value of C, it
3 3
would be eliminated in the process anyway. If x /3 + c (rather than just x /3) had been used in the
above example, we would have had
2 2
1 3  1 3  1 3  1 3 3 7
 x dx   3 x  C1   3 2  C   31  C  3 2 1  3
2
  (same as above)
1

Example 04: Determine the (exact) area under the curve y = x from x = 1 to x = 4.
Solution: We have f  x   x,a  1 and b  4  Thus,
4
4
 2x 3/2 
 x dx  
2 3/2 3/2
  4 1
 3 1 3
2
3

  8  1 
14
3

1

The area between the graph of y = x and the x – axis on the interval [1, 4] has been shown to
be 14/3 square units. The area is shown shaded in the following figure:
Fundamental Properties of Definite Integrals
b b
Property 1: a a
f  x  dx  f  u  du

Proof: Let
b

 f(x)dx = F(x) or a f(x)dx = F(b) - F(a). Similarly,


b b b

 f(u)du = F(u) or a f(u)du = F(b) - F(a) therefore a f(x) dx = a f(u) du


This property states that definite integral is independent of the variable used. For
/2 /2
For example,
0 cos x dx  0 cosu du  1
-:174:-
CHAPTER NINE DEFINITE INTEGRATION WITH APPLICATIONS
b b
Property 2: If f(x) is an integrable function, then

a

f  x  dx   f  x  dx
b
b b
Proof: Consider,
a
f  x  dx  F(b)  F(a)    F(a)  F(b)    f  x  dx 
b

This property states that interchange of the limits of integration changes the sign of the integral.
/2 0
For example,

0
cos x dx  1 where as  cos x dx  1
/2
b c b
Property 3: a a c
f  x  dx  f  x  dx  f  x  dx where a  c  b.

c b
Proof: Consider, a f  x  dx  F(c)  F(a) and c f  x  dx  F(b)  F(c)
Adding both results, we get :
c b b


a

f  x  dx  f  x  dx  F(c)  F(a)  F(b)  F(c)  F(b)  F(a)  f  x  dx
c
a

This property states that if c is any real number that lies in the interval [a, b] then integral from a
to b is equal to sum of the integrals taken from a to c and then from c to b. For example, consider

 cos x dx  sin   sin 0  0  0  0


0
 /2 

 cos x dx  sin( / 2)  sin 0  1  0  1 and  cos x dx  sin()  sin( / 2)  0  1  1


0  /2
 /2  

Adding the last two results, we get :


 cos x dx   cos x dx  1  1  0  cos x dx
0  /2 0

Generalization of Property 3
b a1 a2 a3 an b

 f  x  dx   f  x  dx   f  x  dx   f  x  dx 
a a a1 a2


a n 1
f  x  dx 
 f  x  dx,
an

Here, a  a1  a 2  a 3  ...  a n  b
This means that if the interval of integration [a, b] is divided into any finite number of
subintervals, the integration taken over the interval [a, b] is equal to sum of the integrals taken
over the subintervals.
a a
Property 4:
 f  x  dx   f a  x  dx
0 0
Proof: Let u = a – x or dz =- dx. Also if x = 0 the u = a, and if x = a then u = 0. Thus
a 0 a

 f  a  x  dx   f  u  du   f  x  dx (Using the property 1.)


0 a 0
Let us try an example now. Consider
-:175:-
CHAPTER NINE DEFINITE INTEGRATION WITH APPLICATIONS
/2
/2
 cos x dx  sin x 0
0
 sin( / 2)  sin 0  1  0  1

Now consider,
/2 /2 /2

 cos x dx   cos[( / 2)  x]dx   sin x dx  cos x


/2
0  [cos( / 2)  cos0]  [0 1]  1
0 0 0
We observe that a result of both integrals is same. It may be noted that cos (90 o – x) = sin x.
This property states that if the lower limit in any definite integral is zero, the value of the integral
remains unchanged on replacing x by “upper limit minus x “in the integrand.
Before we provide an example, let us consider the geometrical meaning of this property.
Geometrically this property states that “area under the curve y = f(x) from 0 to any real number
x = a is same as the area under the curve between 0 to a when the function f(x) is shifted “a
units|” on the left of origin. This is depicted in the following figure.

0 a –a 0
We observe that the shape of the curve is same but it is shifted a units to the left of the origin.
Thus area will be same. It may be noted that in the second figure, the lower limit is
–a, nevertheless the area is always positive, hence the integral can be taken from 0 to a as the
numerical values of both integrals will be same.
2a a a
Property 5:

0

f(x)dx  f(x)dx  f(2a  x)dx
0

0
2a a 2a
Proof: By property 3,
 f(x)dx   f(x)dx   f(x)dx
0 0 a
2a a 2a a a
But,
0 0
f(x)dx  f(2a  x)dx [using property 4]. Thus, 
0
 
f(x)dx  f(x)dx  f(2a  x)dx
0 0
2a a
Property 6: i.
 f(x)dx  2 f(x)dx,
0 0
if f (2a  x)  f (x)

2a
ii.
 f(x)dx  0,
0
if f (2a  x)  f (x)
2a a 2a a a
Proof: (i)
 f  x  dx   f  x  dx   f  x  dx   f  x  dx  f  2a  x  dx
0 0 a 0 0
[By property 4]

a a a

0

 f  x  dx  f  x  dx  2 f  x  dx
0
 0
[ f (2a  x)  f (x)]

2a a 2a a a
(ii)
 f  x  dx   f  x  dx   f  x  dx   f  x  dx  f  2a  x  dx
0 0 a 0 0
[By property 4]

a a


 f  x  dx  f  x  dx  0
0

0
[ f (2a  x)  f (x)]

-:176:-
CHAPTER NINE DEFINITE INTEGRATION WITH APPLICATIONS
Example 05: (i) We know that cos (2π- x) = cos x, hence
2

 cos x dx  sin x 
2
LHS  0  sin 2  sin 0  0  0  0
0
   

   
  
RHS  cos x dx  cos  2  x  dx  cos x dx  cos x dx  sin x 0  sin x 0  2 sin x 0  2(sin   sin 0)  0
0 0 0 0
We see that LHS = RHS
(ii) We know that sin (2π- x) =- sin x, hence
2

 sin x dx  cos x
2
0  (cos 2  cos 0)  (1  1)  0 . Hence the result.
0
na a
Property 7: If f  x   f  a  x  , then  f  x  dx  n 0 f  x  dx
0

na a 2a 3a na
Proof: 0 f  x  dx  0 f  x  dx  a f  x  dx  2a f  x  dx  ...  (n1)a f  x  dx (1)

nd
In the 2 integral on the right hand side substitute, x = a + u or dx = du.
Now if x = a then u = 0 and if x = 2a then u = a. Thus,
2a a a a

 f  x  dx   f a  u  du   f a  x  dx   f  x  dx
a 0 0 0
rd
Similarly, in the 3 integral on the right side of (1) if we make the same substitution, we get
3a a

2a f  x  dx  0 f  x  dx . Proceeding in this manner we shall see that each integral on the right
a na a
side of (1) is equal to  f  x  dx . Thus,  f  x  dx  n  f  x  dx provided f (x)  f (a  x).
0 0 0
2
Example 06: n cos x dx  n sin x 02  n(sin 2  sin 0)  n(0  0)  0 [ cos(2  x)  cos x]

0
2n 2
2
 cos x dx  n  cos x dx  n sin x 
0 0
0
 n(sin 2  sin 0)  n(0  0)  0. Hence the result.

Property 8:
a a
 If f(x) is an even function of x, then: a f  x  dx  20 f  x  dx
a

 If f(x) is an odd function of x, then: a f  x  dx  0


 Even function: A function f(x) is said to be an even function of x if f(-x) = f(x)
2
For example, f(x) = x and f(x) = cos x are even functions.
-:177:-
CHAPTER NINE DEFINITE INTEGRATION WITH APPLICATIONS
 Odd function: A function f(x) is said to be an odd function of x if f(-x) = – f(x)
3
For example, f(x0 = x and f(x) = sin x are odd functions.
If these conditions are not satisfied, then the given function is neither even nor odd.
Proof: Let f(x) be an even function, that is, f(– x) = f(x). Then
a 0 a

 f  x  dx   f  x  dx   f  x  dx
a a 0
Put  x  u  dx  du, in the first integral. If x  a then u  a and if x  0 then u  0.Thus
a 0 a a a a a a

 f  x  dx   f  u  du  f  x  dx   f  u  du   f  x  dx   f  x  dx   f  x  dx  2 f  x  dx
a a 0 0 0 0 0 0
Now let f(x) be an odd function, that is, f(–x) = – f(x). Then
a 0 a

 f  x  dx   f  x  dx   f  x  dx.
a a 0
Put  x  u  dx  du, in the first integral. If x  a then u  a and if x  0 then u  0.Thus
a 0 a a a a a


a
     
f  x  dx   f  u  du  f  x  dx  f  u  du  f  x  dx  f  x  dx  f  x  dx  0. This proves the result.
a 0 0 0 0 0
2 3
Example 07: We know that f(x) = x is an even function and f(x) = x is an odd function. Hence
we may see that
4 4 4
 x3 
4
 x3  2
 
1 128 128
LHS  x dx     [64  (64)] 
2
, RHS  2 x 2dx  2    (64  0)   LHS = RHS
4 
 3  4 3 3
0
 3 0 3 3
4 4
 x4 

3 1
Also, x dx     (256  256)  0
4  4  4 3
With the help of these properties we are now solving some important definite integrals.
Example 08: Prove the following:
/2 /2
sinx π
i  
0 sinx + cosx
dx =
4
 ii   ln  tanθ + cotθ  dθ = πln2
0
/2
sinx
Solution: (i) Let
0 sinx + cosx
dx 1


2 sin   x   a a 
2 
I 0  
dx since
  
f  x  dx  f  a  x  dx 

sin   x   cos   x   0 0 
 2  2 
2
cos x
I 0 cos x  sin x
dx  2

Adding 1 and  2 , we have

-:178:-
CHAPTER NINE DEFINITE INTEGRATION WITH APPLICATIONS
2 2
sin x  cos x  
  1dx   x     0  
2
2I  dx  0
0 cos x  sin x 0
2  2
2
sin x 
I

0 cos x  sin x
dx 
4
(ii) Let
2 2 2  sin 2   cos2  
 sin  cos  
I 0 ln  tan   cot   d  
0
ln    d 
 cos  sin   0
 ln 
 sin  cos  

 d

2 2 2
 1 
I 0 ln   d 
 sin  cos   0 ln 1  ln  sin  cos    d   0 ln sin   ln cos  d
2 2 2

0
I   ln sin  d  0 0
ln cos  d  2 ln sin  d

 2 2 
since ln cos  d  ln cos      d  ln sin  d
2


 0
 0
2  0




I  2   ln 2    ln 2
 2 
BETA AND GAMMA FUNCTIONS
Beta Function
1
If m and n are positive, then the definite integral x m1 1  x n1 dx is called the Beta function
0
of first kind and is denoted by B  m,n  (read as “Beta m, n”). Thus,
1
n 1
B  m,n   x m1 1  x 
 dx, m  0,n  0
0

Beta function is also called Eulerian (after Euler, a great mathematician) integral of first kind.
There are some important properties of beta functions which will help to solve important
problems of definite integration. These properties are listed below.
(i) Symmetry of the Beta function, that is; B(m, n) = B(n, m)
1
Proof: By definition, B  m,n   x m1 1  x n 1 dx, m  0,n  0

0

Changing x by 1 – x, we get
1 1 1
m1 n1 m1 n1 m1

B  m,n   1  x  1 1 x  
dx  1  x   x  dx  x n1 1  x  
dx  B  n,m 
0 0 0

Hence, B m,n   B n,m 

(ii) If both m and n are positive integers, then B  m, n  =


 m – 1! n – 1!
 m + n –1!
1
By definition, B  m,n   x m1 1  x n 1 dx, m  0,n  0

0

-:179:-
CHAPTER NINE DEFINITE INTEGRATION WITH APPLICATIONS

Let u  x m1  du   m  1 x m2 dx and dv  1  x n1 dx  v  


1 x 
n

n  1
Now using the formula of integration by parts, we get
n 1
 1 x   1  x  m 1 x m2dx  0  0  m 1 1 x m2 1  x n dx
1 n
    n   

B  m,n    x 
m 1  


n  1  n
0 0 0

1
m 1 m11 n11 m 1

1 x  B  m 1,n 1 1



dx 
 
 x  

n
0
n

Changing m to m + 1 and n to n + 1, we have: B  m 1,n 1  m  2 B  m  2,n  2 


n 1
Substituting this value of B  m  1,n  1 into 1 , we get
m 1 m  2
B  m,n    B  m  2,n  2   2
n n 1
Generalizing from 1 and  2 , we have

 m 1 m  2  m   m 1
B  m,n    Bm   m 1 ,n   m 1
n  n  1 n   m  2 

 m 1 m  2  1 B 1,n  m 1 3


B  m,n  
n  n 1  n  m  2 
 
1 1
n  m2 n  m2
But, B 1,n  m 1  x 0 1  x 
0

dx  1  x 
0
dx

1
1  1  x nm2    1 1
B 1,n  m 1        0 1  
n  m 1  0 n  m 1 n  m 1
Thus from (3), we get

B  m,n  
 m 1 m  2 1 
 m 1! (4)
n  n 1  n  m  2  n  m 1  n  m 1 n  m  2   n 1 n
Multiplying the numerator and denominator of (4) by  n  1! Therefore

B  m,n  
 m 1! n 1! 
 m 1! n 1!
 n  m 1 n  m  2   n 1 n  n 1!  n  m 1!
Corollary: B 1,n  = 1 n
Substituting m = 1 in the above-proved equation, we get

B 1,n  
11! n 1!   n 1!   n 1!  1
 n 11! n! n  n 1! n
2

 8 – x 
–1 3
3
Example 01: Evaluate dx
0

2 2 3
Solution: Let x 3  8z  x  2z1 3  dx  z dz  Also x  0  z  0; and x  2  z  1
3
Therefore,

-:180:-
CHAPTER NINE DEFINITE INTEGRATION WITH APPLICATIONS
2 1 3 1
1 3 2 2 3 2
1
1 3
0  8  x3 




dx  0 8  8z  3 30 
z dz  81 3 1  z  z 2 3dz

1 1
2 1 1 3 1 1 1 2 1 1 1 2
3 20  30 
  z 2 3 1  z  dz  z 3 1  z  3 dz  B  , 
3 3 3

Gamma Function

x x n 1dx is called the Gamma function and is
If n is positive, then the definite integral
0 e

denoted by   n  (read as “Gamma n”). Thus,

  n   e x x n1dx, where n  0
0

Gamma function is also called Eulerian integral of second kind or it is also called the
“Generalized factorial function.”
Recurrence Formula of Gamma Function
Show that Γ  n  =  n – 1! and Γ 1  1 where n is a positive integer.


By definition:   n   e x x n1dx, where n  0.
0
n–1 –x
Using integration by parts formula by taking u = x and v= e , we get
b 
x n 1


 n 1 x n2  e x  dx   Lim ex   n 1 e x x   dx
n  1 1
  n    x n1e x 0  
0
b 0 0

bn1
  Lim  0   n 1   n 1  0   n 1   n 1   n 1   n 1 
b   eb

When n is a positive integer, then by repeated application of above formula, we get


(n)  (n  1)(n  1)  (n  1)(n  2)(n  2)  (n  1)(n  2)(n  3)(n  3)

 (n  1)(n  2)(n  3)...3.2.1(1)  (n  1)!.1  (n  1)! (1)


To prove that  1  1, we have proved that   n   n   n  1 .
Putting n  1, we get  1  (1  1)!  0! 1

7 5 5 5 3 3 5 3 1 1


For example, (5)  4!  24 and                  
2 2 2 2 2 2 2 2 2 2

In case, if n is negative number, we may evaluate the gamma function for negative values of n by
using the above recurrence formula as follows:   n 1    n  /  n 1 . For example,

  4     3 /  4   (2) / (3)(2)  (1) / (3)(2)(1)  (0) / (3)(2)(1)(0)  

This shows that for every negative integer n, Γ  n  is infinite. .However, if n is not a negative
integer; we use the same formula to evaluate its value. For example,
 7  7  5  7 5  3  7 5 3  1  7 5 3 1  1 
                 
 2  2  2  2 2  2  2 2 2  2  2 2 2 2  2 
 7  7.5.3.1  1 
Thus     
 2  24 2

-:181:-
CHAPTER NINE DEFINITE INTEGRATION WITH APPLICATIONS
Relation between Beta and Gamma Functions
In this section we shall develop a relation between Beta and Gamma functions.
Γ  m Γ  n 
Prove that Β  m,n  = , m > 0, n > 0
Γm + n
Proof: From part (ii) section 8.2.1, we have proved that
B  m,n  =
 m – 1! n – 1! = Γ(m)Γ(n)
 n + m – 1! (m + n)
Example 01: Prove that: Γ  1  = π
2
1 1
  
   
2
Proof: Now, Β  m,n  = Γ m Γ n . Put m = n = 1/ 2, we get:   ,    2   2     1  
 1 1 
Γm + n 2 2  1   2 
1 1 1 1
Now be definition, we have   1 , 1   x 2 1  x  21 dx  x 2 1  x  2 dx
1 1 1

 2 2 
0

0
Putting x = sin2θ so that dx = 2sinθcosθ dθ

Also when x  0    0; when x  1     Therefore,
2
2 1 1 2

  ,   sin 2   2 1 sin 2    2sin  cos  d 2
1 1 1 1
 
2
  sin  cos  d
 2 2 0  
0
sin  cos 
2 2
  1  2   1
     2
  2 
 1d  2 0  2  0       
2  2
0

Example 02: Prove that: e-x2 dx = 1 π
 0
2

Solution: Let z  x 2  dz  2x dx  dx  1 dz  Also if x  0  z  0and if x    z  


2 z
   
1 z 1 1 2 1
dz   ez  z  dz   ez  z  2 dz
2 1 1 1
 e dx  
Therefore, x
e
0
20 z 20 20

1 1 1  
1 
since   n    ex x n 1, here n  and      
1
    
2 2 2  0
2 2 
π2
Example 03:  sin p x cosq x dx, where p > -1,q > -1
0

Put z  sin 2 x  dz  2sin x cos x dx.Also x  0  z  0; x    z  1, and


2
cos x  1 sin x  1  z  Therefore
2 2
2 2 2 p1 q1

  sin x cos x  sin x cos x dx   sin x  2 cos x  2 sin x cos x dx


p1 q1
sin p x cosq x dx  2 2

0 0 0
 p 1   q 1 
 2 p1  2 p1  
q1 1 1 1 q11 1  p  1 q  1  1  2   2 

 z2 1  z  2  dz 
2 2 
z2 1 z  2 dz  
2  2
, 
2  2  p 1 q 1 
 
2 
0 0
 2
π2
π
Example 04: Prove that
0 tanθ dθ =
2
2 2 2
sin 
Solution: Let I 
 tan  d   d   sin
12
 cos1 2  d . Putting p = 1/2 and q = –1/2,
0 0
cos  0
we get
-:182:-
CHAPTER NINE DEFINITE INTEGRATION WITH APPLICATIONS
 1 2  1   1 2  1 
 
1  2   2  1   3 4  1 4 1 2  
I        since  3 4  1 4   2
2  1 2  1 1 2  1  2  1 2 1 2
 
 2 2 
IMPROPER INTEGRALS
Recall that the area under the graph of y = f(x) between x = a and x = b is given by
b

a f  x  dx, provided the function f(x) is continuous and non – negative between a and b.
Here the interval between a and b can be extended infinitely far in either direction. The three
possible types of intervals are shown next. The corresponding integrals are called “improper
integrals.”
Interval Integral

a,  a f  x  dx
b

 , b   f  x  dx


 ,   f  x  dx


An integral with finite limits a, b (say) can also be considered as an Improper Integral if the
integrand becomes infinite within the finite interval of integration. For example, the are the
improper integrals as the first integral is infinite at x = 1 and 2nd integral is infinite at x = 3.
2 3
1 1

0
x 1
dx and
0

x 3
dx


In order to motivate a definition for the improper integral f  x  dx , let us consider the following
a
b
definite integral: a f  x  dx The following figure shows the area under the curve f(x) for larger

and larger values of b.


 m


Thus, the improper integral from a to  can be defined as: f  x  dx  Lim f  x  dx,  m  a
a m a 
provided that f is continuous on the interval a,  
 
1
Example 01: Evaluate the following improper integrals:  i  1 xdx  ii  1 x 2 dx
 b
b
 x3 2 
Solution: (i)
 xdx  Lim   x 
1/2
dx  Lim 
b  3 2 
2
  Lim b  1
3 2 3 2 2
 
 Lim b3 2  1    
1
b
1  1 3 b 3 b
Hence the given integral does not exist.
 b
1
b
2  x 1  1 
(ii)  dx  Lim   x  dx  Lim     Lim   1    0  1  1
1x
2 b 
1
b    1 1 b  b 
Note: The integral evaluated in part (i) is divergent, since it has no finite numerical value. On the
other hand, the integral in part (ii) is said to be convergent, since it has a finite value. There is
indeed a finite area under the curve.
-:183:-
CHAPTER NINE DEFINITE INTEGRATION WITH APPLICATIONS
b
Improper integrals of the type
 f(x) dx

are defined and evaluated in similar manner. Thus;
b b

 f(x) dx 

Lim
m  m  f(x) dx provided that f is continuous on the interval  , b  
2
dx
Example 02: Evaluate

  4  x 
2

2
2 2  4  x 1  2
dx 2  1  1 1  1
Solution:

  4  x 
 
m  mLim
m

4  x  dx  Lim 
m 

1


a
 Lim    Lim  6  4  m   6
m  4  x  a m  

Improper integrals of the form
 f(x) dx

are evaluated in the way involving both types of

 m 
improper integrals we have used above. Thus,

 f(x) dx   f(x) dx  m f(x) dx
provided that function f(x) is continuous on the interval  ,   The number m used in the two
integrals can be any real numbers preferably 0 or 1. If either (or both) of the two integrals
diverges, then the original integral also diverges. Otherwise, the original integral converges.
 
x x2 dx
Example 03: Evaluate the improper integrals: (i)
  x4 +1

dx (ii)
 xe
-

x 1
 x 4 1 dx  Substituting x  z  2xdx  dz  x dx  2 dz  Then
Solution: (i) Consider 2

 x4 1 dx  2  z2 1  2  z2 1 dz  2 tan z  2 tan  x 
x 1 dz 1 1 1 1 1 1 2

 0  0 a
x x x x x
Now,

 x  1
4
dx 
 x  1
4
0

x 1 a a x  1 
dx  4 dx  Lim 4 dx  Lim 4 dx
a 0 x  1  
0 a
1  1  1 1
 Lim  tan 1 x 2   Lim  tan 1 x 2   Lim tan 1 0  tan 1 a 2   Lim tan 1 a 2  tan 1 0
a  2  a a  2  0 a 2 a
2

x 1   
Thus,   x +1 dx  2 0   2    2   0  0
-
4

1
(ii) Consider xe x2 dx  Substituting x 2  z  2xdx  dz  xdx  dz  Then
 2

xex dx  ez dz  ez  1  ex . Now let
1 1 1
 
2 2
2 2 2
 0  0 b

 xe
 x2
 xe
 x2

 x2

 x2
dx  Lim xe x dx
2
I dx  dx  xe dx  Lim xe
  0 a a b  0

 1 
a   2  
2 0
a
 1 
b  
2 
2 b
 Lim   e x   Lim   e x 
0

1
2 a   2 1
2 b
2
Lim e0  ea  Lim e b  e0   
1 1 1 1
Thus, I   1  0    0 1     0
2 2 2 2
REDUCTION FORMULAS
Definition: A reduction formula is a formula that connects a given integral with another integral
which is of the same type but of a lower degree or of a lower order, or is otherwise easier to
evaluate.

-:184:-
CHAPTER NINE DEFINITE INTEGRATION WITH APPLICATIONS

 sin 
n
To find a reduction formula for x dx and cos n x dx
Method:
 Write sinn x as sinn1 x sin x as
 Integrate by parts taking sin n1 x as first function.
 Replace cos2 x by 1 sin2 x
 Transpose the given integral in sin n x form and finally solve the given integral.
 
Example 01: Evaluate sin n x dx and cosn x dx. Hence, find sin 6 x dx


Solution: Let In  sin n x dx  sin n1 x sin x dx
Integrating by parts, taking u = sinn-1x and v  sin x
In
 
 sin n –1x sinx dx = –sin n –1x cosx +  n – 1 sin n – 2 x cos 2 x dx

= –sin n –1x cosx +  n – 1 sin n – 2 x 1  sin 2 x  dx



= –sin n 1x cosx +  n – 1  sin n 2 x dx   n – 1  sin n x dx
– –

= –sin n 1x cosx +  n – 1  sin n 2 x dx   n – 1 I n


– –

In   n – 1 In = –sin n 1x cosx +  n – 1  sin n 2 x dx


– –

In 1+ n – 1 = –sin n –1x cosx +  n – 1 In – 2


1
Thus, In  sin n x dx = –sin n –1x cosx +  n – 1 In – 2  (1)
n
This is the required reduction formula.


1
cosn –1x sin x +  n – 1 I 
Similarly, In  cosn x dx =
n n–2 

Now, putting n = 6 in (1), we get


–sin 5 x cos x 5

I6  sin 6 x dx = + I4 . Putting n  4 now and apply the same formula
6 6
–sin 5
x cos x 5  –sin 3 x cos x 3 
= +   I2  . Apply the same formula once again
6 6 4 4 
–sin 5 x cos x 5 5  –sinx cos x 1 
=  sin 3 x cos x    I0 
6 24 8 2 2 
–sin 5 x cos x 5 5 5
=
6 24 16 
 sin 3 x cos x – sinx cos x 
16
sin 0 x dx
5
–sin x cos x 5 5 5
16 
=  sin x cos x – sinx cos x 
3
1 dx
6 24 16
–sin 5 x cos x 5 5 5
=  sin 3 x cos x – sinx cos x  x  c
6 24 16 16

 tan x dx and  cot x dx


n n
To find a reduction formula for
Method:
 Re-write tann x as tann-2 x tan2 x
 Replace tan 2 x by sec2 x 1
 Integrate only the first integral on right hand side using the formula
f  x  , n  1
n+1

       
n
f x f  x dx =
n +1
-:185:-
CHAPTER NINE DEFINITE INTEGRATION WITH APPLICATIONS
tan n 1x

 d 
Thus,
 tan n 2 x sec2 xdx =  tanx  = sec2 x 

,  Note :
n –1  dx 

  tan x dx
4
Example 02: Find a reduction formula for tan n x dx . Hence evaluate

Solution: Let In = tan n x dx = tan n – 2 x tan 2 x dx = tan n – 2 x sec2 x – 1 dx


  
tan n –1x

In = tan n – 2 xsec2 xdx – tan n – 2 xdx =
n –1 
– In – 2

which is the required reduction formula. Putting n = 4, we get


tan3 x tan3 x tan3 x 3 3

 sec x –1 dx = 3 –  sec xdx + 1 dx = 3 – tanx + x + c


tan x tan x
 tan 2 x dx = 2 2
I4 = – I2 = – –
3 3 3
cot n –1x

Similarly, In = cot n x dx = –
n –1
– In – 2

To find a reduction formula for secn x dx and


  csc x dx
n

Method:
 Re-write sec n x  sec n  2 x sec 2 x
n–2
 sec x dx = tanx
2 2
 Integrate by parts, takin u = sec x, v = sec x and using
2 2
 Replace tan x by sec x – 1
n
 Transpose the given integral that occurs on right in the form sec x

 sec x dx Hence evaluate  sec x dx


n 5
Example 03: Find a reduction formula for


Solution: Let In = secn x dx = secn – 2 x sec2 x dx 
Integrating by parts by taking u = secn – 2x and v = sec2 x.
In = secn – 2 x tanx –  n – 2  secn – 2 x tan 2 xdx = secn – 2 x tanx –  n – 2  secn – 2 x sec2 x – 1 dx
 
In = secn – 2 x tanx –  n – 2   sec xdx   sec n n –2

x dx = secn – 2 x tanx –  n – 2  In +  n – 2  I n – 2
In   n  2  In  sec n 2
x tan x   n  2  In 2   n 1 In  secn 2 x tan x   n  2  I n 2
secn 2 x tan x n  2
In   I (1)
n 1 n  1 n 2
which is the required reduction formula.
sec3 x tan x 3
Putting n= 5 into (1), we get I5   I3 (2)
4 4
Putting n = 3 into (1), we get
sec x tan x 1 sec x tan x 1 sec x tan x 1
I3 
2
 I1 
2 2
 sec x dx 
2 2 2 
 ln  sec x  tan x 
3
Equation (2) becomes: I5  sec x tan x  3  sec x tan x  1 ln  sec x  tan x
4 4 2 2 
sec3 x tan x 3 3
  sec x tan x  ln sec x  tan x 
4 8 8

 sin x cos x dx
p q
Example 04: What are the integrals with which can be connected by reduction

formulae?

 sin x cos x dx can be connected with any one of the following six integrals:
p q
Solution:

 i   sin p-2 x cosq x dx  ii   sin p x cosq-2 x dx  iii   sin p+2 x cosq x dx


 iv   sin p x cosq+2 x dx  v   sin p-2 x cosq+2 x dx  vi   sin p+2 x cosq-2 x dx
-:186:-
CHAPTER NINE DEFINITE INTEGRATION WITH APPLICATIONS

 sin x cos x dx , we may


p q
Thus, in finding a reduction formula for

 Decrease or increase by 2 the index of sin x leaving that of cosx unchanged as in (i), (ii)
 Decrease or increase by 2 the index of cos x leaving that of sin x unchanged as in (iii), (iv)
 Decrease the index of sinx by 2 and increase that of cos x by 2 as in (v)
 Increase the index of sin x by 2 and decrease that of cos x by 2 as in (vi)
But we cannot increase or decrease by 2 the indices of both sin x and cos x in the same formula.

 sin x cos x dx with any one of the above six integrals


p q
Method for connecting

Step 1: Take P  sin 1 x cos1 x, where λ is smaller of the two indices of sin x and μ is smaller
of the cos x in the two integrals which are to be connected.
dP
Step 2: Find and express it as a linear function of the two integrands whose integrals are
dx
being connected.
Step 3: Integrate both sides with respect to x, transpose and solve for the given integral.

  
Example 05: Connect sin p x cosq x dx with sin p-2 x cosq x dx . Hence evaluate sin 4 x cos2 x dx

Solution: Let P  sin p21 x cosq 1 x. Differentiating both sides with respect to x, we get
dP
  p  1 sin p2 x  cos x  cosq1 x  sin p1 x  q  1 cosq x   sin x 
dx
  p 1 sin p2 x cosq2 x   q  1 sin p x cosq x   p 1 sin p2 x cos q x cos 2 x   q 1 sin p x cosq x
  p 1 sin p2 x cosq x 1  sin 2 x    q  1 sin p x cos q x
 
  p 1 sin p2
x cos x   p 1 sin p x cosq x   q  1 sin p x cos q x
q

  p 1 sin p2 x cosq x   p 1  q  1 sin p x cosq x


  p 1 sin p2 x cosq x   p  q  sin p x cosq x
Integrating both sides with respect to x, we get

 
P   p 1 sin p2 x cosq xdx   p  q  sin p x cosq xdx

 p  q   sin p x cosq xdx  P   p 1  sin p2 x cosq xdx (1)

which is the required reduction formula.


To evaluate sin 4 x cos2 x dx put p = 4 and q = 2 in 1 , we have

sin 2 x cos2 x 3
 sin 4 x cos2 x dx  
6
 sin 2 x cos2 xdx
6  (2)

Again putting p = 2 and q = 2 in (1) we have


sin x cos3 x 1 sin x cos2 x 1
 sin 2 x cos2 xdx  
4
 sin0 x cos2 xdx  
4  4
 cos2 xdx
4 
sin x cos x 1  1  cos 2x 
3 3
sin x cos x 1

4
 
4  2   dx  
 4
 1  cos 2x  dx
8 
3
sin x cos x 1  sin 2x 
  x 
4 8 2 

-:187:-
CHAPTER NINE DEFINITE INTEGRATION WITH APPLICATIONS

 sin x cos x dx in (2), we have


2 2
Putting this value of

sin 3 x cos3 x 1  sin x cos3 x 1  sin 2x  


 sin 4 x cos2 x dx  
6
 
2  4
 x 
8

2  
sin 3 x cos3 x sin x cos3 x 1  sin 2x 
   x 
6 8 16  2 
APPLICATIONS OF INTEGRATION
This section includes a variety of applications of anti-differentiation. In each instance, the value
of the constant C will be determined for the specific situation. When this is done, the resulting
anti-derivative is called a particular anti-derivative or may be thought of as particular solution.
Example 01: A company finds that the marginal cost when x units of merchandise is produced is
50 – 0.08x dollars. If the fixed cost (overhead) is $700, determine
(a) the cost of producing x units (b) the cost of producing 10 units
Solution: As we know that marginal cost is the derivative of cost function. Using C(x) for the
cost function, we have: C(x) = 50 – 0.08 x
Integrating both sides with respect to x, we get
0.08 2

C(x)   50  0.08x  dx  50x 
2
x  c  50x  0.04x  c (1)

Since fixed cost (overhead) is $700, we know that the cost of producing zero units is $700. In
other words, C(0) = 700. This information can be used to find the constant C.
C(0)  50  0   .04  0   C  700  0  0  C  C  700 . Thus,
2

2
 using 1 
2
(a) C(x) = 50x – 0.04 x + 700 (b) C(10) = 50(10) – 0.04(10) + 700 = $1196
Example 02: A toy rocket is shot vertically upward from the ground with an initial velocity of
300 feet per second. The acceleration due to gravity is -32feet per second per second-negative
because it is downward. (No other acceleration is applied to the rocket.)
(a) Find a formula for the rocket’s velocity t seconds after the launch.
(b) Find a formula for the rocket’s distance above the ground at any time t.
Solution: (a) As we know that the acceleration is the derivative of velocity, so, a = dv/dt
The acceleration is given as –32, so we have: dv/dt = –32

Integrating both sides with respect to t, we get: v   32  dt  v  32t  C (1)
To determine C, use the fact that the initial velocity is 300 feet per second. This mean v = 300
when t = 0. Substituting these two numbers into the equation v = –32t + C, we get
300 = – 32(0) + C giving C = 300
Thus, we have v = –32 t + 300  using 1 
(b) We also know that the velocity is the derivative of distance. That is;
v = ds/dt = – 32 t + 300
.

Integrating both sides with respect to t, we get


32
s   32t  300  dt   t 2  300t  C  s  16t 2  300t  C (2)
2
To find C, note that at the beginning (when t = 0) the rocket’s distance s above the ground is zero,
because it is shot upward from the ground. Substituting 0 for t and 0 for s into (2), we get:

-:188:-
CHAPTER NINE DEFINITE INTEGRATION WITH APPLICATIONS
0 = – 16(0)2 + 300(0) + C giving C = 0
2
Thus, (2) becomes: s = – 16 t + 300t
Example 03: To test learning, a psychologist asks people to memorize a long sequence of digits.
Assume that the rate at which digits are being memorized is dy/dt = 5.4 e– 0.3t, where y is the
number of digits memorized and t is the time in minutes.
a. Find y as a function of t, which gives the number of digits memorized after t minutes.
b. How many digits will be memorized after 5 minutes?
Solution: (a) Here dy/dt = 5.4 e–0.3t
Integrating both sides with respect to t, we get
5.4 0.3t

y  5.4 e0.3t dt 
0.3
e  C  18e0.3t  C (1)

To find C, we use the fact that in the beginning (when t = 0), the number of digits memorized is
zero (that is, y = 0). Thus, y = 0 when t = 0 and (1) gives
0  18e0.3t  C  18 1  C  C  18
Thus, the number of digits memorized (y) as a function of time (t) is
–0.3t
.y = 18 – 18e  using 1  .

(b) After 5 minutes (t = 5), the number of digits memorized is


y  18 18e-0.3t  y  18 18e1.5  y  18 18 .2231 or y  14
Approximately, 14 digits will be memorized after 5 minutes.
Example 04: If the marginal cost when x units is produced is 100 – 0.5x dollars and the overhead
cost is $40, what is the cost of producing 10 units?
Solution: Here C` (x) = 100 – 0.5x. Integrating both sides, we get
0.05 2

C  x   100  0.5x  dx  100x 
2
x  c  C  x   100x  0.25x 2  c (1)

Since overhead is $40, we know that the cost of producing zero units is $40. In other words,
C(0) = $40. This information can be used to find the constant c.
2
C(0) = 100(0) – 0. 25(0) + c or 40 = 0 + 0 + c giving c = 40
2
Thus, C(x) = 100 x – 0.25x + 40 [using (1)]
2
Now the cost of producing 10 units is: C(1) = 100(10) – 0.25(10) + 40 = $1015
Example 05: Let R(x) be the revenue a company receives from the sale of x units of its product.
If the marginal revenue R`(x) is 100–0.2x dollars, determine (a) R(x) (b) the revenue from the
sale of 20 units, assuming there is no revenue when zero units are sold.
Solution: (a) Since, R`(x) = 100 – 0.2x. Integrating both sides with respect to x, we get
0.2 2

R  x   100  0.2x  dx  100x 
2
x  c  R  x   100x  0.1x 2  c (1)

It is assumed that there is no revenue when zero units are sold, that is, when x = 0,
R(0) = 0. Therefore, from (1), we get: 0 = 100(0) – 0.1(0) + c or 0 = 0 + 0 + c giving c = 0
Thus, (1) becomes, R(x) = 100x – 0.1x
(b) The revenue from the sale of 20 units will be
2
R(20) = 100(20) – 0.1(20) = 2000 – 40 = $1960
-:189:-
CHAPTER NINE DEFINITE INTEGRATION WITH APPLICATIONS
Example 06: Determine the profit function P(x) that corresponds to the given marginal profit
-0.01x
P`(x) = 70 – e , P(0) = – $30.
-0.01x
Solution: Given that P`(x) = 70 – e . Integrating both sides, we get

  70  e  dx  70x  0.01
1
Px  0.01x
e0.01x  c  P  x   70x  100e0.01x  c (1)

Since, P(0) = – 30, equation (1) becomes: – 30 = 70(0) + 100.e0 + c giving c = – 130
Hence, (1) becomes P(x) = 70x + 100 e–0.01x – 130
Example 07: A ball is shot vertically upward from the edge of a building with initial velocity 352
feet per second. The building is 768 feet tall. Acceleration due to gravity is -32feet per second per
second.
(a) Determine the equations that describe the velocity of the ball and its distance from the ground.
(b) How far above the ground is the ball after 6 seconds and how fast is it going then?
Solution: (a) We know that the acceleration is the derivative of velocity. Therefore,
.a = dv/dt = –32 [since a = -32 feet / sec2 ]


Integrating both sides, we get: v  32 1dt  v  32t  C (1)

To find C, use the fact that the initial velocity is 352 feet/sec.
This means v = 352 when t = 0. Thus, (1) becomes, 352 = – 32(0) + C giving C = 352
Thus, we have v = –32t + 352 (2)
Also, we know that velocity is the derivative of distance, that is;
v = ds/dt = –32t + 352
. [from (2)]
Integrating both sides, we get:
t2
s   32t  352 dt  32 2  352t  C  s  16t 2  352t  C (3)

To find C, note that at the beginning (when t = 0) the ball’s distance s is 768, because it is shot
upward from the edge of the building 768 feet high. Substituting 0 for t and 768 for s into (3), we
get: 768 = – 16(0) + 352(0) + C giving C = 768.
2
Thus, s = – 16t + 352t + 768 (4)
Hence the equation that describes the velocity of the ball is v = – 32t + 352 and its distance from
2
the ground is s = – 16t + 352t + 768.
2
(b) At t = 6 sec, we have from (4), s = – 16(6) + 352 + 768 = – 192 + 352 = 544 feet
Also from (2), we have v = – 32(6) + 352 = – 192 + 352 = 160 ft/sec.
Example 08: A tourist accidentally drops his camera from the top of a cliff that is 576 feet above
the water below. Assume the acceleration due to gravity to be -32feet per second per second.
(a) Determine the velocity v(t) of the camera at any time t during its fall.
(b) Determine s(t), the height of the camera above the water at any time t during its fall.
(c) How fast is the camera falling 4 seconds after it is dropped?
(d) How long will it take the camera to hit the water? (Hint: What is the value of s when the
camera hits the water?)
Solution: (a) As we know that the acceleration is the derivative of velocity. That is;
.a = dv/dt = – 32 [since, a = – 32 ft/sec]

-:190:-
CHAPTER NINE DEFINITE INTEGRATION WITH APPLICATIONS


Integrating both sides, we get: v  32 1dt  v  32t  C (1)

Initially, the velocity was zero, that is; when t = 0, v = 0, therefore (1) becomes,
0 = –32(0) + C giving C = 0
So equation (1) becomes, v = – 32 t (2)
(b) Also, we know that velocity is the derivative of distance, that is;
.v = ds/dt = – 32t [ from (2)]
2
Integrating both sides, we get s  32 t  C  s  16t 2  C (3)
2
2
Since it is given that s = 576 feet, so at t = 0, (3) becomes, 576 = –16(0) + C giving C = 576
2
Thus, equation (3) becomes , s = – 16t + 576 (4)
(c) At t = 4 sec, (2) becomes, v(4) = – 32(4) = – 128.
Thus, the camera is being fallen 128 feet/sec fast, 4 seconds after it is dropped.
(d) Since when the camera hits the water, its distance will vanish, so from (4) we have
2 2
0 = – 16t + 576 or t = 36 giving t = 6
Thus, the camera will take 6 seconds to hit the water.
Example 09: A woman gets into her car and then drives it with a constant acceleration of 22 feet
per second per second.
(a) Determine the velocity function.
(b) Determine the distance function.
(c) How far does the car go in 6 seconds?
Solution: (a) We know that the acceleration is the derivative of velocity. That is;
.a = dv/dt = 22 [ since a = 22 feet / sec2 ]
Integrating both sides with respect to t, we get: v=   22 dt = 22t + C (1)

Initial velocity of the car was zero when t = 0, so (1) becomes, 0 = 22(0) + C giving C = 0
Substituting C = 0 into (1), we get v = 22t, which is the required velocity function.
(b) Now, we know that velocity is the derivative of distance, that is; v = ds/dt = 22t


Integrating both sides with respect to t, we get: s  22 t dt  11t 2  C  s  11t 2  C (2)
At the beginning, the car covers no distance, that is; when t = 0, s = 0, so (2) becomes
0  11 0   C  C  0
2

Substituting C = 0 into  2 , we get s = 11t


2
(3)
(c) At t = 6 sec, (3) becomes, s = 11(6)2 = 11(36) = 396 feet. Thus, the car goes 396 feet far in 6
seconds.
Example 10: On the moon the magnitude of the acceleration due to gravity is less than on the
earth; it is approximately –5.3 feet per second per second. Consider a ball thrown upward from
the surface of the moon with a velocity of 120feet per second.
(a) Obtain a function that gives the velocity of the ball at any time t.
(b) Determine a function that shows the distance of the ball from the moon’s surface at any time t.
Solution: (a) We know that the acceleration is the derivative of velocity. That is;
a = dv/dt = –5.3
.

-:191:-
CHAPTER NINE DEFINITE INTEGRATION WITH APPLICATIONS
Integrating both sides with respect to t, we get
 
v   5.3 dt  5.3 1dt  v  5.3t  C (1)
Since initial velocity of the ball is 120feet/sec, that is; when t = 0, so (1) becomes,
120 = –5.3(0) + C giving C = 120
Substituting C = 120 into (1), we get v = – 5.3t + 120
This is the required function that gives the velocity of the ball at any time t.
(b) Now, we know that velocity is the derivative of distance, that is; v = ds/dt = –5.3t + 120
Integrating both sides with respect to t, we get
t2

s   5.3t 120 dt  5.3 120t  C  s  2.65t 2 120t  C (2)
2
Initially the ball covers no distance, that is; when t = 0, s = 0. Substituting these values into (2),
we get: 0 = – 2.65(0) + 120(0) + C giving C = 0
2
Thus, equation (2) becomes: s = – 2.65t + 120t
This is the required function that shows the distance of the ball from the moon’s surface at any
time t.
Example 11: The height h (in feet) of a tree is a function of time t (in years). Suppose you begin
(t = 0) by planting a 5-foot tree in your yard and the tree grows to maturity according to the
formula: dh = 0.3+ 1 , t > 0
dt t
(a) Determine a formula for the height of the tree at any time t.
(b) Find the height of the tree after 1year, 4years, 9years, and 16years.
Solution: (a) Here dh 1
= 0.3+
dt t
Integrating both sides with respect to t, we get
 1 t1/2

h   0.3   dt  0.3t 
t 12
 C  h  0.3t  2 t  C (1)

At t = 0 and h = 5. Substituting these values into (1), we get


5  0.3  0   2 0  C  C5

Now, (1) becomes h = 0.3t +2 t +5 (2)


which is the required formula for the height of the tree at any time t.
When t = 1, h  0.3 1  2 1  5  h  7.3 feet.
When t = 4, h  0.3  4   2 4  5  h  10.2 feet.
When t = 9, h  0.3  9   2 9  5  h  13.7 feet.
When t = 16, h  0.316  2 16  5  h  17.8 feet.
Example 12: From the collected data, the health office estimates that a flu virus is spreading
2/3
through the country at the rate of 5t + 22 people per day.
(a) If n is the number of people who have the flu at any time t, where t is the time in days,
complete the equation dn/dt = …
(b) If 50 people had the flu at the beginning of the outbreak, determine an equation that expresses
n as a function of t.
(c) How many people have the flu after 8 days?

-:192:-
CHAPTER NINE DEFINITE INTEGRATION WITH APPLICATIONS
2/3
Solution: (a) Here dn/dt = 5t + 22
53

 5t  22 dt  5 5 3  22t  C


Integrating, n  t
23
 n  3t 5 3  C (1)

(b) Since n = 50 and t = 0, therefore, from (1) 50  3  0 5 3  C  C  50


5/3
Therefore equation (1) becomes n = 3t + 50 (2)
1×5 1
(c) Since t = 8, from (2) n = 3 t  3 +50 = 38 3×5 +50 = 332  +50 =146
Thus, after 8 days 146 people have the flu.
Example 13: Flu epidemic is spreading at the rate dn/dt = 180t -6t2 where n is the number of
people who are sick with flu on any particular day t after the outbreak started.
(a) Determine an equation for n as a function of t. Assume no one has the flu at the beginning
(when t = 0).
(b) How many people have the flu the tenth day after the outbreak begins?
2
Solution: (a) Here dn/dt = 180t – 6 t . Integrating both sides with respect to t, we get

 180t  6t  dt  180  2 t  6  3 t  C
1 2 1 3
n 2
 n  90t 2  2t 3  C (1)

It is assumed that no one has the flu at the beginning, that is; n = 0 when t = 0. Therefore,
2 3
0 = 90(0) – 2 (0) + C giving C = 0
2 3
Thus, (1) becomes n = 90t – 2t (2)
(b) Since, t = 10, therefore, from (2)
n = 90 10  - 2 10  = 90×100 - 2×1000 = 9000 - 2000 = 7000
2 3

Hence, 7000 people have the flu, the tenth day after the outbreak begins.
Example 14: A colony of 200 bacteria is introduced to a growth-inhibiting environment and
grows at the rate of dn/dt = 30 + 2t, where n is the number of bacteria present at any time t
(t is measured in hours).
(a) Determine a function that gives the number of bacteria present at any time t.
(b) How many bacteria are present after 3 hours?
Solution: (a) Given that dn/dt = 30 + 2t
Integrating both sides with respect to t, we get
1

n   30  2t  dt  30t  2  t 2  C  n  30t  t 2  C (1)
2
At the beginning, that is; when t = 0, 2000 bacteria were present. Therefore,
2
2000  30  0    0   C  C  2000
2
Equation (1) becomes n = 30t + t + 2000 (2)
2
(b) Since t = 3, from equation (2), we have: n = 30(3) + (3) + 2000 = 2099
Thus, 2099 bacteria were present after 3 days.
0.2t
Example 15: The weight of a mold is growing exponentially at the rate of dw/dt = e
milligrams per hour. How much will the mold weigh in 10 hours if it weighs 70 milligrams now?
0.2t
Solution: Given that dw/dt = e
Integrating both sides with respect to t, we get
-:193:-
CHAPTER NINE DEFINITE INTEGRATION WITH APPLICATIONS

  e  dt  0.2 e  C
1 0.2t
w 0.2t
 w  5e0.2t  C (1)

Since at the beginning it weighs 70 mg, that is; when t = 0. Therefore, from (1), we have
0.2(0)
70 = 5 e + C giving C = 65
Substituting it into (1), we get w = 5e0.2t + 65
At t = 10, w  5e0.2(10)  65  w  5 7.389  65  w  102
Thus, the mold will weigh 102 milligrams (approx.) in 10 hours.
Example 16: The rate of change of the temperature T inside a furnace after x minutes (0 ≤ x ≤ 20)
is dT/dx = 2x + 15 degree/min. Assume the temperature inside the furnace is 200 F initially.
(a) Find the formula for the temperature at any time x.
(b) What is the temperature inside the furnace after 14 minutes?
Solution: (a) Given that dT/dx = 2x + 15. Integrating both sides with respect to x, we get
1

T =  2x +15 dx = 2× x 2 +15x + C or T = x 2 +15x + C
2
(1)
o
Initially, the temperature was 200 F, that is; when x = 0, T = 200, therefore,
200   0   15  0   C  C  200
2

Thus, (1) becomes T = x 2 +15x + 200


(b) Since; x = 14, therefore, T  14 2  15 14   200  196  210  200  T  606F .
o
Thus, after 14 minutes, the temperature inside the furnace will be 606 F.
Example 17: Plaque builds up on the inside walls of an artery reduces the diameter of the artery
(and thus reduces the blood flow). Suppose that an artery has a diameter of .4 centimeter and the
–0.001 t
length of diameter (D) is decreasing at the rate of dD/dt = – 0.03e cm/year.
(a) Determine D as a function of t.
(b) What will the diameter of the artery be after 10 years?
–0.001 t
Solution: Given that dD/dt = – 0.03e . Integrating both sides with respect to t, we get

  0.3e  dt  0.3 0.001


0.001 t 1
D e0.001 t  C  D  30e0.001 t  C (1)

At the beginning when t = 0, the diameter D was of 0.4 cm. Therefore,


0.4  30e0.001(0)  C  0.4  30  C  C  29.6
Thus, (1) becomes D = 30e-0.001 t - 29.6
(b) After 10 years, D  30e0.001(10)  29.6  D  29.7  29.6  D  0.1
Thus, after 10 years the diameter of the artery will be 0.1centimeter (approx.).
Example 18: After t hours of production, a coal mine is producing coal at the rate of
2
dc/dt = 30 + 2t – 0.03t . tons of coal per hour. Find a formula for the total output of the coal mine
after t hours of production.
2
Solution: Given that, dc/dt = 30 + 2t – 0.03t . Integrating both sides with respect to t, we get

 30 + 2 t - 0.03t  dt = 30 t + 2× 2 t - 0.03× 3 t + C


2 1 2 1 3
c=

or, c  t  = 30t + t 2 -0.01t 3 + C (1)

-:194:-
CHAPTER NINE DEFINITE INTEGRATION WITH APPLICATIONS
c  0   30  0    0   .01 0   C  C  0
2 3
Initially, when t = 0, there was no coal, therefore,
Thus, (1) becomes c  t  = 30t + t 2 -0.01t 3
Example 19: The rate at which atmospheric pressure P changes as the height x above sea level
changes is dP / dx = -3.087e-0.21x (P is measured in pounds per square inch and x is in miles, which
means that dP/dx is in pounds per square inch per mile.) Determine P as a function of x. At sea
level P is 14.7 pounds per square inch.
Solution: Here dP / dx = -3.087e-0.21x . Integrating both sides with respect to x, we get

  3.08e  dx  3.08 .21


0.21x 1 0.21x
P e C  P  14.7e0.21x  C (1)

Since at sea level, x = 0, P = 14.7, therefore, 14.7  14.7e0.21(0)  C  14.7  14.7  C  C  0


Thus, (1) becomes P  14.7e0.21x pounds/square inch.
Example 20: The rate of the change of the area of a circular region with respect to its radius is
dA/ dr = 2πr . Use this fact, and the fact that A = 0 when r = 0, to determine the area of a circular
region when the radius is 4 centimeters.
Solution: Given that , dA/ dr = 2πr .Integrating both sides with respect to r, we get
1

A   2r  dr  2 r 2  C  A  r 2  C
2
(1)

Now when r = 0, A = 0, then 0    0  C  C  0


2

using r = 4cm
2
Thus (1) becomes, A  r 2  A  16 cm .
Example 21: The rate of change of the volume of a spherical balloon with respect to its radius is
dV / dr = 4πr 2 . Use this fact, and the fact that V = 0 when r = 0, to determine volume of the
balloon when its radius is 6 centimeters.
Solution: Here dV / dr = 4πr 2 . Integrating both sides with respect to r, we get

 
2 4
V = 4πr dr = πr 3 + C (1)
3
When r = 0, V = 0, therefore, 0  4   0 3  C  C  0. Thus, (1) becomes V = 4 πr3
3 3
4 4
r = 6 cm, therefore, V = π  6  = π  216  = 288π  904 cm3
3
Now
3 3
Example 22: A manufacturer estimates that marginal cost of a certain production process is given
by C  x  = e0.01x + 3 x , where x units are produced. What does it cost to produce 10 units if the
cost of producing 4 units is $ 2000?
Solution: Here C  x  = e0.01x + 3 x . Integrating both sides with respect to x, we get

 e +3 x  dx = e0.01x dx + 3 x dx =
1 0.01x 1 32
C x  =  
0.01x
e +3× x +C
0.01 32
 C  x   100e0.01x  2x3 2  C (1)
Since it is given that C  x  = $2000, when x  4 units, then (1) becomes

2000  100e0.01(4)  2  4 
32
 C  2000  104  16  C  C  1880
Thus, (1) becomes C  x  =100e0.01x + 2x3 2 +1880 (2)
-:195:-
CHAPTER NINE DEFINITE INTEGRATION WITH APPLICATIONS
At x = 10, C 10   100e0.01(10)  2 10   1880  C 10   111  32  1880  2023
32

Hence the cost to produce 10 units is $2023 (approx.).


Volume of Solid of Revolution
In this presentation you will see how a definite integral can be used to find the volume of a solid
of revolution. Such a solid is produced by revolving a plane region (an area) about a line (such as
the x – axis). Consider the plane region shown shaded in the following figure 1. Now, imagine
revolving this region about the x – axis. As it spins around the x – axis, it sweeps out a three –
dimensional figure – a solid of revolution. See figure (2).
y y

.fig. 1 fig. 2

O x O x

This particular solid is cone – shaped. A formula for finding the volume of such solids of
revolution can be obtained in a manner similar to the way the basic area formula was obtained.
Consider a function y = f(x) that is non – negative and continuous on an interval [a, b]. Divide
this interval of width (b – a) into n equal subintervals of width Δx. We then have:
x  (b  a) / n
Let us draw the rectangles as we did in the topic of “Definite Integral as area under a curve.” The
heights of the rectangles are f(x1), f(x1), f(x1), … and so on. See the figure (3).
As the region is spun around the x – axis, each rectangle generates a cylinder. For example, the
first rectangle with width ∆x and height f(x1), generates a cylinder having radius f(x1), and height
∆x. See the following figure 4.
y
. y = f(x) fig. 3
y = f(x)
fig. 4

x ∆x x x
A rectangle generates a cylinder
2
The volume of a cylinder is πr h where r is the radius and h is the height. Here r = f(x1) and
2
h = ∆x. Thus, the volume of the first cylinder generated is π f(x1) ∆x.
2
The volume of the second cylinder is π f(x2) ∆x.
The total volume generated by revolving all rectangles is
f  x1  x  f  x 2  x  ...  f  x n  x
2 2 2

  f  x i 
2 The rectangles generate cylinders
In summation notation, we have: V  x
i 1
This volume is approximately the volume of the solid of revolution. The larger n becomes, the
smaller ∆x becomes, and better is the approximation. The exact volume of the solid is the limit of
this sum as n → ∞ or as ∆x → 0.

-:196:-
CHAPTER NINE DEFINITE INTEGRATION WITH APPLICATIONS

 f  x  x, the entire solid 


2
V  Lim
x 0 i 1

The limit on the right is the definite integral shown next.


The volume V of the solid produced by revolving the region bounded by y = f(x) and the x – axis
b


(between x = a and x = b) about the x – axis is: V   f  x  dx , provided that function f(x) is
2

continuous on [a, b].


2
Example 23: Find the volume of the solid of revolution obtained by revolving the curve y = x
about the x – axis between x = 1 and x = 3.
b


Solution: Here V   f  x  dx . Substituting the values, we get
2

a
3 3
V    x 2  dx   x 4dx     x5    35 15     242   48.4
1 3 1 1
 
2

1 1 5 1 5 5
2
Thus, the volume of the solid of revolution obtained by revolving the curve y = x about the
x – axis between x = 1 and x = 3 is 48.4 π cubic units.
Example 24: Find the volume of the solid of revolution obtained by revolving the curve y = 1/ x
about the x – axis between x = 1 and x = 2.
b


Solution: Since V   f  x  dx . Substituting the values, we get
2

a
2 2 2 2 2
1 x 1 1 1   1 1
1

V     dx   x 2dx  
x
  1
 1 1

 
x 1
   1       
 2   2 2

Hence the volume is π /2 cubic units.


Example 25: Find the volume of the solid of revolution obtained by revolving the curve
2
y = x + 1 about the x – axis on the interval [0, 3].
b


Solution: Here V   f  x  dx . Substituting the values, we get
2

a
3 3 3
V = π  x +1 dx = π  x + 2x +1
1 2 
 
2
2 4 2
dx = π  x5 + x 3 + x 
0 0  5 3 0

1 2 3 
 3 + 3  3 + 3- 0  = π  48.6 +18 + 3 = 69.6π
5
V = π
5 
Hence the volume is 69.6 π cubic units.
Area between the Two Curves
We know that one of he use of definite integration is to find the area under the curve y = f (x)
between the x-axis and the ordinates at say x = a and x = b. The definite integration is also useful
in finding the area between the two curves y = f(x) and y = g(x). This is shown graphically as
under:

-:197:-
CHAPTER NINE DEFINITE INTEGRATION WITH APPLICATIONS
f(x) f(x)

g(x) g(x)

Fig. (1)
a b a Fig. (2) b
In the first figure (1) we se that the curves do not intersect whereas in the second figure the curves
intersect each other at x = a and x = b. In either case, the area between theses curves is computed
b
by using the formula: A
 f  x   g  x dx
a
2
Example 26: Determine the area of the region enclosed by y = x and y = x.
Solution: Since the limits are not given we have to solve two equations simultaneously to get
x = 0 and x = 1. The graphs are shown in the following figure. Therefore,
1 1

  x – x  dx =  2 x – 3 x 0 = 2 – 3 = 6 sq.units
2 1 2 1 3  1 1 1
Area =
0

3
Example 27: Determine the area of the region enclosed by y = x and y = x.
Solution: Solve the two equations simultaneously we obtain x = 0, x = –1 and x = 1.The required
area in fact is the union of areas of two different regions. The first region starts from x = –1 to
3
x = 0 where the curve y = x lies below the curve y = x . The second region starts at x = 0 and
3
ending at x = 1where the curve y = x lies below the curve y = x. Graph of the above curves is
shown as under:
0 1 0 1

  x – x  dx +   x – x  dx =  4 x – 2 x -1 +  2 x – 4 x 0
Thus the required area is: Area = 3 3 1 4 1 2  1 2 1 4 
1 0

1 4 1 2 1 2 1 4 1 1 1 1 1
Area = 0 – 0 –
4
 –1 + 2  –1 + 2 1 – 4 1 – 0 + 0 = – 4 + 2 + 2 – 4 = 2 sq.units
The figure is shown on the next page.

-:198:-
CHAPTER NINE DEFINITE INTEGRATION WITH APPLICATIONS

3 2
Example 28: Determine the area of the region enclosed by y = x and y = x .
Solution: We have y  x3 and y  x 2  x3  x 2  x 2  x 1  0  x  0,1 .Hence the limits
3
are from x = 0 to x = 1. In this interval, the graph of the curve y = x lies below than that of
1 1

x 
2 2 3 1 1  1 1 1
y = x , so the required area is: Area = -x dx =  x3 - x 4  = - = sq.units
0 3 4 0 3 4 12
Scale: On X-axis 1 small square = 0.2 unit and on Y-axis 1 small square = 1unit
8
7
6
5
4
3 2
y=x
2
y=x3
1
Y-axis

0
-1
-2
-3
-4
-5
-6
-7
-8
-2 -1.8 -1.6 -1.4 -1.2 -1 -0.8 -0.6 -0.4 -0.2 0 0.2 0.4 0.6 0.8 1 1.2 1.4 1.6 1.8 2
X-axis

2
Example 29: Determine the area of the region enclosed by y = 5x – 6 and y = x .
Solution: Solving the two equations simultaneously, we see that x = 2 and x = 3. If we plot the
curves, we see that the graph of the curve y = 5x – 6 lies above than the curve y = x2. Therefore,
the required area is given by:
3 3
Area =  5x -6- x2  dx =  52 x2 -6x - 13 x3 2 =  52 9 -6 3 - 13  27  -  52  4 -6  2 - 13 8
2
45 8 1
 18  9 10  12   sq.units
2 3 6
[Note: Students are advised to draw the graphs of both functions]
2
Example 30: Determine the area between the curve y = x – 4 and x – axis from x = 1 to x = 2.
Solution: The required area is given by:
2 2 2
Area = 0-  x 2 - 4 dx =   4- x  dx = 4x - 3 x 1 = 8- 3  -  4- 3  = 3 sq.units
 1 3  8  1 5

2

1 1

-:199:-
CHAPTER NINE DEFINITE INTEGRATION WITH APPLICATIONS
The graph is show below

WORKSHEET 09
1.The rate at which petroleum was consumed in Pakistan was approximately c  t   21t  281
million barrels per year from 1983 (t = 0) to 1987 (t = 4). Determine the total amount of
petroleum consumed from 1983 to 1987.
2. A man starts his car and then drives it with a constant acceleration of 18 feet per second per
second. How far does the car go in 5 seconds?
3. Find the volume of the solid produced by revolving about the x – axis the region whose
boundary is given. (Note: y = 0 is the equation of the x – axis.)
4. Determine the area enclosed between the two curves. Graphing is recommended:
(i) y  x  5, y  x from x  0 to x  4 (ii) y  x 3 , y  x 2
(iii) y  1  x 2 , y  x  4 from x  3 to x  1 (iv) y  e x , y  1 and x  1
5. Determine the area enclosed by the curves. In each case, the region enclosed consists of two
regions. A careful study of the graphs is essential.
(i) y  x 3 , y   x: (ii) y  x 4 , y  x 2
6. Let t be the number of years from now. What is the total amount of waste dumped into a lake
by a factory that dumps waste at the rate of 5e-0.1t pounds per year indefinitely?
2 2
cot x sin x
7. Evaluate: i   dx  ii   dx
0 cot x  tan x 0 sin x  cos x
8. Show that:
ln 1  x 
2  1 1
2  ln 2
iii   ln   1 dx  0 iv 
x sin x 1
 i   ln  tan x  dx  0  ii   dx   dx 
0 1  cos2 x 0
4 0 
x  0 1 x 2 8
9. Discuss the convergence of
   2 2
x x2 dx x dx cos x
(i)  1  x 3 dx (ii)  xe dx (iii)  1  x  x
(iv)  x 1
(v)  1  sin x
dx
1 0 1 1 0
  2 2
3 3
10. Evaluate : (i) 3 x
 x e dx (ii)  xe x dx (iii)  sin3 x cos5 2 x dx (iv)  sin
7
x dx
0 0 0 0
2 2
(v)  tan  d (vi)  tan  d
0 0
11. Use the definition of Gamma Function, evaluate the following:
  
2 2x 2
(i)  xe x dx (ii) 4 x
 x e dx (iii)  e h dx
0 0 0

-:200:-
CHAPTER TEN VECTOR ANALYSIS WITH APPLICATIONS

CHAPTER
TEN VECTOR ANALYSIS
WITH
APPLICATIONS
INTRODUCTION
In this introductory section, we shall be concerned with defining some basic and important
concepts of scalars and vectors. Some definitions and concepts will be given which play
considerable important role in the vector analysis.
Definition: A scalar is a quantity having magnitude but no direction along with some unit of
measurement. For example; mass, length, time, temperature, density, distance, area, volume, and
any real number.
Scalars are indicated by letters in ordinary type as in elementary algebra. Operations with scalars
follow the same rules as in elementary algebra.
Definition: A vector is a quantity having both magnitude and direction along with some unit of
measurement, such as displacement, force, velocity and acceleration.
If we are told that a person has moved a distance of 10 m from a point O,
he may be anywhere on the circumference of a circle of radius 10 m with
its centre at O (see fig).
This means that the distance 10 m is not sufficient to locate the position
of the person. Therefore, additional information about direction of motion must be supplied along
with the distance. Now if we say that the person has moved 10 m east of O, then his position is
precisely located and the displacement vector OA or OA is completely determined.
Graphical Representation of a Vector P
Graphically, a vector is represented by an arrow OP (see fig) defining
the direction and magnitude of the vector being indicated by the length a =A b= B
of the arrow. The tail end O of the arrow is called the origin or initial
point of the vector, and the head P is called the terminal point. O
Analytically, a vector is represented by a letter with an arrow over it, such as a or a (see fig) and
its magnitude or length is denoted by | a or |a|.
[Note: We shall denote the vector by bold face letters]
Definition: Two vectors a and b are said to be ‘equal’ if they have the same magnitude and
direction regardless of the position of their initial points. In this case we write a = b.
(see the above fig).
Definition: A vector having the direction opposite to that of vector a but having the same
magnitude is denoted by –a and is called the ‘negative’ of the vector a.
a –a
-:201:-
CHAPTER TEN VECTOR ANALYSIS WITH APPLICATIONS
Definition: The vector of zero magnitude is called the ‘null or zero vector’. In fact, the null
vector is an imaginary vector and it may have any direction.
Definition: The vector whose magnitude is unity is called a unit vector. If we divide the given
vector a by its magnitude then we obtain the unit vector u in the direction of vector a. For
example, the unit vector along the vector u is given by u = a/|a| or u = a/a, where `a` is the
magnitude of the vector a.
This definition of unit vector suggests that any vector a can be represented by the product of a
unit vector u in the direction of a and its magnitude `a`. Symbolically, this is written as: a = a u.
Definition: Two nonzero vectors a and b are said to be collinear if there exists a non – zero a
non-zero real constant `k`, such that a = kb. a
If k > 0 then the vectors a and b have the same direction. 2a
If k < 0 then the vectors a and b have opposite directions. –2a
If a vector a is multiplied by a scalar k, then ka is a vector
parallel to a with length |k| times that ofa. The direction of ka remains unchanged if k > 0 and it is
reversed if k < 0. In the above figure, the multiplication of the vector a by 2 and –2 is shown.
Rectangular Unit Vectors and Their Representation in 3-Dimensions
The vectors .i = [1, 0, 0],j = [0, 1, 0] and k = [0, 0, 1] are known as rectangular k
rectangular unit vectors. These are depicted in the figure shown.
Now any non-zero vector a = [a1, a2, a3] in R can be expressed as:
. a = [a1, a2, a3] = [a1, 0, 0] + [0, a2, 0] + [0, 0, a3] j
= a1 [1, 0, 0] + a2 [0, 1, 0] + a3 [0, 0,1] = a1 i + a2 j + a3 k
. .i
Example 01: Show that the vectors a = [4/3, – 1, 3] and b = [2, –3/2, 9/2] are collinear.
Solution: The given vector a can be written as a = [4/3, – 1, 3] = (–2/3) [2, –3/2, 9/2] = (–2/3) b.
Since, a = (–2/3)b, it implies that vectors a and b are collinear vectors.
Definition: The vectors lying in the same plane are called the
‘coplanar vectors’. And if they do not lie in the same plane they
are called ‘non–coplanar vectors’. The vectors a, b, and c shown
in the adjacent figure are coplanar vectors.
Definition: The vectors intersecting at a single point are called the
concurrent vectors and point where the vectors intersect is called the ‘point
of concurrency’. Three concurrent vectors are shown in the adjacent figure
are concurrent vectors.
Definition: If P1(x1, y1, z1) and P2(x2, y2, z2) are two points in the space
3-D, the vector represented by a = P1P2 = (x2 – x1)i + (y2 – y1)j + (z2 – z1)k and its magnitude

or length is defined as |a| = = (x 2  x1 )2  (y2  y1)2  (z 2  z1) 2 which in fact is the distance
formula and it represents the distance between two points in the space.
Now, if we let, x = (x2 – x1), y = (y2 – y1) and z = (z2 – z1) then magnitude of vector a is given by

a = xi + yj + zk and its magnitude by |a| = a = x 2  y2  z2 .


Definition: If any vector is divided by its own magnitude, a new vector formed is called a unit
vector and is usually denoted by u. Thus, if a = xi + yj + zk then unit vector in the direction of a

-:202:-
CHAPTER TEN VECTOR ANALYSIS WITH APPLICATIONS
Is given by: u = a/|a| = (xi + yj + zk) / x 2 + y2 + z2
Example 01: Find the magnitude of the vector a = 5i – 3j + 9k. Also find the unit vector along a
Solution: The magnitude of the given vector a is: |a|  52  (3) 2  92  25  9  81  115
Also, the unit vector along a is given by: u = a/|a| = (5/ 115 )i – (3/ 115 )j + (9/ 115 )k
. . .

Example 02: Find the vector whose magnitude is 5 and is in the direction of the vector 4i – 3j + k
2 2 2 1/2
Solution: Let a = 4i – 3j + k. then |a| = [4 + (– 3) + 1 ] = 26 . Thus, unit vector in the
direction of vector a is: u = a/|a| = (4/ 26 )i – (3/ 26 )j + (1/ 26 )k
. . .

Definition: Two vectors a = a1i + a2j + a3k and b = b1i + + b2j + b3k are said to be parallel if the
ratio of their components is same. Thus the condition that two vector as shown above are parallel
a1 a 2 a 3
if = = .
b1 b2 b3
This is shown in the adjacent figure.
Example 03: Find the value of λ if the vectors 6i + j – k and λ i – 4j + 4k are parallel.
Solution: Since the given vectors are parallel, so λ = – 4 = 4  λ =  6  – 4  = – 24
6 1 –1
PRODUCT OF TWO VECTORS
When two scalars are multiplied, the result is a scalar, but when two vectors are multiplied, the
result may be scalar or vector. Thus, the product of two vectors is of two kinds, scalar product
and vector product.
Scalar or Dot Product of Two Vectors
The product of two vectors that produces a scalar quantity is known as scalar product of two
vectors. Since this product is shown by placing a dot (.) between the two vectors hence, it is also
known as a Dot Product. For example, if a force F is applied on a body that is displaced through
the vector d then the product F. d is called work done by the force F. It is denoted by:
W=F.d
Definition 1: If a and b are two non-zero vectors at an angle θ, their scalar product is defined as:
a . b = |a| |b| cos θ = a b cos θ, where 0 < θ < π
.

Definition 2: If a = a1i + a2j + a3k and b = b1i + + b2j + b3k are two non-zero vectors then dot
product between a and b is also defined as: a . b = a1b1 + a2 b2 + a3b3
From the two definitions, we conclude that: a b cos θ = a1b1 + a2 b2 + a3b3
 
1  a1b1 + a 2 b2 + a 3b3  a1b1 + a 2 b2 + a 3b3
This implies that: θ = cos   = cos-1  


 ab   a 2 +a 2 +a 2 b 2 +b 2 +b 2 
   1 2 3 1 2 3 

It may be noted that since, a . b = a1b1 + a2 b2 + a3b3 this implies that


2 2 2 2 2
. a . a = a1a1 + a2 a2 + a3a3 = a1 + a2 + a3 = |a| = a
Geometrical Meaning of Dot Product (Projection of a vector)
Let OP and OQ be the vectors a and b respectively.
Let angle POQ be θ. Draw QM perpendicular on OP.
Then, cos θ = |OM| /|OQ|

-:203:-
CHAPTER TEN VECTOR ANALYSIS WITH APPLICATIONS
This implies that : |OM| = OQ cos θ
Multiplying and dividing on right side by |OP|, we get
| OP || OQ |cos  abcos  = (a . b)/|a|
| OM |  (1)
| OP | a
Here, |OM| is known as projection of the vector b on vector a. This is usually, denoted by Pa b.
Since, a . b = b . a, therefore, we can interpret the projection of vector b on the vector a as
“The Projection of vector b on the vector a is given by Dot Product of Two
Vectors a and b divided by the magnitude of the vector a.”
From equation (1) above thus, we can say that dot product between two vectors a and b
is equal to projection of vector b on the vector a multiplied by the magnitude |a| of vector
a. That is, .a . b = (Pb a) |a|
Here, Pb a = (a . b)/|b| therefore, a . b = = (Pa b) |b| as well.
Example 01: A particle is acted on by constant forces 4i + j – 3k and 3i + j – k. The body is
displaced from the point i + 2j + 3k to 5i + 4j + k. Find the total work done by the forces.
Solution: Let F1 = 4i + j – 3k and F2 = 3i + j – k. Then the total force is
F = F1 + F2 = (4i + j – 3k ) + (3i + j – k.) = 7i + 2j – 4k.
If d is the displacement measured from the point i + 2j + 3k to 5i + 4j + k., then
. d = (5i + 4j + k) – ( i + 2j + 3k) = 4i + 2j – 2k
Now, the required work done by the given forces is
W = F . d = (7i + 2j – 4k) . (4i + 2j – 2k) = 7(4) + 2(2) + (– 4) (– 2) = 28 + 4 + 8 = 40 Joule
Example 02: Find the angle between the vectors a = i + 2j – k and b = –i + j – 2k
Solution: Let θ be the angle between the vectors a and b. Then using the above formula,
 
a1b1 + a 2 b2 + a 3b3
θ = cos-1  
2 


a12 + a 22 + a 32 2 2
b1 + b2 + b3 

 
θ = cos 1 1 –1 +  21 +  –1 –2   -1  1 
we have,  = cos   = 60°

 12 +  2 2 +  –12  –12 + 12 +  –2 2  2


 
Useful Results: i .i = 1 . 1 cos 0 = 1 . 1. 1 = 1. Similarly, j . j = k . k = 1
.

o
Also , i . j = 1 . 1 cos 90 = 1 . 1. 0 = 0. Similarly, j . k = k .i = 0
.

Theorem: Two non-zero vectors are perpendicular if and only if their dot product is zero.
Proof: Let the vectors a and b be two vectors that are perpendicular to each other, then
a . b = a b cos 90o = a b (0) = 0
. [because cos 90o = 0]
o
Now, let a . b = 0 then a b cos θ = 0 or cos θ = 0/(a b) = 0; which implies that θ = 90
o
Thus angle between vectors a and b is 90 hence, they are perpendicular to each other.
Example 03: For what values of p the vectors a = 2i – j + 2k and b = 3i + 2pj are perpendicular?
Solution: Since the given vectors are perpendicular, therefore
. a . b = 0  (2i – j + 2k) . (3i + 2pj) = 2(3) + (–1)(2p) + 2(0) = 6 – 2p = 0  p = 3.
Vector or Cross Product of Two Vectors
The product of two vectors that produces a vector quantity is known as vector product of two
vectors. Since this product is shown by placing cross (×) between the two vectors, hence it is also

-:204:-
CHAPTER TEN VECTOR ANALYSIS WITH APPLICATIONS
known as a “Cross Product”.
For example, let F be the applied force and r be the arm.
We define the Momentum or Torque as: τ=F×r
We know that momentum/torque is a vector quantity.
Definition 1: If a and b are two non-zero vectors acting at an angle θ,
their cross product is a vector quantity whose direction is
.

perpendicular to both a and b and whose magnitude is given by


.

|a × b| = |a| |b| sin θ = a b sin θ


It may be noted that a × b = a b sin θ n
Here n is a unit vector in the direction of a × b or perpendicular to both vectors a and b. This is
show in the above figure.
Definition 2: The vector product between two non-zero vectors a and b is also defined as:
ˆi ˆj kˆ
a×b = a a a3
1 2
b1 b2 b3

Theorem: Prove that the magnitude of the cross product of two


non - zero vectors a and b represents the area of parallelogram
having a and b as adjacent sides, that is;
Area of parallelogram = a × b.
Proof: We know that
Area of parallelogram = (Base)(Altitude) =|OA||BM|
Now sin  =|BM| / |OB|  |BM|=|OB| sin 
Therefore, area of parallelogram =| OA || OB | sin θ = |a| |b| sin θ = a × b.
Example 04: If a = 5i – 3j + 4k and b = 19i +7j – k then find a × b.
Solution: a × b = i(3 – 28) – j(–5 – 76) + k(35 + 57) = –25i + 81j + 92k
Example 05: Find the area of a parallelogram whose adjacent sides are i – 2j + 3k and 2i + j – 4k
Solution: Let, a = i – 2j + 3k and b = 2i + j – 4k. Now,
a × b = i(8 – 3) – j(–5 – 6) + k(1 + 4) = 5i + 10j + 5k
.

Now area of parallelogram = |a × b| = 5 + 10 + 5 = 150 = 5 6


2 2 2

NOTE: The area of a triangle = 1/2 (The area of parallelogram) = 0.5(a × b)


Scalar Triple Product or Box Product
Let a, b, and c be three vectors, then the scalar triple product of these vectors is represented by
a . (b × c) or b . (c × a) or c . (a × b)
.

This means a . (b × c) = b . (c × a) = c . (a × b)
These components are computed in the following way.
a1 a2 a3 b1 b2 b3 c1 c2 c3
. a . (b × c) = b1 b2 b3 , b . (c × a) = c1 c2 c3 and c . (a × b) = a1 a2 a3
c1 c2 c3 a1 a2 a3 b1 b2 b3
NB: The three determinants are equal as interchanging two rows will keep the same value of it.
-:205:-
CHAPTER TEN VECTOR ANALYSIS WITH APPLICATIONS
Parallelepiped
A solid figure with six faces that are parallelograms is called parallelepiped. In a rectangular
parallelepiped, the faces are rectangles. If the faces are squares, the parallelepiped is a cube.

Parallelepiped Rectangle- Parallelepiped Cube


Theorem: Prove that scalar triple product of three vectors represents the volume of a
parallelepiped.
Proof: Consider a parallelepiped OABCDEFG as shown in the figure. Let OA = a, OB = b and
OC = c.
Now, the volume of parallelepiped OABCDEFG is
VP = (Area of the base OBDC) . (Height)
VP = |b × c| |AM| (1)
Now, from the triangle OAM, cos θ = |AM|/|OA|
This gives, |AM| = |OA| cos θ = |a| cos θ. Thus,
equation (1) becomes: VP = |a| |b × c| cos θ = a . (b × c). This proves the result.
NOTE: If six prisms (Tetrahedron) of same size are
inter-connected with each other, a parallelepiped is formed. Thus,
VP = 6 VT or VT = VP/6 = [a . (b × c)]/6
Example 06: Find the volume of parallelepiped if
a = –3i + 7j + 5k, b = –3i + 7j – 3k and c = 7i – 5j – 3k
.

are its edges or vertices.


Solution: Volume of a parallelepiped = a . (b × c)
3 7 5
= 3 7 3  3  21  15  7  9  21  5 15  49   108  210  170  272
7 5 3
Since volume is always taken as positive, therefore volume of the parallelepiped is equal to 272
cubic units.
Tetrahedron (Triangular Pyramid)
A solid figure bounded by four triangular faces is called
tetrahedron.
Volume of Tetrahedron
If A  x1, y1,z1  ,B  x 2 , y2 ,z2  ,C  x3 , y3 ,z3  and D  x 4 , y4 ,z 4  and
are the vertices of a tetrahedron then its volume can be found in B the following way:
x1 y1 z1 1
1 x 2 y2 z 2 1
V =
T 6 x 3 y3 z 3 1
x 4 y4 z 4 1

-:206:-
CHAPTER TEN VECTOR ANALYSIS WITH APPLICATIONS
Example 07: Show that the volume of the tetrahedron whose vertices are (0, 1, 2), (3, 0, 1)
(4, 3, 6) and (2, 3, 2) is 6.
Solution: Using the formula for the volume of tetrahedron and putting the given values, we get
x1 y1 z1 1
1 x2 y2 z2 1 1
V =    3  6  2    4  2   8 12   2 3 3  3  12  6   3  6 18  12  6   6
T 6 x3 y3 z3 1 6 
x4 y4 z4 1
Vector Triple Product
If a, b and c are three non – zero vectors, then their vector triple product is usually denoted by
(a× b) × c or by a × (b × c) and is defined as follows:
(a× b) × c = (c . a) b – (c . b) a and a × (b × c) = (a . c) b – (a . b) c
From the above definitions, we see (a× b) × c ≠ a × (b × c) .
Example 08: If a = i + j + 3k, b = –i + 7j – 2k and c = 2i – j + k then verify the formula,
a × (b × c) = (a . c) b – (a . b) c
Solution: Taking left hand side, we get
ˆi ˆj kˆ
b × c  1 7 2 = i(7 – 2) – j(– 1 + 4) + k(1 – 14) = 5i – 3j – 13k
2 1 1

ˆi ˆj kˆ
So, a × (b × c) = 1 7 2 = i(–13 + 9) – j(– 13 – 15) + k(–3 – 5) = –4i + 28j – 8k (1)
2 1 1

Taking right hand side, we get: (a . c) b – (a . b) c


= [(i + j + 3k) . (2i – j + k)]( –i + 7j – 2k) – [(i + j + 3k) .(–i + 7j – 2k)](2i – j + k)
= [1(2) + 1(-1) + 3(1)] ( –i + 7j – 2k) – [1(-1) + 1(7) + 3(-2)](2i – j + k)
= 4( –i + 7j – 2k) – 0(2i – j + k) = –4i + 28j – 8k (2)
From (1) and (2), we see that : a × (b × c) = (a . c) b – (a . b) c
Scalar Product of Four Vectors
If a, b, c and d are four non – zero vectors, then scalar product of (a × b) with (c × d) is usually
denoted by (a × b) . (c × d) is known as scalar product of four vectors. To evaluate this product,
we first compute (a × b) and then (c × d). Finally, we compute their dot product to get the
required scalar product of four vectors.
Vector Product of Four Vectors
If a, b, c and d are four non – zero vectors, then scalar product of (a × b) with (c × d) is usually
denoted by (a × b) × (c × d) is known as vector product of four vectors. To evaluate this product,
we first compute (a × b) and then (c × d). Finally, we compute their vector product to get the
required scalar product of four vectors.
NOTE: Students are advised to compute (a × b) . (c × d) and (a × b) × (c × d) for the vectors
a = i + j + 3k, b = –i + 7j – 2k and c = 2i – j + k and d = 2i + j + 3k
.

-:207:-
CHAPTER TEN VECTOR ANALYSIS WITH APPLICATIONS
VECTOR FUNCTIONS
To each value of a scalar variable t in some range if there, exists a unique vector f in space, then f
is said to be a vector function of t. This is denoted by f(t). If a Cartesian system of coordinates is
chosen, then we can write f(t) = f1i + f2j + f3k
where f1, f2 and f3 are three scalar functions of variable t. They are also the components of f(t)
along x – axis, y – axis and z – axis, respectively. For example, the functions f and g as shown
below are the vector functions of time variable t.
2 2
f(t) = cos t i + t j + sin t k and g(t) = 5t i + t j + sin t k
.

Differentiation of Vector Functions


Let f(t) be continuous vector function of a scalar variable t. Then if the limit
f  t + Δt  - f  t 
lim
Δt 0 Δt
exists, it is called the derivative of f(t) with respect to t. It is denoted by df/dt or f `.
If df/dt exists at t = to, f(t) is said to be differentiable at t = to.
The function df/dt is itself a vector function of t and its derivative, if it exists, is called the second
2 2
derivative of f(t) and is denoted by d f/dt or f ``(t).
3 3 4 4 n n
Similarly, we define higher derivatives d f/dt , d f/dt , … d f/dt .
The vector function f(t) is differentiable if and only if its component functions f1(t), f2(t) and f3(t)
`
are differentiable for all t. and in this case, f `(t) = f1` i + f2`j + f3 k.
.

Vector Rules of Differentiation


Let f(t), g(t) be vector functions and φ(t) be a scalar function of t, then
(i) [f(t) ± g(t)] ` = f `(t) ± g `(t) (ii) [f φ(t)] ` = f `(t) φ(t) + f `(t) φ`(t)
(iii) [f(t) . g(t)] ` = f(t) . g `(t) + f `(t) . g(t) (iv) [f(t) × g(t)] ` = f(t) × g `(t) + f `(t) × g(t)
Note: In (iv) the order of the functions f (t), g(t) and their derivatives is not to be changed because
cross product is not commutative.
3 2
Example 01: A particle moves along the curve x = t + 1, y = t , z = 2t + 5 where t is the time.
Find the velocity and acceleration of the particle at time t = 1.
3 2
Solution: Given that x = t + 1, y = t , z = 2t + 5, let the distance vector be denoted by r’ hence
3 2
r = xi + yj + zk = (t + 1)i + t j + (2t + 5)k. Differentiating with respect to t, we get
.

2 2
dr/dt = velocity = (3t + 0)i + (2t)j + (2 + 0)k = 3t i + 2tj + 2k
.

When t = 1, we have: v = dr/dt = 3i + 2j + 2k


Now, acceleration a = dv/dt = 6ti + 2j + (0)k = 6ti + 2j. When t = 1, we get a = 6i + 2j
Integration of Vector Functions
Let vector function f(t) = f1(t)I + f2(t)j + f3(t)k where its components f1(t), f2(t) and f3(t) are
continuous in a specified interval. Then the equation

 f(t) dt  i f1 (t) dt  j f2 (t)dt  k  f3 (t) dt defines as indefinite integral of f(t). If there exists a
d
vector g(t) such that f(t) = dg(t)/dt, then,  f(t)dt   dt  g(t) dt  g(t)  c
-:208:-
CHAPTER TEN VECTOR ANALYSIS WITH APPLICATIONS
. c being the constant of integration and is independent of t.
The definite integral between limits t = a and t = b can be written in such a way
b bd
 f (t)dt =   g(t)  dt =  g + c a = g(b) + c – g(a) – c = g(b) – g(a)
b
a a dt
Example 02: Determine the vector function which has 2cos 2ti + 2 sin 2t j + 4k as its derivative
and 2i – 3j + k as its value at t = 0.
Solution: Let f `(t) = 2ti + 2 sin 2t j + 4k. Integrating,

 f `(t) dt  2i cos 2tdt  2j sin 2t dt  4k  1. dt  c


 sin 2t   cos 2t 
f (t)  2i    2j   4k  t   c  sin 2t i  cos 2t j  4t k  c
2 
or (1)
 2  
Put t = 0, and f(t) = 2i – 3j + k in (1), we get c – j = 2i – 3j + k giving c = 2i – 2j + k. Thus,
equation (1) becomes after putting the value of c
. f(t) = (sin 2t + 2)i – (cos 2t + 2)j + (4t + 1)k
This is the required vector function.
Example 03: A particle moves such that its acceleration is given by f ``(t) = (t4 + 2t)i + t2j – t k. Find
the velocity and displacement vector given that f `(t) = i + j – k and f(t) = i + 2j + k when t = 0.
. .

4 2
Solution: It is given that, a = dv/dt = f ``(t) = (t + 2t)i + t j – t k.

 f ``(t) dt  i (t  2t) dt  j t 2 dt  k  t. dt  c . This gives,


4
Integrating, we get:

1  1  1 2
f `(t) = v =  t 5  t 2  i +  t 3  j –  t k + c (1)
5  3  2 
. .

Putting t = 0 and f `(t) = i + j – k , we get c = i + j – k. Thus, equation (1) becomes after


. .

simplification:
1  1  1 2 
f `(t) = v =  t 5  t 2  1 i +  t 3  1 j –  t  1 k (2)
5  3  2 
. .

This is the required velocity vector. Now, we know that


1  1  1 2 
v = dr/dt = f `(t) =  t 5  t 2  1 i +  t 3  1 j –  t  1 k
5  3  2 
. .

Here, r is a displacement vector. Integrating and simplifying, we get


 1 6 1 3   1  1 
r = f(t) =  t  t  t  i +  t 4  t  j –  t3  t  k + c (3)
 30   12  6 
. .

3
To find c put t = 0 in (3), we get: r = f (0) = c or i + 2j + k = c. Put this value of c in (3) and
.

 1 6 1 3  1  1 3 
simplify, we obtain: r = f(t) =  t  t  t  1 i +  t 4  t  1 j  t  t  1 k
 30 3   12  6 
This is the required displacement of the particle.
DEL – THE DIFFERENTIAL OPERATORS
3
Let r = xi + yj + zk be the position vector of a point P(x, y, z) in a given region of the space R .
A function f(t) which, associates a unique vector f(r) with each vector r in the given region of
space is called vector point function. We can write f(r) = as f(x, y, z).
-:209:-
CHAPTER TEN VECTOR ANALYSIS WITH APPLICATIONS
A function φ which associates a unique scalar  (r) with each vector r in a given region of space
is called scalar point function. We may write  (r) as  (x, y, z).
Domain of a vector point function is called vector field and domain of a scalar point function is
called a scalar field.
Definition: The differential operator  , called del or “nebla” is defined as
 ˆ  ˆ  ˆ
 i  j k
x y z
This differential operator behaves like a vector, so it is also called a vector differential operator.
This differential operator has an important role in three dimensional physical problems.
The Gradient
Let   x, y,z  be a scalar point function with domain D. Suppose  ,  and  are continuous
x y z
in D. Then the gradient of  written  or grad  , is defined by
  ˆ ˆ    
   ˆi  j  k        ˆi  ˆj  kˆ
 x y z  x y z
Geometrical Interpretation of Gradient
Consider a surface   x, y,z   c . Let P(x, y, z) be any point on this surface. Join OP and let
OP = r. Since, r  xiˆ  yjˆ  zkˆ  dr  dxiˆ  dyjˆ  dzkˆ
Now, the differential of multivariable   x, y,z   c is given by:

d 

x
 
dx  dy  dz  0
y z

 
  ˆ
x
  
y z  
i  ˆj  kˆ  dx ˆi  dy ˆj  dz kˆ  0 
  . dr   0
Since  is the gradient of a scalar field function   x, y,z  that is tangential to the surface
  x, y,z   c and the dot product of  and dr is zero, this implies that  and dr are
perpendicular to each other. Now, dr lies on the surface   x, y,z   c , this shows that grad  is
perpendicular to the surface   x, y,z   c . [This is shown in the following figure]
NOTE: Observe that grad  is seen to be perpendicular to the position vector r. Since, r lies on
the surface   x, y,z  = c hence grad φ is normal to the surface   x, y,z  = c.
Directional Derivative grad    r
The directional derivative of a scalar function   x, y,z  in the P

direction of the position vector r is defined as    P


 . r
(x, y, z)  c
r |r|
Here, ( ) P means the value of grad  is taken at the point P. O
(Surface)

Example 02: Find a unit vector normal to the surface φ  x, y,z  = x3 y2 - z2 yx 2 = 9 at


(1, 2, 1).
   ˆ  3 2 2 2
Solution: First we find   By definition,    ˆi  ˆj  k   x y  z yx  9 
 x y z 

-:210:-
CHAPTER TEN VECTOR ANALYSIS WITH APPLICATIONS
 3 2 2 2  
x 
 ˆi x y  z yx  9   ˆj  x 3 y2  z2 yx 2  9   kˆ  x 3 y2  z 2 yx 2  9 
y z
 ˆi  3x 2 y2  2xyz 2   ˆj 2x 3 y  x 2 z 2   kˆ  2x 2 yz  .
  3x 2 y2  2xyz 2  ˆi   2x 3 y  x 2 z 2  ˆj  2x 2 yzk.
ˆ At (1, 2, 1)

 2 2 3
  2 2 2

 (1, 2,1)  3 1  2  2 1 2 1 ˆi  2 1  2   1 1 ˆj  2 1 11 kˆ  8iˆ  3jˆ  4kˆ
2

Hence, a unit vector normal to the given surface is


 (1, 2,1) 8iˆ  3jˆ  4kˆ 8iˆ  3jˆ  4kˆ

 (1, 2,1)

2 2 2

89

1 ˆ ˆ ˆ
89
8i  3j  4k  
 
8  3  4    
2 2 2
Example 02: Find the directional derivative of 4x y z at the point (1, 2, 1) in the direction of
2i + j + 2k.
Solution: Let f  x, y,z  = 4x 2 y2 z 2 ,P = 1,2,1 and a = 2i + j + 2k. Then the required directional

derivative is given by  and is defined as    P . r 1


r r |r|
     
Now,    ˆi  ˆj  kˆ   4x 2 y2 z 2   ˆi  4x 2 y 2 z 2   ˆj  4x 2 y 2 z 2   kˆ  4x 2 y 2 z 2 
 x y  z  x  y z
= 8xy2 z 2 ˆi +8x 2 yz 2 ˆj+8x 2 y2 zk.
ˆ
Now, at (1, 2, 1) :     8 1 2  1 ˆi  8 1  2 1 ˆj  8 1  2  1 kˆ  32iˆ 16jˆ  32kˆ
2 2 2 2 2 2
P

2iˆ  ˆj  2kˆ 2iˆ  ˆj  2kˆ 2iˆ  ˆj  2kˆ 1 ˆ ˆ ˆ


Also, rˆ 
r
  
3 
 2i  j  2k
3 
r  2  1   2  9
2 2 2

  
Hence, from (1), we have: d  1 2iˆ  ˆj  2kˆ  32iˆ  16jˆ  32kˆ  1  2  32   116    2  32   48
dr 3 3
Example 03: Find the rate of change of  = xyz in the direction normal to the surface
2 2 2
x y + y x + yz = 3 at the point (1, 1, 1).
Solution: It may be noted that rate of change of  is equal to dot product of grad  and the unit
normal vector to the surface  (x, y, z) = c. Now,
 ˆ ˆ 

grad    ˆi  j  k   xyz   yz ˆi  xz ˆj  xy k. ˆ At (1, 1, 1) the rate of change is,
 x  y z 
therefore, grad   ˆi  ˆj  kˆ
The normal to the surface   x 2 y  y2 x  yz 2  3 is
   
grad    ˆi  ˆj  kˆ   x 2 y  y2 x  yz 2  3   2xy  y2  ˆi   x 2  2xy  z 2  ˆj  2yz kˆ
  x y z
At 1, 1, 1 , grad   3iˆ  4jˆ  2k.
ˆ Thus unit vector in the direction of grad  is

grad  3iˆ  4jˆ  2kˆ 3iˆ  4jˆ  2kˆ


û   
| grad  | 9  16  4 29
 3iˆ  4jˆ  2kˆ 
Now, rate of change of  is given by :  grad   .  uˆ   ˆi  ˆj  kˆ .   
9
  29 
 29

-:211:-
CHAPTER TEN VECTOR ANALYSIS WITH APPLICATIONS
2
Example 04: Find the values of m and n such that the surface mx – 2nyz = (m + 4)x will be
2 3
perpendicular to the surface 4x y + z – 4 = 0 at the point (1, –1, 2).
2
Solution: Let the surfaces be  1 = mx – 2nyz – (m + 4)x (1)
2 3
 2 = 4x y + z – 4 (2)
Since the point (1, -1, 2) lies on both surfaces, putting (1, –1, 2) in (1), we get
m  4n  m  4  n 1 (3)
grad 1
Normal to surface 1 at the point (1, 1,2)   (m  4)iˆ  4jˆ  2kˆ
grad 1
grad 2
Normal to surface 2 at the point (1, 1,2)   8iˆ  4jˆ  12kˆ
grad 2
 ˆ  ˆ   
 u se the grad formula grad    i  j  kˆ   and substitute (x, y,z)  (1, 1, 2) 
  x y z  
Since both surfaces are normal hence their dot product is zero. Thus,
  
(m  4)iˆ  4jˆ  2kˆ . 8iˆ  4jˆ  12kˆ  0   8(m  4) 16  24  0  m  5
2 2 2 2 2
Example 05: Find the angle between the two surfaces x + y + z = 9 and z = x + y –3
intersecting at the point (2, –1, 2).
Solution: It may be noted that angle between two surfaces is the angle between their normal at
the point of their intersection.
grad 1
Normal to surface 1  x 2  y2  z2  9 at the point (2, 1,2)   4iˆ  2jˆ  4kˆ
grad 1
grad 2
Normal to surface 2  x 2  y2  z  3 at the point (2, 1,2)   4iˆ  2jˆ  kˆ
grad 2
   ˆ ˆ  
 u se the grad formula grad    ˆi  j  k   and substitute (x, y,z)  (2, 1,2) 
  x y z  
 a1b1 + a 2 b2 + a 3b3
The angle between the two vectors is : θ = cos-1  
 a +a +a
 1
2
2
2
3
2
b1 + b2 + b3 
2 2 2

θ = cos-1 8 / 3 21  54.414o

Example 06: Find the directional derivative of 1/r in the direction of r  xiˆ yjˆ zkˆ .
1/2
Solution: Since r  xiˆ yjˆ zkˆ  r | r | x 2  y2  z2 then   x 2  y2  z 2 
1
r
 1/2 ˆ  2 2 2 1/2 ˆ  2 2 2 1/2 ˆ
 grad(1/ r)   x 2  y2  z 2  i  x  y  z  j x  y  z  k
x y z
3/2 ˆ 3/2 ˆ 3/2 ˆ xiˆ  yjˆ  zkˆ
  x  x 2  y2  z 2  i  y  x 2  y2  z 2  j  z  x 2  y2  z 2  k 
 x 2  y2  z 2 
3/2

 Directional derivative of 1/ r in the direction of r is :

grad(1/ r).r

 
xiˆ  yjˆ  zkˆ . xiˆ  yjˆ  zkˆ 

 x 2  y2  z 2  1
 2 2 2  2
1
 x 2  y2  z 2   x 2  y2  z 2   x 2  y2  z 2   x  y  z  r
r 3/2 1/2 2

-:212:-
CHAPTER TEN VECTOR ANALYSIS WITH APPLICATIONS
Example 07: Find the direction in which the directional derivative of   x 2  y2 / xy at the  
point (1, 1, 0) is zero. Also find the component of velocity of the vector r  (t 3  1)iˆ  t 2 ˆj in the
same direction at time t = 1.
Solution: Consider
  x 2  y2  ˆ   x 2  y2    x 2  y2  ˆ  x 2 y  y3  ˆ  xy2  x 3  ˆ
grad    i    j  k   2 2 
i 2 2  j
x  xy  y  xy  z  xy   x y   x y 
1 1 ˆ 1 1 ˆ
At (1, 1), grad     i   1.1  j  0
 1.1   

 
Thus, directional derivative of the surface   x 2  y2 / xy  c is zero in all directions.

Now, r  (t 3  1)iˆ  t 2ˆj  0kˆ  v   3t 2ˆi  2tj.


ˆ At t  1, v  3iˆ  2j.
ˆ
dr
dt
Thus component of velocity in the same direction of velocity is equal to :
3iˆ  2jˆ
 
3iˆ  2jˆ . 2 2 
3 2
94
94

13
13
 13

2
Example 08: Find the directional derivative of  x2 yz  4xz at the point (1,  2,1)

in the direction of 2iˆ ˆj 2k.


ˆ Find the greatest rate of invrease of .
Solution: Consider

grad  
 2
x 
x yz  4xz ˆi   2
y
 2

x yz  4xz ˆj 
2  2
z  
x yz  4xz kˆ
2


  2xyz  4z 2  ˆi   x 2 z  ˆj  x y  8xz k.
2 ˆ

At(1,  2,1), grad   (4  4)ˆi  ˆj   2  8 kˆ  ˆj  6kˆ
Now consider the vector a  2iˆ ˆj 2k. ˆ Then unit vector in the direction of itself is
a 2iˆ ˆj 2kˆ 2iˆ ˆj 2kˆ
â   
|a| 4 1 4 3
It may be noted that greatest rate of increase of  is nothing but its absolute value.
That is,  
| grad  | ˆj  6kˆ  12  62  37

This is the greatest rate of increase of  .


Finally, the directional derivative of  in the direction of vector a is

  
grad . aˆ  ˆj  6kˆ . 2iˆ ˆj 2kˆ / 3  13 / 3
2x –y + z
Example 09: Find the magnitude of directional derivative of  = 4 e at the point [1, 1, –1]
in the direction towards the point [–3, 5, 6].
Solution: Let
a  [1, 1,  1] and b  [3,5,6]. Therefore, c  b  a  [3 1,5 1,6  1]  [4,4,7]. Now,
grad  

x  
4e2xyz ˆi 

y 
4e2xyz ˆj 

z  
4e2xyz kˆ  4 e2xyz 2iˆ  ˆj  kˆ  
  
At 1,1,  1 , grad   4  e211  2iˆ  ˆj  kˆ  4 2iˆ  ˆj  kˆ . 
-:213:-
CHAPTER TEN VECTOR ANALYSIS WITH APPLICATIONS
Thus, directional derivative of  in the direction of vector c that is, from a to b is
4iˆ  4jˆ  7kˆ 
c ˆ

ˆ ˆ
grad .  4 2i  j  k .
|c |

16  16  49
4
8  4  7
81
20
 .
9
Neglecting negative sign, the required directional derivative is  20 / 9.
2 2
Example 10: Find the magnitude of the directional derivative for the function φ = x/(x + y )
o
along the line making an angle of 30 with the positive x-axis at (0, 2, 0).

Solution: grad     x  ˆi    x  ˆj    x  kˆ  y  x ˆi  2xy ˆj


2 2

x  x 2  y2  y  x 2  y2  z  x 2  y2 
 x 2  y2   x 2  y 2 
2 2

4 ˆ 2.0.2 ˆ 1 ˆ
Thus grad  at (0,2,0) is grad   i j i
42  0  42 4

It may be noted that if a line makes an angle 30o then


hypotenuse is 1, perpendicular will be 1/2 and base will
be 3 / 2 . Thus vector CA = cos 30o i + sin 30o j
 3 / 2iˆ  1/ 2 ˆj . Thus directional derivative of  in the
direction of vector CA is
CA 1 ˆ  3 ˆ 1 ˆ  3
grad .  i . i  j 
| CA | 4  2 2  8
The Divergence of a Vector Function
In the previous section, we have seen that the differential operator  operates on a scalar
function and we got a vector function. In this section we shall see that this operator will convert a
vector into a scalar when it applies on it.
Definition: Let f  x, y,z  = f ˆi + f ˆj+ f kˆ be defined and differentiable at each point (x, y, z) in a
1 2 3

certain region of space. Then the divergence of f , written   f or div f , is defined by


f f f
  ˆ  ˆ  ˆ
 f  
 x y z  
i  j  k   f1ˆi + f 2ˆj+ f 3kˆ  1  2  3
x y z 
Note: If the divergence of a vector function v is zero, that is,  v = 0, then v is called
Solenoidal Vector Function.
Example 11: For what value of constant P, the vector function defined below is solenoid at
(1, 2, 3) where F = 2x 2 yziˆ + x 2 y3ˆj- Pxyz 4 kˆ
Solution: We have

 F  
  ˆ  ˆ  ˆ
 x y z 

i  j  k   2x 2 yziˆ  x 2 y3ˆj  xyz 4 kˆ 


x
2x 2 yz 
y
 2 3
x y 


z 
xyz 4   
 4xyz  3x 2 y2  4Pxyz3

Since F is solenoid, therefore 4xyz  3x2 y2  4Pxyz3 = 0 At 1, 2,3 :

4 1 23  31  2  4P 1 23  0  36  216P  0  P  1 6


2 2 3

-:214:-
CHAPTER TEN VECTOR ANALYSIS WITH APPLICATIONS
Physical Interpretation of Divergence
Though divergence is used in many different subjects where its applications can be seen, we here
produce one of its practical uses in fluid mechanics.
Consider a case of a fluid flow. Let
there be a small rectangular
parallelepiped with sides dx, dy and
dz respectively.
Let V = Vxi + Vyj + Vzk be the
velocity of the fluid at the point
P(x, y, z). Now, the mass of the fluid
through the face ABCD in unit time
Vx = velocity × area of the face
= Vx dy. Dz.
Therefore, mass of the fluid flowing
out across the face PQRS per unit time
= Vx (x + dx).dy dz =  Vx  Vx .dx  (dy.dz)
 x 
Finally, the net decrease in mass of fluid in the parallelepiped is equivalent to the flow along x-
axis per unit time  Vx (dy.dz)   Vx  Vx .dx  (dy.dz)   Vx dxdydz .
 x  x
Similarly, the net decrease in mass of fluid in the parallelepiped equivalent to the flow along y-
V
axis per unit time   y dxdydz .
y
Thus, the net decrease in mass of fluid in the parallelepiped equivalent to the flow along z-axis
per unit time   Vz dxdydz .
z
So, the total decrease in mass of fluid in the parallelepiped per unit time is
 V V V   Vx Vy Vz 
   x  y  z  dxdydz 
 x
   dxdydz  Taking the absolute value  .
 x y z   y z 
 Vx Vy Vz 
    dxdydz
Therefore, the rate of loss of fluid per unit volume =  x y z 
dxdydz
 Vx Vy Vz   Vx ˆ Vy ˆ Vz ˆ 

 x

 
y z
  
  x
i
y
j
z  x  
k  V ˆi  Vyˆj  Vz kˆ  divV  .V .

Thus divergence of a vector point function is nothing but rate of loss of fluid per unit volume.
This holds for non-compressible flow. If the flow is compressible its divergence is always zero,
that is, div V  .V  0 . This is known as continuity equation or conservation of mass. It may
further be noted that compressible flow is that whose density is not constant. On the other hand, if
the flow is compressible, its density is always constant.
Example 12: Find div  r  where r  xiˆ  yjˆ  zkˆ and r  x 2  y 2  z 2
r
Solution: Given that r  xiˆ  yjˆ  zkˆ and r  x 2  y2  z2 . Now,
r 1 2 r y r z
  x
1/2
 x  y2  z 2 .2x    and 
x 2 r y r z r

-:215:-
CHAPTER TEN VECTOR ANALYSIS WITH APPLICATIONS
r  x  y  z
 div      ˆi    ˆj    kˆ (1)
 r  x  r  y  r  z  r 
r x
r.1  x. r.1  x.
 x x  r  r  x . Similarly,
2 2

  
x  r  r r
2 2
r3
  y  r 2  y2   z  r2  z2
   and   . Thus (1) becomes
y  r  r3 z  r  r3


div 
r    x    y    z  3r  x  y  z
   
2 2 2 2


3r 2  r 2 2
 
 2
0
      
 r  x  r  y  r  z  r  r3 r3 r x 2  y2  z2
Example 13: Find div  u r  where r  xiˆ  yjˆ  zkˆ and u  x 2  y2  z2 .
Solution: Given that r  xiˆ  yjˆ  zkˆ so that r 2  x2  y2  z2  u . Now,
       
div  ur    ˆi  ˆj  kˆ  .  uxiˆ   uyjˆ  uzkˆ    ux    uy    uz  (1)
 
 x y z  x y z
   
Now,
x x
 
 u   x 2  y2  z 2  2x   u   2y and  u   2z
y z
 x u
  ux   u  x  u  x.2x  u  2x 2 .Similarly,
x x x
 
 uy   u  2y 2 and  uz   u  2z 2 . Thus equation (1) becomes
x x
  
div  ur   3u  2 x 2  y 2  z 2  3u  2u  5u  5 x 2  y 2  z 2 
Example 14: Find the value of n for which the vector function r n r is solenoid
where r  xiˆ  yjˆ  zkˆ a position vector and r  x 2  y2  z2 .
Solution: Consider
 (xr n )  (yr n )  (zr n )
   
div r n r  . r n r 
x

y

z
1
r 1 2
  x
1/2
Now r  x 2  y 2  z 2   x  y2  z2 .2x 
x 2 r
r x r y r z
Thus,  . Similarly,  and   2
x r y r z r
 (xr n ) n r x
Now,  r .1  x.nr n 1.  r n  nxr n 1.  r n  nx 2 r n 2  Using eq. (2) 
x x r
 (y r ) n
n
 (z r ) n
n
Similarly,  r  ny 2 r n 2 and  r  nz 2 r n 2
y z
   
Thus (1) becomes :div r n r  3r n  nr n 2 x 2  y 2  z 2  3r n  nr n 2 r 2  3r n  nr n  (3  n)r n
Now it is given that vector function r n r is solinoid hence div r n r  0  
 (3  n)r n  0. But r n  0 therefore, n  3  0  n  3
Example 15: Find the directional derivative of div(u) at the point (1, 2, 2) in the direction of the
outward normal of the sphere x2 + y2 + z2 = 9 given that u  x 4 ˆi  y 4 ˆj  z 4 kˆ .

-:216:-
CHAPTER TEN VECTOR ANALYSIS WITH APPLICATIONS
 
 x y z 

Solution: div(u)    ˆi   ˆj   kˆ  . x 4ˆi  y 4ˆj  z 4kˆ  4 x 3ˆi  y3ˆj  z 3kˆ .
 
  

Now grad  div(u)   4 3x i  3y j  3z 2 kˆ  12 x 2 ˆi  y 2 ˆj  z 2 kˆ
2ˆ 2ˆ
  

At (1, 2, 2), div(u)  12iˆ  48jˆ  48kˆ . Now, 
Outward normal of the sphere  grad( )  grad(x 2  y 2  z 2  9)  2(xiˆ  yjˆ  zk)
ˆ


At (1, 2, 2), grad()  2 ˆi  2ˆj  2kˆ  2iˆ  4ˆj  4kˆ   
 Directional derivative of div(u) in the direction of sphere x 2  y 2  z 2  9 at (1, 2, 2) is

 2iˆ  4ˆj  4kˆ    24  192  192  408  68



 12iˆ  48jˆ  48kˆ . 
22  42  42 6 6
Example 16: Show that div r r =  n + 3  r 
n n

Solution: Let ˆ zkˆ and  r  =  x 2 + y2 + z 2 


r = xiˆ + yj+
n n2

Therefore, , divr n r    r n r  
  ˆ  ˆ  ˆ   2 2 2 n/2 ˆ ˆ ˆ 
 x
i  j  k    x  y  z  xi  yj  zk 
y z   
 
 ˆ  ˆ  ˆ   2 2 2 n/2 ˆ

i  j  k    x  y  z  xi   x 2  y 2  z 2  yjˆ   x 2  y 2  z 2  zkˆ 
n/2 n/2
  rn r  
 x y z   
  
  x 2  y2  z 2  x   x 2  y2  z 2  y   x 2  y2  z 2  z
n 2 n 2 n 2

x y z
n 1 n n 1 n
n  n 
   x 2  y 2  z 2  2  x   2x    x 2  y 2  z 2  2     x 2  y 2  z 2  2  y   2y    x 2  y 2  z 2  2 
2  2 
n 1 n
n 

2
 x 2  y2  z 2  2  z   2z    x 2  y2  z 2  2 

n 1 n n 1 n
   
 n

 
x 2  y2  z 2 2 x 2   
x 2  y2  z 2 2  n
 
 
 x 2  y2  z 2  2 y2   x 2  y2  z 2  2 

n 1 n
 
 n

 
x 2  y2  z 2 2 z 2   x 2  y2  z 2 2 

n 1 n
 n  x 2  y2  z 2  2  x 2  y2  z 2   3  x 2  y2  z 2  2
n 1 n
 n  x 2  y2  z 2  2  x 2  y2  z 2   x 2  y2  z 2   3  x 2  y 2  z 2  2
n n n
 n  x 2  y2  z 2  2  3  x 2  y 2  z 2  2  div r n r   n  3  x 2  y 2  z 2  2   n  3  r 
n

Note: If rn r is solenoid, then  n  3  r   0  n  3  0  n  3


n

The Curl of a Vector Function


In the previous section we have discussed the dot product of del operator  with a vector. In this
section we shall discuss the cross product of  with a vector.

Definition: If f  x, y,z  is a differentiable vector field then the curl or rotation of f  x, y,z  is

denoted by Curlf or   f or rot(f ) and is defined as

Curlf   f  
  ˆ  ˆ  ˆ
 x

i  j  k   f1ˆi  f 2ˆj  f 3kˆ
y z 

-:217:-
CHAPTER TEN VECTOR ANALYSIS WITH APPLICATIONS
ˆi ˆj kˆ
     

   ˆ y z ˆ x
i j z  kˆ x y   f3  f 2  ˆi   f3  f1  ˆj   f 2  f1  kˆ
x y z  y z   x z   x y 
f 2 f3 f1 f3 f1 f2
f1 f 2 f3

Note that in the expansion of the determinant, the operators


   must precede f ,f ,f 
, ,
x y z 1 2 3

Definition: Let f  x, y,z  be a vector field. If Curl of f  x, y,z  is a zero vector, that is,
Curlf  0, then f  x, y,z  is called irrotational since the perpendicular velocity is zero.
Example 17: If f =  x + 2y + az  ˆi +  bx - 3y - z  ˆj+  4x + cy + 2z  k,
ˆ find a,b,c, so that f  x, y,z 
is irrotational.
Solution: Given that f =  x + 2y + az  ˆi +  bx - 3y - z  ˆj+  4x + cy + 2z  k,
ˆ
By definition,
    
Curlf   f   ˆi  ˆj  kˆ    x  2y  az  ˆi   bx  3y  z  ˆj   4x  cy  2z  kˆ 
 x y z   
ˆi ˆj kˆ
      

x y z
 î 
y
 4x  cy  2z   z  bx  3y  z 
 
x  2y  az bx  3y  z 4x  cy  2z
     
ˆj   4x  cy  2z   z  x  2y  az   kˆ  x  bx  3y  z   y  x  2y  az 
 x   
ˆ ˆ ˆ ˆ ˆ ˆ
Thus,  f   c  1 i   4  a  j   b  2  k   c  1 i  a  4  j   b  2  k
If f  x, y,z  is irrotational, then  f  0
  c  1 ˆi   a  4  ˆj   b  2  kˆ  0  c  1  0,a  4  0,b  2  0
 c  1,a  4,b  2
Hence the required values are a = 4, b = 2, c = – 1
Two Important Theorems
Prove that: 1. div(curlf )  0 2. curl (grad )  0
Proof: (1) By definition, div (curl f )  ( f )
i j k  / x  / y  / z
   
 i  j  k  .  / x  / y  / z  / x  / y  / z  0
 x y z 
f1 f2 f3 f1 f2 f3
The result is zero as we see that to rows of the determinant are identical.
(2) By definition,
i j k
curl (grad )    ()   / x  / y  / z
 / x  / y  / z

    
  2 / yz   2 / zy i   2 / xz   2 / zx j   2 / yx   2 / xy k 
 (0) i  (0) j  (0) k  0

-:218:-
CHAPTER TEN VECTOR ANALYSIS WITH APPLICATIONS
Physical Interpretation of curl
We know that the relationship between linear velocity v , and the angular velocity w is given by
the formula: v  w  r
ˆi ˆj kˆ
Now, w  r  w1 w2 w 3  ˆi  w 2 z  w 3 y   ˆj  w1z  w 3x   kˆ  w1y  w 2 x 
x y z
Therefore, curl v    v     w  r 
ˆi ˆj kˆ



x

y

z
 
 2 w1ˆi  w 2 ˆj  w 3kˆ  2w

 w 2 z  w 3 y    w1z  w 3x   w1y  w 2 x 
Thus curl of vector field is twice the a angular velocity.

r
Example 18: Find curl   where r is a position vector and r  x 2  y 2  z 2
r
Solution:

i j k
r       z    y     z    x     y    x 
curl     i        j        k      
 r  x y z  y  r  z  r    x  r  z  r    x  r  x  r  
x y z
r r r
1
r.0  z. 2yr 1
 z 2 yz   y  zy  z  y
Now,    2 . Similarly,    2 . Thus,       0
y  r  r 2
r z  r  r y  r  z  r 
r
Continuing this way, we see that each term in the braces is zero. Thus, curl    0
r
Example 19: Show that the vector function
f   y 2  z 2  3yz  2x  i   3xz  2xy  j   3xy  2xz  2z  kˆ is solinoid and irrotational?

Solution: Consider, div f  .f    y2  z 2  3yz  2x     3xz  2xy     3xy  2xz  2z 


x y z
= – 2 + 2x – 2x + 2 = 0
Since div f  0 hence, vector function f is solinoid.
Now consider
ˆi ˆj kˆ
  
curl f    f 
x y z
y 2
 z 2  3yz  2x   3xz  2xy   3xy  2xz  2z 
     
 ˆi   3xy  2xz  2z    3xz  2xy    ˆj   3xy  2xz  2z    y 2  z 2  3yz  2x  
 y z   x z 
-:219:-
CHAPTER TEN VECTOR ANALYSIS WITH APPLICATIONS
  
 kˆ   3xz  2xy    y 2  z 2  3yz  2x    ˆi  3x  3x   ˆj  3y  3y   kˆ  3z  3z   0
 x y 
Since, the curl of vector function f is zero hence vector function f is irrotational.
Note: If a vector function f is irrotational then its scalar potential field  is given by
d  grad ().dr  f.dr, where f  grad()
Example 20: Prove that vector function f   x 2  y 2  x  ˆi   2xy  y  ˆj is irrotational
is irrotational. Hence find its scalar potential function.
Solution: Consider
ˆi ˆj kˆ
  
curl f    f 
x y z
x 2
 y 2  x    2xy  y  0
        
  .0   2xy  y   ˆi   .0   x 2  y 2  x   ˆj    2xy  y    x 2  y 2  x  kˆ
 y z   x z   x y 
 0iˆ  0ˆj   2y  2y  kˆ  0
Since curl of f is zero hence f is irrotational vector function.
To find scalar potential function we proceed as under:
Let f    grad(). Now by definition
   ˆ  ˆ 
d 

x
dx 

y
dy 

z
dz   ˆi 
 x y
j
z 

k  . dxiˆ  dyjˆ  dzkˆ  grad().dr

 f .dr   x 2  y 2  x  ˆi   2xy  y  ˆj . dxiˆ  dyjˆ  dzkˆ 
  x 2  y 2  x  dx   2xy  y  dy. Integrating both sides, we have
    x 2  y 2  x  dx     2xy  y  dy  c
x3 x2 y2
  y x   xy   c. Thus,
2 2

3 2 2
3 2 2
x x y
     2xy 2  c. This is the required scalar potential function.
3 2 2
2
  Operator
Since    i   j   . We
x y z
  2 2 
have  2   .    i   j  k  .   i   j  k     2   2   2 
   2

 x y z   x y z   x y z 

Example 18: Evaluate  (r), where r  x ˆi  y ˆj  z kˆ and r  x  y z


2 2 2 2

 2 2 
Solution: By definition  2 (r)    2   2   2  (r)
2
 x y z 

 
2   r    x  x2  y2 z2 .1 x.(2x)(x 2  y2 z2 )1/2
Thus (r)   
x  x  x  x 2  y2 z2 
 
x 2 (x 2  y2 z2 )
 

-:220:-
CHAPTER TEN VECTOR ANALYSIS WITH APPLICATIONS
(x 2  y2 z2 )2 x 2 ( x 2  y2  z2 )
  2 2 2 3/2
(x 2  y2 z2 )3/2 (x  y z )
2 (x 2  y2 z2 ) 2 (x 2  y2 z2 )
Similarly, ( r)  and ( r) 
y
2 (x 2  y2 z2 )3/2 z 2 (x 2  y2 z2 )3/2
2 (  x 2  y2  z 2 ) (x 2  y2 z2 ) (x 2  y2 z2 )
  ( r)  2 2 2 3/2  2 2 2 3/2  2 2 2 3/2
(x  y z ) (x  y z ) (x  y z )

(  x 2  y2  z 2  x 2  y2  z 2  x 2  y2  z 2 ) (x 2  y2 z2 ) 1
    1r
 
3/2
 x 2  y2  z 2  x  y z
3/2
x 2  y2  z 2 2 2 2

WORKSHEET 10
1. Find the vector whose magnitude is that of the vector 5i – 3j + 9k and is in the direction of the
vector 4i – 3j + k.
2. Find the value of ‘t’ if the vectors 5i – 3j + 9k and 4i – tj + k have same direction.
3. For what value of p the vectors a = [2, 4, -7] and b = [2, 6, p] are perpendicular?
4. Find the area of the parallelogram determined by 3i + 4j and i + j + k.
5. Find the volume of parallelepiped if a = [3, 4, 0], b = [2, 3, 4] and c = [0, 0, 5] are its edges.
6. Find the volume of the tetrahedron whose vertices are the points A(2, -–1, –3), B(4, 1, 3),
C(3, 2, –1) and D(1, 4, 2).
7. The coordinates of a moving particle are given by x = 4t – t2/2, y = 3 + 6t – t3 and z = 3t2. Find
the velocity and acceleration of the particle when t = 2 sec.
8. A particle moves so that its displacement at time t is given by:
x(t) = 2cos t i + 2sin t j + tk. Find the magnitude of the velocity and acceleration of the particle at
t = 0.
9. An acceleration of a particle is given by: f ``(t) = t2i + t2j – (t – 2)k
Find the velocity and displacement vector given that f `(0) = i + j + k and f(0) = 2i +3j +k
10. Define gradient, divergence and curl. If  = 3x yz , find  at (2, 1, 2).
2 2

11. Find a unit vector normal to the surface x3y2 – z2yx2 = 8 at (1, 2, 1).
12. Find a normal vector of magnitude 5 to the surface x2y2 + xy2z = 10 at (1, 2, 1).
13. Find the directional derivative of f = 4x2y2z2 at 1, 2,1 in the direction of 2i + j + 2k.
14. Find the directional derivative of f = x2y2 + y2z2 at 1,1, 2 in the direction of i + j + k.
2 2 2 2
15. Find the value of the constant “C” so that the vector function f = x y zi + (x + Cy )j – xyz k
is Solenoid at (1, 2, 1). (Here we note a vector by f or f )
16. For what value of constant “C”, the vector function f = (3x + y) i + (Cy + z) j + 2z k is
Solenoid.
17. Show that div r = 3 and div(r/r3) = 0 where r is a position vector.
18. Show that the curl of the position vector is zero, that is; Curl r = 0.
19. Prove that r n  n rn-2 r, where r is a position vector.
20. Show that Curl rn r = 0
21. Evaluate: (a) 2 r 2 (b) 2 ln r

-:221:-
OTHER BOOKS OF FARKALEET SERIES

 Applied Calculus
 Linear Algebra and Analytical Geometry
 Differential Equations and Fourier Series
 Complex Variable and Transforms
 Complex Analysis, Probability and Statistical Methods
 Statistical Methods and Estimations
 Numerical Analysis and Computational Applications
 Laplace, Fourier and Z-Transforms
 Linear Algebra & Numerical Methods
 MCQs in Mathematics for F. Sc.-I
 MCQs in Mathematics for F. Sc-II
 Textbook with Key (Mathematics) for F. Sc-I
 Textbook with Key (Mathematics) for F. Sc-II
 Partial Differential Equations with Applications

PRICE: Rs. 310/=

JAHANGIR PUBLICATIONS, LARKANA


Cell: 0333-756-5595, 0304-453-2100

You might also like